Sunteți pe pagina 1din 530

Faculty of Actuaries

Institute of Actuaries

EXAMINATIONS
April 1999
Subject 301 Investment and Asset Management

Time allowed: Three hours


INSTRUCTIONS TO THE CANDIDATE
1.

You have 15 minutes at the start of the examination in which to read the
questions. You are strongly encouraged to use this time for reading only but
notes may be made. You then have three hours to complete the paper.

2.

You must not start writing your answers in the booklet until instructed to
do so by the supervisor.

3.

Write your surname in full, the initials of your other names and your
Candidates Number on the front of the answer booklet.

4.

Mark allocations are shown in brackets.

5.

Attempt all 12 questions, beginning your answer to each question on a


separate sheet.
AT THE END OF THE EXAMINATION

Hand in BOTH your answer booklet and this question paper.


In addition to this paper you should have available
Actuarial Tables and an electronic calculator.

301A99

Faculty of Actuaries
Institute of Actuaries

Describe, giving formulae, two methods by which an index representing


capital movements can be adjusted to produce a total return index, i.e.
including capital growth and divided income, for an equity market.
Discuss the principal limitations of each method.

Define and describe the beta of an equity portfolio.


Give two examples, including formulae, of risk-adjusted return measures
which make use of the beta of a portfolio.

[6]

[6]

Discuss the main factors which influence the choice of risk discount rate to be
used when assessing a capital project.
Describe the additional complications if the assessment were to take account
of separate borrowing and lending rates.
[10]

A call option on a particular share is at the money and is offered to you at a


premium of 10% of the exercise price. The only asset you have is a cash
deposit equal to the call option premium (interest on this cash deposit can be
ignored). Contract profit at expiry is defined as the excess return over cash on
deposit.
Explain, both in words and using a diagram, the relationship between contract
profit and share price.
[6]

(i)

Define the terms initial margin, settlement price and variation margin,
and explain why margin is levied, in the context of exchange-traded
futures.
[4]

(ii)

State the advantages and main disadvantage of dealing in derivatives


on an exchange compared to dealing in the OTC market.
[3]
[Total 7]

(i)

Define volatility for a fixed interest bond, using a formula.

(ii)

Bond A has an annual coupon of 6% and is redeemable in 3 years. Bond


B also has an annual coupon of 6% but is redeemable in 30 years. The
volatility of Bond A is 2.673 and the volatility of Bond B is 13.765.

[1]

Estimate the % change in price for each bond, if the yield curve changes
such that yields at the short end rise by % and yields at the long end
fall by %.
[3]

3012

(iii)

Outline a scenario of current and future inflation which is consistent


with this change in the yield curve.
[3]
[Total 7]

(i)

State a formula relating the rental yield of properties to the dividend


yield on equities. Suggest an alternative formula appropriate for
properties with low rental growth, stating any assumptions.
[3]

(ii)

Explain the difference between portfolio-based indices and barometer


indices for property investments.
Comment on the suitability of each of these types of index for assessing
the performance of a property portfolio.
[3]
[Total 6]

List the taxation factors which an individual will consider in selecting


investments which maximise the after tax return.

[6]

Outline the additional difficulties which face an analyst conducting


fundamental investment analysis of a company based and operating in Latin
America.
[6]

10

A listed company has a long record of strong sales and profits growth relative
to its competitors within an industry where the market has been steadily
expanding. The company has recently reported results which show an
unexpected decline in profits. The share price, which had been performing
well in recent years, has fallen sharply.
You are an investment analyst new to the sector.
Explain in detail the investigations you would conduct and the comparative
factors you would assess in carrying out an analysis of the prospects for the
company and its relative attractiveness within the sector at the current share
price.
[11]

11

Economic growth has been strong for a number of years in a medium sized
developed country. In order to curb inflation the central bank has raised short
term interest rates substantially over the last few months.
Describe how the economic situation may develop over the next two years and
how this is likely to affect the level of bond and equity markets.
[15]

3013

PLEASE TURN OVER

12

(i)

State the relationship between the total return on equities and the risk
free rate of return, expected inflation and the equity risk premium.
[2]

(ii)

Explain the effect on the components of the relationship in (i) at the


point when a country enters into a prolonged recession.
[6]

(iii)

In the light of these poor domestic prospects, resident investors in the


recession-troubled country in (ii) above are considering switching into
shares in a neighbouring country. Owing to inflationary problems in
this neighbouring country, it has very high interest rates and very
strong nominal dividend growth.
Discuss the implications of this investment strategy.

3014

[6]
[Total 14]

Faculty of Actuaries

Institute of Actuaries

EXAMINATIONS
April 1999

Subject 301 Investment and Asset Management

EXAMINERS REPORT

Faculty of Actuaries
Institute of Actuaries

Subject 301 (Investment and Asset Management) April 1999 Examiners Report
Overall, the most disappointing feature was the number of bookwork questions which
were badly answered generally because the candidates gave insufficient information
to award marks. They also failed to use all of the information flagged in the questions.
In the longer questions, few candidates provided more than the basic answers, and so
were unable to distinguish themselves. Comments on individual questions appear in
italics at the end of the solution to each question.

A total return index can be calculated either by an XD adjustment or a yield


adjustment.
XD adjustment assuming dividend or interest payments are reinvested
back into the index on its ex dividend date, i.e. it is added to current market
capitalisation, the corresponding increase in the index value would be the
investment income divided by the base value. The xd adjustment is the total
accumulation of each constituent over the year as each constituent company
declares a dividend. It is returned to zero at the end of each year, and a new
accumulation is started.
The total return index (TRI) at time t is derived from the index (I) using the ex
dividend adjustment (XD) by the formula
TRI(t) = TRI(t 1) * I(t) / [I(t 1) {XD(t) XD(t 1)}]
Limitation: There is an assumption that reinvestment takes place on the ex
dividend date. It is important to ensure that tax and re-investment
assumptions are understood.
Yield adjustment the income received over the 12 months prior to time t is
I(t) * y t where y t is the dividend yield at time t
The total return is obtained by adding the yield adjustment to the capital only
index.
Limitation: Over shorter time periods, the income is estimated on a
proportionate basis; however, this only gives an approximation, as income is
not generally received uniformly over the year.

Bookwork and badly answered - some students mistook two methods to mean two
formulas for the same method (and lost potential marks).

Page 2

Subject 301 (Investment and Asset Management) April 1999 Examiners Report

The beta of a portfolio is a measure of the volatility of the portfolio relative to


movements in the whole market as measured by an index. It is defined as the
covariance of the return on the portfolio with the return on the market index,
divided by the market index return.
The risk adjusted measures which make use of beta are
Treynor Measure measure of reward per unit of systematic risk defined
as
T = (Rp r) / Bp
where Rp is the return on the portfolio
r is the risk free rate of return over the period, and
Bp is the systematic risk in the portfolio
Jensen Measure is a measure of return relative to a benchmark with the
same degree of risk
J = Rportfolio Rbenchmark , where Rbenchmark = r + Bp (Rmarket r)

Standard bookwork well answered.

First determination is whether a real or nominal rate is required.


Real rate is used in conjunction with cash flows which are determined on the
basis of present day money values which exclude the effects of future price
inflation.
Nominal rate used if the cash flows (and the financing payments) take account
of the effects of inflation.
In this case, the rate should be equal to the real rate, compounded with the
assumed average rate of price inflation.
If risk is not wholly allowed for in the cash flows, then it should be built into
the discount rate for the project.
In particular, would expect systematic risk to be allowed for by varying the
discount rate, according to the level of riskiness of the project.
The starting point for the risk discount rate should be based on the cost of
capital. The current cost of raising capital should be based on an incremental
cost allowing for an average of equity and debt capital for the entity (company)
involved.
For debt capital, this will be (index-linked rate + a corporate risk premium
based on companys credit rating) * (1 t) to allow for corporation tax.
For equity capital, this is expected total real return + an allowance for equity
risk premium.

Page 3

Subject 301 (Investment and Asset Management) April 1999 Examiners Report
If the project is deemed to have a higher level of systematic risk, then a higher
rate would apply, considered as an additional risk premium based on other
experience.
Or, by making an arbitrary addition to the rate.
While one cannot be precise in making this adjustment for risk, grossly
distorted rates (due to arbitrary additions due to risk) are unlikely to be
helpful.
On balance, it may be helpful to evaluate the project at two different rates
evaluate the project under both rates, and interpret whether the results are
meaningful.
Complications: Using different rates for net positive and net negative cash
flows will add to the complexity of calculations, in particular when there are a
probability of outcomes.
It will also lead to situations where there are multiple solutions to evaluations
of the internal rate of return on the project.
Straight bookwork relating to a new subject. Few students discussed whether to use
nominal or real interest rates and there was insufficient explanation as to the costing of
the capital. A number of students chose to concentrate on probabilistic and systematic
risk, a very small element of the question.

As the cash is not available to finance the purchase of the shares, on exercise
we must immediately sell the shares.
The contract profit will be determined by the difference between the market
price for the shares and the exercise price.
The most that can be lost is the premium paid.
If the market price rises by 10% then the receipt on exercise and resale
(ignoring transaction expenses) will equal the premium paid i.e. no loss but no
profit.
The contract profit is linearly related to the excess of the share price above
the exercise price.
This is shown diagrammatically as follows:
Contract
profit as a
% of the
premium
+100%

10%
100%
Page 4

20%

% Excess of share
price in three
months time over
exercise price

Subject 301 (Investment and Asset Management) April 1999 Examiners Report
i.e. if the share price does nothing we loose everything, if it goes up 10% we
break even and if it goes up 20% we double our money.
Quite well done with students showing a good understanding of what was required.

(i)

Margin is the collateral which each party to a futures contract must


deposit with the clearing house.
When the contract is first struck, the parties will be required to deposit
initial margin as an amount of cash or other assets (of an acceptable
form) with the clearing house for the exchange.
The required amount is changed on a daily basis to mirror risk
(variation margin).
The closing price for a contract at the end of the days trading is the
settlement price.
Variation margin is calculated by reference to the settlement price.
Margin is levied, so that in the event of a party to a contract defaulting
the clearing house will protect the other party to the contract by
ensuring the contract is fulfilled.
The clearing house will call on the margin deposited to help it fulfil the
contract. In this way, the margin finances the guarantee of the contract
provided by the clearing house.
If the price of a contract moves adversely for a party then the amount
held on the deposit with the exchange will be increased by way of
variation margin. Similarly, profits will be remitted on a daily basis.

(ii)

The advantages of exchange traded contracts compared to OTC


contracts are:
better liquidity
better market transparency
the counter party is the clearing house who will be a good credit
whereas for an OTC contract the counter party is the issuing bank
and the credit risk will depend on the strength of that bank.
The main disadvantage is that exchange traded contracts are
standardised and the particular contract needed may not be available.
In the OTC market, a contract may be constructed for your particular
need.

Poorly answered - a surprising number of students failed to define


the Settlement Price correctly and in general there was some confusion in the
differences between futures and options . Part 2 was handled better

Page 5

Subject 301 (Investment and Asset Management) April 1999 Examiners Report

(i)

If the dirty price of a bond is P and its redemption yield is y, the


volatility V is:
V=

(ii)

1 dP
P dy

For small changes in yield, the approximate proportionate change in


price for a bond is given by:

P
= V y
P
where V is the volatility of the bond and y the change in the yield.
For Bond A, the approximate % change in the price is:
2.673 0.005 100 = 1.3%
For Bond B, the approximate % change in the price is:
13.765 0.005 100 = 6.9%
(iii)

Such a change in the yield curve may occur when short term
inflationary pressures are suppressed by the authorities raising short
term interest rates.
This is reflected in the increased yield on short bonds.
The reduction in yields on long bonds reflects a reduction in the
expected long term rate of inflation.
The action of the authorities over short term inflation may have led
long bond investors to feel more confident for the prospects of long
term inflation.

Broadly well-answered.

(i)

The gap between rental yields and dividend yields can be broken down
into the following components:
(property risk premium expected rental growth) (equity risk
premium expected dividend growth).
For properties with low rental growth, a comparison of rental yield
with the gross redemption yield on conventional bonds may be more
appropriate.
In which case, the gap between rental yields and the bonds gross
redemption yield on bonds should be equal to:

Page 6

Subject 301 (Investment and Asset Management) April 1999 Examiners Report
(property risk premium expected rental growth) inflation risk
premium.
(ii)

Portfolio based indices measure rental values, capital values or total


returns of actual rental properties. An index will produce results
particular to the portfolio underlying its construction.
The barometer type of index aims to track movements in the general
property market by estimating the maximum full rental values of a
number of hypothetical rack-rented properties.
A portfolio based index may be inappropriate if the portfolio underlying
the index differs significantly from the portfolio held. Further,
typically the return on the index is a money weighted return so the
index performance will be effected by the timing of the cash flow of the
underlying portfolio. A barometer index may be unsuitable for portfolio
performance measurement since an investor could not closely match its
movement with an actual portfolio of property holdings.

Part 1 was answered reasonably well. Candidates showed a good understanding of the
main principles involved. However, marks were needlessly lost in part 2 as answers
were too short (and incomplete).

The individuals personal tax circumstances


The rate of tax charged on an investment
Tax allowances, e.g. indexation allowance, tax free limits on ISAs, etc.
The components which make up the total return on the investment
How the tax is charged on these different components of the investment
return
The timing of tax charges
Whether the tax is paid at source, or has to be paid subsequently
The liability of the investor to income and capital gains tax (and other
duties)
The extent to which losses and gains can be aggregated between different
investments or over different time periods for tax purposes.

Bookwork - reasonably well done although poorer candidates rapidly ran out of ideas.

General factors applicable to investment analysis of any company


Management ability to meet them and discuss issues
Quality of product knowledge issue
Prospects for market growth availability of good industry data
Competition identification and familiarity issues
Input costs and retained profits quality of accounting data
History availability of back data and quality of that data
Financial analysis quality of initial data

Page 7

Subject 301 (Investment and Asset Management) April 1999 Examiners Report
Sources of information its quantity and quality, accessibility, language
issues
Valuation local methodology, applicability and quality of data
Near bookwork the better candidates identified that this was more than the standard
question on overseas investment and tailored their answer accordingly. Failure to do
this lost marks.

10

Two of the principal considerations will be whether the market reaction is


knee jerk or based on a change in company expectations. Has the company
issued any statements along with its reported earnings decline?
The company will be valued on growth criteria, as a growing company in a
growing sector.
It is important to identify what has caused the profit decline this means
breaking down the results to determine components adding to the bottom line
decline, e.g.
what has happened to turnover, for this company as an absolute and
relative to sector
what has affected costs of the business are there any special expenses
have there been particular exceptional items, e.g. increase in R&D
expenditure
are there particular write-offs leading to the decline
etc.
It is also important to determine what were the preceding expectations for the
company, and for other companies operating in the same sector.
Is this merely a company specific item or are there sector wide implications
which need consideration?
Are there any particular special features of the company recent staff
changes at senior level, changes of accounting practice, takeover activity,
which may have contributed to the decline.
The balance sheet should be examined for the changes debt increases, cost
of funding debt, etc.
In terms of its relative positioning, the fall in share price may have presented
a buying opportunity relative to the sector if the fall is overdone.
It is important to realise that share price moves are based on prospective
factors.
At the same time, it is fairly often the case that bad news comes through in
several stages so it is important to ensure that if the analysis suggests that the
price fall is overdone, nevertheless there is a risk of further bad news
emerging to take the price lower before it might recover.

Page 8

Subject 301 (Investment and Asset Management) April 1999 Examiners Report
The business sector is not described. Specific sector analysis will also be
required.
This question was answered very poorly. In particular, many candidates became
bogged down in how to analyse a company. This question was about a specific event, i.e.
how to analyse the unexpectedly poor results of a growth company in a growth sector,
and its effect on the share price. Of those candidates who had identified a number of
the appropriate issues, few then went on to give any substance to these issues, or to
suggest how to conduct a detailed analysis of the profit and loss account .

11

Factors to consider are pattern of GDP growth, inflation and the exchange
rate.
GDP Growth: Raising of short rates will take some time (69 months lag) to
affect the real economy.
The time lag and the degree of impact will depend on the nature of borrowings
by the private sector (i.e. short or long fixed or floating in nature).
Eventually, however, the economy will slow; domestic demand will weaken as
corporate profitability is hurt by higher debt servicing costs and consumption
slows.
Inflation will also not react immediately but eventually as domestic demand
slows it will fall.
Initially the exchange rate should rise.
This will help to dampen inflation and economic growth in as much as it will
weaken the competitiveness of the export orientated sector and increase the
level of cheaper imports.
How rapid or prolonged this slowdown becomes depend on the reaction of the
central bank.
If short rates are not cut as inflation falls and the economic slowdown bites,
then under the weight of what will become increasingly high real rates of
interest will be a risk that the economy slips into recession.
Bond Market
The reaction of the bond market should be positive.
How positive and how immediate the reaction will be depends on how quickly
inflationary expectations will start to reduce.
Also in view of the strong growth being experienced prior to rates being raised
it is reasonable to assume that (all things being equal) tax receipts will be
buoyant and the supply of Government Debt will be shrinking.
This favourable supply demand balance should help boost bond prices further
and it is likely that the yield curve will invert.
If high short rates persist and the slowdown gathers pace inflationary
expectations will continue to fall and bonds will continue to perform well.

Page 9

Subject 301 (Investment and Asset Management) April 1999 Examiners Report
Equity Market
Strong growth should have boosted corporate profitability and hence the
equity market is likely to have responded well initially.
As fears of higher interest rates and inflation grow (prior to the first rate rise)
the equity market is likely to have started to discount the coming bad news.
The general level of the equity market will subsequently be affected by the
combined influence of the following:
(i)

How quickly and how severely investors revise down their expectations
of future corporate profitability in the face of a slowing economy and a
strong currency.

(ii)

How much of this is offset by the positive influence of falling bond


yields.

In general lower bonds yields and lower inflation should improve the relative
valuation arguments for equities.
However, if expectations of growth and inflation fall to levels associated with
recessionary conditions any further deteriorations will be negative for equity
markets since they will lead investors to expect a severe corporate
profitability squeeze.
Reasonably well answered, although a number of candidates showed insufficient
knowledge of basic economics . Again, few candidates detailed issues specific to the
requirements of the question.

12

(i)

Let d = dividend yield


g = assumed dividend growth rate
Then d + g = Required risk free real rate of return (RFRR) and expected
inflation (EI) + equity risk premium (ERP).

(ii)

In a prolonged recession the outlook for profits growth will be negative.


This means that expected dividend growth (g) will probably be negative
as well.
Assume that the RFRR and the ERP remain constant (if anything the
RFRR may fall a little and the ERP could rise).
It is also safe to assume that in a recession the EI will also have fallen
and could be negative. However it is unlikely that it will have fallen
anything like the extent that g will have fallen.
Hence, in order to preserve the relationship in (i) d will have to rise;
and the rise could be substantial.
e.g. normal conditions could be
(d) + (g) = (RFRR) + (EI) + (ERP)

Page 10

Subject 301 (Investment and Asset Management) April 1999 Examiners Report
2% + 3% = 2% + 2% + 1%
Say g becomes 3% + EI falls to 2%.
d would need to rise to 4%
Other points
dividends can often be maintained by companies in spite of severe
profits falls
the theoretical relationship is a very long term one and in practice g
may only be negative for a couple of years before returning to a
positive number
(iii)

The economic conditions in the two countries are very different.


The home economy is in deep recession, which would be expected to be
a poor environment for equity investment. The attraction of investing
in the equity market of the neighbouring country is the expected more
favourable level of investment growth.
If inflation is stable (even though high), then it need not be a problem
for the equity market. The more favourable conditions would be an
advantage for investing in the neighbouring country.
However, if the inflationary conditions were unstable, or if the
neighbouring country was intending to act on reducing inflation,
the outlook for the market would be more uncertain, although the
prospects of reducing interest rates should prove to be a market
support.
From the perspective of a foreign investor, a currency mismatch would
be introduced.
In particular, it is likely that with high inflationary conditions, the
currency would devalue.
The extent of devaluation would depend on the level of inflation offset
by the level of real economic growth.
The risk for a foreign investor would depend upon the nature of their
liabilities.
The advantage and disadvantage of the move ultimately will come down
to the trade-off between higher growth reduced by expected currency
devaluation.

This was poorly answered - a surprising number of candidates failed to use their
answer to part 1 to help them with part 2 of the question. Few candidates related Part
3 to the earlier parts of the question.

Page 11

Faculty of Actuaries

Institute of Actuaries

EXAMINATIONS
September 1999
Subject 301 Investment and Asset Management

Time allowed: Three hours


INSTRUCTIONS TO THE CANDIDATE
1.

You have 15 minutes at the start of the examination in which to read the
questions. You are strongly encouraged to use this time for reading only but
notes may be made. You then have three hours to complete the paper.

2.

You must not start writing your answers in the booklet until instructed to
do so by the supervisor.

3.

Write your surname in full, the initials of your other names and your
Candidates Number on the front of the answer booklet.

4.

Mark allocations are shown in brackets.

5.

Attempt all 13 questions, beginning your answer to each question on a


separate sheet.
AT THE END OF THE EXAMINATION

Hand in BOTH your answer booklet and this question paper.


In addition to this paper you should have available
Actuarial Tables and an electronic calculator.

301S99

Faculty of Actuaries
Institute of Actuaries

Describe how the principles underlying the actuarial control cycle can be used
in developing a model to manage an equity portfolio.
[5]

(i)

(ii)

Describe the purpose of an investment trust, its pricing, its legal


structure, and the basis on which it operates.

[5]

Describe how an open-ended investment company (OEIC) differs from


an investment trust.
[2]
[Total 7]

Describe the operation and role of a clearing house in futures trading.

[4]

List the distinctive features of companies classified within the utilities


industry sector.

[5]

Describe the factors influencing the investment strategy of a collective


investment vehicle.

[7]

(i)

Distinguish between barometer and portfolio-based property indices.[2]

(ii)

Discuss the use of portfolio based property indices as a benchmark


against which to assess the investment performance of a directly
invested property portfolio.
[3]
[Total 5]

Outline six different methods an actuary might use to value individual


investments.

(i)

Explain the relationship between the yields of corporate bonds and


government bonds in the same currency.

[6]

[2]

(ii)

Describe briefly how this relationship might be affected by an economic


downturn.
[1]

(iii)

In a stable economic environment, how might the expectation of


sustainable low inflation of under 2% per annum affect the relationship
in (i).
[2]
[Total 5]

A large charitable foundation is considering the acquisition of a substantial


forest in North America for its US$-denominated investment portfolio.
Describe how you would evaluate the investment potential of the forest.

3012

[8]

10

An index fund aims to track the FTSE All-Share index. The fund will match
the index weight in each industry sector, but will not necessarily include every
index stock to achieve the sector weight. For example, in a particular sector
which contains 15 stocks, the fund manager decides to hold only three of the
stocks.
Explain in detail why the fund manager might structure the fund in this
particular way.

11

[7]

You are the investment manager of a large unitised pension fund. The fund
has a strict policy of not holding direct property investments. The marketing
material of the fund states that it aims to outperform the median of a peer
group of pension funds by stock selection alone. The peer group comprises 60
managed funds, a number of which hold property assets. Median performance
data and breakdowns of the asset allocation of peer group funds are published
every three months.
Discuss the asset allocation issues that arise for you in attempting to meet the
marketing claims of the fund.
[14]

12

You are a consulting actuary specialising in the appraisal of capital projects.


An international mining company has approached you to examine one of its
proposed projects. The company is considering an investment of 800 million
to develop a copper mine in a third world country not known for its political
stability. From a technical point of view, the quality of the ore is excellent.
Using the companys technology the copper can be produced for less than 25%
of the current world-market price of copper.
The company has asked you to identify the risks facing the project and to
suggest a method of risk analysis.
(i)

(ii)

13

(a)

Describe the steps necessary to achieve an effective


identification of risks facing the companys project.

(b)

Describe briefly five major risks facing the project.

Describe how the risks of the project can be analysed.

[10]
[4]
[Total 14]

You are the manager of a global equity fund valued at US$1 billion. You wish
to alter the asset allocation, switching some of the portfolio from the UK
market to the US market, with the expectation that the switch will be
reversed in three months time.
(i)

Describe the problems and costs you would encounter, if the switch was
conducted by sales and purchases of individual securities.
[6]

(ii)

Explain how equity index futures might reduce the costs and problems
associated with changing asset allocation on a short term and a long
term basis.
[7]
[Total 13]

3013

Faculty of Actuaries

Institute of Actuaries

EXAMINATIONS
September 1999

Subject 301 Investment and Asset Management

EXAMINERS REPORT

Faculty of Actuaries
Institute of Actuaries

Subject 301 (Investment and Asset Management) September 1999 Examiners Report

The control cycle has five elements.


The initial starting point is the general commercial and economic environment
against which stocks are traded and valued.
In developing a solution consideration has to be given to alternative
investment options (for example the mix of domestic and international
equities), consideration of the liabilities and asset liability matching.
Risks like solvency need to be explored and consideration given to future
solvency levels.
The solution involves devising a model using the tools available together with
a professional approach to ensure that the model is based on sound thought
and clear communication.
Once a model has been built it then has to be tested and the results fed back
into the system to refine the problem and the solution. Close attention needs
to be paid to those factors which caused the model to depart from its expected
outcome.

(i)

Investment trusts fall into the category of collective investment


vehicles (CIVs) which are structures for the management of
investments of a large number of relatively small investors delivering
them a greater level of diversification than they could hope to achieve
with their own resources.
An investment trust company is a public limited company which is
normally quoted on a stock exchange.
Investment trusts have a Board of Directors which is responsible for
the direction of the company and for ensuring the trust adheres to the
investment mandate set out in the offer document to shareholders.
Investors participate in the investment performance of the underlying
assets by purchasing shares in the investment trust. Investment trusts
are closed-end CIVs as they are closed to new money. The shares in
issue change hands on a stock market without new money being raised
by the company.
An investment trust can be geared.
The price of the shares is loosely based on the net asset value of the
investment trusts shares and is driven by supply and demand for the
shares on the stock market on which the investment trust is listed.
Investment trust shares normally trade at a discount to net asset value.
Shares are bought from market markers in at the offer price and sold at
the bid price; thus investors bear a bid/offer spread on purchases.

Page 2

Subject 301 (Investment and Asset Management) September 1999 Examiners Report

The investment mangers are appointed by the Board of Directors,


report to the Board and take the day-to-day investment decisions.
The investment managers receive a fee for their services; usually a
percentage of the assets under management.
The directors of the company receive directors remuneration for their
services.
(ii)

Open-ended investment companies (OEICs), cannot gear.


Unlike investment trusts OEICs are open ended; new shares are
created when new money is invested and existing shares are cancelled
when money is disinvested.
Unlike investment trusts OEICs always trade at net asset value.
Unlike investment trusts OIECs have; entry and exit charges are
explicit.
OIECs can raise new tranches of capital, which attract different
management fees to existing tranches.

Comment: Bookwork, but surprisingly poorly answered.

The clearing house is the central counterparty in futures transactions. When


trades are agreed between participants in the pit or on-screens, the contract
between the parties is novated (i.e. the clearing house becomes the buyer to
every seller and the seller to every buyer).
Details of the trade are registered with the clearing house.
Each party to the trade has a contractual obligation to the clearing house.
In turn the clearing house guarantees performance of each side of the original
trade.
This guarantee removes the credit risk of individual participants.
Each party must deposit collateral or a margin with the clearing house.
There are two types of margin, the initial margin and the variation margin;
both margins serve to reduce the clearing houses credit exposure to the
exchange participants.

Utilities are involved in the supply of continuously demanded services to


households and business premises.
Demand is stable, as the services they provide are, in the main, essential.
Page 3

Subject 301 (Investment and Asset Management) September 1999 Examiners Report

They usually require an extensive physical infrastructure; this tends to make


them capital intensive.
Most utility companies are natural monopolies.
They are usually tightly regulated and therefore prices are vulnerable to
political risk.
They generally have low growth prospects; this leads to a high gross dividend
yield.
Gearing is generally low.
They are largely dependent on the domestic market, although some companies
are diversifying internationally.

Description of investment objectives as set out in the marketing material or


offer documents. For example, if the offer documents specify active
international equity fund management then the manager is expected to be an
active investor in international equities.
Taxation; depending on the tax regime the fund may have to adjust its
investment policy to minimise the impact of taxation on the return of the fund.
For example, a fund that pays no tax on capital gains but tax on investment
income may prefer low yielding high growth stocks.
Legal restrictions e.g. maximum percentage of the fund invested in any one
stock or the maximum percentage of the market capitalisation of an individual
stock that the CIV can hold.
Size of CIV; large CIVs need to hold a larger number of stocks than would say
a private individual so as to avoid having a large percentage of the market
capitalisation of an individual stock.
Peer group positioning; this is particularly important if the manager has to
compete for funds in the market. The key risk for the manager and his clients
is underperformance relative to the competitors.
The need to differentiate between exposure to an underlying market and the
currency of that market and thereby introduce currency hedge strategies.
Cash flow whether the vehicle is open-ended, and subject to cash inflow and
outflow, or whether it is closed-ended with no cash flow.

Comment: appeared to give some candidates problems. This question was not about
liabilities.

Page 4

Subject 301 (Investment and Asset Management) September 1999 Examiners Report

(i)
A barometer index aims to track movements in the property market at
large by estimating the full capital or rental values of a number of hypothetical
rack-rented properties (e.g. local house prices).
Its main use is to highlight short term changes in the market level in
terms of prices, rents and yields.
Portfolio based indices measure rental values, capital values or total
returns of actual properties.
Since current rental income of these properties is fixed until the next
rent reviews, response to movements in rental values will be sluggish.
Capital value growth will be based on property valuations conducted
periodically, because estimation of property value is expensive and
there will be few transactions.
(ii)

A portfolio based index is based on an actual portfolio of properties.


Results from different indices will depend on size, regional spread and
sector mix of the individual properties included.
So, it will be important to ensure that the index used is representative
of the portfolio which you are measuring.
Although, property is unique and portfolios of property are
heterogeneous.
Valuations will be carried out at different points of time, both for the
index and for the portfolio being measured.
Sales of many types of properties are not necessarily revealed to
outside parties, so there are problems of obtaining price data.
Rates of return for the indices will typically be money-weighted.
This will be important if the portfolio being measured is subject to
heavy cash flows.

Historic book value


This is the price originally paid for the asset
Written-up or written-down book value
Book value adjusted for changes since the date of purchase in accordance with
some formula
Market value
May not be uniquely be defined; it can only be known with certainty when a
sale takes place.

Page 5

Subject 301 (Investment and Asset Management) September 1999 Examiners Report
Smoothed market value
Market values can be smoothed by taking some form of average over a
specified period to remove daily fluctuations
Discounted cash flow
Literally discounting the cash flows resulting from holding the asset and
allowing for any residual value or loss on disposal.
Stochastic models
These are an extension of the discounted cash flow method in which future
cash flows, interest rates or both are treated as random variables; the result of
such a valuation is a distribution of values from which the expected value or
other statistic can be determined
Expected utility
Instead of discounted cash flow, the utility of each possible outcome can be
calculated in a stochastic model
Arbitrage value
This is calculated by replicating the investment with a combination of other
investments and applying the condition that in an efficient market the values
must be equal
Comment: Methods which were circular (e.g. depended on market price) were rejected.

Page 6

(i)

Corporate bonds tend to have higher yields than the equivalent


government bonds
due to the higher risks of default
and lower levels of liquidity.

(ii)

The yield spread is expected to widen due to greater fears of


bankruptcies for corporates.

(iii)

Government bond yields will probably be very low, and thus with a
stable economic background, the yield spread is likely to narrow as
investors reduce their risk premia in the chase for higher yields

A forest is a hybrid between a property and an equity. It is a real asset.


Many property features apply, but some areas (e.g. volatility of yield) are more
similar to equities. Features are:
Marketability there will be substantial cost involved in buying and selling
so this is likely to be a long term holding.

Subject 301 (Investment and Asset Management) September 1999 Examiners Report
Unit size the cost of a forest will be substantial, and the number of potential
forests (if buying) or of purchasers (if selling) will not be great.
Location will be critically important. A forest only has investment potential
for its income producing potential. At some stage, this means felling the trees
and selling the timber. It is therefore important that the timber is accessible
and that there is sufficient demand locally.
Uniqueness there is a uniqueness factor. Features which influence this
uniqueness will be the quality and type of timber, demand for the type of
timber, the age of the trees, their appropriateness for harvesting, etc.
Yield and security of income the forests yield potential will depend on the
ability to harvest the timber (depends on age of various sections of the forest),
the cost of harvesting and bringing the timber to its market, the demand for
the timber, and the volatility of that demand.
Demand will also depend on the range of uses e.g. housing or other
construction demand, and the competing courses of supply (not necessarily
timber).
Exposure to economic influence e.g. demand, particularly construction,
which is very cyclical and which is a heavy user of timber.
Expense of maintenance a forest will require looking after. Access routes
need to be maintained, the forest need to be looked after to prevent disease,
harvesting will cost money, and the felled timber needs to be prepared for
transport, or brought to market.
Investment characteristics can be changed by the owner, e.g. it might be
possible to do so by introducing new types of trees which can alter (over the
longer term) the quality and type of timber and the speed of harvesting.
There may be risks attached, environmental, legal or political.
Comment: Rather poorly answered. Many candidates failed to appreciate that this
investment had many property characteristics.

10

The fund manager only has to track the performance of the index.
So replicating the index is not essential.
Investing in many stocks and so having relatively small individual holdings in
each stock will result in high dealing costs (necessary each time the relative
sector weightings change) that will reduce the performance of the fund and so
cause underperformance relative to the index.
Research has shown that, after overall market movements have been taken
into consideration, the share price movements of companies within industrial
groupings tends to correlate more closely with each other than with
companies in other industries, so holding three instead of fifteen may well
replicate the performance of the sector.

Page 7

Subject 301 (Investment and Asset Management) September 1999 Examiners Report
The share price movements reflect the changes that have occurred in the
operating environment, and such changes affect companies in the same
industries in similar ways.
The specific sector may only represent a small percentage of the index and
within that sector the three stocks the manager proposes to use may represent
a substantial proportion of the total market capitalisation of the sector.
Stratified sampling of the performance of this sector may have shown that the
performance of the three stocks in question is a very accurate measure of the
performance of the sector as a whole.
Sampling may enable the fund to choose its timing in addressing whether or
when to replicate changes to the underlying index.
Comment: Surprisingly many candidates thought that the index fund would structure
its holdings sampling in order to outperform.

11

The first issue is how to match the median managers holding in property.
Could hold property shares or units in a property unit trust as a proxy for
property.
Or could hold a mix of equities and bonds on the basis that the return on
property is expected to be somewhere between the two.
Then there is the question of matching asset allocation within each asset class
listed in the question;
for example one must decide if, and then how, to distribute assets by sector
within the UK Equity asset class.
One option is not to try, and instead seek to add value by sector selection,
but this goes against the marketing statement so must be rejected.
The best method is to match a suitable index weighting by sector within each
asset class;
for example, the assets in the UK Equity class can be distributed by sector in
line with the FTSE-All Share Index sector weightings.
Another issue is that there may be a time lag between the end of the quarter
and the date that the quarterly data is published.
Hence the information that the investment manager has available to rebalance
the portfolio at the end of each quarter may already be out of date.
The difficulty is twofold.
First, other managers, who are seeking to add value by asset class allocation,
may alter their asset class allocation at some point during the quarter. Clearly
this alters the median manager asset class allocation, but the investment

Page 8

Subject 301 (Investment and Asset Management) September 1999 Examiners Report
manager has no knowledge of this until seeing the published data at the end of
the quarter.
If enough managers do this then the change could be substantial.
There is no easy way to deal with this; the investment manager could look to
see if there are any surveys done on asset allocation say, monthly, but these
may not be reliable.
It is inevitable that part of the investment performance relative to the
benchmark will be explained by difference in asset allocation
although these should be evenly distributed between positive and negative
effects and over a longer period should average zero.
Secondly, if the unitised fund either outperforms or underperforms relative to
that asset class because of superior/inferior stock selection, then it will be
respectively overweight or underweight in that asset class.
The best way of dealing with this is to use new money coming into the fund (or
withdrawals from the fund) to rebalance.
When rebalancing, the cost of dealing needs to be considered, which could act
as a drag on performance if it is too high.
Comment: The most poorly answered question. Again, this question was not about
liabilities. Candidates greatest problem was failure to answer the question asked,
which related to meeting the marketing claims of the fund. Instead, they largely
treated this as a question on property. The property aspects of the solution commanded
few marks.

12

(i)
(a)

There are five steps:


Make a high-level preliminary risk analysis to confirm that the project
does not obviously have such a high risk profile that it is not worth
analysing further.
Hold a brainstorming session of project experts, senior internal staff at
the company and external mining investment experts who are used to
thinking strategically about the long-term.
The aim is to identify project risks that are both likely and unlikely, to
discuss these risks and their interdependency, to attempt to place a
broad initial evaluation on each risk, both for frequency of occurrence
and probable consequences if it were to occur and to generate initial
mitigation options and discuss them briefly.
Carry out a desktop analysis to supplement the results from the
brainstorming session, by identifying further risks and mitigation
options using a general risk matrix, researching similar projects
undertaken by the sponsors or others in the past and obtaining the

Page 9

Subject 301 (Investment and Asset Management) September 1999 Examiners Report
considered opinion of experts who are familiar with the details of the
project and the outline plans for financing it.
Carefully set out all the identified risks in a risk register with cross
reference to other risks where there is interdependency.
Ensuring that upside risks as well as downside risks are covered.
(b)

List of risks (not exhaustive other risks are acceptable too) words in
bold represent a broad categorisation of the risks identified:
Perhaps the biggest risk is that the mine will be expropriated by the
government of the third world country after the company has
invested in developing the mine. Political
The government may impose/increase taxes on the output of mine or
increase its annual fee for the licence to work the mine. The
company is in a very weak negotiating position once it has invested
in the mining operation it may be better to pay increased
taxes/licence fees than pull out of the country. Political/Financial
The world market price of copper might fall from its present levels
reducing the overall payback from the project and increasing the
time to discounted break-even between initial investment and the
future cashflows. Economic
The mine could flood, be covered in by a mud slide or be prevented
from operating as a result of some other natural disaster. Natural
A part of the output from the mine may be mis-appropriated or
stolen either at the mine or in transit to the world market. Crime
The initial reports on the quality of the mines ore may prove to be
incorrect so that the costs of exploitation are higher than originally
anticipated. Business
It may not be as easy as was first envisaged to recruit suitable local
labour, put together the infrastructure to exploit the mine and move
the output to a port for shipment to the world market. Project

(ii)

The purpose of risk analysis is to ascertain the frequency of occurrence


and the consequences if the risk event occurs.
The analysis should concentrate on independent risks.
A guide to frequency of occurrence can usually be got from experts in
each risk. The analysis should be supplemented by a study of the
available statistics.
The financial consequences of the event should be expressed in terms of
present-day money values. They may consist of a range of possible

Page 10

Subject 301 (Investment and Asset Management) September 1999 Examiners Report
values. A mid point or a probability distribution could be used for
further analysis.
The expected NPV of a risk occurring in a future year can be obtained
by multiplying the probability of occurrence in that year by the NPV of
the incremental impact on the cash flows of the project if the event
were to occur in that year.
Risks should then be prioritised for further analysis. Risks with low
NPVs may be discarded by allowing for them in a general contingency.
Risks with high NPVs and risks which would be disastrous but which
have low probabilities of occurrence should be retained for further
analysis.
Comment: Well answered.

13

(i)

(ii)

The main problems in moving monies between markets for both long
and short term switches are as follows:

the costs (commission, bid-offer spread, purchase taxes, etc.) in


switching between markets and reversing the switch within a few
months can be significant

short term switches may upset a strategic profile of stocks in a


market

the operational aspects of carrying out a decision can be slow with


the consequent loss of some of the benefits of the decision

the back office may be over stretched leading to a higher risk of


errors if several stocks are being bought and sold at once

lack of liquidity and depth in the underlying markets can reduce


flexibility

the taxation impact of selling shares which have shown significant


appreciation in price may be unacceptable in terms of cost

For a short term switch, bid/offer spreads, commissions and any lack
of liquidity and depth will cost the manager four-fold in a round trip
between two markets.

Using stock index futures the manager can adjust and subsequently readjust the portfolios exposure between the two markets at a
significantly lower cost.
No tax is crystallised on equity capital gains and the long term profile
of the fund remains intact.

Page 11

Subject 301 (Investment and Asset Management) September 1999 Examiners Report
The investment decision can be executed immediately to catch all the
anticipated movements in both markets whereas some of the benefits of
the decision could be lost because of the time taken to process sales in
the underlying stocks in the relevant markets.
In the case of a short term switch there would be of the order of four
contract notes to be processed for each stock in one of the markets
(assuming say 25 stocks are held in each market this would run to 100
contract notes to be processed); using index futures only four contract
notes need to be processed.
Futures markets are often more liquid than the market in the
underlying stocks so it is possible to deal in size without moving the
market. Stock index futures avoid the need to trade the underlying
stocks and thereby avoid the movement in the market prices of stock
associated with trading large volumes.
In a very large investment house it may be virtually impossible to make
substantial asset allocation switches without the use of futures.
For a long term switch between markets, stock index futures can also
be very useful.
The switch can be achieved by selling stock index futures in the market
the manager wishes to reduce his exposure to (say the US) and buying
stock index futures in the market to which the manager wished to
increase his exposure (say the UK).
With this strategy, the manager is protected from falls in the US
market because losses on the underlying securities are made up by
gains on the short position.
Gains on the UK market accrue to the fund through the long futures
position.
Having locked in his strategic asset profile the manager can now
comfortably proceed with stock switching and unwind the futures
positions appropriately as he proceeds. The fact that the manager does
not have to sell large volumes of stock quickly, should allow the
manager to do individual stock deals on more advantageous terms.
This strategy allows the manager to ensure he locks in his long term
view without losing the market opportunity while trying to fine tune
his stock selection and switching process.
As equity transactions are spread over a longer time period the
pressure on the back office is reduced.
Comment: Reasonably well answered, although it did differentiate candidates.

Page 12

Faculty of Actuaries

Institute of Actuaries

EXAMINATIONS
10 April 2000 (am)
Subject 301 Investment

Time allowed: Three hours


INSTRUCTIONS TO THE CANDIDATE
1.

You have 15 minutes at the start of the examination in which to read the
questions. You are strongly encouraged to use this time for reading only but
notes may be made. You then have three hours to complete the paper.

2.

You must not start writing your answers in the booklet until instructed to
do so by the supervisor.

3.

Write your surname in full, the initials of your other names and your
Candidates Number on the front of the answer booklet.

4.

Mark allocations are shown in brackets.

5.

Attempt all 12 questions, beginning your answer to each question on a


separate sheet.
AT THE END OF THE EXAMINATION

Hand in BOTH your answer booklet and this question paper.


In addition to this paper you should have available
Actuarial Tables and an electronic calculator.

301A2000

Faculty of Actuaries
Institute of Actuaries

(i)

List the advantages of investing via an investment trust rather than a


unit trust.
[3]

(ii)

Explain the term discount to net asset value in the context of an


investment trust. Give three reasons why an investment trust might
stand at a premium, rather than a discount, to net asset value.
[3]
[Total 6]

(i)

(a)

List the investment characteristics of property.

(b)

State with reasons how you would expect the return on property
to compare to the return on index linked Government bonds. [6]

(ii)

State with reasons whether you would expect industrial property or


office property to be the higher yielding investment.
[4]

(iii)

Explain the factors which you would expect to influence the difference
in performance between individual property company shares in a low
inflation environment.
[4]
[Total 14]

An equity market index is often broken down into separate industrial sector
components.
(i)

Explain the advantages of comparing shares within their industrial


sectors.
[3]

In analysing each of its equity portfolios, an investment management company


compares the dividend yields of its holdings with those of the index. To do
this, the company:

ranks the holdings by increasing yield

splits the portfolio into five equal numbers of holdings (quintiles)

calculates the weighted average yield of each quintile, using the market
value of shareholdings as weights

The company conducts the same exercise for the index, and compares the
quintile averages of each portfolio with the other portfolios it manages and
also with the quintile averages of the index.
The company carries out the same analysis using earnings growth rates and
also price to earnings ratios.
(ii)

3012

Explain what the company expects to achieve from comparing portfolios


in this way.
[3]
[Total 6]

One of the stated objectives for the management of an equity portfolio is that it
shall aim to have a beta of 1.2 relative to the market index.
(i)

Explain the term beta, and the meaning of a value of 1.2.

(ii)

Describe the investigations you would make to determine the value


added by the fund manager to the portfolio, assuming full data is
available.
[3]

(iii)

List six reasons why the performance of the portfolio might differ from
that of the index.
[3]
[Total 8]

(i)

Two options have identical characteristics except that one is an


American option and the other is a European option.
Explain which option will have the greater market price.

(ii)

(iii)

(iv)

Explain the difference between a futures contract and an option


contract.
Explain why only a small proportion of exchange traded futures
contracts reach delivery, and describe the practical steps taken to
achieve this result.

[2]

[4]

[3]

Explain two types of risk faced by the counterparties in an over the


counter swap arrangement. Discuss the steps which can be taken by
either counterparty to minimise these risks.
[4]
[Total 13]

You are the investment manager of a fixed interest unit trust. For several
years your remit has been to invest solely in government securities with a
term to redemption of less than 7 years. In recent times, total returns from
the unit trust have declined and competition has intensified. The chief
investment manager has asked you to consider ways in which the total return
of the portfolio might be improved.
Describe what could be done to improve the total return of the portfolio.

[2]

[6]

An index manager offers a fund which tracks the local equity index using full
replication. As a consequence of a merger of a local company which is an index
constituent with a foreign company (which is not an index constituent), the
index weight of the local company will increase from 3.5% to 6% of the index.
Outline the steps which the index manager must conduct to maintain the
tracking ability of the fund.

3013

[4]

PLEASE TURN OVER

(i)

State the principal aims of the authorities which regulate investment


management and securities industries.
[3]

(ii)

Define a self-regulated regime and a statutory regime. Discuss their


main differences.
[3]
[Total 6]

(i)

List four factors concerning the liabilities which should be taken into
account when determining investment strategy.
[3]

(ii)

Explain the significance of the covariance of the return for an


individual investment with the return of the portfolio, when selecting
individual investments for a portfolio.
[2]

The director of a small business has managed her company from a wheelchair
for many years. She has decided to retire, aged 65, to look after her husband,
aged 70, who is suffering from Alzheimers disease (a progressive brain
disorder). They have no dependants. Their principal source of wealth, other
than their residence which is specially designed for wheelchair access, is tied
up in the business. She has agreed to sell the business for a cash sum of 1
million, which will be used to provide for their future.
(iii)

10

(a)

Describe the information you would require in order to enable


you to assess their financial liabilities.

(b)

Propose an asset mix which is appropriate to meet these


liabilities, giving reasons for the weighting and choice of each
asset.
[9]
[Total 14]

Over the last 12 months, the dividend yield on a share has doubled so that it is
now twice that of the market.
(i)

Discuss possible underlying reasons for the doubling of the yield.

(ii)

State three accounting ratios you would use to conduct further analysis,
explaining the relevance of each ratio, in the specific circumstances of
this share.
[3]
[Total 6]

3014

[3]

11

For several years, the equity investments of a pension fund have been
allocated between two index tracking funds. One fund tracks the
performance of the largest 100 listed global companies (ranked by market
capitalisation). The second fund tracks the largest 100 domestic companies
(ranked by market capitalisation) listed in the major developed country in
which the pension funds liabilities are domiciled.
Over the years, the domestic fund has performed better than the global fund.
You have been asked to advise whether the equity allocation to the two funds
should be re-balanced.
Discuss briefly the issues which you would consider and the calculations you
would make before deciding whether it is appropriate to re-balance the
holdings in the two funds.
[6]

12

(i)

You intend to build a risk model for the purpose of capital project
evaluation. Define risk and describe how you would incorporate risk
into your model.
[3]

For many years the inhabitants of two islands have travelled between the
islands in boats and car ferries. A major construction company is planning to
build to a toll bridge as a link between the two islands. They have asked you
to conduct a risk analysis of the project.
(ii)

Describe the steps which you would take to identify the risks facing
this project. Include three examples specific to the toll bridge in your
answer.
[4]

(iii)

List four methods of mitigating risk in a capital project.


For each of the four methods listed, include an example of a risk which
is specific to the toll bridge project, and which could be mitigated in
this way (provide a different risk example for each of the four
mitigation methods).
[4]
[Total 11]

3015

Faculty of Actuaries

Institute of Actuaries

EXAMINATIONS
April 2000

Subject 301 Investment

EXAMINERS REPORT

Faculty of Actuaries
Institute of Actuaries

Subject 301 (Investment) April 2000 Examiners Report


Overall, this might be described as a relatively normal examination in terms of the
range and quality of candidates responses. Bookmark questions fell into two categories
those where candidates largely obtained near full marks, and those where many
candidates gained only half marks, due to the incompleteness of their answers. As a
consequence, the major determinant of success for many individuals was the quality of
their answers to those questions that examined higher level skills. Detailed comments are
as follows:

(i)
Although question says list, marks should only be awarded for stating the
advantage, e.g. gearing is not an advantage the advantage is that it enables
outperformance when the market is rising. Marks attribution. Alternatives in the
last bullet only count once.

(ii)

The gearing of investment trusts should enable them to outperform


unit trusts in bull markets.

Investment trust schemes may be bought at a discount to net asset


value and if the discount narrows this should be a source of
outperformance relative to unit trusts.

Investment trust can provide equivalent income stream at a discount.

Investment trust management charges are usually lower than for unit
trust.

Investment trusts can invest in a wider range of assets than unit


trusts.

Investment trusts may have a better tax position than unit trusts.

An investment trust is not required to hold cash for liquidity to satisfy


redemptions (OR is not obliged to buy or sell at potentially
unattractive prices just because of cash inflow or outflow).

The discount to net asset value per share is defined as:


net asset value - market price
net asset value

expressed as a %age.
This may become a premium because:

Page 2

(a)

the value of assets may be historic and due for re-rating,

(b)

investors in the trust may be barred from direct entry to the


markets in which the trust is invested and they may be prepared
to pay premium in order to gain the exposure they desire,

Subject 301 (Investment) April 2000 Examiners Report


(c)

investors in the trust may anticipate the trust management adding


value on top of the current market prices of the trusts investments,

(d)

the sector covered by the trust may suddenly become fashionable


and the trust rise in price as a consequence.

Q1 A straight-forward question and generally well handled. Many candidates


failed to gain full marks because they answered part (i) with a list of words rather
than including a brief explanation of each advantage.

(i)

The investment characteristics

real asset expected to provide a hedge against unanticipated inflation;


a running yield typically between that available on equities and
bonds;
rental income subject to infrequent rent reviews, which may be
upwards only;
very unmarketable;
high dealing costs;
security of income depends on the quality of the tenant;
susceptible to Government controls;
buildings suffer from obsolescence but land always likely to have some
value;
unit size is large;
each property is unique;
no central market with quoted property prices
valuation is a matter of professional judgement
investment characteristics can be changed by the owner/marriage
value.

In comparison with index linked Government bonds, property is less


marketable, less secure and more expensive to manage. Investors would
therefore be expected to require a higher return from property.

(ii)

Relative to office properties, generally, industrial properties;

are cheaper and quicker to build which limits rental growth;


become obsolete more quickly because they are more vulnerable to
deterioration and so have higher depreciation costs as a percentage of
the total value;
are harder to re-let because of specific use:
have a higher content of land value within price;
are more vulnerable to economic recession.

For these reasons, industrial properties would generally be expected to


provide a higher yield than offices. In a low inflation environment,
Page 3

Subject 301 (Investment) April 2000 Examiners Report


interest rates would also be expected to be low.
Rent increases will be difficult to implement, unless properties are in
areas of high demand.
Although broad economic conditions may be favourable, deflationary
problems may cause difficulties for some tenants or particular sectors,
leading to voids.
For property companies with gearing, the cost of servicing debt is likely to
be relatively low although the differential between the interest rates paid
by higher quality and lower quality borrowers may be wide.
Q2 Parts (i) and (ii) were generally well answered with several candidates gaining full
marks. The answers to part (iii) were generally disappointing.
The question asked candidates to consider the difference in performance in a low inflation
environment. Many candidates gave reasons which did not directly relate to this
environment. The correct way to approach the question was to assume that the overall
performance of the two companies would have been broadly equal in a normal
inflationary environment. Hence, candidates were being asked to explain the factors
which might be expected to produce differential performance in a low inflationary
environment.

(i)

Practicality

factors affecting one company in an industry are likely to be relevant to other


companies in that industry

much of the information for companies in the same industry will come from a
common source, and will be presented in the same way

no analyst can expect to become an expert in all areas, so specialisation is


appropriate

the grouping of equities by a common factor adds structure to decisionmaking. It assists portfolio classification and management

Correlation of Performance

(ii)

Research shows that the share price movements of companies within the
same sector are more closely correlated with each other than with companies
in other sectors

The price movements reflect the changes which have occurred in the
operating environment of companies in the same sector.

By comparing the portfolios with each other, these analyses identify the
consistency of the companys management of the portfolios.

Page 4

Subject 301 (Investment) April 2000 Examiners Report


The analyses will also identify style biases (e.g. growth or value characteristics)
in the portfolios, relative to the index.
They will also identify how risky the portfolios are relative the index.
If conducted over time, they will also show how stable is the style of management
of the company.
Q3 Part (i) was straight-forward and well answered. Part (ii) was a practical question
requiring candidates to consider the analysis being undertaken and how it might be used
by the company. The hint to the answer is contained in the earlier part of the question.
The bulk of the answer to part (i) is concerned with consistency of approach. The answer
to part (ii) is also concerned with consistency of approach over time and relative to the
index.

(i)

The Beta of a portfolio is a measure of the portfolios volatility relative to


movements in the whole market. It is usually defined as the covariance of
the return on the portfolio with the return on the market, divided by the
variance of the market return.
A beta of 1.2 means the change in value of the portfolio should be 20%
greater than the change in value of the market.

(ii)
The performance of the portfolio would be compared to the return on the
index. The portfolios target return should recognise the pre specified
level of risk. Using an index representative of the market the portfolio is
invested in, target returns could be calculated on 1.2x the index return.
Quarterly returns for the portfolio could be compared to the quarterly
returns on the target over, say, a five year period. The excess return
would indicate the level of value added by the manager.
(iii)

The performance will differ because the portfolio will be unlikely to


hold stocks and sectors in weights which are wholly representative of
the index.

the portfolios beta over the period may have varied to levels
significantly above or below 1.2 affecting returns

the portfolio may have other objectives/constraints which effect


performance.

The diversification (or lack of it ) may affect volatility of portfolio


returns;

the volume and dealing cost impact of trades in the portfolio

the effects of cash flow

Page 5

Subject 301 (Investment) April 2000 Examiners Report

the impact of tax

the effects of expenses

Q4 The definition of beta was known by almost all candidates. In part (ii) a large
minority of candidates failed to record that the target returns should be calculated as 1.2
times the market index return. The value added by the manager would be calculated by
comparing actual returns with target returns. A worrying number of candidates included
the Economic Value Added formula in their response. EVA is a ratio used to measure the
underlying profitability of a company. It is irrelevant for an equity fund. Part (iii) was
reasonably well handled.

(i)

An American option is one that can be exercised on any date before


expiry. A European option can only be exercised at expiry. Assuming all
the other terms and conditions of the two options are identical, the
American option will have the greater market price. Its value includes
scenarios which can profitably be exercised prior to expiry, as well as at
expiry.

(ii)

A futures contract places an obligation on the buyer to buy/the seller to


sell, an asset on an agreed basis in the future. For an option contract, the
buyer of the contract has the right to buy/sell (depending on whether the
option is a call or put) an asset on an agreed basis in the future. The
seller of the contract must sell/buy the asset if the purchaser of the option
so elects. Under the futures contract the asset will change hands unless
the option is closed out.

(iii)

Delivery is the settlement process in the futures markets. Most positions


in exchange traded futures are closed out before delivery by taking an
opposite position as this is a simpler process than making or taking
delivery. The counter party to each futures position is the exchanges
clearing house. The buyer of futures contract can close out his position by
selling an equivalent contract and thereby reducing his net position with
the clearing house to nil. Without the exchanges clearing house, delivery
could only be avoided by dealing direct with the original party with whom
the position was opened.

(iv)

Each counterparty faces two kinds of risk Market Risk and Credit Risk.
Market risk is the risk that market conditions will change, so that the
present value of the net outgo under the agreement increases. The
market maker will often attempt to hedge market risk by entering into an
offsetting agreement.
Credit risk is the risk that the other counterparty will default on its
payments. This will only occur if the swap has a negative value to the
defaulting party so the risk is not the same as the risk that the
counterparty would default on a loan of comparable maturity. Credit risk
can be mitigated by evaluating and monitoring the creditworthiness of the
counterparty, and to seek deposit back margining arrangements to limit
the degree of exposure.

Page 6

Subject 301 (Investment) April 2000 Examiners Report

Q5 A straight-forward book work question with many candidates scoring high marks.
Many did however demonstrate that they remained somewhat confused by the mechanics
of exchange traded futures and options. Future candidates may find it helpful to think
through in advance of the examination the cash flows and contracts/ obligations entered
into at purchase, during the time over which the contract is held and at the time the
contract is closed/ sold/ expired. For example, many candidates failed to appreciate the
consequence of margin calls at the expiry of an exchange traded cash settled futures
contract.

This answer is more fully developed than would be expected of a candidate.


However, the question does ask candidates to describe and not merely list the
improvements, so they have to show how the proposal will improve return. The
question states this is a fixed interest unit trust, and so an unqualified proposal to
invest in equities is not an acceptable improvement - see last bullet.

Benchmark to bond indices. Whilst not directly designed to improve


performance, investing in securities which are designed to produce
average performance when compared with the competition should
indirectly ensure that the returns are never far from average. Internal
expense savings might allow you to reduce charges to unit holders thereby
improving overall return to unit holders.

Investing in listed corporate bonds. The yield will be higher than


government securities of comparative term reflecting the additional credit
risk, reduced marketability and any special terms and conditions.

Investing in longer term securities in a positive yield curve environment.


Obviously this strategy carries risks should interest rates rise. However,
all else being equal, when your view of future movements in the yield
curve is neutral, longer dated securities offer a higher yield in a positive
yield curve environment.

Trading securities in a positive yield curve environment as their term to


maturity reduces. Similar point to item 4 above. Having invested in the
longer dated securities, they should be sold from time to time as they
reduce in term in a positive yield curve environment. The capital gain
from the sale allows the trust to pick up a further increase in yield
providing the proceeds can then be reinvested in the then longer dated
securities.

Ensuring that investment is not made into stocks which are particularly
attractive to other types of investors. Some types of investors e.g.
individuals and/or foreign investors will bid up the price of a bond as to
them it is more attractive generally for tax reasons. It could also be
relatively more attractive for regulatory reasons. There is little point in
investing in these bonds if other bonds with suitable terms and conditions
are available to you.

Page 7

Subject 301 (Investment) April 2000 Examiners Report

Anomaly switching refers to monitoring a full range of potential


investments and continuously comparing their prices. In so doing it may
be possible to identify investments which appear relatively cheap and for
which no explanation can be found. These investments could be
purchased in the belief that their price will correct to previous relative
levels.

Policy switching refers to anticipating future changes in the shape of the


yield curve. Policy switching can be extremely profitable if the market is
correctly anticipated.

Investing in asset backed securities. Similar to investing in corporate


bonds. Also similar is investing in local authority bonds, except that many
people believe that local government authorities are extremely unlikely to
default on their borrowings as they have a certain amount of government
backing and they have tax raising powers.

Underwriting new issues for a fee can be attractive if the trust is happy to
hold the issue being underwritten. Markets can move quickly and it can
happen that pricing will change significantly from the time of
underwriting to the time of issue. Hence there is the risk of not being able
to sell on the investment.
Investing in convertibles or preference shares, if the objective of the trust
grants permission to do so.

Q6 The question was generally well answered.

This is a practical question there are marks for what needs to


be done and marks for the precise timing of when things
happen. This is about how the portfolio is changed, not how
the index is calculated. There are no derivative solutions!
Essentially the fund will offer index tracking with its current
portfolio up to the close of business on the day before the
merged company forms part of the index.
In order to maintain the index tracking, the manager has to
buy the additional index weight (i.e. 2.5% of the value of the
fund) in the stock
at its index opening price on the first day when it comprises
6% of the fund.
At the same time, the manager has to dispose of the stocks
which no longer comprise the 2.5% of the index which the
merged company now occupies.

Page 8

Subject 301 (Investment) April 2000 Examiners Report


These disposals also need to take place at market opening
prices, in order to match the index movement.

Q7 The question was apparently fully understood by almost all candidates with most
candidates realising that the index fund manager must buy additional shares in the local
company and hence sell shares in the correct proportions in the other companies making
up the index. However, most candidates failed to secure full marks as they failed to
describe the critically important timing of events necessary to track the index.

(i)

The financial markets of all developed economies are regulated to a


greater or lesser extent. The principal aims of the regulation are most
likely to be:
1.
2.
3.

To maintain confidence in the financial system


To protect consumers of financial products
To promote efficient and orderly markets

(ii)

A self-regulatory system is organised and operated by the industry


participants without government intervention.
A statutory system is one in which the government sets out the rules and polices
them.
The main differences between a self-regulatory system and a statutory system
are as shown below:
1.

Public Confidence A Statutory system is less open to abuse by industry


participants and may command a higher degree of public confidence. In a
self-regulatory system, the closeness of the regulator and the firms and
individuals it is regulating could lead the regulator to regulating in favour
of the industry. Even if this were not actually so, the public might believe
it to be so and thereby undermining one of the principal aims of
regulation, namely, the promotion of public confidence in the financial
system.

2.

The direct and indirect costs of Regulation In a self-regulatory system


the regulators are made up of industry participants. As such they will
have the best incentive to operate the system so as to minimise the costs
of regulation. The statutory system includes independent regulators who
do not have this direct incentive.

3.

Flexibility Regulators in a self-regulatory system should be much more


quickly aware of the need to change regulation to suit changes in the
markets.

4.

Enforceability A Statutory system is more easily enforceable. It


includes legal recourse. Under a self-regulatory system, a firm or
individual might expelled but there will be no legal compulsion for that
firm or individual to cease trading.
Page 9

Subject 301 (Investment) April 2000 Examiners Report

5.

Market Knowledge It is likely that the regulators in a self-regulatory


system will have the greatest knowledge of the market. It is contended
that these regulators are more likely to introduce rules and systems which
work as intended.

6.

Relationships with the Regulator For any system to function it is


important that the firms and individuals co-operate with the regulator. It
may be easier for market participants and the regulator to co-operate in a
self-regulatory system as compared with a wholly independent regulator
under a statutory system.

Q8 A straight-forward book work question with many candidates scoring full marks.

(i)

The four accrued liability aspects to be considered when determining


investment strategy are:

nature of the liabilities;


currency of the liabilities;
term of the liabilities;
the level of uncertainty, in timing and amount, of the liabilities.

[Not required part of answer] By definition, the matching strategy will


most closely (as is practicable) match the liabilities by nature, currency
and term. It would also respond as well as is possible to the uncertainty
in the liabilities.
(ii)

The covariance of the return for an individual investment with the return
on the portfolio indicates to what degree the two move in synch. If there
is a low covariance then the investment would have attraction as a
diversifier for the portfolio as its inclusion in the portfolio would reduce
the overall risk (i.e. volatility of return) of the portfolio, subject to how
volatile the investment return is for the individual investment.

(iii)

(a)
The answer to (iii)(a) has to refer to each of the 4 bullet
points in part (i) and address those points. Sample solution below gives
idea of type of answer expected and level of marks to be gained. NOTE
following discussion at Examiners Review Meeting we felt previous answer
needed re-focussed as shown below.
Require to determine the nature of the liabilities. Are they real,
how do they relate to inflation? What is their amount? What is the
couple lifestyle and outgoings, e.g. mortgage interest or capital
repayments ? Are there any other sources of income, e.g. the
director might undertake consultancy within the business.
Are the liabilities denominated in the domestic currency.

Page 10

Subject 301 (Investment) April 2000 Examiners Report


What life expectancy can be assumed? Is the term be limited by a
factor depending on expectation of joint life. Do the two disabilities
impact on this?
What other aspects might affect the uncertainty concerning the
assets, e.g. cost escalation of healthcare and support may not be
pure RPI, may need periodic capital drawdown for medical needs,
etc. What is the likely pattern of costs, e.g. does it escalate with
age? Can the residence be sold, a last resort, when full time
residential care is needed?
Is there a continuous need for disinvestment?
(b)

There are a wide range of potential answers to this question. My


interpretation of Propose means examine possibilities and explain
your reasoning. The main elements and marks are as follows (other
answers possible)
There is a sufficient amount of funds to provide a mix of assets,
which can be tailored into the liability needs. Need to think about
some ready income, some stable income sources and some real
assets to provide growth of value for likely escalating income
needs.
Among asset classes, would conventionally think of cash, property,
fixed interest and index linked bonds, UK and overseas equity.
Might also consider impaired life annuity or long term care
product.
Conventional fixed interest will provide a high level of income to
provide for living expenses, but capital will be protected in nominal
terms only.
Index linked will provide some protection against inflation (though
maybe not health care cost inflation).
Equities will provide the only likely means of providing escalation
of capital values at a rate which keeps pace with cost of care.
However, capital values of equities are volatile and there is a
continuing need for disinvestment. A degree of overseas
investment will diversify risk, subject to introduction of currency
volatility.
Property is unsuitable inadequate spread, already exposed
through existing residence.
Cash is required to avoid steady liquidation of investments.

Page 11

Subject 301 (Investment) April 2000 Examiners Report


A suitable mix might be (other answers acceptable)
Domestic Equities
International Equities
Fixed Interest
Index Linked
Cash

30-40%
15-25%
20-35%
0-10%
5-10%

Total Equities 45-65%

Re-balance periodically to maintain broad proportions and review


mix over time as conditions change.
Q9 Part (i) was well handled
The responses to parts (ii) and (iii) were overall disappointing. Many candidates simply
provided a virtually generic information list without regard to the specific circumstances
of the question. Candidates were awarded full marks if they categorised their information
list by the four factors noted in part (i) and if the information requested was relevant to
the circumstances.
Turning to part (iii) most candidates appreciated that the couple concerned may well have
relatively heavy requirements for cash and that some investment in assets offering
protection from inflation was required to defray increasing health care costs. High marks
were awarded to candidates who also appreciated that for so long as the couple remained
alive, health care costs were likely to escalate considerably in advance of inflation as their
collective health deteriorated.

10

(i)
The answer has to address the realistic problem. Unreasonable to expect
company prospects to have improved so that dividend doubles and not expect share
price to react positively as well. Note question also says yield = 2 * market yield;
this is not a dot.com doubling an insignificant dividend.
If the dividend yield has doubled, then either the dividend has been
substantially increased or the share price has fallen dramatically, or a
combination of these factors.
If the dividend has been increased substantially, then one would expect
the share price to have risen as well, which would tend to reduce the yield.
This would imply that the dividend increase would likely have been
accompanied by a statement concerning a change in dividend policy, for
example an increase in the payout ratio or a special dividend, i.e. a one
off event.
The share price may have dropped dramatically due to a profits warning
or a results statement. There may be fears of a dividend cut.
May be an unfashionable second tier share subject to persistent selling
but nothing fundamentally wrong.

Page 12

Subject 301 (Investment) April 2000 Examiners Report


(ii)

Accounting ratios need to be relevant. Choose from Payout ratio or


dividend cover or income cover to check sustainability of dividend
Net asset value to check whether share good value at this price.
PER is acceptable if it is qualified as a monitoring of the trend in PER
over time.

Q10 The question was generally satisfactorily answered. Many candidates


could easily have scored higher if they had included an explanation of the
relevance of the ratios given to the specific circumstances of the share.

11

This is a question about the prospective view of the suitability of the


alternative options and how they relate to the liabilities of the pension
fund. The portfolios are tracking portfolios so performance in-line with
benchmark is assumed.
You have not been asked to consider the absolute levels of the market, only the
reallocation. All else held equal, the funds should be rebalanced from time to
time in accordance with the asset allocation strategy adopted for the pension
fund.
The extent and timing of the rebalancing depends on the estimates of near and
mid term expected total returns of each tracker fund.
The near to mid term future expected capital growth for each tracker fund should
be calculated using simple fundamental relationships for the companies making
up the index in each country. These relationships might be based on, for
example, the price/earnings ratio, the dividend yield and for non-financial
companies, the ratio of stock-market value to net worth. When added to dividend
yield estimates, calculations such as this help the investor to decide the near to
mid term expected total return from each market.
Given estimates of future total return, the overseas market can be considered
relatively cheap if:
Expected return in local currency + expected depreciation of home currency > expected
return in home country
The near to mid term expected returns of the funds and the value of
diversification are the two key factors to consider in the extent and timing of the
rebalancing decision. Other issues to consider are as follows:
(i)

The current equity allocations of other comparable institutional investors


should be reviewed. [The trustees may consider that it is a desirable
constraint that performance be not too far away from that of other
comparable funds].

Page 13

Subject 301 (Investment) April 2000 Examiners Report


(ii)

Net returns from overseas investment can be different because of global


market inefficiencies including taxation and regulatory frictions. [For
example, irrecoverable taxes and withholding taxes, if any, must be
considered as a cost which might otherwise be avoided or delayed. This is
not likely to be the case here because both countries are major and well
developed countries].

(iii)

Mismatching of assets and liabilities by country for the overseas tracking


fund investment is a relatively minor risk if the liabilities are long term
and real in nature. [It is not necessary to consider additional currency
hedges. Real liabilities can be approximately matched by real
investments in a different currency. The linkage between inflation and
exchange rates described by the purchasing power parity theory reduces
the long term exchange rate risk for real investments.]

(iv)

Cash flow from the two funds is unlikely to be significantly different but
should be reviewed in line with the cash flow needs of the pension fund.

(v)

Depending on the size of the holding and the actual arrangements with
the tracking funds, there may be some albeit remote possibility of shifting
market prices (both on the sale of the existing portfolio and on the
purchase of the new assets).

(vi)

Any sale and repurchase will involve dealing costs; these costs must be
considered but are unlikely to be of any significance to the decision.
[Similarly any difference in ongoing fund charges should be considered but
is highly unlikely to be significant.]

(vii)

It will be necessary to liquidate the investments to rebalance the funds.


This may take time. The pension fund might be exposed to being
underweight in the equity markets for a short time.

Q11 The response to the question was generally poor. The question asked the candidates
to consider the issues and calculations to make before deciding whether it is appropriate
to re-balance the holdings between the global index fund and the domestic index fund.
Candidates were not asked to consider the wisdom of investing in one or other of the
funds nor the reasons for the difference in past performance. Put at its most simple, it
is generally agreed that optimal risk adjusted performance requires a well diversified
investment portfolio. Having made a number of different investments in the past, the
practical problem of monitoring and potentially re-balancing those investments to
maintain the original investment objectives arises.

12

(i)

Risk is the uncertainty of timing and volatility of future cash flows. Risk
includes both upside and downside risk.
Probabilistic risk means risk that can be eliminated (or average out) by
investing in a number of similar projects. Systematic risk is risk that
cannot be eliminated by investing in the same type of project many times,
nor by diversification. Probabilistic risk should be allowed for by specific

Page 14

Subject 301 (Investment) April 2000 Examiners Report


risk identification and analysis. Systematic risk should be allowed for by
varying the discount rate used in the model.
(ii)

(iii)

The steps necessary to achieve an effective identification of the risks


facing the project can be summarised as follows.
1.

Make a high level preliminary risk analysis to confirm that the


project appears fundamentally viable.

2.

Hold a brainstorming session of project experts including all


relevant internal and external people who have experience of this
type of project and who are used to thinking strategically about the
long term. Seek to identify risks, both likely and unlikely, to
discuss their likely interdependence and to attempt to place a
preliminary evaluation of each risk both in terms of likely
frequency, and distribution of amount. Generate potential
mitigation options. Risks identified at this time might include:

The political risk of the two islands and the likelihood of the
respective governments seeking to confiscate the bridge or
impose maximum charges

The risk of the ferries dropping prices to extremely low levels

The risk of the road system being let go so as to make the


bridge unusable

The risk of other bridges or tunnels being built either by one of


the governments or by another private entity

(Other reasonable risks are equally acceptable.)

3.

Carry out a desktop analysis to supplement the results of the


brainstorming session. Identify additional risks and proposed
mitigation options. Develop a general risk matrix for the project.
Research other similar projects and obtain expert opinion where
available.

4.

Set out the identified risks in a risk register with cross references
to show interdependencies.

5.

Ensure both upside and downside risks are catered for

For the risks identified any of the following mitigation options are
acceptable provided the option is feasible for the risk.
1.
2.
3.
4.
5.

Avoid the risk by redesigning the project


Reduce the risk by redesigning the project
Reduce the uncertainty through further research
Transfer the risk to another entity e.g. Appoint a sub-contractor
Insure the risk
Page 15

Subject 301 (Investment) April 2000 Examiners Report


6.

Share the risk with another party and particularly with a party
who is capable of mitigating the risk through expert control

Q12 A straight-forward book work question with many candidates scoring full and near
full marks.

Page 16

Faculty of Actuaries

Institute of Actuaries

EXAMINATIONS
11 September 2000 (am)
Subject 301 Investment and Asset Management

Time allowed: Three hours


INSTRUCTIONS TO THE CANDIDATE
1.

You have 15 minutes at the start of the examination in which to read the
questions. You are strongly encouraged to use this time for reading only
but notes may be made. You then have three hours to complete the paper.

2.

You must not start writing your answers in the booklet until instructed to
do so by the supervisor.

3.

Write your surname in full, the initials of your other names and your
Candidates Number on the front of the answer booklet.

4.

Mark allocations are shown in brackets.

5.

Attempt all 11 questions, beginning your answer to each question on a


separate sheet.

AT THE END OF THE EXAMINATION


Hand in BOTH your answer booklet and this question paper.
In addition to this paper you should have available
Actuarial Tables and an electronic calculator.

301S2000

Faculty of Actuaries
Institute of Actuaries

(i)

Explain four methods of valuing a corporate bond that is convertible into


ordinary shares of the company after some future date.
[7]

(ii)

Discuss briefly how all four methods might be used to assess the issue
price of a new convertible loan stock.
[3]
[Total 10]

Discuss the limitations and disadvantages of assessing managers on their


investment portfolio performance.

[8]

(i)

State the key characteristics of money market instruments.

[1]

(ii)

Outline the reasons why a pension fund might find investment in money
market instruments attractive.
[4]

(iii)

Explain why pension funds would not normally invest a large proportion
of their assets in money market instruments.
[3]
[Total 8]

In a small European country, the performance and the asset allocation split by
country of all the major funds are published on a quarterly basis about 6 weeks
after the quarter end. One of the fund managers manages a relatively small
portfolio of international equities. His portfolio mandate excludes domestic
equities whereas his benchmark is the median return of managers running funds
which do not have this constraint. In terms of style, the managers competitors
are very active.
(i)

Discuss the difficulties facing the fund manager in seeking to manage the
portfolio of international equities within the terms of his mandate and the
benchmark set.
[7]

(ii)

Suggest ways for overcoming these difficulties.

[3]
[Total 10]

Discuss the characteristics and uses of two equity indices used in the United
States and two equity indices used in Japan.

[8]

Discuss reasons for using a notional portfolio approach in valuing the assets of a
pension fund in an actuarial valuation.
[7]

3012

A global asset management company has to date provided only active asset
management. The directors of the company are concerned that it is losing
market share to passive asset managers. It is considering the launch of an
international equity index tracker fund. The fund will aim to track the FT/S&P
World index with a tracking error of 0.5%. The company will need to invest
25million in the fund in order to begin tracking the index from the launch date.
The fund will have a management charge of 0.2% p.a.
You have been retained by the board of the company to carry out a preliminary
investigation of the viability of the project.

(i)

List the information you would need from the directors concerning project
evaluation before drafting your report.
[3]

(ii)

Explain the issues which your report to the directors might cover.
[8]
[Total 11]

A fund manager has just taken responsibility for a domestic equity portfolio that
has been managed on an index tracking basis using full replication. The portfolio
comprises 3% of the quoted capitalisation of the home country. He wishes to
manage the fund on an active basis, reducing the number of securities in the
portfolio from 500 to approximately 100 stocks, but will maintain index weight in
each major industrial sector in the index.
Discuss the issues involved in implementing this proposal.

10

[7]

(i)

Explain why investors demand a higher yield on factories compared with


that on offices and shops.
[4]

(ii)

Discuss the factors that the property manager of a pension fund might
consider when assessing the suitability of a factory for inclusion in the
portfolio.
[3]
[Total 7]

The government of a country has decided to introduce regulation of its financial


services industry, although it has not yet decided whether the system will be
statutory or self-regulated. It will appoint an executive responsible for
implementing the system and acting as Regulator in its initial phase. The three
candidates for the appointment are:

a lawyer experienced in drafting legislation


a market practitioner operating in another developed market
an actuary who operates as an investment consultant

Discuss the skills which each of the candidates would be likely to bring to this
role, explaining how these might differ depending on whether a statutory or selfregulated system is introduced.
[12]

3013

PLEASE TURN OVER

11

(i)

Explain the difference between credit risk and market risk in the context
of a fixed-for-floating interest rate swap and describe the conditions
necessary for a credit loss to occur.
[3]

(ii)

Explain why an investment bank which has arranged two offsetting


single currency interest rate swap contracts still faces credit risk.

(iii)

[3]

UKL, a British software firm, wishes to borrow US dollars at a fixed rate


of interest. USCorp, a US arms manufacturer, wishes to borrow sterling
at a fixed rate of interest. The companies have been quoted the following
fixed rates of interest by INVInc, a global investment bank:
Company

US dollar rates

Sterling rates

UKL
USCorp

6.0% p.a.
5.2% p.a.

10.0% p.a.
9.6% p.a.

Design a swap that will give an apparent gain of 0.16% per annum to
UKL and USCorp without exposing them to foreign exchange risk within
the swap and discuss the risks faced by INVInc in the solution you
propose.
[6]
[Total 12]

3014

Faculty of Actuaries

Institute of Actuaries

EXAMINATIONS
September 2000

Subject 301 Investment and Asset Management

EXAMINERS REPORT

Faculty of Actuaries
Institute of Actuaries

Subject 301 (Investment and Asset Management) September 2000 Examiners


Report
Question 1
Question (i) was, in the main, answered well; part (ii) proved problematical with few
candidates scoring well.
Question 2
Question 2 was one of the better answered questions on the paper with most candidates
scoring well.
Question 3
Question 3 parts (i) and (ii) were answered well and most candidates made a reasonable
attempt at part (iii), while most candidates made reference to the length of the pension
funds liabilities few referred to the fact that the liabilities were real, in addition fewer
pointed out that pension funds can bear a higher level of risk.
Question 4
The answers to this question were very variable, a number of candidates made the
assumption that the competitor funds would only invest in domestic equities while
others seeing that the competitors were described as active automatically assumed that
they would perform well.
Question 5
This question was answered well.
Question 6
Most candidates were able to score well on this question though a sizeable minority did
not seem to know what a notional fund was.
Question 7
This question was poorly answered, those that treated the question as a capital projects
question scored well on part (i) but often fell down on part (ii) as they just regurgitated
the standard capital project answer without any attempt to adapt it to the project in
question. Those candidates who did not treat the question as a capital projects question
were still able to pick up some marks in part (i) but, in general, were unable to make
much headway in part (ii).
Question 8
This question produced a varied response, some candidates were able to score very
highly while others failed to score well. A number of candidates who didnt score well
felt that 3% of the domestic equity market was insignificant; while others assumed that
because the number of stocks was being reduced, the size of the fund was also being
reduced. Others failed to grasp the significance of maintaining the sector weights. In
addition some candidates were under the mistaken belief that funds with a mandate to
track the index had to replicate the index within the portfolio.

Page 2

Subject 301 (Investment and Asset Management) September 2000 Examiners


Report
Question 9
This question was answered well.
Question 10
In this question most candidates scored well in what was a book work question.
Question 11
While most candidates picked up a number of marks on this question there were a
sizeable minority who did not score well. Many candidates seeing the word actuary
immediately assumed that the examiners were expecting that candidates should
conclude that the actuary was the ideal candidate and hence were prepared to attribute
every ideal characteristic to the actuary while dismissing the other two candidates.

Overall Comment
Overall the examiners were disappointed with the overall level of knowledge displayed by
candidate. There were many opportunities to pick up easy marks by reproducing book
work, however few candidates took full advantage of these opportunities. Where
candidates were expected to think laterally or apply their knowledge the answers were, in
the main, poor. The examiners would encourage careful thought about the special
circumstances of some of the questions being asked since these are the areas where
candidates can demonstrate their higher level skills.
There was a continuation of the disturbing trend noted in recent years of discussing "the
liabilities" in circumstances where it did nothing to answer the question being asked. Also
of concern was a minority of candidates who indiscriminately quoted the Actuarial
Control Cycle, while the Actuarial Control Cycle is important it should only be quoted
when relevant. The examiners are concerned that some candidates may hold the mistaken
belief that mentioning these two points will automatically score them marks, when in fact
it will only do so in relevant circumstances.

Page 3

Subject 301 (Investment and Asset Management) September 2000 Examiners


Report

(i)
There are four main methods.
Discounting the future income stream
Consider the convertible bond as having two components:

A fixed income stream until the date of conversion and


A stream of dividends starting at the date of conversion

The date of conversion is then taken as the date for which this present value will
be maximised
This is calculated as the first date at which the dividend income on conversion
would exceed the initial fixed income from the bond
It is necessary to make the following assumptions when using the method:

The rate of dividend growth on the underlying equity stock


The investors required rate of return

Comparison with the share price


A convertible can be valued by considering it as equivalent to the underlying
share plus an extra amount of income for the period until conversion
Thus the value of the convertible is taken as the market price of the underlying
share plus the discounted value of the income received in excess of ordinary
dividends in the period before conversion.
Comparison with loan stock or preference share price
The minimum value of the convertible is the current value of the loan stock or
preference share ignoring the option to convert.
Option valuation method
The three methods above assume conversion at a certain date or ignore the
conversion option and so ignore the value of the option to convert
Using option pricing theory a convertible can be valued as a loan stock or
preference share plus the value of the option to purchase the underlying shares.

The quoted market price of the convertible would also be considered if it were
available

Page 4

Subject 301 (Investment and Asset Management) September 2000 Examiners


Report
(ii)
The comparison with loan stock price or preference share price method gives a
floor on the price and the manager would need to consider carefully if the price is
less than this figure
The option valuation method gives an upper limit on price but this upper limit is
heavily influenced by the share price volatility assumption used in the option
valuation method.
The other two methods act as checks on the previous two methods ensuring that
the results are reasonable.

Past performance is not a guide to the future.


There is a random element in investment returns.
It may be difficult to determine how much a fund managers results are due to
process and method and how much are due to luck.
Techniques that proved successful in one set of economic circumstances or
market conditions may not work so well in other economic circumstances or
market conditions in the future.
In the long term, higher risk portfolios are expected to produce higher returns.

Investment portfolio performance measurement needs to be adjusted for the level


of risk before comparisons can be made between managers.
Investment portfolio performance measurement needs to be adjusted for
differences in fund objectives and fund constraints before comparisons can be
made between managers. In practice, this is very difficult to do.
Investment portfolio performance measurement needs to consider the differences
between time-weighted and money-weighted rates of return to adjust for cash
flows, etc.
Determining the frequency of performance measurement calculations requires a
delicate balance between assessing performance frequently enough to spot
problems early and correct them and avoiding spurious conclusions based on very
short measurement periods.
Performance measurement impacts fund manager behaviour.
For example, a manager whose performance is measured against the median of
her peers is unlikely to take asset allocation decisions that lead to her being
significantly different to that of her peers.
This may not be in the best interests of the fund managers client.

Page 5

Subject 301 (Investment and Asset Management) September 2000 Examiners


Report
Users of performance measurement services must balance the value of the
service against the cost.
For some assets, like property, valuation is difficult, expensive and in the end
somewhat subjective.
Detailed and frequent valuations of such assets is inappropriate.

(i)
The key characteristics of money market instruments are:

Highly liquid
Stable capital values

(ii)
Pension funds might hold cash as a source of liquidity to meet outgoings, as a
temporary holding strategy to avoid exposure to asset classes that are falling in
value.
Some economic conditions might make cash temporarily attractive to pension
funds.
Generally speaking rising interest rates will depress both bond and equity
markets; a pension fund may wish to shorten the duration of its bond portfolio
and move some money out of the equity markets and into cash.
At the start of a recession when equity markets might suffer from low growth
and when bond yields may rise from an oversupply of government paper a
pension fund manager may wish to put part of her portfolio in cash.
If the domestic currency is weak or expected to weaken further, a pension fund
investment manager may place part of her portfolio in foreign money market
instruments of a safe haven currency.
Cash may be held due to a contribution being paid which is awaiting investment.
Cash may be held as part of a derivative strategy.

(iii)
Historically cash has underperformed other asset classes over long periods of
time.
Pension funds can bear the higher level of risk associated with equity
investments in return for the higher levels of return where there is no need to
realise assets in the short term.

Page 6

Subject 301 (Investment and Asset Management) September 2000 Examiners


Report
Cash is not a good match for the nature of pension fund liabilities which tend to
grow in line with salary inflation.

(i)
It will be difficult for the manager to know how the asset allocation of the
median manager or the median return between the dates the information is
published which could be up to a month after a quarter end.
This makes it very difficult for the manager to assess the level of risk she is
taking relative to the benchmark.
The size of the managers fund may be small compared with his competitors
making it difficult to get cost effective exposures to some international equity
markets.
The mandate constraint no domestic equities may be a particularly difficult
one in times when the local market is performing very well and the managers
competitors are heavily exposed to the local market as part of their
international equities portfolio.

Overseas investment expertise may be expensive in a small European


country.
For a relatively small fund, the costs of custody and settlement may be very
high relative to the returns generated.
The manager will have to deal with the different accounting conventions,
language problems and time zones.
There are considerable fixed overheads in managing a portfolio of
international equities which may detract from the return or reduce the
managers margin.
Currency risk

(ii)
Grow the size of funds under management to make it move economically viable
to operate an international equity portfolio this could done by acquisition or
organically and to include domestic equities.

Get the benchmark changed to say the relevant FT/S&P World Index
excluding the country in question.

Get the asset allocation and performance data produced more frequently
like monthly.

Use index derivatives to cut the cost and increase the speed of asset
allocation.

Page 7

Subject 301 (Investment and Asset Management) September 2000 Examiners


Report

Act as an asset allocator across countries and contract out the investment
management to an index manager to track the chosen indices.

The Dow Jones Industrial Average is made up of 30 shares.


It is unrepresentative of the US market.
It is an unweighted arithmetic average index.
Movements in the index are a quick guide to daily movements in the prices of
shares mainly in the industrial sector.
S&P 500 index (also called the S&P Composite index) is made up of 500 shares.
It is a weighted arithmetic average index.
Its constituents are a broad cross-section of all the sectors in the US market.
It is suitable for the performance measurement of a funds portfolio of US
equities.
Nikkei Dow Industrial Average 225 (commonly know as the Nikkei 225) consists
of 225 shares.
It is an unweighted arithmetic index.
It is not representative of the Japanese market as it had changed little since its
inception. (Note: actually, this index has been revised quite considerably to bring
its structure more into line with the broad market, but the Core Reading does not
yet reflect this).
It is nonetheless a widely used indicator of short term movements in the
Japanese equity market.
Topix (Tokyo Stock Exchange First Selection Index) comprises approximately
1,100 shares.
It is a weighted arithmetic index.
Constituents represent the leading companies in the market.
The index is suitable for use in performance measurement.
MSCI should be allowed as an index (market cap weighted arithmetic, wide
sector and stock coverage, multi-currency versions, not readily available but good
for performance measurement).
FT/S&P Actuaries World Index (Japan) should get credit and the key points are

Arithmetic weighted average;

Quoted indifferent major currencies as well as yen;

Stocks not available to foreign investors excluded;

Good/suitable for performance measurement

Nikkei 300 should be treated in the same way as Nikkei 225 but comment should
be made that it is a broader and more representative index of the domestic
market.

Page 8

Subject 301 (Investment and Asset Management) September 2000 Examiners


Report

A notional portfolio is a tool to help smooth the results of actuarial valuations of


pension fund assets over time and thus avoid the volatility associated with a
market value approach to asset valuation.
Notional portfolios tend to be made up of assets that reflect the broad
characteristics of the liability profile.
It tends to be influenced by the actuarys desire not to change elements of the
actuarial basis unnecessarily.
It is a discounted income approach to the valuation of assets where the
assumptions used in valuing the assets are consistent with those used in valuing
the liabilities.
The approach produces a different ratio of the market value of each asset to its
calculated value in actuarial valuations.
The result of the valuation will be influenced by the assets held by the
investment manager on the valuation date.
However, the actual distribution of the funds assets at the valuation date
represents the investment managers views of short and long term prospects for
each asset class. The investments are replaced as a result of changes in that
view.
If the actuary replaces the actual assets with a notional portfolio, there is no risk
that the valuation process stands between the investment manager and the best
investment policy for the fund.
Using a notional portfolio also removes the problem of having to estimate cash
flows on each individual asset and reduces the amount of calculation required.
The bookwork should be properly explained.

(i)
What criteria have been established by the directors for assessing the viability of
projects under the following headings:

Financial results expected (internal rate of return, net present value or


payback period)
The consequences for the company if these results are not achieved
Achieving synergy with other projects undertaken by the company
Satisfying political and regulatory constraints within and outside the
company
Use of scarce capital and/or management resources in the best way possible
The extent of the upside potential in the project?

Page 9

Subject 301 (Investment and Asset Management) September 2000 Examiners


Report
(ii)
The capital investment of 25 million can only be repaid as new money is raised
from investors as 25m is the minimum fund size for such a portfolio.
The only source of income appears to be the fund management charge.
However, this will grow in line with inflows of new money to the fund and the
increase in the value of the fund due to investment performance.
The expenses consist of the costs of running, selling and marketing the fund.
Some discussion of the ability of the existing distribution channels of the
company would be required in order to assess their ability to sell passive/index
fund management.
There does not seem to be much scope for significant upside in the project given
the 0.2% p.a. management charge. However, the fund may be needed to hold
funds which are moving a core to passive mangers and retaining a satellite fund
with active managers.
As an alternative answer: The key issues in realising the upside from the project
are economies of scale and persistency of the business. Overheads are more fixed
in nature and hence the business would become very profitable beyond a certain
minimum fund size. Persistency is also important in recouping acquisition costs.
The risks in the project and the possible risk mitigation strategies.
The political issue of moving from active management to passive management
and its impact on the moral of individual fund managers.
Detailed analysis tends to be expensive. So more complex evaluations will be left
until it is clear that the effort is justified.
Draw the attention of the directors to the fact that there may be alternative
projects which make better use of the scarce resources available.
The directors attention should be drawn to the sensitivity of the results to
varying assumptions as to new cash inflows, investment returns, lapses and
sales, operational and marketing expenses.

8
Although there is no change to the sector weights, this is a very large change to the
portfolio, turning over a large proportion of the stocks.
Given the 3% of market cap size, this is also likely to involve substantial turnover in
terms of the domestic market.
The main problems will be:

Page 10

Subject 301 (Investment and Asset Management) September 2000 Examiners


Report

Moving prices against the fund the size of the portfolio means stocks being sold are
likely to fall in value, stocks being bought are likely to rise.

Judging the short term success of the switch will be difficult the funds own
demand will push up the price of the stocks retained.

Considerable time will be taken to effect the change if prices are not to be moved too
much against the fund.

There are difficulties in ensuring that timing of deals is advantageous.

Dealing costs will reduce the return on the fund after taking account of expenses.

There is a possibility of crystallising capital gains which may be subject to tax.

There may be some infringement of asset percentage restrictions.

The diversification of the portfolio will reduce and so the risk will increase.

This places more weight on stock selection, and so on research and analysis
(although this is not necessarily a disadvantage, given the active nature of the
manager).

Some answers may cover compliance issues. The following may also be considered:

What do product rules say, allow and what are product holders expectations?

What implications are there for staff requirements to manage it?

Are the fund management charges going to charge to cover the extra cost of active
management?

How will the performance record and measurement be dealt with?

(i)
Factories are cheap and easy to build. So an under-supply in the market can be
quickly corrected. Thus the potential for upside growth is limited.
Factories tend to become obsolete more quickly thus requiring a higher level of
refurbishment expenditure compared with offices and shops.
Compared with shops and offices, the site value of factories tends to be
significantly lower. So deterioration of the building leads to depreciation costs
that are a high proportion of total value.
Manufacturing industry is particularly vulnerable to economic recession and
bankruptcy of a tenant will result in a rent void. This applies particularly where
the building is unsuitable for any other use.

Page 11

Subject 301 (Investment and Asset Management) September 2000 Examiners


Report
The fabric of factories is more vulnerable to deterioration compared with shops
and offices.
(ii)
The quality of the tenants covenant, the price and the rental income of the
property.
The state of repair of the factory.
The ability to relet the property to another manufacturer or convert it to some
other use in the event of a void.
The general economic outlook for the economy including, inflation, interest rates
and the exchange rate.
The outlook for the sector of the economy in which the factory operates.
Location of factory in term of access to transport and a steady supply of suitably
skilled labour.
The existing property holdings of the pension fund; will the factory add to the
concentration by geographical area or property type or will it improve the
diversification of the fund.
The potential for rezoning or development of the factory site.
The size of the deal.
Nature of lease and tenants responsibilities.

10

(i)
Credit risk is the risk that a counterparty to the swap transaction will default on
its obligations to exchange cashflows.
For a credit loss to occur the counterparty must default when the swap has a
positive value to the non-defaulting counterparty.
Market risk is the risk that the value of a swap may shift from being an asset in
the books of the counterparty to being a liability of the counterparty due to
changes in interest rates.
(ii)
Interest rates will change over the life of the swap so that to the bank one of the
offsetting swaps is an asset and the other one is a liability.
If banks counterparty to the swap which is an asset in the banks books defaults,
the bank must still honour its commitments to the counterparty to the swap
which is a liability in the banks books.
The banks still has a credit risk even though its market risk has been offset
because it has separate contracts with each counterparty.
It is important that the candidate differentiates properly between credit risk and
market risk.

Page 12

Subject 301 (Investment and Asset Management) September 2000 Examiners


Report
(iii)
US$ 5.2%

US$ 5.84%
Stg 10%

UKL

INVInc

USCorp
US$ 5.2%

Stg 10%

Stg 9.44%

The total benefit that can be split between the parties is 40 basis points. If UKL
and USCorp get a benefit of 16 basis points each, INVInc should get the
remaining 8 basis points.
swap diagram split as follows: for getting the correct apparent gain to UKL and
USCorp and ensure that UKL and USCorp dont have any foreign exchange rate
risk
Risks faced by INVInc:

11

INVInc faces credit risk if either counterparty default when the swap has a
positive value to INVInc

INVInc faces foreign exchange risk; it earns 64 basis points in US$ and pays
54 basis points in sterling this is in effect market risk.

Self-regulation is a system organised and operated by the participants in a


particular market. There is little or no government intervention in the market.

In statutory regulation, the government sets the rules and polices them.
The lawyer
Likely to be skilful in drafting government legislation i.e. framework will be
legally tight
Will be close to government sources and able to understand government agenda
Likely to be aware of known abuses (e.g. possible reasons for introduction)
Should understand capabilities of local practitioners and consumers
Independent - not aligned to market
Will be viewed as consumer friendly
Close to aligned statutory bodies, e.g. competition authorities
Likely to be able to tap into similar international regulatory networks
May lack the knowledge of the detailed workings of financial markets
Under market practitioner
May lack consumers confidence due to closeness to industry.
The market practitioner

Page 13

Subject 301 (Investment and Asset Management) September 2000 Examiners


Report
Likely to have good knowledge of other countries systems
Will have practical awareness of likely risks and potentially bad practice, e.g.
cartels, insider activity, etc.
Likely to be seen as reasonably friendly to industry needs
Likely to be persuasive to companies on need for change since close to them
Good awareness of quick win benefits, i.e. greatest inherent dangers
Likely to understand cost / benefit trade-off from companies perspective
Will have rapid response attitude to market practitioner needs
Seen as independent from government
The actuary
Should have reasonable insight into market practice, wider international
knowledge
Not aligned to business, so can be seen as consumer friendly
But also seen as independent from consumer by business
But appreciative of cost / benefit trade-off
Should be technically up to speed with innovation and its risks for regulation
Can liase with international professional network
Should adopt practical but strict guideline approach, e.g. freedom with
publication
Statistical gatherer to make cartels less likely.

Page 14

Faculty of Actuaries

Institute of Actuaries

EXAMINATIONS
10 April 2001 (am)
Subject 301 Investment and Asset Management

Time allowed: Three hours


INSTRUCTIONS TO THE CANDIDATE
1.

You have 15 minutes at the start of the examination in which to read the
questions. You are strongly encouraged to use this time for reading only
but notes may be made. You then have three hours to complete the paper.

2.

You must not start writing your answers in the booklet until instructed to
do so by the supervisor.

3.

Write your surname in full, the initials of your other names and your
Candidates Number on the front of the answer booklet.

4.

Mark allocations are shown in brackets.

5.

Attempt all 11 questions, beginning your answer to each question on a


separate sheet.
AT THE END OF THE EXAMINATION

Hand in BOTH your answer booklet and this question paper.

In addition to this paper you should have available


Actuarial Tables and an electronic calculator.

301A2001

Faculty of Actuaries
Institute of Actuaries

Two countries have announced a full review of their respective regulatory


regimes for the investment management and securities industries. Country A is
a substantial industrialised nation with a mature stock exchange. Country B is
a substantial emerging market economy with a relatively small stock exchange
and an underdeveloped capital market.
For each country, state with reasons the type of regime that might be most
suitable.

[6]

(i)

Describe briefly the money market.

[2]

(ii)

List the financial instruments that are traded and the other transactions
that are typically undertaken in the money market.
[2]

(iii)

Outline the reasons why an investment manager would purchase these


investments.
[3]
[Total 7]

(i)

State two expressions for the expected real total return to a domestic
investor on an overseas asset.

(ii)

[4]

The real return on equities over the ten years to end December 1999 was
nearly twice the average real return over the last century.
Give reasons why the most recent experience may not be a reliable guide
to future performance.
[8]
[Total 12]

List the possible uses for market indices and state the appropriateness for
Japanese equities of using the Nikkei Dow 225 and Topix indices against these
uses.
[6]

Compare the main features of investments in offices and shops.

Mr and Mrs X are both aged over 70 and own an apartment. Mr and Mrs X will
sell the apartment to their daughter Mrs Y on the basis that Mr and Mrs X can
live in the apartment rent-free until they are both dead.

[4]

(i)

List the factors you would take into account when calculating the price
Mrs Y should offer for the purchase of the apartment.
[6]

(ii)

State with reasons the other factors relevant to Mrs Ys consideration of


this investment.
[4]
[Total 10]

301 A20012

You are the investment consultant to the trustees of a fund that has its assets
split amongst several different fund managers. One of the trustees has written
to you and his letter includes the following information relating to one of the
managers:

Total Return Index


Fund Value (including new money)

1 January
2000

1 July
2000

31 December
2000

1,000
$1bn

850
$1.15bn

1,050
$1.45bn

He states, allowing for the $250m of new money received by this manager on
1 July 2000, this portion of the fund has grown by $200m and therefore the
manager has achieved a return of 20% over the year which compares to a 5%
return on the index. This clearly indicates we should reallocate monies away
from the other managers to this manager.
(i)

Comment on the calculations the trustee has made and the comparison he
has drawn.
Suggest a comparison you believe to be more appropriate.

(ii)

[5]

State with reasons the other factors you would suggest the trustee
considers before allocating more money to this fund manager.
[5]
[Total 10]

At October 2000 the assets of a fund stood at US$5bn and they were equally split
between the US, Europe and the rest of the world. Within each region the fund
had exposure to diversified investments. The fund manager expected returns in
the short term on European investments to exceed returns on US investments
and considered a tactical switch.
(i)

State two reasons why the fund manager might have expected higher
returns in the short term from European investments compared to US
investments.
[2]

(ii)

State two alternative ways the fund manager could have implemented
this switch and comment on the advantages and disadvantages of each
approach.
[4]
[Total 6]

(i)

Outline three different ways of financially evaluating a capital project. [3]

(ii)

List the advantages and disadvantages of each method.

(iii)

The evaluation process has revealed that the project managers will have a
number of options at various stages of the project depending on the
success of the project at that stage.

[3]

Describe how these options could be incorporated into the evaluation


model.
[2]
[Total 8]

301 A20013

PLEASE TURN OVER

10

You have recently completed a regular review of a large defined benefit pension
fund. There has been a recent and major decline in the market value of the
funds assets. The company is keen to avoid increasing its contribution rate both
now and in the future. With this objective in mind, the company has asked you
to report on the following matters.
(i)

The fund surplus calculation has been made using the market value of
assets.
Discuss the appropriateness of such a potentially volatile measure of
value given the very long term nature of the future liabilities, including a
reference to alternative asset valuation approaches.
[5]

(ii)

The assets of the fund are invested in a range of equity, fixed interest and
property investments without any formal regard to the matching of the
assets to the liabilities.
Describe briefly the principles that might be employed by the fund to
better match the value and timing of the assets with the value and timing
of the liabilities. Outline the difficulties of trying to implement such a
strategy in the case of a pension fund.
[4]

(iii)

For the past several years the funds investment objective has been to
achieve an investment return in any given calendar year that is in the top
quartile of returns earned by comparable pension funds. The funds
investment managers have met this objective in four out of the past five
years. The funds investment managers are not constrained as to their
choice of investments.
State with reasons a proposed replacement objective or change to the
current objective and/or any constraints that could be introduced in order
to help to meet the companys objective of not increasing its contribution
rate to the fund.
[6]
[Total 15]

11

A large general industrial company has a fast developing information technology


and computer services division. This division is expected to generate the
majority of the companys earnings over the next five years and the companys
broker has sent your fund management team a note recommending they buy the
companys latest stock issue. As the sector analyst, your colleagues have asked
you to comment on whether the company is likely to outperform its industry
sector.
(i)

List the principal features of general industrial companies.

(ii)

Discuss briefly the factors that will affect the investment performance of
all the companies in the sector.
[4]

(iii)

Comment on the brokers recommendation, including any further


information you would require.
[8]
[Total 16]

301 A20014

[4]

Faculty of Actuaries

Institute of Actuaries

EXAMINATIONS
April 2001
Subject 301 Investment and Asset Management
EXAMINERS REPORT

Faculty of Actuaries
Institute of Actuaries

Subject 301 (Investment and Asset Management) April 2001 Examiners Report

Country A is likely best suited to a mixed regime involving statutory regulation,


self-regulation and codes of practices.
A system is already in place. New system should be a development of the old
rather than a step change. This would minimise costs, avoid unnecessary
confusion and maintain confidence in the system.
Part self-regulation because the country is a substantial nation with a mature
stock market. It therefore has skilled and experienced market participants who
can help to ensure good use of market knowledge, relatively rapid response to
changing market needs and help to minimise the costs of the system.
Part statutory regulation because it should help to maintain public confidence
and minimise abuse of the system.
Country B may also adopt a mixed regime involving statutory regulation, selfregulation and codes of practice. However, statutory regulation is likely to be a
far more important component of the regime than for Country A. (A full
statutory regime is also acceptable.)
Mainly statutory regulation because market development relies on the confidence
of both domestic and foreign investors, the promotion of efficient and orderly
markets and the protection of consumers. Market participants are likely to be
relatively scarce and inexperienced. Self regulation could therefore lead to
mistakes, fraud and consequent inefficient markets and loss of confidence.
Partly self-regulation as regulatory systems should develop over time and
eventually the depth and experience of the market participants will grow. The
system adopted now should allow for increasing emphasis on self-regulation in
the future.
Bookwork Question - Overall tackled fairly well. Most candidates got the basic
marks available but few candidates covered the other relevant points which were
specific to this particular question i.e concerning the establishment and
development of a regulatory system where none existed and the further
development of one where some structure is already in place.

Page 2

(i)

It is where short term liquid instruments are traded. The money market
is not a physical market. It relies on screens trading systems. Its
participants are government and official dealers/ market makers.
Sometimes the unofficial inter-bank market is also considered to be a part
of the money market.

(ii)

Sale and purchase of treasury bills (or other short term government debt
usually a note issued at a discount)
Sale and purchase of other eligible bills (CDs, CP, FRNs)
Repurchase agreements for the bills
Lending and borrowing of short term funds in the inter-bank market

Subject 301 (Investment and Asset Management) April 2001 Examiners Report

(iii)

Liquidity to meet unexpected cash flow


Matching near term future expected cash outflow needs
Investment restrictions Hedging leveraged investment (e.g. futures,
partly paid share) to avoid leveraging the fund
Short term investment pending an anticipated near term future
investment
Defensive position Expect longer term rates to rise in near future
Recently received funds awaiting investment
Investing in an overseas money market in anticipation of a near term
relative decline in the local currency

Straightforward Question - In general done well by candidates.

(i)

Expected return = Initial income yield


+ Expected capital growth
+ Expected appreciation of exchange rate of overseas currency over
domestic currency
expected inflation
and
Expected return = Initial income yield
+ Expected income growth
+ Change in yield
+ Expected appreciation of exchange rate of overseas currency over
domestic currency
expected inflation
or
Expected return = Risk free real rate + Risk premium for type of asset +
currency risk premium

(ii)

Reasons why the most recent performance may not be a realistic guide to
future performance are:

we have had a long bull run and market valuations are high price
earnings ratios high, dividend yields low
returns on capital and equity have improved to high levels
demographics have pushed savings into longer term investments
demand greater than supply
political climate has been more favourable to business
economic factors have been favourable as long term valuation/interest
rates have fallen
alternative investments have been in shorter supply gilt sales
reduced

Page 3

Subject 301 (Investment and Asset Management) April 2001 Examiners Report

inflation has fallen to lower levels and been more stable than
historically
labour has been more accommodating on wages and flexible working
practices
technology the new paradigm has taken valuations to higher
levels

Few candidates scored well on this question. In the first part many candidates failed to
write out logical expressions often confusing the risk premium, income and expected
growth components. Many left out the expected inflation component completely.
The second part was tackled poorly by many candidates where many seemed to miss the
thrust of the question. There was very little discussion about the structural reasons
which might account for the unusually strong returns seen over recent years.

4
Nikkei Dow Industrial
Average 255
Measure of short-term
market movements
Market history
Future trend analysis
Performance benchmark
Valuation of notional
portfolio
Sub-sector analysis
Basis for index funds
Basis for derivatives

Most used

Tokyo Stock Exchange


First Section Index
(TOPIX)
Yes

Limited
Yes
Not really
Yes

More representative
Yes
Yes
Yes

Limited coverage
Limited coverage
Limited coverage

Yes
Yes
Yes

Very straightforward question which was done surprisingly badly by many


candidates. Often there was little or no attempt to link the general uses of indices
to the two specific cases in the question. Many candidates only gave a description
of the two indices rather than linking their description to these uses ie answering
the question which was asked.

Page 4

The average unit size for shops is smaller than for offices.
Whilst the general location of an office is important, the precise position within
that location is not critical.
For shops, the exact position has a major influence on the rent that can be
charged.
Rent is normally a small proportion of an office tenants outgoings whereas rent
is usually a big proportion of the total cost for a retailer.

Subject 301 (Investment and Asset Management) April 2001 Examiners Report

For offices, there is a wide range of prospective tenants with no concentration on


any particular industry.
The rents on shops are dependent on the profitability of the retail sector.
Offices do depreciate to the extent that old offices can become obsolete.
Usually for shops, the fabric of the building represents a small proportion of the
value and the retailer is responsible for fitting the shop out; for these reasons
there is little depreciation for the freeholder provided the basic shell of the shop
is sound. For the investor, shops usually have lower running costs than offices.
Initial yield is generally lower for shops than offices
Basic bookwork question which was, on the whole, well answered.

(i)

A discounted cash flow calculation could be used to determine the price to


offer. Factors required for this calculation are:

term of the lease and the ground rent, if leasehold


current market price for the apartment
current rental level appropriate for the apartment + expected rental
growth in order to calculate the income forgone.
the expected period until the apartment becomes vacant allowing for
the health of Mr and Mrs X
the expected growth in market price over the period until the
apartment becomes vacant
the transaction costs, both for the current purchase and the resale,
other costs e.g. insurance, maintenance etc.
tax
the rate of return required from such an investment

The calculation could be carried out on alternative assumptions to test for


sensitivity e.g. using different rates of growth in the market price,
allowing for different periods until both Mr and Mrs X are dead etc.
Consideration should also be given to factors that could dramatically
affect the value of the apartment e.g. the risk of development blight,
changes in the taxes on property investments, the possibility of
redevelopment etc.
(ii)

This investment is real in nature,


it is expected to be of medium to long term,
its marketability would be poor,
there is no income,
it may entail a significant amount of management and there is a high
level of uncertainty about the term and return of the investment.
Are these characteristics consistent with Mrs Ys objectives for her
investments?
Is the size of the investment convenient and is this opportunity
competitive relative to alternatives available to her?
Would Mrs Y like to live in the apartment?

Page 5

Subject 301 (Investment and Asset Management) April 2001 Examiners Report

Many candidates treated this as a pure property question and included everything they
could recall regarding residential property investment. Often the main points
surrounding the price calculation, ie market price less the present value of expected
income foregone became obscured or even missed.
In part (ii) the points about the part this particular investment would play in the overall
investment portfolio of Mrs Y were often missed.

(i)

The trustee has calculated the return on the index as:


(1,050/1,000 1) 100 = 5%.
This is a time weighted return which is unaffected by the incidence of the
funds cash flow.
The trustee has calculated the return on the fund as:
((1,450 250)/1,000 1) 100 = 20%.
The result of this calculation is affected by the significant amount of new
money received mid year when the market was at a relative low.
When this result is compared to the return on the index of 5% the
difference may be due in large part to the effect of the cash flow and hence
it is not possible to draw a conclusion about the value added by the fund
manager.
A more appropriate comparison would be either:

the time weighted return on the index compared to the time weighted
return on the fund, or

the money weighted return on the index allowing for the funds cash
flow compared to the money weighted return on the fund

The time weighted return on the fund is calculated as:


((1,150 250)/1,000 1,450/1,150 1) 100 = 13.5%.
This return of 13.5% on the fund compares to the return on the index of
5%.

Page 6

Subject 301 (Investment and Asset Management) April 2001 Examiners Report

(ii)

The other factors to be considered are:

risk; the analysis takes no account of the degree of risk undertaken by


the manager

what has the manager done; it would be necessary to carry out further
analysis to determine how the fund manager has achieved this result
and in particular, whether his actions were consistent with what he
said he was going to do

competition; the result would need to be put in the context of the


returns achieved by other managers with similar briefs

credibility; the period observed is too short for the result to be given
much credibility and a longer period would need to be analysed

strategic allocation; the initial allocation between the managers will


have been determined to give effect to the long term investment
strategy and revisions to the managers allocations would need to be
consistent with the long term strategy

appropriateness of the index; is the index appropriate for the portfolio


held by the manager

On the whole this was done well. Most candidates explained the weaknesses of the
Trustees approach and of the money-weighted return methodology in particular.

(i)

(ii)

Note: the question is seeking a common sense statement and so many


different reasons could be put forward which will score marks. However,
the reasons should be short term factors i.e. not fundamental structural
changes which might alter the long-term strategy. Possible reasons
include:

belief that the euro was oversold and would outperform the US$

concerns that US interest rates would rise and undermine US


securities markets whilst European interest rates had peaked

in connection with a thematic move away from high tech sectors to


which there may be a greater exposure in the US

expecting higher short term inflation in Europe resulting in higher


nominal growth

Two alternative ways of implementing this switch are:


(a)

by selling actual holdings in the US and buying more European


securities and then later reversing the trades, or

Page 7

Subject 301 (Investment and Asset Management) April 2001 Examiners Report

(b)

leaving the underlying holdings alone and reducing the US


exposure and increasing the European exposure by a derivatives
overlay e.g. selling appropriate US futures contracts and buying
appropriate European futures contracts and then later reversing
the trades

The principal advantages for (a) are that the exposures can be focussed in
the desired way and where the exposure is indirect i.e. via unitised funds,
the switch may be affected relatively quickly. However, where direct
investments are held the switch may involve buying and selling many
securities that will be relatively time consuming, administratively
complex and expensive.
The principal advantages for (b) are that the switch could be implemented
relatively quickly and cheaply without disturbing the underlying holdings.
However, there may not be any suitable instruments for altering exposure
to specific areas. The derivatives may not perform in line with the
underlying investments (although arbitragers should avoid significant
discrepancies). There is an additional element of credit risk in respect of
the counter party to the derivative contracts.
Tax considerations may result in one method producing a higher net
return than the other for a given market movement.
On average this was tackled well. However many candidates seem to assume (wrongly)
that the market impact of a large derivatives deal is negligible.

(i)

Net Present Value (NPV) The NPV is the discounted value of the
positive and negative cash flows at a chosen rate of interest. If
positive, the project might be viable.
Investor internal rate of return (IRR) The IRR is the interest rate
that equalises the net present value of the positive cash flows and the
negative cash flows of the project. If the IRR is equal to or greater
than the chosen hurdle rate rate then the project could be viable.
Payback Period (PP) The length of time before the capital expended
on the project is recouped from the net revenues without discounting
the cash flows.

(ii)

Page 8

IRR is a rate of return rather than an absolute amount. Hence, IRR


may be a better way of comparing several projects of differing sizes.
IRR assumes that surplus funds earn interest at the same rate as the
return on capital. This can lead to multiple IRR solutions although
usually only one solution is a reasonable answer. Worse, the
assumption that surplus funds earn interest at the IRR cost of capital

Subject 301 (Investment and Asset Management) April 2001 Examiners Report

(iii)

can be over optimistic and significant if the project turns significantly


cash positive at a given point.
NPV and IRR do not give any guidance regarding the timing of the
cash return on the investment. Payback period does.
Payback period is only a partial evaluator as it does not consider what
happens to the project post the payback point. However, if the project
is well behaved then payback period gives a very good guide to the
cash needs and long term potential of the project.
NPV allows surplus cash to be invested at a separate investment rate.
NPV requires the discount rate to be chosen in advance. Choosing the
discount rate can be difficult and relatively arbitrary. The problem is
one of presentation. The discount rate is an assumption and the
assumptions are sometimes ignored when sponsors are considering the
results of the project evaluation. As IRR effectively blends NPV and
discount rate together, IRR can sometimes be a better way of
presenting results to sponsors.

Options are uncertain and can be modelled in the same way as risks are
incorporated into the stochastic model.
If the option has significant impact to the project and for example leads
the project from one stage to another stage then it is useful to model each
stage separately and then together.
[Alternatively an answer which suggests that the options be evaluated
using option pricing formulae is acceptable if the answer notes the
difficulty of choosing the assumptions for the calculation and the answer
describes how the result would be incorporated into the evaluation model.

Many candidates scored well here. Those who have grasped the overall concept of capital
projects coped well with both the straight book-work parts and the later parts on the
application of the theory. In particular most candidates who progressed to the third part
described well a method of coping with options.

10

(i)

The valuation method used for the assets should be consistent with the
valuation method used for the liabilities. If the market value of assets is
used then that would generally imply that the liabilities should be valued
using a consistent rate of discount as no market value of liabilities will
exist.
The valuation method of the assets should also take into account the
purpose of the valuation. If the valuation is concerned with the break up
of the fund then clearly the market value and even less than the market
value may be the most appropriate value.
When valuing the fund on a going concern basis one might prefer to value
assets in a different way to avoid a misleading result and to ensure that
assets and liabilities are valued in a consistent way.

Page 9

Subject 301 (Investment and Asset Management) April 2001 Examiners Report

One possible method is to adopt a rolling average or smoothed value of


assets and to correspondingly value the liabilities using relatively stable
assumptions. The choice of the smoothing technique is of course
subjective.
Alternatively, the assets might be individually valued using a discounted
cash flow approach and the liabilities would be valued using the same rate
of discount. This approach involves a great deal of calculation. Also it is
very difficult to estimate the future cash flows of some investments.
Further this approach would not smooth away short term departure from
the long term asset allocation benchmark.
A third approach would be for the actuary to manufacture a notional
portfolio of assets and value these assets on a discounted cash flow basis
at a long term stable rate of discount. The liabilities would be valued at
the same rate of discount. The notional portfolio could be based on the
long term investment allocation benchmark and perhaps with some
recognition for the current asset allocation. This approach would be
consistent with liabilities, not calculation intensive and would be
consistent with valuing the fund on a long term and going concern basis.
(ii)

The liabilities are extremely long term. The actual timing of the
payments are unknown.
Further the in-service liabilities increase as pensionable salaries increase
and the pensions in payment are likely to also be increasing to some
degree in line with inflation. The amounts of the future liabilities are
therefore unknown.
Full matching of investment receipts and payment outgo is therefore not
possible.
Even immunisation is not a practical approach as the liabilities are real,
unknown and very long term in nature.
The principles of immunisation might be employed and particularly so if
the companys contribution rate objective is a long term one. Over the
long term shares are generally considered to be the most appropriate
asset to meet long term real liabilities.

(iii)

The relatively small surplus has been caused by a recent and major
decline in the market value of the assets. Hence, the fund is likely to have
a healthy surplus if a notional portfolio method is employed. Hence if the
valuation assumptions prove to be broadly correct in aggregate in the
future then the current contribution rate and the current investment
strategy are both appropriate.
Nevertheless the current investment objective does not take any regard of
the companys desire to limit its contribution rate to the fund. In effect

Page 10

Subject 301 (Investment and Asset Management) April 2001 Examiners Report

the company is seeking to reduce the relative risk of the investment


strategy probably at the expense of some future returns performance.
The trustees should require the investment managers to adopt an asset
allocation benchmark with a majority of shares and some property
reflecting the large size of the fund and its real liabilities.
The trustees should require that the market value of the assets remain
within a pre-agreed range around the benchmark. As market values
change and these ranges are exceeded steps would be taken to rebalance
the portfolio within a reasonable time frame given costs and market
opportunities.
The trustees could consider constraining the investments to domestic
assets only although the diversification and additional expected return is
likely to more than outweigh the associated risks of future return.
Further, if the companys liabilities are solely domestic then it may be
reasonable to constrain the fund to avoid investment in foreign shares and
fixed interest investments. The benefits of further diversification may not
outweigh the currency risks. This constraint would be expected to reduce
both the risk and returns from the fund.
Overall not done particularly well. Many candidates coped with the first part, the second
was less well tackled and the third was, in general, badly done. In covering the second
part many candidates confined themselves to simply stating the mathematical conditions
for immunisation. In the third there was a wide variety of attempts but few candidates
successfully tied together the ideas sought in the first two parts with the practical
situation outlined in the question i.e. practical constraints leading to better matching of
assets and liabilities and a more stable contribution rate going forward.

11

Principal features of general industrial companies as per Unit 4 (3.2.3) plus


comment on it being one of FTSE economic groups.
Factors that will affect the investment performance of all the companies in the
sector as per Unit 4 (3.1.2) i.e. resources, markets and structure with particular
reference to general industrial companies
Comment on the brokers recommendation, including any further information
required will include:
Company broker so bias/vested interest and so need to also consider other
independent sources for recommendations, industry outlook, company
announcements, views of suppliers and competitors
Fundamental analyses of company covering management, product, market
growth, competitive position, and accounting data. May need company visit

Page 11

Subject 301 (Investment and Asset Management) April 2001 Examiners Report

Earnings not the same as profits are other parts of business in decline, not
receiving investment or are earnings cyclical and temporarily depressed
What is point in economic cycle
Investor demand for IT sector high but volatile with limited profitability or
dividend records. Assets generally intangible. Share price could (already) reflect
possibility of rerating or break-up.
Other companies in sector may have similar plans
What is purpose of share issue is this the latest of many, perhaps against the
market trend? What are other investors perceptions of future capital and
dividend growth and risk
What is existing house exposure to company and policy regarding stock weights
in portfolios? Does the recommendation come as part of an underwriting
commitment on preferential terms?
Few candidates scored highly here. Parts (i) and (ii) were done OK but full marks were
rare. Part (iii) was not well done with most candidates being far too narrow in their
perspective and few covering all the relevant areas i.e. gather general information, do
your own analysis looking at all divisions of the company (not just the new IT area), look
at general market factors and take account of your own portfolio circumstances.

Page 12

Faculty of Actuaries

Institute of Actuaries

EXAMINATIONS
12 September 2001 (am)
Subject 301 Investment and Asset Management

Time allowed: Three hours


INSTRUCTIONS TO THE CANDIDATE
1.

You have 15 minutes at the start of the examination in which to read the
questions. You are strongly encouraged to use this time for reading only
but notes may be made. You then have three hours to complete the paper.

2.

You must not start writing your answers in the booklet until instructed to
do so by the supervisor.

3.

Write your surname in full, the initials of your other names and your
Candidates Number on the front of the answer booklet.

4.

Mark allocations are shown in brackets.

5.

Attempt all 10 questions, beginning your answer to each question on a


separate sheet.
AT THE END OF THE EXAMINATION

Hand in BOTH your answer booklet and this question paper.

In addition to this paper you should have available


Actuarial Tables and an electronic calculator.

301S2001 (28.2.01)

Faculty of Actuaries
Institute of Actuaries

Bank A and Bank B are two investment banks operating in the swaps market
around highly developed and liquid bond markets in a country. A and B enter a
swap agreement under which A lends cash to B in return for borrowing
securities.
(i)

Describe briefly the credit and market risk exposures faced by Bank A
under the swap agreement.
[2]

(ii)

Describe briefly the steps that could be taken by A to mitigate the risks in
(i) above.
[4]
[Total 6]

Chart 1 below shows how the risk level of a portfolio of equities varies with the
percentage of the portfolio invested outside of the countries of the European
Monetary Union (Eurozone).
(i)

Discuss briefly the risk implications of Chart 1 for a Eurozone investor


who wishes to invest in equities outside the Eurozone but who has
decided not to hedge the foreign currency exposure.
[2]
Chart 1

100% Hedged
Unhedged

20.0%

Risk level (volatility of annual returns)

19.5%
19.0%
18.5%
18.0%
17.5%
17.0%
16.5%
16.0%
15.5%
15.0%
14.5%
14.0%
13.5%
13.0%
0%

10%

20%

30%

40%

50%

60%

70%

80%

90%

100%

Percentage of equity portfolio invested outside of Eurozone

(ii)

Compare and contrast your answer to part (i) with the case where the
Eurozone investor decides to fully hedge the foreign currency exposure in
the portfolio.
[4]
[Total 6]

301 S20012

(i)

State the main aims of regulation in financial services markets.

(ii)

Discuss briefly the different ways a regulatory regime can be established.


[4]
[Total 6]

(i)

State briefly how you would expect returns from short-term and longterm fixed-interest government bonds to compare with original
expectations:
(a)
(b)

if inflation turns out to be lower than expected


if yields are falling more than expected

[4]

(ii)

In a particular country, over the last ten years government bonds have
outperformed property. Suggest briefly possible reasons for this.
[6]
[Total 10]

(i)

Outline the features of:


(a)
(b)

[2]

unit trusts
investment trusts

[3]
[3]

(ii)

State two differences between unit trusts and open-ended investment


companies.
[1]
[Total 7]

(i)

Describe briefly four methods of valuing an equity investment.

[4]

(ii)

For each method, comment on the main drawbacks.

[4]

(iii)

State with reasons your approach to valuing:


(a)
(b)

a company operating in a number of different industries


a property company

[4]
[Total 12]

Discuss how economic influences have an impact on the property market.

(i)

Define immunisation and state the conditions for it.

(ii)

Describe briefly the problems in putting immunisation theory into


practice.
[7]
[Total 11]

301 S20013

[13]

[4]

PLEASE TURN OVER

You are the finance director of a motor car manufacturer who owns the freehold
on the manufacturing facility. A property developer has offered to purchase the
freehold. Outline the factors that you would need to take into account in
deciding whether to recommend to your board the offer.
[10]

10

You are an independent adviser to a French-based fashion clothing retailer,


which is considering setting up a new internet subsidiary to sell its clothing
range into the US market. Currently it has no distribution outside France.
(i)

Discuss the methods you would use to determine whether to pursue this
venture.
[7]

(ii)

Identify the major risks involved in launching the subsidiary together


with ways that these risks might be mitigated.
[6]

(iii)

Describe how you would choose the discount rate to be used in evaluating
this project.
[6]
[Total 19]

301 S20014

Faculty of Actuaries

Institute of Actuaries

EXAMINATIONS
September 2001
Subject 301 Investment and Asset Management
EXAMINERS REPORT

Introduction
The attached subject report has been written by the Principal Examiner with the aim of
helping candidates. The examiners are mindful that a number of interpretations may
be drawn from the syllabus and Core Reading. The questions and comments are based
around Core Reading as the interpretation of the syllabus to which the examiners are
working. They have however given credit for any alternative approach or interpretation
which they consider to be reasonable.
The report does not attempt to offer a specimen solution for each question - that is, a
solution that a well prepared candidate might have produced in the time allowed. For
most questions substantially more detail is given than would normally be necessary to
obtain a clear pass. There can also be valid alternatives which would gain equal marks.
K Forman
Chairman of the Board of Examiners

20 November 2001

Faculty of Actuaries
Institute of Actuaries

Subject 301 (Investment and Asset Management) Sept 2001 Examiners Report

Examiners Comments
The examiners are becoming increasingly concerned that candidates are relying more
and more on straight bookwork to provide them with enough marks to pass this paper.
When a question requires application of the bookwork most candidates seem to feel that
writing out the bookwork will gain them marks, in general this is not the case. For
instance in question 10 part (iii) the question asked Describe how you would choose the
discount rate to be used in evaluating this project. Most candidates stated in their
answers that if the project were risky they would add a margin to their discount rate to
reflect the risk. From the question it was obvious that the project was riskier than the
retailers normal operations and therefore this should have been stated and the answer
framed on that basis.
Overall the examiners were disappointed by the level of understanding shown by
candidates.
Question 1
This question was poorly answered with most candidates gaining only a few marks. The
question was straightforward. However most candidates had not grasped the underlying
principles behind the swaps market and therefore were unable to provide satisfactory
solutions.
A large number of candidates suggested opening another position with a third bank. By
suggesting this they failed to understand that this strategy not only would cost money,
but also would open Bank A up to another set of risks.
Question 2
The answers to this question were marginally better than those seen in question 1.
However few candidates provided much in the way of comparisons as required in part
(ii).
A number of candidates spent time explaining why the chart looks as it does when the
question did not request that information.
Question 3
Part (i) was answered well. However in part (ii) many candidates seemed to regard
unregulated markets and voluntary codes of conduct as forms of regulated markets.
Question 4
In part (i) candidates seldom differentiated between real and nominal returns. There
was also confusion concerning returns & prices.
Part (ii) was, in general, answered satisfactorily.

Page 2

Subject 301 (Investment and Asset Management) Sept 2001 Examiners Report
Question 5
This was straightforward bookwork and, in general, allowed candidates to score well.
Question 6
Those candidates who read the question generally did well. However many chose to
answer a question on the valuation of a portfolio of equity investments. The latter course
produced answers in which market value, smoothed market value & utility value were
given.
If further clues as what was required were needed, part (iii) should have provided them.
Question 7
This question was not answered well. It required candidates to comment on how the
supply and demand of property would be impacted by changes in the economy. It also
required a description of the linkage between the economic changes and the property
market e.g. a buoyant economy means that tenants will require more space thus driving
up rents and hence property values. A surprising number of candidates managed to
answer this question without mentioning rents or rental values at all.
Question 8
This was answered very well. It was straightforward bookwork.
Question 9
This was not answered well. Many candidates failed to appreciate that they were the
finance director of the motor manufacturer and therefore would have knowledge of the
motor industry. In addition few considered the impact of the proposed transaction on the
business.
Question 10
The question received a varied response. Part (i) asked candidates to discuss which did
not mean list. Part (ii) required candidates to identify risks associated with that project
not projects in general. Therefore statements like market risk was not sufficient to gain
a mark. Part (iii) has already been discussed in the opening remarks.

Page 3

Subject 301 (Investment and Asset Management) Sept 2001 Examiners Report

(i)
Bank A has two credit exposures:

In the first, Bank B may default on the termination date and thereby
not return the cash in exchange for the bonds.

In the second, the issuer of the bonds might default during the period
of the repo agreement.

Bank A faces the risk that the market value of the bonds acquired as
security will drop below the amount of cash lent plus the interest thereon.
This risk increases with the duration of the bonds held as security.

(ii)
Under a repurchase agreement the lender of cash (Bank A in this case)
acquires full title to the securities handed over as collateral.
The risk is really that the value of the collateral at the time of default is
less than the repurchase price.
That risk is greater for bonds with high duration values than for bonds
with low duration values.
To mitigate this risk one might:
(a)

Insist on low duration bonds.

(b)

Insist on margining the transaction each day.

(c)

Insist on initial margin to cover, say 99%, of expected daily price


changes.

If the issuer of the collateral defaults, the lender of cash (Bank A in this
case) still has recourse to the counterparty (Bank B in this case) who must
buy the bond back at the agreed repurchase price.
If the margining provisions are in place the lender of cash (Bank A in this
case) could call for more collateral if the existing collateral were to be fall
significantly in price because of a default.
This risk could be further reduced by insisting on AAA-rated or AA-rated
(depending on the degree of comfort one is looking for) government bonds.

Page 4

Subject 301 (Investment and Asset Management) Sept 2001 Examiners Report

(i)

The investor can lower the risk level of the portfolio by investing almost
any percentage of the portfolio outside the Eurozone.
The risk level falls steeply as the percentage invested outside the
Eurozone rises to approximately 50%.
As the percentage invested outside the Eurozone rises above 50%
(approximately) the risk level begins to rise again.

(ii)

With the possible exception of 100% invested outside the Eurozone where
the foreign currency exposure is not hedged, the unhedged and 100%
hedged graphs both show that increasing the percentage of the portfolio
invested outside the Eurozone reduces the risk of the portfolio whether
the foreign currency exposure is hedged or not.
For small percentages (less than 10%) invested outside the Eurozone, the
reduction in risk is similar whether the foreign currency exposure is
hedged or not.
However the reduction in risk varies in a U-shaped fashion with the
percentage of invested outside the Eurozone when the foreign currency
exposure is unhedged.
By contrast, the reduction in risk increases dramatically with the
percentage invested outside the Eurozone when the foreign exchange risk
of the portfolio is fully hedged.
A massive reduction in risk can be achieved by investing all of the
portfolio outside the Eurozone and fully hedging the foreign exchange
exposure.
Risk falls from about 19.5% to 13.75%.
Not much additional reduction in risk reduction is achieved where the
foreign exchange risk is fully hedged when the percentage invested
outside the Eurozone is above 80%.

(i)

The principle aims of regulation in financial services markets are:

to correct market inefficiencies and promote efficient and orderly


markets

to protect consumers of financial products

to maintain confidence in the financial system

Wherever possible, regulation should not place an unacceptable burden of


cost on either the providers or consumers of financial services.

Page 5

Subject 301 (Investment and Asset Management) Sept 2001 Examiners Report
(ii)

Self regulation:

organised and managed by the providers

can work because knowledge of markets high and arguably best placed
to react to changes in the markets

threat of government intervention and potentially associated


bureaucracy means great incentive to succeed

can be viewed as too close to the industry and consequently might


suffer (perhaps unfairly) from low public confidence

Statutory regulation:

organised and managed by the government

lack of direct market knowledge can mean more costly, less effective
and less flexible

distance from industry should control abuse and hence lead to more
confidence from consumers (however, high profile failures can destroy
confidence just as rapidly as in self regulated markets)

Commonly, both self and statutory regulation operates in different


sections of financial services industries. For a successful system of
regulation, the professionalism of the providers and intermediaries is also
paramount.

(i)

Nominal yield = risk free yield + expected future inflation + inflation risk
premium.
(a)

Real returns would rise for both short and long-term bonds.
Nominal returns would rise more for long-term than short-term
because capital appreciation would be greater.

(b)

Real returns would fall because expected future inflation and risk
premium would be constant.
Nominal returns would rise for both because of price appreciation.

Page 6

Subject 301 (Investment and Asset Management) Sept 2001 Examiners Report
(ii)

Change in supply and demand for bonds


drawing up price (down yields)
change in taxes on property
lower economic growth property demand lower
rental increases lower
lower inflation lower rental increases
change in investment regulations
change in planning regulations property management more
difficult
change in supply of appropriate properties

(i)

(a)

trusts in legal sense


not subject to company law
stated investment objectives
open-ended
manager expands / contracts fund by creating / cancelling units
no borrowing
priced at net asset value

public company
closed-end fund
capital structure like any company
stated investment objective
can borrow (subject to limits)
normally listed on stock exchange
quoted price different (may be) to net asset value

(b)

(ii)

OEICS governed by company rather than trust law.


Entry / exit charges are explicit.

Page 7

Subject 301 (Investment and Asset Management) Sept 2001 Examiners Report

(i)

Discounted Dividend Model


This method values the share as the discounted value of the future
dividends. Dividend growth, the required rate of return and timing
assumptions are needed.
Price Earnings Ratio
With an estimated value for future earnings per share and an appropriate
price earnings ratio, a price can be calculated by multiplying the two
together. The price earnings ratio is best estimated using known results
for similar companies
Dividend Yield
In a similar way to the method involving the price earnings ratio,
estimated dividends can be divided by an appropriate dividend yield to
give an estimate of the share price.
Net Asset Value
The net asset value is a measure of the price based essentially on the
realisable value of the assets. This method can be useful when a company
is not making profits.
Value Added Methods
A share price can be calculated by adding together the accounting value or
book value of a share and the added value being generated by the assets
tied up in the company. The economic added value can be calculated as:
net operating profit after tax - (companys weighted
average cost of capital capital retained for shareholders)

(ii)

Discounted Dividend Model

assumptions needed for required rate of return and dividend growth

level and shape over time of assumptions crucial to correct valuation

Price Earnings Ratio

Page 8

need reliable estimate of earnings (finding suitable peer group to set


price earnings ratio challenging - company may be abnormal for good
reasons)

consistency required between figures for meaningful results

Subject 301 (Investment and Asset Management) Sept 2001 Examiners Report
Dividend Yield

need reliable estimate of dividends

dividend yield too must be reliable and free from distortions due to
recent abnormal distributions

Net Asset Value

heavy reliance on correct valuation of assets (not necessarily


appropriate to use accounting figures)

particularly if using statistic to estimate break-up value of assets


(ii)

Value added methods

(iii)

(a)

need to determine average weighted use of capital


book value needs to be determined by suitable method
The group would be best valued as the sum of the value of its parts.
Because assumptions are required to estimate the value, it would
be very difficult to get meaningful results without breaking the
company up into the different business units.
This is likely to be straightforward as available information will be
broken up in a similar way.

(b)

Property company assets are dominated by the properties under


management and the intellectual capital built up in the employees
managing the portfolio.
On wind-up, the property would be expected to have a broadly
unchanged value.
A common valuation method used for valuing property company
shares would be based on the net asset value of the company.

Page 9

Subject 301 (Investment and Asset Management) Sept 2001 Examiners Report

Economic influences have an impact on the property market in three interrelated areas: occupation market, supply of property, and investment market.
The interaction between occupational demand and supply of property for rent
determines the market level of rents.
The property investment market determines the capital value of rented property.
Occupation market
Any factor which has an effect on economic activity will affect occupational
demand for property.
Tenant demand is closely linked to the buoyancy of trading conditions
and GDP.
Economic growth increases demand for commercial and industrial premises.
However, the impact of economic growth will not necessarily be uniform across
the different property sectors
or throughout regions of the country.
Levels of employment on the service sector tend to influence occupier demand for
offices very significantly.
New patterns of economic activity, domestically and globally, change demand
patterns.
If an example given. (no extra marks for more than one example).
Supply of Property
The peak of the property development cycle does not coincide with the peak of the
business cycle.
Development may be frequently restricted by local planning authorities.
Property takes time to develop.
Therefore the time lag between gaining consent for a property development and
completing construction of it, frequently results in substantial amounts of stock
coming on to the market as the economy slows down.
A slow down in the economy, coupled with rising real interest rates, is harmful to
the property development industry.
The property investment market
The state of the property investment market relies to a significant extent on the
occupancy market, as it is the latter that provides the investment income and the
potential for rental growth.
Property investment returns have been a good hedge in the long run against
unexpected inflation.
Assuming that there are no other external influences, freeholders should be able
to increase rents with inflation so that the real value of the rent is not
compromised.
However, for properties which have infrequent rent reviews inflation erodes the
real value of the rental stream.

Page 10

Subject 301 (Investment and Asset Management) Sept 2001 Examiners Report
Higher real interest rates should lead to a lower valuation of future rents and
therefore lower capital values.
The relationship between interest rates and property yields is unclear in the
short term.
In the longer term, high long term yields tend tom push up property investment
yields, other things being equal.
The sources of investment money, and whether they are positive or negative in
cash flow terms, are important in determining the state of the property
investment market.
The main sources are: institutional investors, public/private property companies
using bank debt, and international investors (all three required for the mark).
When overseas investors are significant purchasers of property the exchange rate
will have an effect on demand levels.

(i)

Immunisation is the investment of the assets in such a way that the


present value of the assets minus the present value of the liabilities is
immune to a general change in the rate of interest.
The conditions are:
(a)

present value of asset-income equals the present-value of the


liability outgo

(b)

the mean term of the value of the asset-income must equal the
mean term of the value of the liability-outgo

(c)

the spread about the mean term of the value of the asset-income
should be greater than the spread of the value of the liability-outgo

These conditions could be expressed as formulae if preferred.


(ii)

Immunisation is generally aimed at meeting fixed monetary liabilities.


Many investors need to match real life liabilities.
By immunising, the possibility of mismatching profits as well as losses is
removed
apart from a very small, second order effect
The theory relies upon small changes in interest rates.
The fund may not be protected against large changes.
The theory assumes a flat yield curve and requires the same change in
interest rates at all terms.
In practice the yield curve does change shape from time to time.
In practice the portfolio must be rearranged every day
to maintain the correct balance of equal discounted mean term

Page 11

Subject 301 (Investment and Asset Management) Sept 2001 Examiners Report
and to maintain the correct balance of greater spread of asset proceeds
Assets of a suitably long discounted mean term may not exist.
The timing of asset proceeds may not be known.
And the timing of liability outgo may not be known.

The factors that would need to be considered are:


(i)

What price is being offered, is it reasonable given sites use position etc?

10

What have similar sites fetched in recent sales?


What is book value, will there be a disposal gain, how will it be treated
in pcl account and are there tax implications?

(ii)

What will leaseback terms be (if applicable) rental, duration,


covenants, refurbishment?

(iii)

What alternative sites are there if not leasing back, cost of these, cost of
moving, restrictions?

(iv)

What development probabilities are being foregone, could planning


consent be obtained to increase value?

(v)

What will repurchase rights be and what will basis be?

(vi)

What mortgages are outstanding and what will impact be?

(vii)

Is property used to secure loans or treated as such in credit rating?


What impact would sale have?

(viii)

What will impact on workforce be of sale, change of facility?

(i)
The main purpose of the initial appraisal is to ascertain whether the
project is likely to satisfy the minimum criteria that have been established
by the parent company for projects to proceed.
The main criteria are likely to be financial.
Other possible criteria:
Achieving synergy or compatibility with other projects undertaken by the
parent company or any other of its subsidiaries.

Page 12

Subject 301 (Investment and Asset Management) Sept 2001 Examiners Report
Satisfying political constraints, both within and without the parent
company.
Net present value (NPV)
Internal Rate of Return (IRR)
Payback period.
Another measure which is sometimes used is receipts/costs ratio, this is
defined as the ratio of the NPV of gross revenues to the NPV of the capital
and running costs.
As IRR can give multiple solutions it is less popular than NPV.
The NPV method would yield a satisfactory result is the answer was
positive when an appropriate discount rate was used.
The result of the IRR, the payback period and the receipts/costs ratio
would be regarded as satisfactory if they exceeded the pre-set level of the
parent company.
Following this analysis a sensitivity analysis should be conducted in order
to ascertain how sensitive the result is to varying the parameters around
their most likely levels.
If the results proved very unsatisfactory then the subsidiary, as planned,
should not be launched.
If the results show a satisfactory outcome then a proper risk analysis
should be conducted.
In the initial stages corporation tax should be ignored to reduce the
complications, it will need to be included in the later stages of the
analysis.
(ii)
Risk
Language
Web security i.e. risk that
data supplied to the
retailer is not secure.
Credit Card Fraud
Fashion i.e. the
subsidiary may be
offering the wrong type of
clothing for the American
market

Mitigation
Employ people who are bi-lingual.
Employ software firm to advise on suitable
package.
Ditto.
Make sure thorough market research has
been conducted and do not offer high
fashion unless one is certain that it will
sell.

Page 13

Subject 301 (Investment and Asset Management) Sept 2001 Examiners Report
Stock i.e. ensuring there
is enough stock to meet
demand without meaning
there is an excess of stock.
Supply i.e. ensuring that
the clothes will be
delivered to the customer
within the promised
timescale.
Returns i.e. how will
unwanted goods be
handled.
Competition i.e. what are
other e-tailers doing and
how will it impact on this
subsidiary.
Presence i.e. how will
customers be attracted to
the website and will they
recognise the subsidiaries
name.
Pricing i.e. is the price
charged competitive with
other retailers both on the
internet and in the shops.
Currency i.e. how will
moves in currency affect
the prices the subsidiary
can charge for its goods
and how will it affect the
profitability of the
company when expressed
in Euros.
Market risk i.e. the stock
market may regard the
subsidiary as very risky
and put a lower valuation
on the whole group.
(iii)

Market research.

Investigate the companies offering third


party delivery services and choose one that
can fulfil the companys needs.
Set up a suitable internal system to monitor
returns.
Monitor competition.

Advertise.

Monitor prices.

It might be possible to hedge the currency,


at least in part.

Explain to investors what the plans are and


what strategies are being put in place to
minimise the risk and maximise the return.

Assume that a real discount rate is required however a nominal discount


rate can be used if inflation is allowed for in the cash flows.
The project might be considered as high risk given it is the first operation
for the retailer outside its home market and few, if any, retailers
operating via the internet are profitable; therefore the project should be
appraised on a higher discount rate than would be considered for projects
exhibiting normal degrees of risk for the parent company.
The discount rate used should reflect the discount rates used by other
start up dot com businesses however these rates may be hard to

Page 14

Subject 301 (Investment and Asset Management) Sept 2001 Examiners Report
determine and therefore an arbitrary addition to the discount rate the
parent company normally uses may be the only solution.
The determinants of the normal discount rate are:
The current cost of raising incremental capital for the parent company in
order to finance the subsidiary i.e. what is the rate of return that needs to
be earned on the capital if existing shareholders are to be no better off and
no worse off.
The cost of debt capital should be taken as the cost in real terms of new
borrowing for the parent company, by taking a suitable margin over the
current expected total real return on index linked bonds, having regard to
the companys credit rating, and multiplying by (1 - t), where t is the
assumed rate of corporation tax. The cost of equity capital should be taken
as the current expected total real return on index linked bonds plus a
suitable margin to allow for the additional return which equity investors
seek to compensate them for the risk they run.

Page 15

Faculty of Actuaries

Institute of Actuaries

EXAMINATIONS
10 April 2002 (am)
Subject 301 Investment and Asset Management

Time allowed: Three hours


INSTRUCTIONS TO THE CANDIDATE
1.

Enter all the candidate and examination details as requested on the front of your answer
booklet.

2.

You have 15 minutes at the start of the examination in which to read the questions.
You are strongly encouraged to use this time for reading only, but notes may be made.
You then have three hours to complete the paper.

3.

You must not start writing your answers in the booklet until instructed to do so by the
supervisor.

4.

Mark allocations are shown in brackets.

5.

Attempt all 11 questions, beginning your answer to each question on a separate sheet.
AT THE END OF THE EXAMINATION

Hand in BOTH your answer booklet, with any additional sheets firmly attached, and this
question paper.
In addition to this paper you should have available Actuarial Tables and
your own electronic calculator.

301A2002

Faculty of Actuaries
Institute of Actuaries

Define the following terms:


(a)
(b)
(c)
(d)

Capital cover
Certificate of deposit
Beta value
Systematic risk

(i)

Outline:

(ii)

[6]

(a)

the classical system of corporation tax

[2]

(b)

the imputation system of corporation tax

[2]

List the factors regarding the taxation system that investors need to consider
before deciding on the most appropriate investment strategy.
[2]
[Total 6]

A national sports body is considering building a new national sports stadium together
with an office and shopping mall complex on the site of the now disused previous
national stadium, situated in the suburbs of the countrys capital city. As well as
providing some of the finance itself, the national sports body is reliant on both the
national government and commercial financial institutions to fund the project.
(i)

Discuss the steps necessary to achieve an effective identification of the risks


facing the project.
[12]

(ii)

Suggest what the major risks may be.

(i)

Enterprise Value (EV) of a company is defined as the sum of the market


capitalisation of the equity and the net debt of the company. Two companies
in the same industry trade on different enterprise value to sales ratios. Explain
why this difference may exist.
[4]

(ii)

Explain why it might be preferable to look at earnings before interest, tax,


depreciation and amortisation (EBITDA) rather than basic earnings per share
when comparing two companies operating in a global industry.
[3]

(iii)

(a)

Define what is meant by the weighted average cost of capital (WACC).


[2]

(b)

Explain why one company might have a higher WACC than another
and what implications this has for the two companies.
[2]
[Total 11]

301 A20022

[3]
[Total 15]

Outline the main factors that will influence the asset allocation to bonds, equities and
cash in setting a long-term investment strategy.
[5]

You are given the following market data and information about a pension fund wholly
invested in domestic equities:
Date

Market Value
of fund

Domestic
Share Index
(Capital only)

Dividend Yield
on Domestic
Share Index

31 Dec 00
31 Mar 01
30 June 01
30 Sept 01
31 Dec 01

2,400
2,400
2,700
3,000
2,800

1,000
1,069
1,184
1,198
1,120

2.6
2.9
2.8
2.6
2.8

Period

Contribution
Income/Outgo

Investment
Income

Q1
Q2
Q3
Q4
(i)

(a)

37
20
125
-35

Stating any assumptions you make, calculate:


(1)
(2)
(3)

35
40
40
45

the money weighted return for each period and over the year
the time-weighted return for each period and over the year
the index returns over the same period
[9]

(ii)

Comment on the returns

[3]

(iii)

Compare the investment income actually received by the fund with the
investment income that would have been received if the fund had been
invested in the index.

[2]

(iv)

Using the information from the answers to (ii) & (iii), what conclusions might
you draw about the stock selection policy of the fund?
[3]
[Total 17]

(i)

Describe the different ways in which a futures exchange could manage its
credit risk exposure.
[7]

(ii)

Discuss the credit risks faced by participants in the over-the-counter (OTC)


derivatives market and how these can be reduced.
[5]
[Total 12]

301 A20023

PLEASE TURN OVER

(i)

Outline the basis of construction for the FTSE Actuaries Share Indices

(ii)

List the uses to which these indices can be put.

[2]

[3]
[Total 5]

An institutional investor based in a certain country has the equivalent of $500m in


assets backing long-term, real liabilities denominated in the currency of that country.
The fund has no current direct or indirect exposure to property investments.
The manager of the fund feels that commercial property in that country will increase
in value in the next few years at a rate significantly above that of any other asset class.

10

11

(i)

Outline the difficulties faced by the manager in gaining direct commercial


property exposure.
[4]

(ii)

Describe briefly the difficulties the manager would face in using a property
unit trust to gain exposure to commercial property.
[2]
[Total 6]

A private investor, not subject to tax, wishes to construct a portfolio consisting of


bonds and equities. The investor proposes to use the long-term historical returns on,
and standard deviations of, the two asset classes as estimates of the expected future
returns and risk of his portfolio. Based on this data, he wishes to construct a portfolio
with the minimum risk.
(i)

Discuss the suitability of his estimates of expected returns and risk levels. [3]

(ii)

Discuss the parameters that will influence the return on the portfolio and the
mix between bonds and equities needed to obtain the minimum risk portfolio.
[For this part of the question, you may assume that the historical returns and
standard deviation are accurate estimates of expected future returns and risk
levels.]
[8]
[Total 11]

(i)

List four economic factors which influence the level of Government bond
yields.
[2]

(ii)

Discuss the additional influences which lead to different yields being available
on:
(a)
(b)

(iii)

301 A20024

corporate bonds
equities

[2]

Comment on how the real yield premiums on equities over Government and
corporate bonds might be expected to move over a period of recession.
[2]
[Total 6]

Faculty of Actuaries

Institute of Actuaries

REPORT OF THE BOARD OF EXAMINERS ON


THE EXAMINATIONS HELD IN
April 2002

Subject 301 Investment and Asset Management

Introduction
The attached subject report has been written by the Principal Examiner with the aim of
helping candidates. The examiners are mindful that a number of interpretations may
be drawn from the syllabus and Core Reading. The questions and comments are based
around Core Reading as the interpretation of the syllabus to which the examiners are
working. They have however given credit for any alternative approach or interpretation
which they consider to be reasonable.
The report does not attempt to offer a specimen solution for each question that is, a
solution that a well prepared candidate might have produced in the time allowed. For
most questions substantially more detail is given than would normally be necessary to
obtain a clear pass. There can also be valid alternatives which would gain equal marks.

K Forman
Chairman of the Board of Examiners
25 June 2002
Faculty of Actuaries
Institute of Actuaries

Subject 301 (Investment and Asset Management) April 2002 Examiners Report

EXAMINERS COMMENT
While the overall pass rate was in line with previous years; many more might have passed if
focused on the question that was asked. Many candidates instead answered the question they
thought should be asked or just wrote all they knew on a particular subject without reference
to any question either real or imaginary. These approaches generally do not yield many marks
while candidates reduce the time they have to answer other questions.
As in previous years the examiners were disappointed in the candidates ability to apply
bookwork or to exercise any degree of judgement.
There was also a tendency assume that whenever an institution was mentioned examiners
automatically meant a life company was involved and the discussion soon centred around
free assets and the like. This is an examination in investment and asset management and
therefore and institutional shareholder could be a pension fund, an investment trust, an OEIC,
a unit trust, a private client manager, a general fund or a life company.
Question 1 was bookwork and should have provided well-prepared candidates with six
marks however many candidates had trouble pinning down the definition of a certificate of
deposit.
Question 2, again this was bookwork and was generally answered well.
Question 3 was, in the main answered well.
The answers to Question 4 varied considerably, few candidates pointed to the fact that in part
(i) the sales figure did not measure profitability. While in part (ii) the basis of the answer was
given in the question and yet many candidates failed to produce a satisfactory answer. Part
(iii) was reasonably well answered.
Most candidates gained a few marks on Question 5 however a number of candidates spent
some time discussing the liabilities without paying due attention to the assets and the returns
and risks associated with them.
Candidates who read Question 6 and thought about their answer before starting on the
calculations were well rewarded in that they made the right assumption regarding cash flows,
namely that they were received at the end of the period. Those candidates who assumed that
cash flows were received mid way through the period had trouble calculating the timeweighted return and usually changed their assumption in order to perform the calculation.
While these candidates did not lose marks it did mean that they had more calculations to
perform.
In general most candidates produced figures that were not far from the correct solutions,
candidates were not penalised for making arithmetic mistakes providing the underlying
formulae were correct.
Parts (ii), (iii) & (iv) of question 6 were very poorly answered, if candidates had made
mistakes in part (i) and had made reasonable comments in parts (ii) and (iv) then they could

Page 2

Subject 301 (Investment and Asset Management) April 2002 Examiners Report

receive full marks for those sections. In general the comments made demonstrated a poor
grasp of what the figures that been calculated actually meant.
In part (iii), some candidates calculated total returns or in some cases capital returns, many
candidates failed to appreciate that the yield figure on the domestic share index was an annual
yield.
Many candidates failed to read Question 7 and therefore produced unfocused answers. This
usually involved looking at the question from the prospective of a participant in the futures
exchange rather than the futures exchange itself. These answers scored a few marks usually
for mentioning initial and variation margins.
Part (ii) was poorly answered.
Question 8 was bookwork and most candidates received full marks or near full marks.
Question 9 demonstrated the either candidates dont read the question or that they do but fail
to grasp the significance of the facts given in the question.
Many candidates saw no contradiction in mentioning that sufficient properties may not be
purchased in order to provide a diversified property portfolio. At the same time they assumed
that the full $500m is invested in property thereby assuming the investment manager would
not want overall diversification between the various asset classes.
Candidates seem to lack any sense of what constitutes a large fund or a small fund and to
what extent liquidity would play a part in allowing a switch into property to take place.
What is basically the application of bookwork was not answered well.
Question 10 produced the poorest answers despite the examiners exercising a significant
degree of flexibility where candidates had chosen a different interpretation in answering part
(ii). Candidates confused the word parameters for factors however the examiners gave credit
for a well-constructed discussion of the factors rather than the parameters.
In general candidates seemed at a loss to use what knowledge they possessed to provide an
adequate answer.
In Question 11 parts (i) and (ii) were answered well with the answers to part (iii) being more
variable.

Page 3

Subject 301 (Investment and Asset Management) April 2002 Examiners Report

(a)

A calculation made for loans issued by companies.


The capital cover is the number of times that the assets of the company
(excluding intangibles and after notionally paying current liabilities) cover the
amount of the loan (including prior ranking loans).

(b)

A certificate issued by a bank


showing that a stated sum of money has been deposited for a specified time
at a specified rate of interest.
Certificates of deposit can be traded (i.e. sold) by the original depositor

(c)

A measure of a stocks volatility relative to movements in the whole market.


Usually defined as the covariance of the return on the stock with the return on
the market, divided by the variance of the market return.

(d)

The risk of the individual share relative to the overall market which cannot be
eliminated by diversification.
It is measured by the Beta factor.
A share with a Beta greater than 1 is said to be aggressive i.e. the share is
expected to do better than the market when prices rise.
Conversely, a share with a Beta less than 1 is a defensive stock,
i.e. its price will be expected to fall by less than the market when
prices fall.

(i)

In the classical system company profits are taxed twice.


Once in the hands of the company and once in the hands of the investor.
The investor may pay both income tax and capital gains tax.
The tax rates paid on income and capital gains may be different.
Under the imputation system the investor will receive their distributions after
the company has paid some or all of their tax liability on the distribution.
The sum paid by the company to the government is imputed to the investor.
If the investor is not liable to tax they may be able to reclaim some or all of the
tax paid.
For some investors there may a further tax liability on the distribution.

(ii)

The factors to be considered are:


The total rate of tax on an investment and how it is split between income and
capital gains.
The timing of the tax payments e.g. whether the tax is deducted at source or
has to be paid subsequently.
To what extent losses or gains can be aggregated over different investments
and time scales.
The extent to which tax deducted at source can be reclaimed.

Page 4

Subject 301 (Investment and Asset Management) April 2002 Examiners Report

(i)

Step 1
Make a high-level preliminary risk analysis to confirm that the project does
not obviously have such a high risk profile that it is not worth analysing
further.
A clear risk is that the finance cannot be raised
The government may decide that the capital required is too great to justify
politically.
The commercial financial institutions may doubt the ability of the national
sports body to successfully manage the project to a successful conclusion.
(these two marks for these or any other reasonable issue that each of the
financial backers might have)
It would be important for the project to understand clearly the positions of
these two parties before going further.
Step 2
Hold a brainstorming session of project experts and senior internal and
external people who are used to thinking strategically about the long
-term.
The aim will be to identify project risks, both likely and unlikely,
to discuss these risks
and their interdependency,
to attempt to place a broad initial evaluation on each risk,
both for frequency of occurrence
and probable consequences if it does occur,
and to generate initial mitigation options and discuss them briefly.
Step 3
Carry out a desktop analysis to supplement the results from the brainstorming
session,
by identifying further risks and mitigation options,
using a general risk matrix,
researching similar projects undertaken by the sponsor or others in the past
(including overseas experiences),
and obtaining the considered opinions of experts who are familiar with the
details of the project and the outline plans for financing it.
Step 4
Carefully set out all the identified risks in a risk register,
with cross references to other risks where there is interdependency.
Step 5
Ensure that upside risks as well as downside risks are covered.

Page 5

Subject 301 (Investment and Asset Management) April 2002 Examiners Report

A risk matrix could be used for the above purposes


with column headings relating to the cause of risk
and the rows relating to the risks in successive stages of the project

(ii)

The major risks are


(a)
(b)
(c)
(d)
(e)
(f)

finance not available from government


planning permission not being granted
returns will not be sufficient to attract investors
sports body not financially strong enough
wrong position
cost will be too great

Other risks will also be acceptable as there is no absolutely correct answer to


this part.

(i)

The sales figure will represent the value of the sales made by the company in a
particular year. The sales figure does not give any information regarding how
profitable the company is whereas the enterprise value represents the total
value of the company. The value that investors place on a company will be a
reflection of the return they expect the company to make. Therefore a
company with a high value of sales but low profit margins is likely to have a
lower EV compared to a company with the same value of sales but a high
profit margin.
The company with the low profit margin may have a higher than expected EV
if investors expect the profitability of the company to rise in the near future.
Given both companies are in the same industry the most likely reasons for one
having a higher EV to sales ratio than the other are:
(1)
(2)
(3)
(4)
(5)

(ii)

Basic earnings per share are calculated after taking into account:
(1)
(2)
(3)
(4)

Page 6

Management ability.
A more efficient capital structure.
The possibility of a take over bid.
size or liquidity ability to trade the stock
change of capital structure over the period

Interest on loans.
Tax on profits.
Depreciation of fixed assets.
Amortisation of goodwill.

Subject 301 (Investment and Asset Management) April 2002 Examiners Report

Each of these items can introduce distortions when trying to compare one
company with another.
If an investor wishes to make a comparison of companies operating in a global
industry, then the tax rate will vary from country to country as will the ideal
capital structure. Equally the way accounting rules treat depreciation and
amortisation can distort an investors view on the attractiveness of one
company compared to that of another.
(iii)

(a)

The weighted average cost of capital is defined the aggregate return


required by the providers of debt and equity capital, allowing for the
effects of tax and the risks borne by the capital providers.

(b)

The WACC of a company is a function of its capital structure, the costs


of financing in the debt markets, the tax rate applicable and the
companys beta. All these items may change from company to
company, even within the same industry.
A company with a high level of debt will have a greater proportion of
its WACC based on debt financing than a company with a low level of
debt. Equally a company with a high beta will have a greater
proportion its WACC based on its cost of equity finance.

Before we could construct a portfolio for an investor we would need to have some
idea of the nature and term of the liabilities of that investor.
This will determine the level of risk that the investor can afford to take in trying to
meet the liabilities.
Historically cash has been the asset with the lowest level of risk and return. Where
stability of capital values is prerequisite, cash is the appropriate low-risk asset but the
price of this low risk is low returns in the longer term relative to equities or bonds.
Where the investor can afford to take a somewhat higher level of risk in meeting the
liabilities, bonds may be the appropriate asset class because of its slightly higher riskreturn profile.
For investors who are looking for high or real returns and who can take bear the
volatility of equities, then an equity portfolio may be appropriate.
Benchmark or peer group comparison.

Page 7

Subject 301 (Investment and Asset Management) April 2002 Examiners Report

(i)

Contributions and investment income all occur on last day of each quarter.
The index returns need to be calculated on a similar basis using the yield at the
end of each quarter.
Time weighted return
Q1

Q2

Q3

Q4

(1.54)

11.67

6.48

(5.50)

10.64

Money weighted return


The quarterly returns are the same but the annual return is derived from
2,400 (1 + i) + 37(1 + i)+ 20(1 + i) + 125(1 + i) = 2,835
(1.54)

11.67

6.48

(5.50)

10.25

Index time weighted return


R(t) = ((I(t)*(1 + Y(t)/4))/I(t - 1) - 1)*100
where R(t) = total return for period t
I(t) = index value at time t
Y(t) = yield on index at time t
7.68

(ii)

11.53

1.84

(5.86)

15.14

Money and time-weighted are same for each quarter because of assumption
but annual is different and reflects time of cash flow v market movements.
Both under perform the index by a considerable amount
The first quarter is the period accounting for all the under performance
There is strong out performance in Q3
Given the difference in income, capital return for the fund have been very poor

(iii)

Period
Q1
Q2
Q3
Q4
Total

Page 8

Fund
Index
Income Income
35
40
40
45
160

18.6 [2,400*1,069/1,000*0.029/4]
18.6
17.8
19.6
74.6

Subject 301 (Investment and Asset Management) April 2002 Examiners Report

(iv)

As can be seen the fund was invested in stocks that yielded over twice the
average for the index.
It is likely that high yield stocks under performed in the year in question as
overall the fund under performed the index by a considerable margin.
The fund manager may have a yield requirement. If this is the case then
perhaps a different index should be used to monitor performance.

(i)

The exchange protects its credit exposure to participants in the futures market
in several different ways.
Trades can only be cleared by members of the exchange with clearing status.
Clearing status involves authorisation, having certain minimum capital and
operational standards.
Institutional investors, corporates and individuals who wish to effect futures
transactions must have their trades cleared by members of the exchange with
clearing status.
Clearing members of the exchange must pass on at least the initial margin
requirement and the variation margin calls to their clients. They can of course
pass on higher margin requirements.
The exchange imposes initial margin requirements on clearing firms (and
hence their clients) as part of the procedure of entering into a futures contract.
Typically, the initial margin requirement would provide the exchange with
sufficient capital (usually with 99.5% certainty) to weather an adverse price
movement in the futures contract in the event that the client/clearing member
defaulted.
The credit exposure of the client/clearing member to the exchange varies with
the value of the futures contract. For example, where the client has a short
futures contract on the FTSE 100 index and the index rises the clients
exposure to the exchange via the clearing member increases.
Variation margin is also required where the initial margin falls below a
threshold specific to the contract. This provides collateral movement from the
client to the exchange that varies with credit exposure of the client/clearing
member to the exchange.
Price movement limits allow the exchange to suspend trading in a contract if
its price moves up or down by more than set limits. Such limits allow the
exchange to limit its credit exposure to clearing members/clients in the event
of sudden moves in the price of the futures contract.
The margin requirements for speculators may be different to those for hedgers
as speculators may not have the underlying asset to deliver.

Page 9

Subject 301 (Investment and Asset Management) April 2002 Examiners Report

Exchanges usually reserve the right to increase the margin requirements if


they deed it fit. This was one of the problems that a well-known company in
the oil futures market (Metallgesellschaft) suffered.
Close contract if margin not produced.
(ii)

In the OTC markets, there is generally1 no central counterparty that acts as the
buyer to every seller and the seller to every buyer. Hence the credit risk of the
counterparty must be assessed both initially and on an ongoing basis.
Settlement of OTC contracts tends to take place at the expiry of the contract so
significant credit exposures can build up for the counterparties.
In addition, because of the nature of forward contracts they can swing from
being an asset on a balance sheet to being a liability. So there can be
significant and sudden swings in credit risk.
Some participants in the OTC market call for collateral posting to the party
with the credit exposure to reduce credit risk.
1

Note that some swaps contracts have a central counterparty for all trades e.g. certain swaps
cleared with the London Clearing House (LCH).

(i)

These indices are calculated on a weighted arithmetic average market


capitalisation basis with weights reflecting free float in bands.

(ii)

Any 6 of the 8 points listed in Unit 12, 2 on page 4.

(i)

Prime commercial property comes in large unit sizes.


With a portfolio of $500m, even if the manager wishes to invest as much as
10% of the portfolio in commercial property, this amounts to only $50m.
It is unlikely that the fund would get a diversified portfolio of direct property
covering shops, offices and industrial properties across different geographical
areas of the country for $50m.
The manager would need to hire in or outsource the property investment
portfolio to a manager with property investment management skills. This will
add to the lead-time in getting the portfolio set up.
There is also a considerable lead time in finding commercial properties
suitable for institutional portfolios, negotiating terms and effecting the legal
conveyance of the property to the institutional investor.
The commercial property market may have moved significantly in the time it
takes to put the last two things in place.

Page 10

Subject 301 (Investment and Asset Management) April 2002 Examiners Report

The costs of dealing in property are very high compared with say international
equities or bonds. It is difficult to generate good investment returns from
tactical property investment.
The valuation of property investments is very different from say that of
equities or bonds which are quoted on a stock exchange. The manager would
need to consider the systems necessary to produce fund valuations when direct
property investments are held.
(ii)

It may be difficult for the manager to redeem units in a property unit trust
quickly. For example, property unit trusts may reserve the right to defer
cancellation of units for periods of six months or so.
Property unit trusts often maintain a significant amount of cash in their
portfolios in order to be able to meet redemptions. So the investment is a
combination of cash and commercial property.
A pure commercial property unit trust may not exist.
He might represent a large percentage of a fund and therefore create liquidity
problems.

10

(i)

The returns from and risk of bonds and equities varies depending on the time
period over which they are measured.
For example, the geometric mean of nominal returns on UK Gilts (as quoted in
the Core Reading for Subject 301) were 5.1% per annum over the period
1899-2000, 13.1% per annum over the period 1975-2000 and 12.6% per
annum over the period 1990-2000.
The investor would need to have some idea of his investment time horizon.
Then examine the returns and risks over a number of periods of similar length
to his time horizon to get a feel for the stability of returns and risk levels for
that that time horizon.

(ii)

As equities have in the past outperformed bonds over any long period of time,
the higher the proportion of equities in the fund the higher the expected return.
The risk of a portfolio consisting of two asset classes is not a linear
combination of the risks of the two asset classes.

Page 11

Subject 301 (Investment and Asset Management) April 2002 Examiners Report

The risk of the portfolio, n, is given by the equation:


n = {p2 b 2 + (1 p)2 e2 + 2p(1 - p) b e } Equation 1
Where n is the expected risk of the portfolio, b is the historical risk of bonds,
e is the historical risk of equities and is the coefficient of correlation
between bonds and equities.
Equation 1 can be minimised with respect to p to determine the proportion in
bonds to give the minimum risk.
The value of p that minimises Equation 1, will determine the expected return
on the portfolio.
A key determinant of the minimum risk level is the coefficient of correlation
between bonds and equities.
If bonds and equities were perfectly correlated the risk of the portfolio would
be a linear combination of the two risks.
As returns on bonds and equities are not perfectly correlated, the risk will be
somewhat lower than that of a linear combination of the risks of bonds and
equities.
return will be a combination of allocation to bonds and equities
and the actual returns of binds and equities
actual returns will most likely differ from expected because of changes in
risk free (real) rate of return
inflation
risk premiums
exchange rates
economic/corporate profit growth
supply/demand constraints
If objective is to match particular liabilities, nature and term will dictate type
of assets
Specific types of bonds and equities may be necessary to minimise risk e.g.
corporates, overseas, passive management, growth/value.

Page 12

Subject 301 (Investment and Asset Management) April 2002 Examiners Report

11

(i)

Inflation
Short-term interest rates
Fiscal deficit
The exchange rate
Institutional cash flow
Other economic factors

(ii)

(a)

Corporate bonds are unlikely to carry the same security as loans made
to a government. The yield differential will be based on the
differences between the borrowers.
The risk of default requires investors to demand high yield. The
default risk (and hence the additional premium) increases with
pressures on profits.
Liquidity will also be an issue. Corporate bonds are less liquid than
gilts, again leading to investors demanding additional yield premium.
Add to this, the potential impact of world events causing investors to
move to ultra secure investments will lead to an increase in the yield
premium over government bond yields.

(b)

Demand features dominate the drivers of the level of the equity market.
Central to this will be investors expectations for corporate profits.
Investors will reflect the level of risk (over government securities) they
are willing to take. They will accept a lower premium if they are
confident about future corporate profitability.
Investor views on economic growth, interest rates and inflation
expectations as well as general market confidence will drive the equity
market.
As growth starts to slow, equity markets start to fall in anticipation of
corporate profitability being lower.
Demand shifts towards bond based securities and government bonds in
particular.
Company insolvency has increased the risk of corporate bonds. The
yield premium on corporates over government bonds increases. Actual
defaults also serve to drive underperformance relative to government
bonds.
Short-term interest rates (expected and in time actual) reduce as the
government looks to stimulate demand. As economic confidence
begins to recover, investors are more willing to accept equity risk and
corporate default risk. After a period of underperforming the bond
markets, equity returns improve.

Page 13

Faculty of Actuaries

Institute of Actuaries

EXAMINATIONS
11 September 2002 (am)
Subject 301 Investment and Asset Management

Time allowed: Three hours


INSTRUCTIONS TO THE CANDIDATE
1.

Enter all the candidate and examination details as requested on the front of your answer
booklet.

2.

You have 15 minutes at the start of the examination in which to read the questions.
You are strongly encouraged to use this time for reading only, but notes may be made.
You then have three hours to complete the paper.

3.

You must not start writing your answers in the booklet until instructed to do so by the
supervisor.

4.

Mark allocations are shown in brackets.

5.

Attempt all 13 questions, beginning your answer to each question on a separate sheet.
AT THE END OF THE EXAMINATION

Hand in BOTH your answer booklet, with any additional sheets firmly attached, and this
question paper.
In addition to this paper you should have available Actuarial Tables and
your own electronic calculator.

301S2002

Faculty of Actuaries
Institute of Actuaries

You are the investment manager of a unitised fund which holds a diversified portfolio
covering all major asset categories. Equity markets have recently received a severe
shock. There are concerns that your fund may experience a very high level of
encashments and it has been suggested you should protect the fund by selling futures
contracts.
State the main comments you would make in reply to this suggestion.

[4]

A large pension fund holds a substantial diversified portfolio of US equities that is


managed by a specialist manager. The managers brief is to modestly outperform over
rolling three-year periods relative to a benchmark index. The return on his portfolio
for the latest twelve months was -20% as measured in US$ whilst the benchmark
index fell 25%. The chairman of the trustees has written to you as the funds
investment advisor stating this return is a disaster and that the fund has lost a fortune.
He argues the manager should be sacked immediately and a claim commenced against
the manager for the loss the fund has suffered.
Outline the main points you would make in your reply and identify what further
information you would require.

[5]

(i)

State the different characteristics of offices and residential properties.

[4]

(ii)

A unitised fund invests in commercial properties. There are no derivative


contracts available which are relevant to the properties in which the fund
invests.
(a)

Explain the particular difficulties that large encashments cause such a


fund.

(b)

State two alternative approaches this fund could take to manage large
encashments and describe any adverse aspects associated with them.
[6]
[Total 10]

By the end of 2001 the global economy had slowed to the point where it wasnt
growing. The monetary authorities in many countries cut short term interest rates to
historically low levels.
Explain what the monetary authorities were hoping to achieve and how equity and
bond markets could be expected to react.
[6]

State the formula used to incorporate liabilities in portfolio theory, defining all
components.
Explain briefly how the formula may be used in practice.

301 S20022

[2]

[2]
[Total 4]

(i)

Explain why a pension scheme might consider investing in non-government


debt securities.
[4]

(ii)

State the various factors that may influence the difference in yield between
government and non-government debt securities.
[2]

(iii)

Describe briefly how risks associated with an individual corporate bond may
be reduced.
[4]
[Total 10]

You have been called as an expert witness to advise on a suitable discount rate to
calculate the value of a lump sum award to a 50-year old individual in compensation
for his claim for loss of earnings following an injury at work. The amount of the
award is based on the annual earnings lost and the number of years out of work and
makes use of a discount rate in order to create a present value. In prior cases the
discount rate has been the real yield available on an index of long dated index-linked
government securities with the individual stocks weighted by their respective market
capitalisation.
(i)

Give reasons why the yield on a long dated index-linked government securities
index could be an inappropriate discount rate at this time.
[2]

(ii)

Discuss whether the average yield on an unweighted index of all index-linked


government securities available in the market could be a more appropriate
discount rate.
[2]

(iii)

It has been suggested that a more reasonable discount rate would be the
expected return above inflation on a portfolio of mixed assets.
Discuss the type of assets that a typical individual investor would hold in such
a portfolio and the factors you would need to consider in determining an
appropriate discount rate.
[4]
[Total 8]

You have been asked to develop an index against which to judge an equity portfolio.
(i)

Set out the formula for measuring the relative changes in the constituents
share prices.
[2]

(ii)

Describe briefly the reasons for only including the level of free float of
shares available for investment.

[2]

(iii)

Modify your formula to take account of the free float.

[2]

(iv)

State two limitations of your formula.

301 S20023

[2]
[Total 8]

PLEASE TURN OVER

A project sponsor has decided to build a small power plant to supply steam and
electricity to a paper mill. The sponsor has asked your bank to lend money to the
special purpose company which will own the power plant.
Outline the various ways that the bank may be able to reduce its risk in respect of both
repayment delay and default. Your answer should consider features of both the loan
and the project.
[10]

10

(i)

Describe briefly the three main corporation tax systems.

(ii)

State with reasons which system(s):


(a)
(b)
(c)

11

(i)

might be expected to encourage dividend payments


might be expected to encourage companies to diversify
might be expected to be favoured by tax exempt investors

[3]

[1]
[1]
[1]
[Total 6]

State a formula relating the expected return with the required return for
conventional government bonds.
State another formula relating the expected return with the required return for
property.
[2]

12

(ii)

List the main simplifying assumptions made to derive the above formulae. [2]

(iii)

Discuss briefly why actual returns over a 12 month period may differ markedly
from the expected returns for:
(a)

a portfolio of government bonds

(b)

a portfolio of property leased to the same government

[4]
[Total 8]

You are an analyst working for a merchant bank that is responsible for the first public
share offering of a company. You have been asked to quote a price range for the share
offer.
(i)

Describe the process you will follow to determine the price range.

[4]

(ii)

List the information you will require to value the company.

[3]

(iii)

Give two reasons why you might recommend a price range lower than the fair
market value.
[2]
[Total 9]

301 S20024

13

You are given the following total return data for a fund and relevant indices:

Equities
Fixed Interest Bonds
Index Linked Bonds

Year 1
Index Fund

Year 2
Index Fund

Year 3
Index Fund

+31% +35%
+14% +13%
+11% +12%

-2% +2%
+17% +14%
+17% +16%

+24% +26%
+1% +2%
+7% +7%

The funds strategic benchmark was set at the start of year 1 as 60% Equities, 20%
fixed interest bonds and 20% index linked bonds; it was not rebalanced. The fund
manager adopted a strategy of 50% equities, 40% fixed interest bonds and 10% index
linked bonds at the start of the period and did not rebalance. Ignoring the funds cash
flows and stating any assumptions you make:
(i)

Calculate the total return on the fund and the strategic benchmark over the
period and state the relative performance.
[3]

(ii)

Estimate how much of the funds relative performance is attributable to stock


selection and how much is attributable to asset allocation and comment on
your results.
[9]
[Total 12]

301 S20025

Faculty of Actuaries

Institute of Actuaries

EXAMINATIONS
September 2002
Subject 301 Investment and Asset Management
EXAMINERS REPORT

Introduction
The attached subject report has been written by the Principal Examiner with the aim of
helping candidates. The examiners are mindful that a number of interpretations may be
drawn from the syllabus and Core Reading. The questions and comments are based around
Core Reading as the interpretation of the syllabus to which the examiners are working. They
have however given credit for any alternative approach or interpretation which they consider
to be reasonable.
The report does not attempt to offer a specimen solution for each question that is, a
solution that a well prepared candidate might have produced in the time allowed. For most
questions substantially more detail is given than would normally be necessary to obtain a
clear pass. There can also be valid alternatives which would gain equal marks.
K Forman
Chairman of the Board of Examiners
26 November 2002

2.12.02

Faculty of Actuaries
Institute of Actuaries

Subject 301 (Investment and Asset Management) September 2002 Examiners Report

In general candidates did well on bookwork questions, made some reasonable attempts at
knowledge parts but were disappointingly poor on the application parts of questions.
The paper had thirteen questions, more than have been set previously. As a consequence,
however, the largest mark for a question was 12. Also the larger number of questions meant
that more of the syllabus was covered but this should not have affected students ability to
obtain a pass.
Specific comments on each question follow at the end of the individual solution.

If there is a heavy encashment then the fund will need cash to pay out in respect of the
redeemed units. The main way cash can be generated is by selling the investments
held by the fund. Derivatives i.e. futures and options, will not generate much cash
e.g. if futures are sold. Although they may lock into a particular level for selling,
either markets or specific investments, this is not a particular issue as the funds unit
price is linked directly to the market prices for the funds underlying investments.
Using derivatives to reduce market exposure may produce a performance advantage
but this is not a solution to the funds cash flow problem unless the fund can borrow
against the receipts from the derivatives contracts. However, the markets may have
already adjusted to (or overreacted to) the recent severe shock with the result that
reducing market exposure will not actually bring in performance benefits. Whether or
not the fund is allowed to use derivatives is a point that would need to be confirmed.

This question was badly answered as candidates focussed on futures and derivative features
rather than whether the suggestion was a suitable approach.

Whilst the return on the portfolio for the twelve months is a large negative return, this
needs to be compared to the benchmark set for the manager and over a time period
consistent with his brief i.e. 3 years. Over the twelve months referred to the US
equity markets has fallen significantly e.g. the return on the S&P 500 index was
27%. Thus, even over the short time period quoted, the managers performance may
not be poor relative to his benchmark index; indeed he may have outperformed his
benchmark.
A claim against the manager may be justified if the fund has suffered a loss as a result
of the manager not implementing his brief properly. However, this does not appear to
be the case on the information provided.
To address the question fully we would need details of the brief set, the history of the
returns achieved by the manager and details of the portfolios construction overtime.
If the trustees were concerned about nominal returns on the portfolio then it would be
appropriate to review the investment strategy.

Most candidates covered the obvious points but many were not able to identify what further
information would be required and what issues needed to be looked at.

Page 2

Subject 301 (Investment and Asset Management) September 2002 Examiners Report

(i)

The different characteristics of investments in offices and properties are


summarised as:
Offices
Available in very large units.
Companies

Unit Size
Type of tenant
Security of rental income

Effect of political factors

Can be high if rent a small


part of tenants outgoing,
improved if office is multi
let.
Not that affected.

Sensitivity to level of
economic activity

Sensitive to corporate
spending but not directly to
consumer spending.

Depreciation

Offices can become obsolete.

Location

General location important


but precise location within a
general area is not critical.

(ii)

Residential
Relatively small units.
Individuals, or companies at
the luxury end of the market.
Relatively higher risk as
single tenant per unit.
Historically more affected
than other property sectors
e.g. through rent controls and
laws protecting tenants
rights.
Sensitive to factors effecting
consumers disposable
incomes although a falling
housing market may see
increased demand for rental
accommodation.
Whilst houses may need
refurbishing they do not
usually become obsolete.
Precise location may be
critical.

(a)

Property is illiquid and hence there is a problem raising cash to pay the
encahsing investors. The forced sale of one of the funds properties
may harm the interests of the remaining investors.

(b)

The fund could maintain a high level of liquidity. The principle


disadvantage of this option is that in anticipation of possible
encashments, the funds exposure to property is diluted and this will
impact on the funds return.
An alternative is to contractually impose a restriction saying large
encashments will be delayed for a period e.g. 12 months, at the option
of the manager of the fund. This enables the manager to sell properties
in an orderly manner at a reasonable price. However, investors may be
unhappy that they cant get their money back when they want.

Part (i) was bookwork and was well answered.


Reasonable attempts were made at (ii) although a number of candidates chose to ignore the
information given in the question regarding derivatives.

Page 3

Subject 301 (Investment and Asset Management) September 2002 Examiners Report

By reducing interest rates the monetary authorities were seeking to stimulate


consumer and corporate spending by making money cheaper to borrow and by
reducing the attractiveness of short term savings.
Of concern to the authorities would be the risk of economic growth being too strong
giving rise to inflationary pressures. This was not the case.
The yields on short dated bonds would be expected to fall in line with the cuts in short
term rates by the authorities. The yields on long dated bonds would also be expected
to fall provided investors were comfortable about the expected levels of future
inflation.
The level of equity markets would be influenced by:

the improved prospects for economic growth, if it was believed that the cuts in
interest rates would stimulate growth

the reduced level of bond yields resulting in an increased value for future
corporate profits, and

fluctuations in the equity risk premium

The first two factors would serve to increase the equity market, the latter factor would
undermine this to a degree if investors saw an increased risk to equity investment.
Candidates could explain what governments were trying to achieve and covered most of the
points with regard to short-dated securities but were poorer on long-dated and equity
implications.

S = A xi (1 + Ri ) - L ,
i =1

where S
A
xi
Ri
L

is the surplus at the end of the period;


is the value of the assets at the start of the period;
is the proportion invested in security i;
is the return on security i;
is the projected value of the liabilities at the end of the period.

Mean-variance portfolio theory can then be applied to minimise the variance of the
surplus for a given expected return, treating the liability as a negative asset.
In practice it will be necessary to decide how to place values on the liabilities and to
determine, not only the expected value of the liability at the end of the period, but also
its variance and covariances with the assets. One way of doing this is to use a
stochastic asset liability model.
Candidates either knew this part of the course or did not. Those that did scored close to full
marks.
Page 4

Subject 301 (Investment and Asset Management) September 2002 Examiners Report

(i)

General references to how to set investment strategy and link of fixed


liabilities to fixed assets
Maturing schemes (longevity, closed to new entrants) generally has led to a
secular trend to bonds
Liabilities matched by bonds, particularly for international accounting
standard
Limited supply of government debt, lack of new issues anticipated, market
tightly held at long end
Companies issuing bonds in favour of equity, UK corporate market now
bigger than gilt
Utility companies issuing index-linked bonds
Investors want to diversify
Higher yield, limited credit risk
Scope for active management from greater diversity of issues
Could be self-investment driven
Could be a match for bulk transfer

(ii)

The forecast strength of the economy during the term of the loan
Corporate prospects, credit quality, during the term of the loan
Marketability

(iii)

Floating charge over some or all of the assets of the company


Fixed charge over a given asset in addition to ranking in a wind up
Collateral provided
Financial covenants e.g. income cover
Prior ranking debt (e.g. subordinated debt will rank behind senior debt)
Rights in a technical default
Restrictions on further borrowing
Parent co. guarantees (e.g. loan issued by a subsidiary)
Third party guarantees (e.g. insurance credit wrap)

It was disappointing that candidates did not do well on this question. In part (i) many
obvious points were missed while in (ii) only credit quality and marketability tended to be
covered. Very few candidates got the point of (iii) which is extremely disappointing as this is
an important facet of non-government bond valuation.

(i)

Market may be distorted due to limited supply and excess demand from other
investors. Investing in such securities may not be what a reasonable person
would do given size of award. Claimant may expect earnings growth greater
than prices.

(ii)

Current and future market structure dependent on supply more than demand
from any individual or group of investors and has developed due to historical
and political funding considerations. A low risk portfolio for an individual
would produce a stream of income and capital payments that would leave
them no worse off had the injury not occurred. Matched portfolio would have
mixture of dates including very short.
Page 5

Subject 301 (Investment and Asset Management) September 2002 Examiners Report

(iii)

Tax, dealing costs and charges, cashflow needs, security, wealth enhancement
mix of cash, bonds, property and equity biased towards UK market. Could
be individual shares but more likely to be collectives [301:16 individual
investment considerations].

Reasonable attempts were made at (i) and (ii) but in (iii) few comments were made about the
factors that need to be considered. Most spent too much time commenting on appropriate
asset allocation.
P

(i)

wi Pii,0,t
I (t ) = K i
wi
i

where I(t) is the capital index at time t;


Pi,t is the price of the ith constituent at time t;
Pi,0 is the price of the ith constituent at time 0 the last time at which
there was a capital change;
wi is the weight applied to the ith constituent;
K is a constant related to the starting value of the index at time 0.
(ii)

(iii)

Removes strategic holdings, represents investable universe so more rational


for index-tracking management.

I (t ) =

Ni,t Pi,t
i

B(t )

where Ni,t is the number of shares issued for the ith constituent at time t;
B(t) is the base value, or divisor, at time t.
B(t) is obtained from B(t - 1) through the chain-linking process.
(iv)

Formula assesses capital values only it does not allow for income
receivable either by time or amount. Different companies will have different
distribution policies. Formula takes no account of the charges (management
or dealing costs, commissions and duties) or taxes (income or capital gain)
associated in running the clients portfolio. These could be significant if there
are major changes in the makeup of the portfolio due to such events such as a
change in assessment if the free float.

This was another bookwork question that was in the main well done.

Page 6

Subject 301 (Investment and Asset Management) September 2002 Examiners Report

Project risks and their mitigation

Construction risk
Power supply agreement with the paper mill
Operating/ maintenance agreement with the project sponsor
Residual value guarantee from a credit worthy entity
Raw materials supply agreement (water, coal, gas, sludge etc.)
Amount of equity
Amount of subordinated debt
Full project appraisal (incl. Identification and analysis of risks)
Cash flow
Liquidity account a minimum of 6 to 12 months interest
EBITDA/ Interest cover
Risk transfer to sub-contractors, raw material suppliers etc..
Risk insurance full all risks cover
Risk sharing potentially with the paper mill
No dividends period

Loan terms and conditions

Timing of the loan, pre or post construction

Loan guarantee from the sponsor

Loan guarantee from the builder during the construction period

Term of loan the power plant will have a very long useful life. As a lender
your interest in the project is limited to the term of the loan. In this case, the
sponsor is likely to have a contract to supply heat and power to the mill for a
period of say 10 to 15 years. The sponsor is likely to ask for a loan term equal
to the power supply agreement term.

Restrictions on further borrowings

Principal repayment schedule likely to be as fast as the cash flows will


allow for the first say three to five years. Decreasing the leverage in the
special purpose company will allow the sponsor to refinance the loan at lower
cost. It is unlikely that the sponsor will wish to repay the whole of the debt.
Debt is less expensive than equity.

Page 7

Subject 301 (Investment and Asset Management) September 2002 Examiners Report

Financial covenants the main financial covenant will be a coverage such as


EBITDA to interest expense ratio of at least 1.5. Lender may have rights to
take control at this point. Also restrictions on further borrowing. Interest
cover

Fixed security over all assets

This question was poorly answered by most candidates. There was a tendency to write down
all they knew about project management and not look at it in the context of the question
posed. It was disappointing that the guidance in the question on what needed to be looked at
was ignored.

10

(i)

Classical A companys profits are taxed twice: once in the hands of the
company and once in the hands of the shareholder. The shareholder may be
subject to tax on dividends and/or capital gains arising from increases in the
share price.
Split-rate Similar to the classical system excepting that different tax rates
may be levied on retained profits and distributed profits. The system might be
used in conjunction with a system that taxes investors income and capital
gains at different rates.
Imputation A system designed to enable a companys profit to be taxed
once rather than twice. Dividends paid from taxed profits are paid to
shareholders together with a tax credit. The rules vary greatly and can be
quite complex but it is often the case that the tax credit received is sufficient to
offset the tax due on the net dividend. Also, lower taxed investors can often
reclaim the tax credit.

(ii)

(a)

All else equal under the imputation system dividends are taxed once.
Retained profits lead to increased share prices and capital gains.
Hence retained profits in an imputation system might be effectively
taxed twice. The imputation system encourages dividends. Split-rate
would be an alternative depending on the tax rates being levied.

(b)

The classical and potentially the split-rate systems are more likely to
encourage retained profits and capital gains. Companies are more
likely to grow and hence are more likely to become conglomerates.

(c)

All else equal the dividend imputation system should be most attractive
to tax exempt investors as it potentially allows them to reclaim the tax
paid by the company.

(Where different answers were possible marks were given for either solution)
Candidates scored well on this question in the main with most getting close to full marks on
(i). There were mixed results on part (ii) tending to indicate that candidates may have
learned the tax regimes but not fully understood them.

Page 8

Subject 301 (Investment and Asset Management) September 2002 Examiners Report

11

(i)

Conventional Government Bond


Gross redemption yield = required risk free real yield + expected inflation +
inflation risk premium
Property
Rental yield + expected growth in rents = required risk free real yield +
expected inflation + property risk premium

(ii)

Assumptions:
All investors want a real rate of return
All investors have the same time horizon for investment decisions
All investors have the same tax position
Reinvestment rates equal the expected total return from each asset

(iii)

The theoretical relationship is a very long term one. Actual returns over a
relatively short period often are different from those expected.
Many differences will relate to inaccurate estimation of the variables.
Short term changes in supply and demand etc. will cause markets to diverge
from fair value.
Accrual of income and expenses can be difficult. The amount and timing of
future rent reviews, major repairs etc.. is often unknown.
Property valuations are often done infrequently whereas bonds are valued in
real time.

Reasonable attempts were made at part (i) but candidates did not score as well on (ii)
missing out the time, tax and real return assumption points. Part (iii) also had mixed results.

12

(i)

[Reference 301: Unit 10, section 2, 301:4:3.1, 17:1,3.7, 19:2,3] Need to


construct a model of the company which allows future cashflows and earnings
to be estimated. This will take account of management ability, quality of
product, prospects for market growth, competition, input costs, retained
profits, company, sector and market history. From this a share value can be
derived using a discounted dividend model or sector price/earnings ratio. It is
likely that the price range will be set lower than the most optimistic forecasts
to encourage take-up by other investors

(ii)

Financial accounts and accounting ratios, dividend and earnings cover,


revenue and expenditure and so profit variability and growth, levels and
structure of borrowings, levels of liquidity and lines of credit available, growth
in asset values (which may be mainly intangible e.g. intellectual capital),
comparative figures for other companies (similar in terms of market or stage
of development), public statements made by the company and trade/press
comment, listing information and criteria for the exchange on which the stock
Page 9

Subject 301 (Investment and Asset Management) September 2002 Examiners Report

is to be launched, opinions of company management, competitors, customers,


suppliers and market analysts.
(iii)

[Reference Unit 10, section 3 and Unit 11] Economy or trade cycle may effect
short-term market sentiment and so valuation of companies generally or
specific. Liquidity in the market may be poor so investors need to be
encouraged to switch out of existing holdings to fund purchase, particularly if
there is a high level of new issuance generally. Improved investor sentiment if
issue rises on opening (good for future business of bank). Avoids take up of
stock underwritten so improves profitability of deal for the bank.

Another question where candidates failed to make use of all the information given to them, in
this case that it was a first public offering. Parts (i) and (ii) were marked together as many
candidates made the comments expected in part (i) when answering (ii) and vice-versa. Part
(iii) was not well answered.

13

The table below shows the results:

Equity
Fixed Interest
Index Linked
Total

Fund Wt Index Wt

Fund
Return

50.00% 60.00%
40.00% 20.00%
10.00% 20.00%
100.00% 100.00%

73.50%
31.40%
39.01%
53.21%

Index
Return

Asset
Alloc

Sector

Total

59.19% -0.89% 7.16% 6.26%


34.71% -3.11% -1.33% -4.43%
38.96%
1.13% 0.01% 1.13%
50.25% -2.87% 5.83% 2.96%

(i)

The fund has outperformed its benchmark by 3%, having returned 53% against
the benchmarks 50%.

(ii)

Asset allocation gave a negative contribution of 3% whilst stock selection


made a positive contribution of 6%. Over weighting fixed interest was a poor
decision which was further compounded by poor stock selection in this sector.
Equity asset allocation was negative but this was more than made up for by
excellent stock selection. Under weighting index-linked also produced a
positive contribution.

It was disappointing not to see logical application of theory in this question and many
candidates obviously dived straight in to answering it without taking in all that was required.
Many missed the point that there was no rebalancing. Arithmetical errors were not penalised
but use of wrong formulae resulted in the loss of about one third of the marks being awarded.

Page 10

Faculty of Actuaries

Institute of Actuaries

EXAMINATIONS
8 April 2003 (am)
Subject 301 Investment and Asset Management

Time allowed: Three hours


INSTRUCTIONS TO THE CANDIDATE
1.

Enter all the candidate and examination details as requested on the front of your answer
booklet.

2.

You have 15 minutes at the start of the examination in which to read the questions.
You are strongly encouraged to use this time for reading only, but notes may be made.
You then have three hours to complete the paper.

3.

You must not start writing your answers in the booklet until instructed to do so by the
supervisor.

4.

Mark allocations are shown in brackets.

5.

Attempt all 12 questions, beginning your answer to each question on a separate sheet.
AT THE END OF THE EXAMINATION

Hand in BOTH your answer booklet, with any additional sheets firmly attached, and this
question paper.
In addition to this paper you should have available Actuarial Tables and
your own electronic calculator.

301A2003

Faculty of Actuaries
Institute of Actuaries

Outline the key issues to be considered when forming a long-term investment strategy
for a charitable trust.
[5]

(i)

Define a generalised formula for a total return index suitable for property
performance measurement purposes, stating any assumptions you would need
to make.
[2]

(ii)

Outline the problems in constructing such an index for quarterly publication.


[4]
[Total 6]

Over the last five years, a general industrial company has restructured itself,
principally through acquisition into an information technology and communication
services company. The company has borrowed heavily to support its activities. Due to
a recession in the market, the company is struggling to cover its debt repayments and
also needs further capital to complete research into a new component that the
management believes will generate 60% of revenues over the next 10 years. The
company has recently appointed a debt management consultant who has approached
the bank with a proposal to swap its loans to the company for equity.
(i)

Set out the principal characteristics of this company.

(ii)

Outline the factors that will affect the banks preference for debt or equity
investment in the company stating any further information you would require
in order that a decision may be made.
[8]
[Total 12]

(i)

Define rack rent and marriage value.

(ii)

List reasons why the rack rent might be higher than the rent currently being
received.
[2]

(iii)

Describe a situation where a tenant may have no option but to pay a rent
higher than the rack rent.
[2]
[Total 6]

301 A20032

[4]

[2]

List the problems that may be encountered with overseas investments.

(i)

State reasons why an individual might be attracted to collective investment


vehicles or policies, rather than direct holdings.
[3]

(ii)

List the key differences between an investment trust and a unit trust.

(iii)

Explain why an investment trust would typically have a more volatile share
price than the offer price of a unit trust.
[4]
[Total 11]

[5]

[4]

An investment bank is pricing a 15 year swap using its internal cost of capital of 8%
per annum. In return for a swap premium, the bank will pay its client the excess
interest payable on a variable rate loan of LIBOR + 100bps above a fixed rate loan of
5.5% (the client will pay the bank in the event of the fixed rate loan giving rise to
higher payments than the variable rate loan). These payments are made at the end of
each quarter. The capital payment under the clients loan is not covered by the swap
arrangement.
(i)

Assume LIBOR is initially 4%, and remains at this level for the first 7 years of
the term, and is 6% thereafter. Calculate the present value of the cashflows
covered by the swap based on a principal sum of 10 million.
[6]

(ii)

Explain why in practice the bank will charge more than the present value
calculated in (i) above.
[5]
[Total 11]

(i)

Describe a classical split rate system of corporation tax.

[2]

A government of a developing country has approached you as an investment


consultant to advise them on restructuring the taxation system to encourage longer
term investment in companies listed on their local stock exchange.
(ii)

301 A20033

Describe methods of taxation that would encourage longer term investment.


[4]
[Total 6]

PLEASE TURN OVER

Lifeco is a major insurance company operating mainly in its domestic market. In


order to raise additional finance, it has decided to issue a new convertible bond. The
details of the convertible bond are:

Coupon 2.75% payable annually.

Term 5 years.

Conversion price 125% of the share price at issue at any time before redemption
provided the share price has increased by 30% at any time since issue.

The company may redeem the convertible after 4 years, and thereafter at the
option of the company, at par, providing the price exceeds 120% of the conversion
price.

(i)
(ii)

Explain why Lifeco might choose to raise additional finance by issuing a


convertible bond.

[3]

Explain why Lifeco might have set the redemption terms it has.

[2]

The share price at issue is 1.00 and the current prospective dividend yield is 3%.
The dividend of the company over the past year grew by 15%. Having analysed the
prospects for Lifeco, you estimate that the dividend growth rate experienced in each
subsequent year will fall by 2% p.a. in each of the next 6 years, before ultimately
stabilising at 3% p.a.

10

(iii)

Estimate the value of the convertible bond to an investor who requires a return
from equities of 6.09% p.a. Ignore taxation. State your assumptions.
[9]

(iv)

After 4 years the market price has just reached 1.25. The company made an
open offer to repurchase the convertible bond at 101.6 per 100 nominal.
Suggest possible reasons why the company has made this offer.
[4]
[Total 18]

An investment bank is establishing a series of bond indices to be used in combinations


as benchmarks for bond portfolios which it manages.
List the features of bonds that should be taken into account in establishing these
indices.
[4]

11

A developing country is reviewing the extent and form of regulation of its investment
markets because of complaints about the cost of regulation.
Explain the economic costs of regulation.

301 A20034

[5]

12

An insurance company operates a unitised with-profit fund. Valuations are carried out
at each month end. These valuations are available 3 weeks after each month end and
form the basis for the market value adjustment factors used for adjusting surrender
values.
The appointed actuary is concerned that because of the market volatility there has
been adverse selection against the with-profits fund. You have been asked to submit a
proposal to update valuations and investment performance on a more frequent basis.
In particular you have been asked to be able to provide an update immediately after an
adverse market movement.
The unitised with-profits fund has been divided into separately managed funds for
each asset class as follows:
Asset class
Fixed interest medium term corporate bonds
Fixed interest medium term gilts
UK equities
Non-UK equities
Property
Cash
Commercial Mortgages

% of the fund
18%
4%
45%
15%
15%
2%
1%

Outline the investigations you would undertake and set out the points you would make
in your reply to the appointed actuary.
[11]

301 A20035

Faculty of Actuaries

Institute of Actuaries

EXAMINATIONS
April 2003
Subject 301 Investment and Asset Management
EXAMINERS REPORT

Introduction
The attached subject report has been written by the Principal Examiner with the aim of
helping candidates. The examiners are mindful that a number of interpretations may
be drawn from the syllabus and Core Reading. The questions and comments are based
around Core Reading as the interpretation of the syllabus to which the examiners are
working. They have however given credit for any alternative approach or interpretation
which they consider to be reasonable.
The report does not attempt to offer a specimen solution for each question that is, a
solution that a well prepared candidate might have produced in the time allowed. For
most questions substantially more detail is given than would normally be necessary to
obtain a clear pass. There can also be valid alternatives which would gain equal marks.
Mrs J Curtis
Chairman of the Board of Examiners
17 June 2003

Faculty of Actuaries
Institute of Actuaries

Subject 301 (Investment and Asset Management) April 2003 Examiners Report

Legal constitution of charity and objectives and constraints


Duties, authorities and responsibilities of trustees and other parties in process
Experience and resources available to trustees to decide, implement and monitor
External advice available
Differences between strategy and tactics
Objectives and their prioritisation look at purpose of charity, operating
considerations, need for income and capital, existing commitments and plans
Size and appraisal of current portfolio
Broad theoretical alternative options to consider including expected returns and risks
for equities, bonds, cash/other and property (may have operational use as well as
investment aim)
Secondary issues e.g. domestic/overseas split, nature of bonds, choice of benchmark,
tactical limits, timing
Risks affecting fund and stakeholders and their priority/relevance short and long
term income requirements, fund raising, inflation, capital protection, statutory returns,
general market volatility, currency, peer group comparisons, ethical or SRI
considerations
Impact of tax exempt status on expected returns and risks
Attitude to risk of trustees and contributors
Implementation factors nature of existing and proposed investments, timing, liquidity,
income requirements, dealing frequencies, costs of reconstruction and ongoing
management
Existing and prospective compliance requirements (could Myners and trustee
legislation be extended to charities)

Overall this was done reasonably well by candidates although few scored high marks.
Most got the obvious points re the fund objectives, restrictions and attitude to risk but few
went further and developed their answers into the more detailed points which were needed to
score better.

(i)
(ii)

Page 2

(i)

General formula and assumptions as per unit 12, section 1 plus allowance for
factors referred to in 12.4 for portfolio based indices including costs

(ii)

Issues as per 12.4:


Uniqueness of property lack of homogeneity and available comparators,
definition of sector , investability restricted with large lot sizes indivisible,
rules required for changes to index constituents, impact of obsolescence and
redevelopment or other change of characteristics (e.g. tenancy) on constituents
Cashflow pattern for deciding typical yield
Subjectivity, timing and variation of valuation methods and cost of frequent
appraisal
Actual return only known at point of sale with price kept confidential
Infrequency of sales in some sectors

Most candidates scored well.


The standard points re the difficulties in valuation (subjectivity, frequency, cost, etc.)
were made by most candidates. Few however, went on to explain sufficiently how
these and other points created problems when constructing a quarterly index. This
was the essence of the question.

Subject 301 (Investment and Asset Management) April 2003 Examiners Report

(i)

(i)

Principal features of IT and communication (and service/ non-general


industrial companies by implication) companies as per Unit 4 (3.2.3, 3.2.5 and
3.2.8) plus general comment on current market trend.

(ii)

Consider prospects for equity and bonds generally plus specifically factors that
will affect the investment performance of all the companies in the sector and
the particular company concerned
Look at level of existing debt and equity (if any) exposure to this and sector
generally, security available, expected return, costs of restructure, business
plan, priorities on income and capital repayment, levels of cover, actions of
peers, bank policy and experience, statutory requirements, secondary
marketability
Look at existing capital structure and change proposed
Look at actual swap terms proposed
Looking for formal recommendation (yes or no) with argument and comment
on the consultants recommendation, including any further information
required. Points to cover include:
Consultant may be biased (has vested interest) and so need to also consider
other independent sources for recommendations, industry outlook,
company announcements, views of suppliers and competitors
What is negotiating strength are we lead lender?
Fundamental analyses of company covering management, product, market
growth, competitive position, and accounting data. May need company
visit
Earnings not the same as profits what is real impact of new product and
how close to completion is it
What is point in economic cycle and general outlook for sector/market
Investor demand for IT sector low and burnt fingers but volatile with
limited profitability or dividend records. Assets generally intangible.
Other companies in sector may have similar plans or products with better
prospects
What are other investors perceptions of future capital and dividend growth
and risk

(ii)

Surprisingly badly done for a largely bookwork question. Very few candidates scored
full marks here although most got at least some of the points.
Not well done by the majority of candidates. Few made the various points about
looking around on a broader basis to establish the health of the sector etc. and even
fewer made the point that revenues are not the same as profits.

(i)

Rack rent is the rent that would be received from a property if it were subject
to an immediate open market rent review.
Marriage value is the value added by combining several different interests in a
property.

(ii)

Sale and leaseback situation, so rent is below market rate


Rack rents have increased since the last review

Page 3

Subject 301 (Investment and Asset Management) April 2003 Examiners Report

Rent linked to an index (e.g. RPI) between infrequent full reviews, leading to
drift over time
Government controls on permissible rents
Strong covenant of tenant, leading to lower rent
(iii)

The tenant may have purchased a lease with an upwards only rent review
clause. Rents have since fallen, but the tenant is unable to react to changed
conditions without surrendering the lease.

Parts (i) and( ii) were done well, but many dropped some marks in part (iii).

Unrecoverable taxes (withholding tax)


Accounting information may be less reliable
Accounting standards may be different compared to the domestic market
Less information may be available than in the domestic market
.and this may be less timely
Language problems may exist/translation delays
Time delays may exist
Market regulation may be weaker in some countries than others
Political changes may result in changes in how overseas investors are treated
Marketability many overseas markets have low liquidity or concentrated
ownership (e.g. cross shareholdings)
Currency complications
Ownership restrictions may exist in certain industries (e.g. airlines, utilities)
All adds to overall costs and the level of admin expertise required

Standard bookwork question which was answered well in most cases.

(i)

Can invest small sums


Diversification possible with small sums
Specialist expertise
Simpler to arrange (e.g. salesman/bank teller)
Tax advantages in some cases
Smoothed investment return (with profits)

(ii)
Investment trust
Closed fund
Price may be above or below net asset
value of underlying assets
Company governed by company law
Price may be more volatile due to
discount varying over time
Can borrow (gearing possible)
Shares traded on a stock exchange

Page 4

Unit trust
Open fund
Always priced at net asset value
Trust governed by trust law
No discount, so volatility
follows underlying assets only
Not permitted to borrow
Units bought/sold by trust
manager

Subject 301 (Investment and Asset Management) April 2003 Examiners Report

(iii)

Discount The discount will vary over time, due to market sentiment about
the managers and their investment style.
Marketability Investment trust shares are often less liquid than the
underlying investments, whereas the unit trust manager guarantees that units
will always be marketable (with a few exceptions, e.g. property unit trusts).
Unquoted investments Investment trusts typically have higher holdings in
unquoted or unmarketable assets, for which market values are not readily
available.
Gearing Investment trusts are often geared, so they will have a higher
volatility than the value of the underlying assets.
Closed vehicle When demand is rising for a particular investment trust,
new shares cannot be created unlike in a unit trust where the manager would
issue new units. The converse also applies when a trust falls out of favour.
This creates additional volatility.

Most candidates scored well on parts (i) and (ii). In part (iii) few candidates scored well. The
basic points that gearing and changes in the level of discount would introduce volatility were
often made but most did not explain them sufficiently. Very few mentioned the three other
factors at all.

(i)

Discount rate of 8% p.a.


i(4) = 7.77% p.a.
( 4)
( 4)
PV variable interest rate loan = 10, 000, 000 5% a + 7% v 7 a
7
8

= 5,096,491
( 4)
PV fixed rate loan = 10, 000, 000 5.5% a = 4,846,680
15

Premium = 5,096,491 4,846,680 = 249,811

Annuity1
Annuity2
PV

i4

0.0777

Fixed
8.812

Variable
5.360
3.452

0.485

0.510

PV 10m 4,846,680 5,096,491


Difference 249,811

Page 5

Subject 301 (Investment and Asset Management) April 2003 Examiners Report

(ii)

The bank is taking a credit risk in the event of a client default...


but the bank will only lose out if the lost future quarterly payments are
expected to be paid from the client to the bank
This risk is much more significant if the fixed rate is lower than the initial
variable rate
which is more likely if the yield curve is downwards sloping
The bank may have entered into offsetting transactions, so it will be exposed
to interest rate fluctuations if the client defaults
The calculation does not allow for the expenses of setting up a swap (e.g. legal
expenses, sale and negotiation)
nor the cost of any additional regulatory capital required
The bank is exposed to an interest rate risk
which the client is not
This risk can be valued using a Black-Scholes approach, or a proprietary
model
The bank will have priced the swap based on the clients current credit rating,
and is locking into the rate for a 10 year period. This may result in capital
strains if the clients credit rating worsens and more capital is required to back
the swap

(i)

Many candidates scored quite well here with the majority understanding the basic
idea as to the cash flows involved. However, an alarming number made elementary
arithmetic mistakes resulting in odd answers which should have raised alarm bells. A
few demonstrated that they have little or no idea how to value a simple annuity!

(ii)

Not well done in general. The points on credit risk, interest rate risk, expenses and
margin for profit were generally the only ones made and frequently only some of these
were made.

(i)

(ii)

company taxed at one rate on retained profits and another on distributed


profits
an investor is subject to income tax on the whole of distributions and
capital gains tax arising from increases in the share price.

Each investor, individual or institutional, will attempt to maximise after tax


returns and will therefore attempt to find tax-efficient investments.
The government wants to encourage longer term investment, however, it
would prefer to avoid a taxation system that curtails an efficient market from
operating
Purchase & Sales of investments
Tax on purchase of investments
Tax on sale of investment
Both of these will discourage frequent purchase and selling of investments
Income (Dividends etc.)
Income is effectively a withdrawal of value from an investment, so to
encourage longer term investment the government could:

Page 6

Subject 301 (Investment and Asset Management) April 2003 Examiners Report

Tax income at a higher rate


Reduce tax on income reinvested in same company
Capital
Higher rate of tax on gains for investment held for a short period, say a month
Reducing rate of tax on gains or proportion of gains taxable depending on the
length the investment is held
Reduced or deferred tax on gains where capital is reinvested
Taxing capital gains only when an asset is disposed of
(i)

Most scored well here.

(ii)

It seems that many candidates overcomplicated this part. Many answers developed
ideas designed to encourage capital investment on the part of companies e.g.
encouraging retained earnings, etc. Few made the simple points re transaction taxes
and stepped CGT rates. Very few introduced any ideas designed to encourage reinvestment of gains or income.

(i)

The option premium reduces the cost of raising capital.


Convertible delays any dilution of the existing ordinary share capital.
Conversion only occurs if the share price increases sufficiently.
Interest payments on the convertible loan stock are normally made out of pre
tax profits, unlike dividends on ordinary shares. This may appeal to gross
investors.
Lifeco may be unable to issue additional shares because either a rights issue
would be extremely unpopular with shareholders or because it is constrained
by restrictions on the issue of further share capital.
Lifeco may be constrained by restrictions that prevent it from issuing further
conventional loan stock.

(ii)

By maximising the option premium it reduces the cost of raising additional


capital.
The dividend yield is higher than the coupon rate so setting a hurdle price will
delay exercise of the convertible from the first point the price exceeds the
conversion price.
The limitation on when Lifeco can redeem the convertible increases the option
value because provided the convertible exercise conditions have been satisfied
there is effectively a call option operating.

(iii)

Assume dividends are paid annually in arrears.


The optimal conversion date occurs when the price of the ordinary share first
exceeds the conversion price (1.00 1.25 = 1.25)
Dividend growth rate year 1, 13%, year 2, 11%, year 3, 9%, year 4, 7%, year
5, 5% and therafter 3%, assuming that a dividend yield of 3% is maintained.
Price at issue: 1.00
1 year
1.00 113% = 1.13
2 year
1.13 111% = 1.2543
Page 7

Subject 301 (Investment and Asset Management) April 2003 Examiners Report

2.5 years
3 years
4 years
5 years

1.2543 1.09.5 = 1.310


1.2543 1.09 = 1.367
1.367 1.07 = 1.463
1.463 1.05 = 1.536

The earliest we can convert is about 2.5 years when we anticipate that the
trigger price will be above 1.30, i.e. forego the dividends in the first 2 years
and receive the coupon instead.
100 nominal of loan stock converts to 80 shares.
3
4
+ 1.13*1.11*1.09*1.07v6.09%
2.75a2 6.09% + 80*0.03*(1.13*1.11*1.09v6.09%
5
+1.13*1.11*1.09*1.07 *1.05v6.09%
)
5
+80*(0.03*1.13*1.11*1.09*1.07 *1.05 / 0.03)v6.09%
=

5.035 + 8.2626 + 91.4363 = 104.73 plus option premium/time value


Assuming that conversion does not occur is equal to: 100 nominal of loan
stock pays annual interest of 2.75. The value as a loan stock is:
2.75a5 + 100v5 @6.09% = 85.97

Thus, the value of the convertible loan stock will be equal to 104.73%, plus an
element of time value.
(iv)

The company wants to avoid a dilution of existing share capital.


It is more capital efficient to pay the invested out of pre-tax profits rather than
dividends out of post tax profits.
The capital loss of purchasing the bond at 101.6 and cancelling it at 100
could be used to reduce the companys corporation tax.
This is akin to the company repurchasing its own shares, without the
regulatory difficulty this might entail.
The company believes its shares are undervalued

Poorly done in general. Very few candidates really grasped what was involved. This was
especially noticeable in the very few marks gained (in general) from parts i) ii) and iv). Most
of the marks which were gained were scored in section iii) where many candidates had a go
at calculating the value of the convertible. However, in many cases, these attempts were
undermined by a basic misinterpretation of the terms involved e.g. often candidates missed
the point that conversion price was 125% of the share price at issue i.e. that every 100
nominal of loan stock would become 80 shares or that the dividend growth rate dropped by
2% each year i.e. 13, 11, 9, 7, 5, etc.

10

Page 8

Coupon/yield
Credit rating
Duration
Term

Subject 301 (Investment and Asset Management) April 2003 Examiners Report

Size of issue
Liquidity
Country of origin
Exchange traded on e.g. Eurobonds
Type Fixed/Index linked/Hybrid
Quite well done on the whole although few achieved full marks.

11

All forms of regulation have a cost.


The aim of regulation should be to maximise the benefits and minimise cost so that
the benefits outweigh the costs
Direct costs arising from regulation:
Administering the regulation
Compliance for regulated firms
Other economic costs of regulation:
Altering behaviour of consumers, who may be given a false sense of security
and a reduced sense of responsibility for their own actions.
An undermining of the sense of professional responsibility amongst intermediaries
and advisors.
A reduction in consumer protection mechanisms developed by the market itself.
Reduction in product innovation
Reduction in competition

A fairly standard bookwork question which was tackled quite well by the majority.

12

The investment mix and performance information of the unitised with-profit (UWP)
fund unitised at any will be between 3 and 7 weeks out of date
The actual investment performance of the UWP fund will depend on:

The actual asset mix at any point in time, and


The actual investment performance of each separately managed fund

The actual investment mix at any time will depend on the relative investment
performance of each separately managed fund and the net inflow/outflow of money
from each fund.

Page 9

Subject 301 (Investment and Asset Management) April 2003 Examiners Report

To avoid very frequent trades and money movements most fund operate an account
for everyday cash movements, for example premiums, claims and expenses, and on a
periodical basis will transfer larger amounts between the account and the investment
funds.
The investment manager may make a tactical investment switch of money between
individual funds that could significantly alter the investment mix. A procedure is
required for the investment manager to notify where money is moved into or out of
the individual funds. This will allow the reported investment mix to be adjusted on a
more frequent basis.
The investment performance of each fund, and fund valuations are not known on a
daily basis, so more frequent data is not available. Therefore it will be necessary to
estimate the investment performance between valuations and to update the
performance when the actual investment mix and performance data becomes
available.
The investment performance needs to be monitored on a daily basis.
The required accuracy of the fund performance estimate depends on importance of the
sub-fund, and the volatility of the investment performance, i.e. there is a need to be
more accurate for UK equity that forms 45% of the UWP fund than the Cash that
forms 2% of the fund.
For each of the segregated fund the appropriate indices to be investigated and
compared with previous actual investment performance to determine appropriate
investment performance indicators are:
UK Equities
An insurance company is likely to have a broad spread of UK equity investment
covering both small and large companies.
Therefore the FT-SE All Share index investment performance should be investigated
It will be necessary to compare the actual investment held by the UK equity fund to
FT-SE All Share in order to verify that this is a suitable index, rather than using a
weighted combination of sub-indices
Non-UK Equities
This fund could hold investment in any non-UK equity. The actual investment mix
by country needs to be investigated.
Available international equity indices that could be investigated include FTSE World
Indices series and Morgan Stanley Capital International Indices series as they both
cover both developed and emerging markets.

Page 10

Subject 301 (Investment and Asset Management) April 2003 Examiners Report

Based on the actual investment mix by country it is likely that no one index will be
suitable, but that a weighted combination reflecting the actual distribution by country
or region will be required
It is important that the price indices and XD adjustments are in the domestic currency,
i.e. already adjusted currency fluctuations.
Fixed interest medium gilts
The FTSE Actuaries Government Securities UK Indices provides a series of price
indices sub-divided by term
Investigate the current holdings and amounts of those holdings and compare these
with each category of the price indices to determine appropriate weights.
Check with the investment manager whether a short term policy switch has been
undertaken in case the investment holdings being analysed are not representative.
It is unlikely that any one index will be appropriate by itself however; gilts only form
about 4% of the whole UWP fund, so there will be less impact of a simplification to
use one index that broadly represents the fund.
Property
Investigate the current holdings of the property fund to determine the level of direct
property holding to indirect property holding (property shares)
For the indirect property proportion a sub-index of FTSE actuaries share indices is
likely to be appropriate
Direct property investment tends to have a stable performance over a short period
because valuations are only carried out only periodically for each property
Investigate the recent monthly past performance and, say calculate an average
performance for use.
If the recent performance has not been reasonably stable because of the effect of
property revaluations, then a procedure for the investment manager to notify you of
significant property revaluations will be required.
Fixed Interest medium corporate bonds
A suitable benchmark for a bond portfolio can be more complicated because they are
constructed subject to specific constraints such as duration or credit rating.
Many different series of international bond indices are produced, mostly by brokers.
The exact calculation methods and input data vary and no single series is likely to be
suitable.
To estimate the fund performance between valuations it would be suitable to use the
benchmark combination of indices

Page 11

Subject 301 (Investment and Asset Management) April 2003 Examiners Report

Cash and commercial mortgages


Both these funds only constitute a very small part of the UWP fund, so we do not
have to do anything too complicated.
The fund performance is unlikely to be volatile for either cash or commercial
mortgages, so could use the average performance from say, the pervious three months
Undoubtedly the question which caused the most difficulty in the paper. The vast majority of
answers concentrated almost entirely on whether or not more frequent valuations were
necessary or even desirable. There were many suggestions as to what else would be done to
stop selection against the office e.g. closing the fund, arbitrary MVAs, etc. and a lot of
suggestions as to how the fund assets could be restructured so as to remove or reduce
volatility, e.g. switch it all into bonds or cash!
However relatively few candidates answered the question actually asked i.e. how to provide
more frequent valuations and estimates of performance especially after sudden adverse
market moves.

Page 12

Faculty of Actuaries

Institute of Actuaries

EXAMINATIONS
15 September 2003 (am)
Subject 301 Investment and Asset Management

Time allowed: Three hours


INSTRUCTIONS TO THE CANDIDATE
1.

Enter all the candidate and examination details as requested on the front of your answer
booklet.

2.

You have 15 minutes at the start of the examination in which to read the questions.
You are strongly encouraged to use this time for reading only, but notes may be made.
You then have three hours to complete the paper.

3.

You must not start writing your answers in the booklet until instructed to do so by the
supervisor.

4.

Mark allocations are shown in brackets.

5.

Attempt all 11 questions, beginning your answer to each question on a separate sheet.
AT THE END OF THE EXAMINATION

Hand in BOTH your answer booklet, with any additional sheets firmly attached, and this
question paper.
In addition to this paper you should have available Actuarial Tables and
your own electronic calculator.

301S2003

Faculty of Actuaries
Institute of Actuaries

Describe three instruments that are used in the money markets.

[6]

(i)

Define the term equity risk premium.

[1]

(ii)

Describe why a change in the equity risk premium will alter the valuation of
equities.
[3]

(iii)

Give examples of two events that would alter the equity risk premium.
[2]
[Total 6]

A major institutional investor has decided to make a direct investment in property.


List the advantages of large offices relative to other types of property.
[5]

An institutional investor has conducted an asset-liability modelling exercise, and as a


result has decided to invest a proportion of its portfolio in the Global Equity asset
category via an external fund manager.
List the decisions that need to be made by the institutional investor with regard to this
investment in Global Equity (but not about the choice of fund manager).
[8]

You are an analyst at a firm of global equity investment managers. Your firms
approach to stock picking is based on fundamental share analysis and your day-to-day
work involves carrying out this analysis for a particular sector of the global market.
(i)

List the sources of information you would expect to use in your work.

[5]

(ii)

Describe the process you would expect to go through in order to establish


whether a stock should or should not be part of your clients portfolios.
[5]

One of your stock recommendations held across all your clients portfolios has seen a
substantial fall in its market price over a calendar quarter. Your analysis suggests that
the price now represents extremely good value. Based on this view, your
recommendation to colleagues would be to substantially increase the allocation to the
stock.
(iii)

Suggest possible reasons for the difference between your and the markets
view of the fair price for the stock and outline the risk faced by your firm if
your recommendation is made and adopted.
[3]

You have added to your holding and the price continues to fall.
(iv)

301 S20032

Describe briefly what actions the firm may take when reassessing the position.
[3]
[Total 16]

List the ways in which costs can arise when an investment regulatory system is
developed.

[6]

(i)

Outline the process used by futures exchanges to remove the credit risk of
individual participants.
[2]

(ii)

Table 1 below shows part of the operation of a margin account for a short
position in two gold futures contracts. The initial margin is US$2,000 per
contract and each contract is for delivery of 100 ounces of gold. The closing
futures prices are in US$ per ounce of gold.
Copy the table into your answer book and fill in the entries in the blank
columns.
Table 1
Operation of the margin account for two gold futures contracts.
Date

Futures price at
which contract
is entered into
on: May 3rd
May 3rd
May 4th
May 5th
May 6th
May 7th
May 8th

Closing
Futures
Price
US$

Daily Gain
(Loss)

Cumulative
Gain (Loss)

US$

US$

Margin
Account
Balance
US$

400
396.5
399.4
400.4
399.7
405.9
397.9
[6]

(iii)

Describe briefly two major differences between the trading of currency futures
contracts on an exchange and the trading of forward currency contracts in the
Over-the-Counter (OTC) market.
[2]
[Total 10]

(i)

Write down a formula for the after tax return for an investment whose
dividend is d and capital gain is g.

[1]

(ii)

State any assumptions that you have made in this formula.

[1]

(iii)

Describe what other factors an individual will need to take into account in
practice to determine his after tax return.
[3]
[Total 5]

301 S20033

PLEASE TURN OVER

An open-ended investment company (OEIC) invests the capital of its shareholders in a


portfolio of not less than twenty bonds issued by US companies that have a MMM,
non-investment grade, credit rating. All bonds are denominated in US Dollars. On
average, five percent (5%) of bonds with a credit rating of MMM default over a
12-month period. The share price of the OEIC is always equal to its net asset value
per share.
Over the last five years none of the bonds held by the OEIC have defaulted and the
share price of the OEIC has increased by 14% per annum compound with a standard
deviation of return of 5% per annum. Over the same period the risk-free rate of
interest has been 4% per annum.
(i)

State the formula for and calculate the Sharpe ratio for the risk-adjusted
performance of the OEIC over the last five years.

[2]

Over the same period, an index of US government bonds and an index of MMM-rated
US companies gave total returns (capital growth and income) of 8% and 2% per
annum respectively. Both of these indices are made up of about twenty bonds which
have a similar coupon and maturity profile to the OEICs portfolio.
(ii)

Suggest reasons why the investment companys portfolio has out performed:
(a)
(b)

the US government bond index and


the index of MMM-rated US companies.
[4]

(iii)

10

Discuss the usefulness of the Sharpe ratio in assessing the risk-adjusted


investment performance of this manager and comment on the skills of the
manager of the investment companys portfolio.
[5]
[Total 11]

A city in a developing country is contemplating bidding to host the Olympic Games in


2016. History shows that the costs involved in hosting such events have been much
greater than expected. Your merchant bank has been asked to advise the committee
overseeing the bid.
(i)

Outline the procedure you would adopt in establishing the feasibility of the bid
by the city.
[8]

(ii)

List the risk factors you would include in your risk matrix.

(iii)

Describe briefly the ways in which the bid could be financed and the factors
that need to be taken into account.
[5]
[Total 17]

301 S20034

[4]

11

Following the death of a relative, you have inherited a sum of money worth
approximately five times your net annual salary. Your personal circumstances are as
follows:

You are working and belong to an occupational pension scheme.


You have ten years to work before normal retirement.
You are married with no children.
You are a home owner with an accompanying mortgage.
Your mortgage has five years to run and currently monthly mortgage payments
amount to around one-third of your net monthly income.
You have cash savings amounting to approximately one years net salary.
You use credit and store cards to manage cash flow on a monthly basis.
You are in good health but have for some years taken part in a private healthcare
scheme.

(i)

Using the details above and stating any additional necessary assumptions,
describe your liabilities and your assets excluding your inheritance.
[4]

(ii)

Comment on the degree of matching between the two.

(iii)

Suggest possible uses for your inheritance commenting briefly on the


suitability of the various options.
[4]
[Total 10]

301 S20035

[2]

Faculty of Actuaries

Institute of Actuaries

EXAMINATIONS
September 2003

Subject 301 Investment and Asset Management

EXAMINERS REPORT

Introduction
The attached subject report has been written by the Principal Examiner with the aim of
helping candidates. The examiners are mindful that a number of interpretations may
be drawn from the syllabus and Core Reading. The questions and comments are based
around Core Reading as the interpretation of the syllabus to which the examiners are
working. They have however given credit for any alternative approach or interpretation
which they consider to be reasonable.
The report does not attempt to offer a specimen solution for each question that is, a
solution that a well prepared candidate might have produced in the time allowed. For
most questions substantially more detail is given than would normally be necessary to
obtain a clear pass. There can also be valid alternatives which would gain equal marks.
J Curtis
Chairman of the Board of Examiners
25 November 2003

Faculty of Actuaries
Institute of Actuaries

Faculty of Actuaries

Institute of Actuaries

EXAMINATIONS
September 2003
Subject 301 Investment and Asset Management
EXAMINERS REPORT

Faculty of Actuaries
Institute of Actuaries

Subject 301 (Investment and Asset Management) September 2003 Examiners Report

Candidates appeared better prepared than has been the case previously and as a
consequence we had a higher pass rate. Bookwork was done reasonably well but poorer
candidates are still unable to apply the theory to problems and even fewer appear able to
draw implications and conclusions out of results. We continue to be concerned that
candidates do not answer the question set. A good example of this was Q3 where candidates
were asked for advantages but many gave us a mind dump of all they knew about property
investment. Whilst they invariably pick up a number of points, the amount that candidates
write and the time taken to answer such a question could be put to better use.

Any three of the instruments given below are acceptable.


Deposits The simplest of all money market instruments. In the deposits market
banks simply take and lay off deposits from each other.
The market is liquid over a wide range of maturities, although primarily less than one
year.
Treasury Bills A treasury bill is a promise to repay a set sum of money by the
treasury at a specified date in the future, normally not longer than 91 days.
T bills are issued by way of auction and trade a discount to their face value.
They are not part of the Governments funding programme per se but are much more
an instrument of monetary policy.
Bills of Exchange A bill of exchange is an instrument which is drawn and issued
by the seller of goods to the buyer, specifying the amount to be paid, either
immediately or at some point in the future.
Once the bill has been accepted by the buyer the bearer of the bill is entitled to the
proceeds at maturity.
The accepted bill may then be discounted provided the purchaser is willing to take on
the credit risk of the bills acceptor, which is the promise of repayment.
Commercial Paper is a promissory note issued by a company at a discount to the
face value with maturities normally up to twelve months.
It may be issued via the auspices of a bank guaranteed programme or directly into the
market.
Certificates of Deposit it is, in effect, a securitised bank deposit.
They carry a fixed coupon rate and have a maturity of up to five years.
Like any other security, the certificate can be traded enabling the deposit holder to
realise the deposit through the sales proceeds and not by withdrawal.

Page 3

Subject 301 (Investment and Asset Management) September 2003 Examiners Report

Short Term Gov Bonds With their status as a relatively risk free investment the
money markets also trade in short term (usually less than five years) Government
bonds.
The Government is the issuer and the bonds usually pay a twice yearly coupon, they
are highly liquid.
Floating rate notes which are linked to the short-term money markets and are
widely traded.
They are issued highly rated corporates, usually banks since they carry very limited
credit risk.
This was bookwork and candidates scored reasonably well.

(i)

The equity risk premium is the extra return that the overall stock market or a
particular stock must provide over the rate of a risk free asset, normally
treasury bills, to compensate for the additional risk being taken.

(ii)

The equity risk premium is used in the calculation of the weighted average
cost of capital (WACC) for a stock.
The WACC is given by ke*E/V + kd*D/V
The cost of equity is given as ke = kf + *(km kf)
Therefore when the equity risk premium, km kf , rises so does the WACC.
The WACC is used in discounted cash flow valuations, the higher the WACC
the lower the valuation, therefore increasing the equity risk premium increases
the WACC and thus decreases valuations. The converse is true for a fall in the
equity risk premium.

(iii)

Any two reasonable events that would change the risks associated with
holding equities in general or a particular sector are acceptable including
terrorism, major accounting scandals, oil crises, war etc.

In hindsight this question might have been better phrased to point candidates to the sort of
answer shown in the solution. However marks were given for more general answers. It was,
however, a concern that candidates generally went for low-key events rather than shocks in
answering (iii).

Page 4

They can be purchased in large units.


Rental income tends to be secure as rents normally represent a small part of the
tenants outgoings.

Subject 301 (Investment and Asset Management) September 2003 Examiners Report

There is a wide range of prospective tenants, with no concentration on any particular


industry.
Many large offices are multi-let. This spreads the risk of tenant default
and helps provide comparable evidence in the rent review process.
For offices in large towns there are usually comparable properties for the purpose of
reviewing rents.
A typical 25 year institutional lease has 5 yearly upward only rent reviews.
When let to a single tenant on a full repairing and insuring lease, management costs
are relatively low (must have both first and second parts of sentence to score marks
no marks for only part of the answer)
There is a wide range of different properties available.
Precise location not critical
so long as easily accessible by car and public transport.
Bookwork, but see comment in introduction.

Choice of benchmark.
Need to decide whether benchmark should be a market cap index benchmark
such as FTSE All World or MSCI World
or a regional composite benchmark
which takes for example equal weightings in the regional markets of North
America, Europe, and the Far East
The proportion of domestic equity in the Global mandate must be decided
This could be 0% if a separate domestic equity mandate is to be managed
or set at a larger percentage of the total mandate if a higher proportion of domestic
equity than implied by the choice of benchmark is required
Need to decide whether to include Emerging Market Equity within the Global Equity
mandate
because often a Global Equity mandate is taken to mean Developed World only
Purpose of allocation (match liabilities or investing excess over liabs.)
And the extent to which the fund manager will be permitted to diverge from the
benchmark in terms of region, or country, or sector, or stock.
Need to consider style (value, growth, large/small cap, passive/active etc.)
Regional/sector approach

This was a more poorly answered question. Some candidates failed to accept that the
decision had been made and the question was about issues of pre-implementation.

(i)

Company accounts
Company trading statement
Visits to company
Management meetings
Financial press
Trade papers
Competitor intelligence

Page 5

Subject 301 (Investment and Asset Management) September 2003 Examiners Report

Stock broker papers


Stock exchange information
Government sources of statutory information
(ii)

Your role will involve stock analysis within your sector. Additionally, you
will be involved in assessing the prospects for your sector within the overall
economy.
You will attempt to establish whether a stock is, according to your analysis,
under or over valued by the market.
Part of the assessment will involve construction of a model to help estimate
future cashflows in earnings. Your success will depend upon the quality of
your model and the quality of the data that goes into the model.
The modelling process will deliver data on a variety of financial features of
the company and its market place. This will help identify the key drivers to
profitability.
Further analysis can then be targeted at the most important areas.
Cashflow analysis can be used with economic projections to assess the
robustness of the profit stream.
The output will be an indication of the fundamental value of a share given
your assumptions and input data.

(iii)

Potential for Differences


Model No model can expect to be perfect. A subtle but important feature
might be missing or not working properly.
Input Data The model can be only as good as the information on which it is
based. Again, something might be missing or incorrect.
Assumptions Even the perfect model would give a share value different to
that of the market if assumptions about future inflation, required rate of return
or perceived level of risk are different to market expectation.
The risk for your company of continuing to increase exposure to the
underperforming stock is essentially the risk of you being wrong and the
issues associated with the underperformance that would arise.

(iv)

Ask another analyst in the firm to review the stock and make a
recommendation.
Cut your losses, sell the position and put it down to experience.
Seek a meeting with the companys management to discuss the share price
weakness and what plans they may have to address it.

Page 6

Subject 301 (Investment and Asset Management) September 2003 Examiners Report

Examine if any derivative options are available to add value.


A number of candidates were unable to set out the day-to-day work of an investment analyst.
Whilst (i) was bookwork and reasonably done, (ii), (iii) and especially (iv) were poorly
answered overall. One or two candidates did, however, give very full answers.

Direct costs arise in administering the regulation and in compliance for the regulated
firms.
Other economic costs can arise:

an alteration in the behaviour of consumers, who may be given a false sense of


security and a reduced sense of responsibility for their own actions
an undermining of the sense of professional responsibility amongst intermediaries
and advisors
a reduction in consumer protection mechanisms developed by the market itself
reduced product innovation
reduced competition
education
development of additional administration systems
cost of government regulation

Thankfully most candidates appear to have studied the regulation units well and gave
reasonable answers to this question.

(i)

(ii)

When two traders deal, a contract is created.


The floor traders acting for the parties involved fill out clearing slips.
The clearing slips are matched by the exchange.
Details of the trade are registered with the exchanges clearing house.
In turn the clearing house guarantees each side of the original bargain.

Award 1 mark for getting the correct amount of initial margin $4,000.
Table 1
Date

Futures price at
which contract
is entered into
on: May 3rd
May 3rd

Closing
Futures
Price
US$

400
396.5

Daily Gain
(Loss)

Cumulative
Gain (Loss)

US$

US$

700

700

Margin
Account
Balance
US$

4000
4700
Page 7

Subject 301 (Investment and Asset Management) September 2003 Examiners Report

May 4th
May 5th
May 6th
May 7th
May 8th

(iii)

399.4
400.4
399.7
405.9
397.9

580
200
140
1240
1600

120
80
60
1180
420

4120
3920
4060
2820
4420

In the forward foreign exchange markets, contracts can be tailored by the


investment bank to suit the needs of the client in terms of maturity date,
contract size, choice of currencies, etc.
In the futures markets currency futures contracts are standardised as to
maturity date, contract size, currency pairs, etc.
In the OTC market participants need to pay much closer attention to credit risk
than participants in the currency futures market where at least some1
participants have the guarantee of the exchange clearing house.

(i) The use of the word process in this question appears to have caused some confusion to
candidates. Many wrote about margin etc. rather than the mechanics of the market.
(ii) Was done reasonably although common mistakes were to use only one contract and one
ounce of gold. Also some subtracted when they should have added. Such mistakes were only
penalised once and so only a couple of marks were lost on this section.
(iii) Was reasonably answered although a number of candidates did get the differences back
to front.

Only general clearing members of the exchange get the guarantee of the exchange clearing house. All other
participants are dependent on their futures broker for performance of the contract.

Page 8

Subject 301 (Investment and Asset Management) September 2003 Examiners Report

(i)

r = d*(1 ti) + g*(1 tc)

(ii)

d and g are assumed to be dividend yield and capital return as a percentage of


initial investment respectively
d is assumed not to be re-invested if payments have occurred during the period

(iii)

The overall tax system e.g. tax rates and exemptions.


Particular rules for individual types of asset
The investors own status (individual or particular type of institution).
The investors financial position.
The tax efficiency of the vehicle used to hold the assets.
To what extent losses or gains can be aggregated between different
investments or over different time periods for tax purposes.
Whether the tax is deducted at source or has to be paid subsequently.
The extent to which tax deducted at source can be reclaimed by the
investor

(i) and (ii) were straightforward with reasonably good answers being given. (iii) was poor
with answers not focusing on specifics.

(i)

The formula is S = (Rp r)/p.


The Sharpe ratio is (14 4)/5 = 2.

(ii)

(iii)

(a)

Except in times of stress in credit markets, one would expect the return
on a portfolio of corporate bonds of this credit rating to give higher
returns than a similar portfolio of government bonds for credit risk and
liquidity reasons alone.

(b)

In relation to the MMM-rated corporate bond index, the manager


seems to have been particularly good at avoiding MMM-rated
companies that defaulted and at picking up the higher yield on those
that did not default.

None of the bonds held by the company has ever defaulted. If one of the
bonds had defaulted and if it had no recovery value, then the fund could lose
up to 5% of its value.
As the company has not experienced any defaults the standard deviation is not
really a good measure of the risk of the companys risk.
The Sharpe ratio is likely to fall substantially if the investment company were
to experience a default on one of the bonds in its portfolio.

Page 9

Subject 301 (Investment and Asset Management) September 2003 Examiners Report

On average, the manager should have had at least five (at least 1 per year of
exposure) defaulting bonds in his portfolio over the five-year period.
Either the manager has been very lucky or he is very skilful at choosing
MMM-rated corporate bonds that dont default.

While most candidates could write down the formulae, (ii) and (iii) were good examples of
candidates being unable to explain results and comment upon them. Where candidates found
themselves under time pressure, we suspect that this was the question left to the end.

10

(i)

Order to determine the feasibility of the Olympic bid the process should be
divided up into a number of steps:
Step 1
Make a high-level preliminary risk analysis to confirm that the bid does not
obviously have such a high risk profile that it is not worth analysing further.

A clear risk is that the finance cannot be raised


The government may decide that the capital required is too great to justify
politically.
It would be important to determine where the finance was likely to come from
and who would be managing the process.
Who would pay for the initial bidding costs in the event that the bid were
unsuccessful?

Step 2
Hold a brainstorming session of project experts and senior internal and
external people who are used to thinking strategically about the long-term.
The aim will be to identify project risks, both likely and unlikely,
to discuss these risks
and their interdependency,
to attempt to place a broad initial evaluation on each risk,
both for frequency of occurrence
and probable consequences if it does occur,
and to generate initial mitigation options and discuss them briefly.
Step 3
Carry out a desktop analysis to supplement the results from the brainstorming
session,
by identifying further risks and mitigation options,
using a general risk matrix,
researching previous Olympic bids and the problems that were encountered.

Page 10

Subject 301 (Investment and Asset Management) September 2003 Examiners Report

and obtaining the considered opinions of experts who are familiar with the
details of the project and the outline plans for financing it.
Step 4
Carefully set out all the identified risks in a risk register,
with cross references to other risks where there is interdependency.
Step 5
Ensure that upside risks as well as downside risks are covered.
A risk matrix could be used for the above purposes
with column headings relating to the cause of risk
and the rows relating to the risks in successive stages of the project

(ii)

Items that should be included in the risk matrix:


1 Mark for each risk, maximum 8 marks.
Is climate suitable for the games?
Does the City have a pollution problem?
Can the Citys infrastructure cope with the massive inflows?
Is the Government keen to see the City win?
Can security issues be addressed?
The attitude of the international community towards the country.
Will the time zone of the country mean that all the major TV networks are less
interested?
Are the local population likely to be interested?
Will the facilities be used after the games?
Natural disasters (earthquake, volcanic eruption etc.)

(iii)

To answer this part one needs to consider by whom and how the project might
be financed. In terms of who might finance it, the following might do so:

government
local authority/city council
sports bodies
private finance/venture capital
public company
a combination of the above

How it can be financed is by:

tax revenue/borrowing by government


local tax/borrowing by council/city
loans (secured or unsecured) or mortgages

Page 11

Subject 301 (Investment and Asset Management) September 2003 Examiners Report

debentures
equity capital
combinations of the above

One needs to look at the income sources, the outgoings and the end usage to
which facilities might be put. Not just the stadiums but accommodation (the
athletes village) and new transportation links need to be considered.
Different costs might be financed in different ways. i.e. the infrastructure
could be built, used and then transferred to a sports body or company with
them putting up the finance (PPI).
Government might build transport links because it suited their long-term
development plans.
A university might like the accommodation and other facilities after the event
and so be interested in being a partner in the project.
This is a routine bookwork questions and candidates scored well on it. However (iii) was not
as well answered as we would have hoped with few candidates being able to articulate more
than a few ways.

11

(i)

Assets
The house is part owned and may well represent a valuable real asset. Value
will be heavily dependent on desirability of property and also local/national
housing market.
Pension provision is partially funded. This may be a defined benefit or
defined amount of money invested on your behalf.
The cash savings represent a further asset with future interest a likelihood.
Future income is an asset. This could be assumed to be a non-decreasing
stream up until retirement at which point it will probably drop.
The insured healthcare is an asset which will demonstrate piece of mind
value at all times and actual value in the event of qualifying ill health.
Liabilities
The outstanding mortgage debt repayment is a liability for the next five years.
Payments may be at a fixed rate or variable. Depending on the contract, final
payment may clear the debt or trigger a demand for further loan repayment
(perhaps to be settled with cash).
All future living expenses (including pension contributions if required) are a
liability and will be real in different ways.

Page 12

Subject 301 (Investment and Asset Management) September 2003 Examiners Report

The healthcare premium is a liability although the policy would be likely to be


renewable with the option to cease payment.
Store card debt could create additional liability if delayed payment led to
interest being incurred.
Tax (income, inheritance) should be factored into the liabilities as appropriate
and any tax relief added on to the assets.
(ii)

Real income stream can be considered to be some sort of match for real
expenditure stream.
Close matching is not possible. Income will move with pay policy of
employer or pension terms in retirement whereas variety of inflationary
features impact on expenditure (price inflation, interest rate movements,
insurance premium inflation).
The mortgage repayments may well deliver final loan settlement. If further
repayment were required, other assets held (potentially the cash or cash
generated by moving house) would need to be used for payment.

(iii)

The investor would have to consider their attitude to risk. A low risk investor
might seek to match out liabilities as far as possible (e.g. paying off mortgage
as soon as practical and then investing for retirement income).
A more aggressive investor might weigh up the opportunity cost of any money
used now to deflect liabilities and choose to invest instead if the expected
return was attractive (e.g. investing in new assets if expected return exceeded
mortgage rate).
Regardless of the extent of the liabilities paid off, gearing investment to likely
retirement date (normal or early) would probably be desirable in order to
reduce impact of drop in income. A phased move from higher risk/higher
expected return assets (e.g. equities) to low risk, lower expected return assets
(e.g. bonds) over the period to retirement might be suitable.
Other factors to consider include:

Tax position (the need to potentially pay inheritance tax/income tax)


The desire to invest in tax efficient investment products
Tax rate may drop by virtue of reduced earnings on retirement
The amount of money available to invest (may preclude direct investment)
The costs associated with investing

This question was well answered and unlike in other questions, solutions were well set out
and covered most of the points outlined in the solution. There were additional suggestions to
those set out in (ii), some of a more flippant nature, such as blowing it on a world cruise,
which received marks provided they were suitably commented upon.
Page 13

Faculty of Actuaries

Institute of Actuaries

EXAMINATIONS
20 April 2004 (am)
Subject 301

Investment and Asset Management

Time allowed: Three hours


INSTRUCTIONS TO THE CANDIDATE
1.

Enter all the candidate and examination details as requested on the front of your answer
booklet.

2.

You have 15 minutes at the start of the examination in which to read the questions.
You are strongly encouraged to use this time for reading only, but notes may be made.
You then have three hours to complete the paper.

3.

You must not start writing your answers in the booklet until instructed to do so by the
supervisor.

4.

Mark allocations are shown in brackets.

5.

Attempt all 9 questions, beginning your answer to each question on a separate sheet.

AT THE END OF THE EXAMINATION


Hand in BOTH your answer booklet, with any additional sheets firmly attached, and this
question paper.
In addition to this paper you should have available Actuarial Tables and
your own electronic calculator.

301

A2004

Faculty of Actuaries
Institute of Actuaries

You are an institutional asset manager domiciled in a developed country.


(i)

State with reasons the risk factors you would consider before investing in a
Government Bond issued by your country.
[6]

(ii)

State with reasons what additional risk factors you would consider when
investing in emerging market debt.
[6]
[Total 12]

A portfolio comprises the following assets:


European equities to the value of 750m.
European bonds to the value of 650m.
Cash (including margin account) to the value of 150m.
350 short contracts of Dec 04 UK FTSE100 Index Future currently priced at 4225.
150 long contracts of Dec 04 UK Long Gilt Future currently priced at 106.11.
Calculate the effective fixed income, equity and cash proportions of the fund.

[4]

You work in the investment team of a life insurer which holds a large portfolio of
investments. As part of a diversified property portfolio, you own an industrial unit
with office space. The property is occupied by a single corporate tenant.
As part of the rent review process, the tenant has asked for a rent reduction suggesting
80% of the current rent should be paid.

(i)

Describe the key features of industrial and office properties.

[4]

(ii)

Discuss four different options you have, commenting on the financial viability
for each of the options.
[8]
[Total 12]

(i)

Describe how the principle of the actuarial control cycle applies to the
investment management of a fund.

[3]

(ii)

Describe the different levels of monitoring you would put in place in order to
ensure the arrangements remained appropriate.
[3]
[Total 6]

(i)

Describe what is meant by an Over the Counter (OTC) option.

(ii)

Describe the different ways in which a futures exchange could manage its
credit exposure.
[8]
[Total 10]

301 A2004

[2]

(i)

Describe briefly the method by which weighted arithmetic capital indices are
constructed.
[2]

(ii)

Comment on why the current practice of index construction has moved to


reflect the level of free float of shares available for purchase.
[2]

(iii)

Outline two other methods of constructing indices.

[4]
[Total 8]

Outline the issues you would need to consider in developing an investment strategy
for each of the following investors:
(a)

Contributions to a personal pension plan fund.

(b)

The reserves of a general insurance company.

(c)

The reserves of a large multi-national pharmaceutical company that has just


raised a large sum through a rights issue for future product research and
development.

(d)

The $70 million investment portfolio of a wealthy family.

(e)

A charity.
[15]

A researcher has compiled a data set showing the annual returns (inclusive of
dividends) of the stock market in a developed country for each calendar year starting
in 1903 and ending in 2002 (both the calendar years 1903 and 2002 are included in
the data set). The range of returns for the overlapping 20-year periods in the data set
runs from 4.5% to 15.6%. It has been suggested that you can use this data to forecast
future 20-year returns.
(i)

Comment on the difference between the numerous overlapping and five nonoverlapping periods as forecasts of future returns from the equity market for
20-year periods.
[1]

(ii)

Comment on using the non-overlapping 20-year periods to test the statement


Equities always give a positive return in the long term where 20 years is
regarded as long term .
[3]

(iii)

Outline the main factors that influence equity returns.

(iv)

The researcher has discovered that this market has delivered higher returns
with lower volatility than all of the other major developed markets in the
world over the period from the start of 1903 to the end of 2002. Discuss the
implications of this finding of the researcher for the statement in part (ii) again
assuming that 20 years is regarded as the long term .
[8]
[Total 15]

301 A2004

[3]

PLEASE TURN OVER

The trustees of a charity whose assets have a market value of 1 billion have directed
that 60% of the fund be invested in equities tracking the FTSE Actuaries All Share
Index and 40% be invested in bonds tracking the FTSE Actuaries All Stocks Bonds
Index (the benchmark indices).
During 2003, the fund manager believed that equities would perform better than debt
as growth prospects for the UK economy were improving. Hence at the start of the
year he placed 800 million in equities and the balance of 200 million in short
bonds. He reinvested dividends and coupons in the respective sector as soon as they
were received. New money was invested in the ratio of 80:20 for equities and debt.
The following data has been provided.
FUND VALUE
Equities
Short Bonds

Date
1/1/03
31/3/03
30/6/03
30/9/03
1/1/04

800
885
1,030
1,000
1,150

Return

Q1
Q2
Q3
Q4

Net contribution received from


charity at the beginning of the quarter

200
230
245
230
275

+200
+180
100
150

Benchmark Returns (% per quarter)


Equities
Bonds
Bonds
(income reinvested)
all stocks
<5 years
10.0
2.0
5.7
3.5

9.0
15.0
3.0
2.0

5.0
8.0
+2.0
+6.0

(i)

Calculate money weighted and time weighted rates of return for the overall
fund in 2003 stating any assumptions that are made.
[4]

(ii)

For each quarter allocate the difference between the fund s rate of return and
the return on the benchmark between:
(a)
(b)
(c)

(iii)

that which is attributable to stock selection;


that which is attributable to asset allocation; and
that which is attributable to bond duration selection.

Give brief comments on the performance of the fund manager.

END OF PAPER

301 A2004

[11]
[3]
[Total 18]

Faculty of Actuaries

Institute of Actuaries

EXAMINATIONS
April 2004
Subject 301

Investment and Asset Management


EXAMINERS REPORT

Introduction
The attached subject report has been written by the Principal Examiner with the aim of
helping candidates. The questions and comments are based around Core Reading as the
interpretation of the syllabus to which the examiners are working. They have however given
credit for any alternative approach or interpretation which they consider to be reasonable.

J Curtis
Chairman of the Board of Examiners
5 July 2004
Faculty of Actuaries
Institute of Actuaries

Subject 301 (Investment and Asset Management)

April 2004

Examiners Report

The number of candidates was significantly higher than at previous examinations. This is
likely due to the change in syllabus next year. Unfortunately many of the candidates who
presented themselves were not well prepared as is evidenced by the high level of FB results at
almost one third of the candidates who sat.
The examiners also have to apologise for setting a question that was not fully covered by core
reading. Question 2 asked candidates to value a fund that had derivative contracts as part of
its investments. Core reading does not cover contract sizes explicitly, this being covered in
401. As a consequence candidates were not able to value the derivatives. To offset this, the
pass mark was reduced by 4 marks in effect excluding the question. However candidates who
made an attempt at it gained marks for method and these were included in their overall
marks. We received comments that the exchange rate had also not been given for / . The
examiners believe that candidates should have a basic knowledge of current exchange rates
for major currencies just as they would know current interest rates.
The solutions supplied should not be viewed as being totally comprehensive. Many
questions, especially those involving bookwork, do have additional points for which marks
were awarded.
In terms of each question the following comments will hopefully help candidates.
Q1. The examiners are looking for candidates to frame their answers to each part of the
question, outlining specific points. Many answers are not well framed and appear to be
brain dumps rather than showing to the examiners that the candidate understands the topic.
Q2. This has been commented upon above. Showing method was worth up to 2.5 marks.
Q3. Part (i) was done well. In (ii), whilst the options were listed by many, the financial
implications were not so well covered and cost candidates marks.
Q4. This question was in the main well answered. Where marks were lost it was normally for
failing to articulate all aspects of monitoring.
Q5 & Q6. Both were well answered.
Q7. Candidates did not do well on this question mainly because they did not apply
knowledge. Points were often listed that needed to be considered but solutions were not
framed in relation to appropriate strategies for each type of portfolio. Part (d) was one of
the best examples where candidates appeared not to believe that a family portfolio of $70
million was large and framed answers in terms of assets and liabilities for a average
individual client rather than for one for whom normal liabilities are likely to be a very small
consideration. Part (e) was also not well done with protection of capital and income
constraints seldom mentioned.
Q8. This question was poorly answered as candidates appeared not to think deeply enough
about the issues raised especially for (iv). Part (i) was reasonably straightforward and
answers reflected this. In (ii) answers failed to cover the wider implications whilst in (iii)
explanations were weak on detail.

Page 2

Subject 301 (Investment and Asset Management)

April 2004

Examiners Report

Q9. Part (i) was done reasonably well with candidates knowing the formulae and able to
apply them. In (ii) candidates produced formulae but failed to explain them appearing to
assume that the examiners knew the notation being used. We have discovered that the
notation is used in ActEd tutorials and material. However it is not the same as that used in
core reading and the examiners do not access ActEd material. We would council candidates
to use either formulae that are fully outlined in core reading or preferably even when using
such formulae to provide definitions. Answers to (iii) were mixed but those who scored well
in (ii) tended to collect good marks for (iii).
In marking this question marks were awarded for method as much as correct answers. Thus
showing how to calculate the first quarter s analysis earned about 65% of the available
marks. Any arithmetic errors were taken into account in subsequent quarter calculations.

(i)

The factors that should be considered when investing in a Domestic


Government Bond are:
Interest rate risk
Reinvestment risk
Inflation risk
Yield curve risk
The interest rate risk and the reinvestment risk relate to changes in interest
rates. If interest rates change then bond prices will move in the opposite
direction, similarly if interest rates change then the rate on reinvested income
will also change.
Inflation risk
unexpected inflation will result in the purchasing power of the
payments received being reduced, it may also result in interest rates rising thus
reducing the value of the bond.
Yield curve risk, if the shape of the yield curve changes then this might result
in the price of the bond changing, changes in the shape of yield curve may be
brought about by either changes in the supply or demand for bonds at differing
maturity dates or by changes in the market s view of future interest rates.

(ii)

The additional factors which should be considered when investing in an


emerging market Government Bond are:
Credit risk
Default risk
Spread risk
Downgrade risk
Exchange rate risk
Liquidity risk
Political risk
Event risk
Inflation risk
CPI or RPI benchmark

Page 3

Subject 301 (Investment and Asset Management)

April 2004

Examiners Report

While the above may also be pertinent to the Domestic Bond market, they are
in the main likely to have negligible impact on the decision to invest.
Credit Risk

this can be divided into three separate headings:

Default risk this is the risk that the Government in question defaults on
its obligations; this has occurred in the past either through economic
collapse or a change of Government.
Spread risk this reflects the risk of default of the Government and
represents the extra return an investor requires over treasury bonds to
compensate them for the extra risk. This spread may change and result in
either a profit or loss for the investor.
Downgrade risk
if a rating agency such as S&P or Moodys downgrade
a country s bonds then the price is likely to fall; these agencies will
downgrade if they feel there is an increased chance of default. Equally
they may upgrade a country s bonds if they feel that the country s
economy has improved.
Exchange risk
this is simply the risk that the exchange rate between the
investor s domestic currency and that of the emerging market will change thus
resulting in a profit or loss on the bonds held.
Liquidity risk
this relates to the ability or inability of the investor to convert
the bond into a known amount of cash at short notice. This ability may change
over time and will depend on the size of the issue and its popularity.
Political risk
as mentioned previously there may be a change in
Government and the new Government may be unwilling to honour the
obligations of the previous Government regardless of its ability to pay.
Event risk there are many events that may cause a country to default on its
payments, a devastating earthquake or famine or floods may severely damage
a county s economy, a revolution may bring to power a Government who no
longer wants to honour its obligations.
Inflation the reliability of the CPI/RPI may create an additional risk over
normal inflation risk

Page 4

UK FTSE short has value = 350*10*4225*1.5 = ( 22.2 million)


UK Gilt long has value = 150*100,000*1.0611*1.5 = 23.9 million
Total value portfolio = 750 + 650 + 150 22.2 + 23.9 = 1,551.7 million
Cash = 9.7%, Equities = 46.9% and bonds = 43.4%

Subject 301 (Investment and Asset Management)

(i)

April 2004

Examiners Report

Offices
Leases are typically long-term, full repairs/insuring and have five year
upward only rent reviews.
Wide range of prospective tenants in different industrial sectors and building
often multi-let. This serves to control void risk and provides comparability
when setting rent.
Location must be convenient for staff and customers but precise location not
as important.
Rent will typically be a small proportion of the tenants outgoings.
Obsolescence can be a problem if pace of modernisation slips
Industrial
Precise proximity to labour and communications network is important.
Often industry/tenant specific leaving property vulnerable to rental void.
Can be built relatively quickly and cheaply.
Usage can mean more rapid obsolescence/deterioration.
Rent likely to be more significant part of tenants outgoings.
On the whole, offices lowering yielding than industrial

(ii)

Sale at reasonable price unlikely to be an option if rental dispute not resolved.


Similarly, redevelopment (given significant costs/further investment) very
much a last resort option when other negotiated solutions have failed.
Option to accept the proposal (or a negotiated compromise) on the basis that
not to do so could lead to rental void (tenant moves or goes out of business)
and subsequent increase in management costs and fees.
Rental reduction could be agreed to alongside space reduction. Although
industrial property is unlikely to be immediately useable by a new tenant, the
offices could be sub-let.
Different levels of rental income cashflows can be discounted and compared
with any potential sale proceeds.
Income should be adjusted for outgoings and management costs and
assumptions made about potential future rent increases/decreases.

Page 5

Subject 301 (Investment and Asset Management)

April 2004

Examiners Report

The discount rate is likely to be set by reference to bond yields adjusted for
risk of void.
Potential splitting of tenancy likely to be positive move in terms of controlling
risk.
Discussion with tenant on longer term viability of own business model given
request for rent reduction is vital. Macro and micro economic analysis should
be brought out in these discussions, particularly if new tenant likely to be
required.
Local knowledge of property expert (supplemental to in-house team) likely to
be helpful when discussing future rental levels and potential sale prices. Local
comparisons will be required as well as national examples perhaps drawn from
own portfolio.
Redevelopment option would have to be assessed in the first instance by
reference to local property developments and/or own in-house resources.

(i)

The diagram below represents the actuarial control cycle as applied to


institutional investment arrangements.

Setting clear
objectives
Regular monitoring of
all arrangements in
place relative to the
objectives

Select suitable and high


quality investment
managers to operate the
strategy and structure
defined.

(ii)

Agree a long term


strategic asset
allocation

Building a structure of the


different types of investment
manager that best fits the long
term strategic asset allocation

Monitoring should be focused on the different decision points in the cycle to


ensure departures from targeted outcome identified and assessed.
This should incorporate regular assessment of whether objectives remain
right for the current and expected future situation.
Specific areas to address include:

Page 6

Subject 301 (Investment and Asset Management)

April 2004

Examiners Report

At the investment manager level: quarterly performance and risk monitoring


qualitative analysis on investment managers staff capability and house
methodologies
At the manager structure level: analysis of suitable manager types
(value/growth, large/mid/small cap, active/passive) dialogue/judgement
required on whether overall balance still appropriate.
At the strategic level: risk assessment on degree of matching between assets
and liabilities and judgement/dialogue on whether still acceptable. Regular
pro-active assessment on whether there are new asset classes that should be
considered further.

(i)

An OTC option is a privately negotiated derivative contract offered by dealers


directly to end-users.

(ii)

The exchange protects its credit exposure to participants in several ways.


Trades can only be cleared by members of the exchange with clearing status.
Clearing status involves authorisation, having certain minimum capital and
operational standards.
Institutional investors, corporates and individuals who wish to effect futures
transactions must have their trades cleared by members of the exchange with
clearing status.
Clearing members of the exchange must pass on at least the initial margin
requirement and the variation margin calls to their client. They can of course
pass on higher margin requirements.
The exchange imposes initial margin requirements on clearing firms (and
hence their clients) as part of the procedure of entering into a futures contract.
Typically, the initial margin requirement would provide the exchange with
sufficient capital (usually with 99.5% certainty) to weather an adverse price
movement in the futures contract in the event of a client/clearing member
defaulting.
The credit exposure of the client/clearing member to the exchange varies with
the value of the futures contract. E.g. where the client has a short futures
contract on the FTSE100 index and the index rises, the client s exposure to the
exchange via the clearing member increases.
Variation margin is also required where the initial margin falls below a
threshold specific to the contract. This provides collateral movement from the
client to the exchange that varies the credit exposure of the client/clearing
member to the exchange.

Page 7

Subject 301 (Investment and Asset Management)

April 2004

Examiners Report

Price movement limits allow the exchange to suspend trading in a contract if


its price moves up or down by more than set limits. Such limits allow the
exchange to limit its credit exposure to clearing members/clients in the event
of sudden moves in the price of the futures contract.
The margin requirements for speculators may be different to those for hedgers
as speculators may not have the underlying assets to deliver.
Exchanges usually reserve the right to increase the margin requirements if
they deem it fit.

(i)

The general formula is from Unit 12.

(ii)

Not all shares are freely available for purchase with some being held for long
term strategic/business reasons.
Including these shares within an index can lead to distortions in the
performance analysis, complications for index trackers and other problems.

(iii)

Unweighted/ GDP weighted/ geometric would be acceptable. Appropriate


formulae required.

(a)

The investment objective is to create as large a fund as possible on


retirement
although as retirement approaches some defence against the possibility of
falling interest rates is also desirable.
So likely strategy would involve investment in equities (and possibly bonds as
well for diversification), transferring into long-dated gilts as retirement
approaches.
The key issues should centre around finding the optimum time to begin
transferring
and how regularly to transfer
and in what proportions of the fund.

(b)

Being able to set competitive premiums can depend on achieving a good


investment return on reserves
so the highest return is required
subject to risk of technical insolvency
and maintaining adequate liquidity.
Investment strategy will be a diversified investment strategy using asset types
permitted by solvency technical regulations.
The key issues should centre on producing probabilities of insolvency from
various combinations of assets.

Page 8

Subject 301 (Investment and Asset Management)

April 2004

Examiners Report

(c)

The shareholders will not want to see the money that has been raised invested
in any risky investments as it has been raised for a clear function
for new
product R&D
yet the company will want to work the money as hard as possible since
some of it may not be spent for some time
So the strategy is likely to entail investment in safe, short dated cash
instruments.
The key issues should centre around the probability of capital loss
as more risky investments (e.g. corporate bonds) are considered in order to
raise return

(d)

This portfolio is likely to have no liabilities, restrictions or objectives to


concern itself with
other than to aim for a very well-diversified investment strategy...
to deliver the highest possible return
It can consider investing in all asset types.
The key issues should centre around correlation between asset types
and the risk/return payoff of those asset types.
Absolute returns likely to be preferred.

(e)

Charities generally have capital that they wish to maintain


while aiming to pay their costs and do their charitable work from donations
and the cash flow generated by the capital.
Hence investment in investment grade
bonds suggests itself as a basic strategy
The key modelling investigation should centre around the probability of
capital loss as proportions of high income non-investment grade bonds and/or
high income equities etc are added to the portfolio.

(i)

The overlapping periods are not independent (whereas non-overlapping


periods are independent) and therefore lead to unreliable estimates of the 20year mean return.

(ii)

The sample size (5 non-overlapping periods) is too small to use in the


development of reliable estimates of future 20-year returns from equities.
The range of 20-year returns developed from the 5 independent 20-year
periods is likely to understate the spread of returns that one might see from
this market over 20-year time periods in the future.
It is quite possible that there could be 20-year periods in the future, which
produce negative returns.

(iii)

expectations of future corporate profitability


value of those profits
real interest rates and inflation
perception of riskiness of equities
level of real economic growth

Page 9

Subject 301 (Investment and Asset Management)

April 2004

Examiners Report

supply and demand


taxation
attractiveness of alternative investments

(iv)

Investors tend to diversify their equity portfolios across a number of major


markets to reduce risk through lack of correlation.
20-year returns in other major developed markets were found by the
researcher to be lower than in this market
and if the volatility of returns is also higher
then lower returns and higher volatility will increase the chances of
negative returns over 20-year periods.
As the chances of positive returns over the long-term (20 years) look less
likely in an international portfolio, equity returns look less and less attractive
to the long-term (20-year time horizon) investor unless there is a significant
reduction in the correlation of returns between the major developed markets in
the future.
Investors may therefore wish to lower the equity content of their portfolios and
replace equities by assets that have similar long-term returns as equities but
which are uncorrelated to the returns of equities. This should reduce the
volatility of portfolio returns without sacrificing return.

(i)

Marks given for each formula and answer. This is straightforward application
of Unit 22. MWR =23.6%, TWR = 26.36%

(ii)

Q1.

Equities
Fixed Inc
Total

Fund Wt

B mark Wt

Ind Rtn

Fund Rtn

Asset

Stock

800
200
1000

60
40
100

10.00
(9.00)
2.40

(7.81)
(4.17)
(7.08)

1.52
2.28
3.80

(14.25)
0.97
(13.28)

Relative return is (9.48). Due to duration is 0.2*(( 5.0)


stock selection for bonds = 0.97 0.80 = 0.17.
(iii)

Stock selection main contributor to performance; especially good in Q4


Asset allocation also contributed to the good performance
Duration principal contributor to fixed income performance
Other comments worth marks if valid

END OF EXAMINERS REPORT

Page 10

( 9.0)) = 0.80 and

Faculty of Actuaries

Institute of Actuaries

EXAMINATIONS
21 September 2004 (am)
Subject 301

Investment and Asset Management

Time allowed: Three hours


INSTRUCTIONS TO THE CANDIDATE
1.

Enter all the candidate and examination details as requested on the front of your answer
booklet.

2.

You have 15 minutes at the start of the examination in which to read the questions.
You are strongly encouraged to use this time for reading only, but notes may be made.
You then have three hours to complete the paper.

3.

You must not start writing your answers in the booklet until instructed to do so by the
supervisor.

4.

Mark allocations are shown in brackets.

5.

Attempt all 12 questions, beginning your answer to each question on a separate sheet.

AT THE END OF THE EXAMINATION


Hand in BOTH your answer booklet, with any additional sheets firmly attached, and this
question paper.
In addition to this paper you should have available Actuarial Tables and
your own electronic calculator.

301

S2004

Faculty of Actuaries
Institute of Actuaries

You have been given the following information.


Year 1
Benchmark asset allocation:
Equities Index
Bonds Index

60%
40%

30%
70%

Actual asset allocation of fund:


Equities
Bonds

50%
50%

50%
50%

Investment returns:
Average return for similar funds
Equity index return
Bond index return
Equity return in fund
Bond return in fund

Year 2

7.0%
10.0%
5.0%
9.0%
6.0%

8.0%
12.0%
4.0%
14.0%
5.0%

(i)

List three methods used to assess portfolio performance, and compare the
merits of each method.
[6]

(ii)

Evaluate the past performance for the fund as a whole over the two year
period, using the three different methods.
[5]
[Total 11]

(i)

You are a UK equity analyst and have been asked to prepare a fundamental
analysis of one of the companies you cover. List the quantitative factors that
you would investigate.
[3]

(ii)

You are analysing a company which has just completed a major refinancing.
List which factors you would spend the most time investigating, explaining
why they have been selected.
[3]
[Total 6]

(i)

Explain why margin is levied on exchange-traded futures contracts. Include in


your explanation a description of initial margin and variation margin, and the
process by which these are determined.
[5]

(ii)

Your local currency is the Euro and you wish to actively hedge the currency
exposure in an actively traded 50m US equity holding. Explaining your
reasoning, state whether you would hedge the currency exposure using an
exchange-traded future or an over the counter forward contract, assuming that
the hedge is adjusted weekly.
[3]
[Total 8]

301 S2004

(i)

Outline the factors to be considered before a tactical switch can be made by an


institutional investor from their benchmark position.
[4]

(ii)

Explain what an anomaly switch is, commenting on the risks and the scope for
profits in larger bond markets.
[2]

(iii)

Describe three techniques to identify opportunities for anomaly switches.

(iv)

Describe two methods by which an investor could take advantage of an


anomaly whilst maintaining no overall market exposure (a market neutral
position) to the bond markets.
[2]
[Total 11]

[3]

A developed country is reviewing its market regulations following a number of high


profile company bankruptcies. The country currently has a mixed regime backed by
rigid codes of practice that it believes have contributed to events.
(i)

Describe what is meant by a mixed regime.

[2]

(ii)

There has been a proposal to weaken the rigid codes of practice and to replace
these with codes of principles to be adhered to. Discuss the advantages and
disadvantages of this proposal.
[4]
[Total 6]

(i)

List the advantages of investing through an investment trust rather than a unit
trust.
[2]

(ii)

Define the investment trust term discount to net asset value per share .

(iii)

Discuss three circumstances under which this discount may become a


premium.

[1]

[3]
[Total 6]

You are the consultant to a large defined benefit pension fund that has historically
invested significantly in equities. The new finance director is concerned about the
volatility of company contributions resulting from the strategy.
(i)

Set out the reasons for and against continuing a high equity strategy for the
pension fund.
[3]

(ii)

Discuss the advantages of investing in government bonds as an alternative. [2]

(iii)

State the other factors that the trustees should consider in setting investment
strategy.
[4]
[Total 9]

Explain the advantages of overseas investment for an institutional investor.

301 S2004

[3]

PLEASE TURN OVER

An airline which has an average credit rating for its sector needs to issue debt to pay
for some new aircraft that it expects to be in service for the next 20 years.
An investment bank has suggested the following three types of debt:
Bond A

a 20 year bond with a coupon of 7.0% p.a.

Bond B
a 20 year bond with a coupon of 6.5% p.a. for the first 10 years
with an option for the airline to redeem at par at that point. If the bond is not
redeemed then the coupon increases to 8.0% for the final 10 years.
Bond C
a 20 year bond with an initial coupon of 6.5% p.a.. If the
company s credit rating improves relative to its initial rating the coupon
reduces to 6.0% p.a. and if the credit rating deteriorates from the initial rating
the coupon rate increases to 7.5% p.a. Alterations apply immediately for the
period of the change in rating.
Compare the three bonds from the risk management perspective of the airline
company.

10

11

[12]

(i)

Describe four financial risks faced by an institutional investor.

[4]

(ii)

Describe methods for controlling each of these financial risks.

[6]
[Total 10]

The electricity distribution network of a country enjoys a natural monopoly. The


Electricity Market Authority is responsible for ensuring that the price that the network
operator charges for distribution is reasonable.
(i)

List the key factors, treating the electricity distribution network as a capital
project, that affect the long-term cashflow and rates of return of the electricity
distribution network.
[6]

The government believes that the electricity distribution network is making too much
profit.

12

(ii)

Discuss the measures that may be introduced to limit the profits being made.
[6]
[Total 12]

(i)

Give a generalised formula for a total return index suitable for property
performance measurement purposes, defining all terms used.

(ii)

Outline the problems in constructing such an index for quarterly publication.


[3]
[Total 6]

END OF PAPER

301 S2004

[3]

Faculty of Actuaries

Institute of Actuaries

EXAMINATIONS
September 2004
Subject 301

Investment and Asset Management


EXAMINERS REPORT

Introduction
The attached subject report has been written by the Principal Examiner with the aim of
helping candidates. The questions and comments are based around Core Reading as the
interpretation of the syllabus to which the examiners are working. They have however
given credit for any alternative approach or interpretation which they consider to be
reasonable.

M Flaherty
Chairman of the Board of Examiners
7 December 2004

Faculty of Actuaries
Institute of Actuaries

Subject 301 (Investment and Asset Management)

September 2004

Examiners Report

Due to the change in the examination system from 2005, we believe that a large number of
candidates entered for this examination to try and obtain a pass that would give them more
flexibility in determining what elections to make with regard to credits for the new
examination system. Consequently there were over 900 entries compared with around 650 in
previous years. There was some evidence that candidates were not as well prepared as they
might have been and this is reflected in the comments below.

(i)

Portfolio
relative to:

Published Indices

Other Portfolios

Benchmark
portfolio

Pros

Easy to do
Data readily available,
and accurate

Gives an indication
of the cost or benefit
of a strategy, relative
to those adopted by
other funds

Benchmark portfolio
can be constructed to
reflect fund
objectives
Can be helpful in
aligning fund
manager s interests
with liability
requirements

Cons

Index may be
inappropriate for
investor s objectives

Comparison may be
inappropriate if other
funds have very
different objectives

General cons

All methods look at past performance only, so are not a reliable guide
to the future
Assessments do not take account of risks taken by managers

(ii)
Year 1

Year 2

Total

Actual

7.50%

9.50%

17.71%

Index
Average
Benchmark

7.50%
7.00%
8.00%

8.00%
8.00%
6.40%

16.10%
15.56%
14.91%

ActualExpected

1.61%
2.15%
2.80%

This was answered well in the main for both the bookwork part (i) and the application part
(ii).

Page 2

Subject 301 (Investment and Asset Management)

September 2004

Examiners Report

(i)
The purpose is to identify and analyse the key factors affecting the future
profitability of a company. This is to determine whether a share is over- or under-valued by
the market.
Investigations
Financial accounts and accounting ratios
Dividend and earnings cover
Profit variability and growth by looking at sources of revenue and expenditure
Level of borrowing
Level of liquidity
Growth in asset values
(Both for the company and also for similar companies in this sector and
outside the sector)
(ii)

As there has been a refinancing, many of the above investigations will no


longer be relevant. This particularly affects historical financial information,
rather than prospective forecasts.
Care should be taken with the company s own forecasts, which may give an
excessively optimistic picture of future prospects.
The key investigations to focus on will be:
sources of revenue and expenditure
interest and capital payments on borrowings
cash levels, and cash forecasts
changes in net asset values
quality of management
future earnings forecasts

Part (i) was well answered but (ii) was not with few candidates showing the understanding
that the refinancing would be likely to distort financial ratios etc.

(i)

Margin is the collateral which each party to a futures contract must deposit
with the clearing house. It acts as a cushion against potential losses which the
parties may suffer from future adverse price movements.
In the event of a party to a contract defaulting the clearing house will protect
the other party to the contract by ensuring the contract is fulfilled.
Initial margin is deposited with the clearing house when the contract is first
struck, by both parties to the contract.
To ensure that the clearing house s exposure to credit risk is controlled, the
margin is changed on a daily basis through additional payments/refunds of
variation margin.

Page 3

Subject 301 (Investment and Asset Management)

September 2004

Examiners Report

If the price of a contract falls, the buyer has to put up additional margin whilst
the opposite is true if the price rises.
(ii)

State view
Euro/US dollar forwards are more flexible than futures so can have a longer
term, whereas futures will need to be rolled over periodically (typically on a
quarterly basis.)
A disadvantage is that adjusting the outstanding position weekly will be
simpler with futures than with forwards as futures can be bought and sold
easily on an exchange, whereas forwards can only be bought and sold from
investment banks
this may result in an imperfect hedge being maintained at
times.
Transaction costs may be an issue
Counter-party risk

Both parts reasonably answered.

(i)

Authority and advice to switch


are approvals in place/required
Expected extra return to be made relative to additional risk (if any)
Are there portfolio/mandate constraints on the changes that can be made
Expenses of making the switch
that a profit will be made on return to
neutral position
Problems of switching a large portfolio of assets

(ii)

This is a switch between stocks with similar volatility, taking advantage of


temporary anomalies in price.
Generally a relatively low risk strategy,
but the widespread use of computer based analysis by market participants
limits opportunities to profit from significant anomalies in major markets.

(iii)

Any 3 of yield differences, price ratios, price models, yield models.

(iv)

The investor could:


buy the asset which is believed to be under priced and short sell a similar asset
which is correctly priced or overpriced, or
buy the asset which is believed to be under priced and sell a derivative linked
to the benchmark bond for the market.

The examiners were disappointed with the answers to this question. Many candidates
appeared not to have a full grasp of the bookwork and missed numerous points in (i). Part
(iii) was also poorly answered.

Page 4

Subject 301 (Investment and Asset Management)

(i)

September 2004

Examiners Report

In practice many regulatory regimes are not based solely on one system of
regulation, but are instead based on a mixture of systems to enhance the
strengths and address the weaknesses. A mixed regime is one that includes the
following approaches to regulation operating in parallel:
Unregulated markets
Voluntary codes of conduct
Self-regulation
Statutory regulation

(ii)

A rigid code of practice is very precise in what exactly is and is not permitted.
A code of principle is a general guide to the fundamentals but not necessarily
the detail.
Advantages
Codes of principle are more durable to changing market conditions.
Codes of principle should allow the market to operate more efficiently by
allowing product innovation.
Should encourage competition.
Provides greater freedom of action.
There should be less ability to act against the spirit of the regulation.
Disadvantages
Regulation is less precise with more grey areas so market participants will be
less sure that they are operating within the rules, leading to transgressions at
the periphery going unchecked for longer.
Because the rules are less precise there is greater reliance on the regulator
enforcing regulations, investigating suspected breaches and imposing
sanctions.

Candidates knew the answers to (i) but were generally weak on (ii).

Page 5

Subject 301 (Investment and Asset Management)

September 2004

Examiners Report

(i)
The gearing of investment trusts should enable them to outperform unit
trusts in bull markets.
Investment trust schemes may be bought at a discount to net asset value
and if the discount narrows this should be a source of outperformance
relative to unit trusts.
Investment trust management charges are usually lower than for unit trust.
Investment trusts can invest in a wider range of assets than unit trusts.
Investment trusts may have a better tax position than unit trusts.

(ii)

The discount to net asset value per share is defined as:

net asset value market price


net asset value
expressed as a %
(iii)

This may become a premium because:


(a)

the value of assets may be historic and due for re-rating

(b)

investors in the trust may be barred from direct entry to the markets in
which the trust is invested and they may be prepared to pay a premium
in order to gain the exposure they desire

(c)

investors in the trust may anticipate the trust management adding value
on top of the current market prices of the trust s investments

(d)

pay a premium for geared exposure to a rising market

This was well done for parts (i) and (ii) but candidates often had few explanations in (iii).

Page 6

Subject 301 (Investment and Asset Management)

(i)

High equity

September 2004

Examiners Report

reasons why

Highest expected return


Historically successful (until 2000)
Not possible to match liabilities by nature or term
Surplus generation used to improve benefits and reduce company
contributions
Advance credit can be taken in actuarial valuation through equity risk
premium in discount rate
May offer hedge against inflation
May match statutory funding test
May match transfer basis following sale
Common strategy for other schemes (herd instinct)
Reasons why not
Maturity of fund
No match for guaranteed benefits and no surplus
Insufficient income generation
Highest risk and maybe risk is too high for sponsor, especially if scheme
large relative to the sponsor
Doesn t match statutory funding test or accounting assessment
Threat of deflation
Availability of alternatives
As the question asks for strategy i.e. long-term benchmark allocation, no credit
is available for references to short term under or over value of equity markets.
(ii)

Advantages of government bonds


Lowest risk credit, currency and possibly inflation, could improve
sponsor credit rating through lower volatility of contributions
Match or immunise to annuity pricing/buyout solvency or statutory
funding test bases
Liquidity
Income guarantees
Lower dealing and management costs

(iii)

Other issues to consider in setting strategy


Nature and term of liabilities and any changes anticipated
Level of funding and cashflow and any changes anticipated
Statutory funding tests and horizons
Capital market forecasts of returns and risks within asset and liability
model
Stochastic modelling of developing assets and liabilities
Sponsor creditworthiness and tolerance of funding and contribution
volatility

Page 7

Subject 301 (Investment and Asset Management)

September 2004

Examiners Report

Trustees attitude to risk


Market capitalisation or peer group target comparisons
Cost of change and implementation
Good candidates did well on this question but poorer candidates did not give satisfactory
answers to (ii) and appeared unaware of the factors to be covered in (iii).

Benefits of overseas investing


Diversification of market and currency and creates opportunities for tactical asset
allocation
Expands the universe of available opportunities and allows active fund managers
to invest in companies giving the best return/risk profiles, wherever the companies
are listed
Allows access to best of breed and specialist fund managers in regions
Markets may be more inefficient than domestic market increasing outperformance
opportunity
May match real liabilities over the long term
May feature in global market or consensus benchmark

This bookwork question was done reasonably well.

In comparing the three bonds one needs to be mindful of issues that will affect the
sector and the company. The main issues are:
Airlines are cyclical service companies so there will be a greater impact from the
economic cycle.
Airlines are both capital and labour intensive.
Generally subject to tight government regulation and vulnerable to other forms of
political risk (e.g. terrorism).
The domestic market is the most important but they also have international exposure.

Bond A
The bond has a term of 20 years so corresponds to the period of expected use of the
aircraft and there will be a residual value of the aircraft at the end, however, it would
recoup the capital cost over the life of the aircraft not at the end. This should not be
an issue as the company will have other aircraft and borrowings to meet.
The coupon at 7% p.a. is higher than the other bonds because more credit risk has
been transferred to the lender.
The return from the risk transfer is greater certainty over the cost of borrowing.
In a cyclical industry this avoids cost of servicing debt varying and having to
refinance in unknown future economic conditions.

Page 8

Subject 301 (Investment and Asset Management)

September 2004

Examiners Report

Bond B
Again the bond has a term up to 20 years to match the expected use of the aircraft.
However, this can also be considered as a 10 year bond with the option to extend for a
further 10 years.
In the first 10 years the coupon is lower because the credit risk over 10 years is lower
than over 20 years, so the initial cost of the debt is lower.
At the end of 10 years the company can choose to refinance the debt and repay this
loan or to extend it.
If the borrowing costs are lower for a new issue the company will repay this loan.
If the re-financing costs are more than 8% p.a., for example if the company s credit
rating has deteriorated or the general cost of borrowing has increased the company
will extend the debt.
The higher coupon after 10 years is the cost of the option to extend the borrowing
with a known cost.
The disadvantage of the arrangement is that cost of the borrowing is less certain and
may rise during a low in the economic cycle when it can be least afforded.
Bond C
This bond also allows the company to borrow for 20 years.
The initial coupon is 6.5% p.a. and offers the prospect of a lower cost if the
company s credit rating improves, .
but the penalty is a higher cost if the credit rating deteriorates.
This arrangement retains more credit risk with the company.
If economic conditions are difficult and is the reason for the company s current credit
rating then this arrangement may offer the best prospect of a lower borrowing
requirement.
However, the arrangement has the highest liquidity risk as it results in less certain
borrowing costs and
more variable profitability as the cost of servicing the debt is most likely to increase
when profitability is low and to reduce when profitability is high.

Of all the questions this was the one that candidates had most difficulty with and few got
more than half marks. Candidates appeared unable to relate the bonds to the business in the
context of cash-flow, useful life and options.

Page 9

Subject 301 (Investment and Asset Management)

10

(i)

September 2004

Examiners Report

Market risk the risk relating to changes in the value of the portfolio due to
movements in the market value of the assets held
Credit risk the risk that a counterparty to an agreement will be unable or
unwilling to fulfil their obligations, or fails to perform them in a timely
fashion
Operational risk
the risk of loss due to fraud or mismanagement within the
fund management organisation itself
Relative performance risk
institutional investors

the risk of underperforming comparable

Liquidity risk
the risk that though solvent (on a balance sheet basis) either
does not have sufficient financial resources available to meet obligations as
they fall due or can only secure them at excessive cost

(ii)

Controlling Market Risk


To control the risk a measurement is required such as variance of the return on
the portfolio over a specified period of time or the maximum loss that could be
suffered with say a 95% or 99% probability within the timescales. The returns
and losses may be measured in absolute terms or relative to a performance
benchmark.
Using the risk exposure measure, a risk control system can be set on the
acceptable risk, such as limits on the value at risk.
Controlling Credit Risk
The key factors in managing credit risk are:
Creditworthiness of the counterparties
Total exposure to each counterparty
The credit risk can be controlled by placing limits and monitoring credit
ratings and exposures for individual counterparties.
It can also be controlled in derivative transactions by dealing with recognised
exchanges with a clearing house which stands as counterparty to all deals.
Credit risk needs to be controlled across the range of types of assets and not
just by asset class.

Controlling Operational Risk


Operational risk may not generally be as easy to quantify or measure as credit
or market risk.
Page 10

Subject 301 (Investment and Asset Management)

September 2004

Examiners Report

Control of operational risk essentially depends on good management practice


including having established and documented chains of reporting and
responsibility.
Relative Performance Risk
The techniques for monitoring and controlling relative performance risk are
essentially the same as those for controlling market risk except that
performance is measured relative to the performance of the institution s
competitors rather than in absolute terms or relative to the market as a whole.
Liquidity Risk
There are four essential elements to controlling liquidity risk.
Having credit facilities, for example bank overdraft facility or sufficient size to
meet immediate cash requirements.
Having readily realisable assets with a low penalty for forced sales so that
cash can quickly be accessed with a low cost for forced sales.
Monitoring and projecting cash flow so that the cash requirements and asset
realisations can be planned to avoid forced sales.
Having terms of business that control or delay the timing of payments so that
the cash flow and asset realisations can be planned.
Whilst (i) was done reasonably, many candidates failed to pick up the marks available in (ii)
through being unable to describe how the risks could be controlled.

11

(i)

Key Variables
Operating costs including repairs, maintenance, administration, depreciations
and the purchase of lost electricity.
Net interest bearing debt possibly separated into long term debt and short term
debt to fund working capital.
Capital invested equals balance sheet total adjusted to reflect the technical
replacement value of the network together with a number of other small
adjustments.
Implied debt/equity ratio.

Page 11

Subject 301 (Investment and Asset Management)

September 2004

Examiners Report

Key Features
Permitted total return on capital invested based on weighted average cost of
capital formula.
Return on equity calculated using the capital asset pricing model, being based
on a riskless return (annualised return from a government bond with an agreed
term to maturity) plus a margin reflecting the (low) riskiness of the business.

Revenues received should reimburse actual operating costs.

(ii)

The three measures that could be introduced are price controls, tax and
competition. A brief description of the nature and effects of each is required

Part (i) was done reasonably although many failed to get all the major points. Part (ii) was
not done well with few candidates being able to outline a series of measures.

12

(i)

The general formula is:

I (t )

wi
i

Pi ,t
Pi ,0
wi

where I(t) is the capital index at time t;


Pi,t is the price of the ith constituent at time t;
Pi,0 is the price of the ith constituent at time 0
the last time at
which there was a capital change;
wi is the weight applied to the ith constituent;
K is a constant related to the starting value of the index at time 0.
The weights used are the market values of the constituents, usually the market
values of the constituents at time 0.
Total return index is calculated from the capital index described above plus a
Rental Yield adjustment.
The income received over the 12 months prior to time t (measured in index
points) is
I(t)

yt

where yt is the average rental yield at time t.

Page 12

Subject 301 (Investment and Asset Management)

(ii)

September 2004

Examiners Report

The production of reliable indices requires knowledge of the market values of


the constituents of the indices at frequent intervals. There are a number of
problems in obtaining such information for property:
Each property is unique.
The market value of a property is only known for certain when the
property changes hands.
Estimation of value is a subjective and expensive process.
Valuations will be carried out at different points in time.
Sales of certain types of investment property are relatively infrequent.
The prices agreed between buyers and sellers of properties are normally
treated with a degree of confidentiality.
The heterogeneity of property magnifies the problems of obtaining price data.
It is difficult to group properties into usefully homogeneous groups and still
obtain sufficient price data for each group e.g. the underlying portfolio of
properties will vary in size, regional spread and sector weighting (office, retail
etc.).
As the current rental income (the historic yield) is fixed until the next rent
review, any response to movements in rental values will be spread over time.

This was done reasonably although many candidates did not score as well as they should
have due to poor definitions of terms used.
The solutions included in this report should not be viewed as definitive but as indicative of
what was required to score full marks. Where additional points were made these were
awarded appropriate marks. In total there were around 110 marks available rather than
100.

END OF EXAMINERS REPORT

Page 13

Faculty of Actuaries

Institute of Actuaries

EXAMINATION
12 April 2005 (pm)

Subject ST5
Finance and Investment
Specialist Technical A
Time allowed: Three hours
INSTRUCTIONS TO THE CANDIDATE
1.

Enter all the candidate and examination details as requested on the front of your answer
booklet.

2.

You have 15 minutes at the start of the examination in which to read the questions.
You are strongly encouraged to use this time for reading only, but notes may be made.
You then have three hours to complete the paper.

3.

You must not start writing your answers in the booklet until instructed to do so by the
supervisor.

4.

Mark allocations are shown in brackets.

5.

Attempt all 7 questions, beginning your answer to each question on a separate sheet.

6.

Candidates should show calculations where this is appropriate.

AT THE END OF THE EXAMINATION


Hand in BOTH your answer booklet, with any additional sheets firmly attached, and this
question paper.
In addition to this paper you should have available the 2002 edition of the
Formulae and Tables and your own electronic calculator.

ST5 A2005

Faculty of Actuaries
Institute of Actuaries

(i)

(ii)

State the taxation factors and influences on these factors that need to be
considered when selecting investments which maximise after tax returns.
Outline the three main systems of corporation tax.

[6]

[3]
[Total 9]

You are a fund manager managing a portfolio of international equities. As part of a


presentation to trustees you have presented the following table:
Region

Fund Return

Market Return

US
Japan

+5.5%
+9.4%

+5.1%
+8.3%

Market returns are stated with reference to the S&P 500 for the US and Topix for
Japan.
A trustee points out that the Dow Jones Index rose by 6% over the period and the
Nikkei rose by over 10% and therefore the fund has actually underperformed in these
regions.
Outline the points you would make in your response.

[8]

An insurance company has a line of shares held within its shareholders funds. The
shares are in a quoted investment management organisation to whom the insurance
company has outsourced the management of its policyholders funds and, due to good
recent performance, these funds are experiencing significant positive cashflows. The
rest of the shareholders funds are invested in bonds and property. The insurance
company wishes to transfer half the line of stock to the policyholders global equity
fund at a discount to the prevailing bid price and has been required by the regulator to
commission an independent valuation of the discounted share price at which the
transfer should take place.
(i)

State the key objectives of financial regulation.

[3]

(ii)

Explain why the company might wish to transfer the shares to the
policyholders funds.

[3]

(iii)

(iv)

ST5 A2005

(a)

Outline the principal factors to be considered in determining an


appropriate price.

(b)

List the additional information you would consider in setting the price.
[3]

Set out the other business issues that should be considered by the committee
responsible for agreeing the discount on behalf of the policyholders.
[11]
[Total 20]

(i)

Describe how to construct a zero coupon yield curve.

[5]

(ii)

Define the par yield for a bond.

[2]

(iii)

You have the following information for three conventional gilts:


Gilt 1: coupon 6.75%, maturity 1 year, price 102.01
Gilt 2: coupon 9.50%, maturity 2 years, redemption yield 4.90%
Gilt 3: coupon 7.75%, maturity 3 years, price 110.41
Assume that gilts pay annual coupons and that a coupon has just been paid.
Calculate the spot yields for the next 3 years.
[5]
[Total 12]

The table below contains information about a pension fund and index returns. The
benchmark for the fund is an investment that is 50% equities and 50% bonds.
31/12/03
31/12/04
Contributions
Values (m) Values (m)
(m)

Equities
Bonds
Cash
Equity Total Return Index
Equity Index Yield
Bond Total Return Index
Bond Index
Base Rate

600.0
350.0
50.0
1,000.0
3.00%
1,220.0
275.0
3.50

700.0
450.0
50.0
1,115.0
3.12%
1,299.3
280.5
4.00

12.0
63.0
1.0

Investment
Income
(m)
15.0
25.0
1.0

(i)

Defining all formulae used and stating any assumptions made, analyse the
performance of the fund.
[11]

(ii)

Comment on the results of your analysis and any investment features that the
data may suggest.
[9]
[Total 20]

In Nestl v. National Westminster Bank plc [1994] the judge considered that
decisions of trustees should be judged by modern portfolio theory and that the risk
level of the whole portfolio is considered rather than just individual investments.
(i)

Outline how Behavioural Finance challenges this view.


[10]

(ii)

ST5 A2005

Give examples of how trustees could make poor manager selection decisions
based on these behaviours.
[4]
[Total 14]

PLEASE TURN OVER

Spenser & Michael (S&M) is a UK-based food retailer which is well known
throughout Europe and the Far East but largely unknown in the United States of
America. S&M have tried to borrow US$500m at a fixed rate of interest in US
dollars but the interest rates S&M can secure are prohibitively expensive.
S&M have been quoted a five-year fixed rate of 6% per annum for a sterling
denominated loan.
BIM is a US-based food retailer and would like to borrow the sterling equivalent of
US$500m over five years at a fixed rate of interest in sterling. Like S&M, BIM has
been quoted prohibitively expensive rates for a sterling loan.
BIM has been quoted a five-year fixed rate of 5.25% per annum for a US$
denominated loan.
At the time of the transaction, the yield on five-year government bonds is 5.25% in
the UK and 4.75% in the US.
You are the head of the currency swap desk of a global investment bank.
(i)

Describe, using the above information, the factors that will influence the
design of a five-year currency swap.

[8]

(ii)

Design a five-year currency swap for S&M and BIM that will net the global
investment bank 0.45% per annum over the life of the swap while ensuring
that S&M and BIM have no exchange rate risk on their exchange of interest
payments.
[4]

(iii)

Describe the risks that the global investment bank takes on in structuring this
swap for BIM and S&M and how the global investment bank can hedge these
risks.
[5]
[Total 17]

END OF PAPER

ST5 A2005

Faculty of Actuaries

Institute of Actuaries

EXAMINATION
April 2005

Subject ST5
Finance and Investment
Specialist Technical A
EXAMINERS REPORT

Introduction
The attached subject report has been written by the Principal Examiner with
the aim of helping candidates. The questions and comments are based around
Core Reading as the interpretation of the syllabus to which the examiners are
working. They have however given credit for any alternative approach or
interpretation which they consider to be reasonable.

M Flaherty
Chairman of the Board of Examiners
28 June 2005

Faculty of Actuaries
Institute of Actuaries

Subject ST5 (Finance and Investment Specialist Technical A)

April 2005

Examiners Report

Generally candidates were able to make reasonable attempts at most questions.


Whilst there were some incidences of weak knowledge of bookwork, those candidates
who failed usually scored poorly on application and or higher level skill aspects of
questions. The following comments are written to assist candidates with
understanding what the examiners are looking for. The solutions should not be
regarded as complete but as a guide to the amount of knowledge and level expected.
Other reasonable points and interpretations were awarded appropriate marks.
Q1. This was a bookwork question and well answered. Candidates failed to pick up
marks primarily under the influences section of (i). Marks were also lost for
incomplete explanations in (ii) especially under classical and split-rate where thye
shareholder s position was often ignored.
Q2. Another bookwork question that was done well. Points tended to be lost because
of incomplete explanation of points.
Q3. This question was poorly done. Whilst most could attempt (i), the remainder of
the question seemed beyond many candidates as they were unable to demonstrate that
they understood what the issues were and how they needed to be dealt with.
Q4. This was well done although it was interesting that continuous interest rates were
determined rather than yearly rates. Calculation of the price of the 2-year bond
required the use of yearly rates given the information supplied. Where an error was
made in the calculations this was penalised only once and provided this was carried
through marks were awarded for subsequent calculations.
Q5. This type of question gets set on a regular basis and the examiners are always
surprised at the spread of answers that they encounter and the failure to use specific
pieces of information supplied, particularly in relation to part (ii) of this question.
Candidates should realise that all information is there for a purpose usually to assist
them in framing their answers in an appropriate manner. The solution shows the
answer that is most likely to be encountered in the real world. However no penalties
were applied for the more unrealistic assumptions that were often used regarding the
timing of contributions and income. The examiners believe that full analysis should
be undertaken in a question of this nature as illustrated in the solution.
Candidates are encouraged to set out the formulae that they use as this will ensure
that when a mistake is made marks can still be awarded for subsequent calculations.
While many candidates scored well in (i), few candidates were able to attain
significant amounts of marks in (ii). Explanations tended to lack the detail necessary
to attain the marks on offer.
Q6. This question was done well being predominantly bookwork.
Q7. Of all the questions this was the one that candidates had most difficulty with.
Answers in (i) were basically what the examiners view as brain dumps and were
not focused on the question as set. Accordingly poor marks were awarded. Very few
good answers were seen for (ii) but many managed to collect around half marks in
(iii) despite poor attempts at the earlier sections.

Page 2

Subject ST5 (Finance and Investment Specialist Technical A)

April 2005

Examiners Report

The factors to be considered are:


The total rate of tax on an investment.
How the tax is split between different components of the investment return.
The timing of tax payments.
Whether the tax is deducted at source or has to be paid subsequently.
The extent to which tax deducted at source can be reclaimed by the investor.
To what extent losses or gains can be aggregated between different investments or
over different time periods for tax purposes.
Influences on these factors are:
tax rates on capital gains
tax rates on income/dividends
exemptions and allowances against tax
rules on particular assets
investor s own status
investor s financial position
investment vehicle s tax efficiency

(ii)

Classical: A company s profits are taxed twice: once in the hands of the
company and once in the hands of the shareholder. The shareholder may be
subject to tax on dividends and/or capital gains arising from increases in the
share price.
Split-rate: Similar to the classical system excepting that different tax rates
may be levied on retained profits and distributed profits. The system might be
used in conjunction with a system that taxes investor s income and capital
gains at different rates.
Imputation: A system designed to enable a company s profit to be taxed once
rather than twice. Dividends paid from taxed profits are paid to shareholders
together with a tax credit. The rules vary greatly and can be quite complex but
it is often the case that the tax credit received is sufficient to offset the tax due
on the net dividend. Also, lower taxed investors can often reclaim the tax
credit.

Page 3

Subject ST5 (Finance and Investment Specialist Technical A)

April 2005

Examiners Report

While the Dow Jones and Nikkei are indices that are often quoted they are not
particularly representative of their markets. The Dow Jones Index is based on 30
shares and the Nikkei is based on 225 shares.
The S&P 500 and Topix are more broadly based being based on 500 and 1,100 shares
respectively. They are therefore more representative.
Both the DJ and N are unweighted indices, that means that every company has the
same impact on the index.
Both the T and S&P are weighted indices, the weights being the market capitalisations
of the companies, this means that larger companies have more influence on the index
than small companies.
The constituents of the N have changed little since inception whereas the Japanese
stock market has changed significantly.
The DJ is made up of 30 industrial stocks and therefore ignores the impact of other
areas e.g. financials.
The constituents of both the S&P & T are revised regularly and encompass the full
range of companies operating in their respective markets.
Therefore the S&P & T are better indices to use when looking at fund performance as
they better represent the universe from which fund managers can select stocks.

(i)
to correct market inefficiencies and to promote efficient and orderly
markets
to protect consumers of financial products
to maintain confidence in the financial system
(ii)
to remove volatility in the insurance companies solvency margin
to reduce the inherent investment risk of the shareholders investments
get a better price than a public sale
to avoid negative publicity/speculation arising from a public sale
(iii)

(a) Any regulatory or recommended limit on the maximum (and minimum


discount).
Discounts applying to similar private placements or block trades.
(b) Normal daily volume in the shares and transaction prices.

(iv)

Page 4

Being seen to avoid conflicts of interest (who instigated transaction and why).

Subject ST5 (Finance and Investment Specialist Technical A)

April 2005

Examiners Report

Treating customers fairly (suitability of investment; appropriateness of terms)


and so satisfy regulator s Principles of Business.
Is the asset manager involved in the transfer and will they be paid for
managing the holdings thereafter.
Any lock in period that restricts the shareholders dumping the stock at a later
date.
Respective size of the policyholders fund into which the stock is being
transferred,
its benchmark and mandate, concentration of holdings, nature of underlying
investments (direct or commingled funds)
Who are other shareholders
how free is float?
Volatility of share price and impact on portfolio risk

(i)

Starting point is to take the market prices of conventional bonds (e.g. gilts) for
a range of possible maturities.
Starting at the shortest maturity, T1 say, use the observed market price and
solve for the yield. This yield is an approximation for the zero coupon rate for
maturity T1, called R1, say.
Using the next shortest maturity conventional bond maturing at T2, again take
the observed price and using R1 solve for the forward rate starting at T1 for
the period T2 T1. Now solve for the spot rate R2.
Repeat using the next maturity conventional bond until the longest maturity
bond has been used. This fixes the longest spot rate at the maturity of the
longest bond.
Plot the spot rates R(T) against T

to arrive at the zero coupon yield curve.

(ii)

This is the coupon rate that the bond would be required to make the theoretical
value of the bond equal to its nominal value under the prevailing pattern of
zero coupon interest rates.

(iii)

S1: 102.01 * (1 + S1) = 100 + 6.75

S1 = 4.65%

No price given for 2 year gilt so need to calculate:


P2: P2 * (1 + 0.049)2 = 9.5 * (1 + 0.049) + 100 + 9.5

P2 = 108.56

Now solve for S2


108.56 * (1 + S2)2 = 9.5 * (1 + 1f1) + 109.5 where 1f1 is the 1 year forward
rate starting in 1 years time
Page 5

Subject ST5 (Finance and Investment Specialist Technical A)

April 2005

Examiners Report

(1 + S2)2 = (1 + S1) * (1 + 1f1)


S2 = 4.91%
S3: 110.41 * (1 + S3)3 = 7.75 (1 + 2f1)2 + 7.75 * (1 + 1f2) + 107.75
(1 + 2f1)2 = (1 + S3)3 / (1 + S1)

(1 + 1f2) = (1 + S3)3 / (1 + S2)2

S3 = 3.94%
If continuous rates used the answers are S1=4.54%, S2=4.80% and S3=3.86%.
P2=108.32(using continuous rates) is not correct but we do not penalise in other calculations.

(i)

Contributions occur halfway through the period.


Generalised formula for return (each class):
MV(1) = MV(0)*(1+i) + C*(1+i/2) using the normal approximation for a half
year s interest.
[MV(1)-MV(0) - C] / [MV(0) + C/2] = Return (%)
Equity return = (700-600+12)/[600

12/2] = 18.86%

Bond return = (450-350-63)/[350 + 63/2] = 9.70%


Cash return = (50-50+1)/[50 1/2] = 2.02%
Fund total return = (1,200-1000-50)/[1,000 + 50/2] = 14.63%
Bond Index return = 1,299.3/1,220

1 = 6.50%

Equity Index return = 1,115/1,000

1 = 11.50%

Benchmark return = [50 * 1.065 + 50 * 1.1150]/100

1 = 9.0%

In allocating performance the fund weights should be the average weights


based on the denominator of the fund returns but initial market values used as
weights would not be marked as wrong
Equity Asset Contribution = (594/1,025
= 0.20
Bond Asset Contribution = 0.32

Page 6

0.50) * (11.50

9.0)

Subject ST5 (Finance and Investment Specialist Technical A)

April 2005

Examiners Report

Cash Asset Contribution = 0.34 (as it is not part of the index assume return =
actual, although 3.5 to 4.0 could be used and then a stock contribution would
have to be calculated)
Equity Stock Contribution = 594/1,025 * (18.86

11.50) = 4.27%

Bond Stock Contribution = 381.5/1,025 * (9.70

6.50) = 1.19%

The sum of the parts = 5.64% compared with actual 5.63% due to rounding
error.

(ii)

Asset allocation positive in both bond and equity decisions but holding cash a
negative.
Stock selection very positive for both bonds and equities.
Yield on equity investments was 15/594 = 2.53% compared with 3.12% for
index so investment strategy in equities is capital growth orientated or the
timing of purchases and sales was such that a full year of dividend income has
not been received.
Yield on bonds was 25/381.5 = 6.55% compared with 4.5% [ difference
between bond index and total return index] for index. This suggests a
portfolio away from the index possibly in lower quality corporate bonds or
emerging market debt.
Cash return very poor given base rates.
Fund manager should be asked to comment on the strategy.
Strategy should be compared with mandate.

(i)

Behavioural Finance looks at the mental biases and decision-making errors


that affect financial decisions
The following challenge the view of the judge:
Anchoring past experience adjusted to allow for evident differences to
current conditions herd instinct
Prospect Theory how decisions are made when faced with risk and
uncertainty
How questions are framed, especially if of a structured response variety
How risk aversion changes as time-frame or number of opportunities increases
Dislike of negative events influences perceived probability of outcome
Ease of imagining apparent likelihood increases with detail

Page 7

Subject ST5 (Finance and Investment Specialist Technical A)

April 2005

Examiners Report

Ease of bringing to mind


Overconfidence hindsight bias
- confirmation bias
mental account count individual gains/losses rather than netting them
primary/recency effect first or last option to be presented more influential
range of options effect more choice means less decision
status quo preference to keep things as they are
regret effect don t do anything in order to create no regret
ambiguity effect premium for rules

(ii)

Suitable examples/conclusions include:


Recent past performance.
The what, not the how, and subject to manipulation.
Size and reputation.
No one was sacked for buying IBM.
Image.
Advertising, familiarity with brand.
Client list.
Assume the others do the due diligence.

(i)

S&M (quoted + 0.75% over sterling government bonds) has a poorer credit
rating than BIM (quoted +0.50% over US$ government bonds) as evidenced
by the spread over corresponding five-year government rates.
To avoid any exchange rate risk on the exchange of interest rate payments,
S&M will need to borrow at the five-year fixed rate of 6% per annum in
sterling and receive payments at a rate of 6% per annum fixed for five years
from the global investment bank as part of the swap design.
To avoid any exchange rate risk on the exchange of interest rate payments,
BIM will need to borrow at the five-year fixed rate of 5.25% per annum in
US$ and receive payments at a rate of 5.25% per annum fixed for five years
from the global investment bank as part of the swap design.
The difference between the US$ payments by S&M to the global investment
bank and the sterling payments by BIM to the global investment bank provides
the margin for the global investment bank.
However, the global investment bank will probably want to charge a higher
rate of interest to S&M than to BIM to reflect the poorer credit rating of the
former.

(ii)

Page 8

In arriving at its fee the global investment bank would probably wish to tilt the
charges to BIM and S&M to reflect their relative credit ratings. Thus the

Subject ST5 (Finance and Investment Specialist Technical A)

April 2005

Examiners Report

global investment bank may wish to charge S&M a somewhat wider margin
than BIM.
One possibility would be to charge S&M a US$ five-year fixed rate of 5.80%
(1.05% over five-year US$ government bonds) and BIM a sterling five-year
fixed rate of 5.90% (0.95% over five-year sterling government bonds).
(iii)

The global investment bank is left with a residual foreign exchange risk on
each exchange of interest payments between the two parties.
This risk could be hedged by forward foreign exchange contracts.
The global investment bank is also left with credit risk.
Credit risk could be hedged using credit derivatives.

Page 9

Faculty of Actuaries

Institute of Actuaries

EXAMINATION
13 September 2005 (pm)

Subject ST5
Finance and Investment
Specialist Technical A
Time allowed: Three hours
INSTRUCTIONS TO THE CANDIDATE
1.

Enter all the candidate and examination details as requested on the front of your answer
booklet.

2.

You have 15 minutes at the start of the examination in which to read the questions.
You are strongly encouraged to use this time for reading only, but notes may be made.
You then have three hours to complete the paper.

3.

You must not start writing your answers in the booklet until instructed to do so by the
supervisor.

4.

Mark allocations are shown in brackets.

5.

Attempt all 9 questions, beginning your answer to each question on a separate sheet.

6.

Candidates should show calculations where this is appropriate.

AT THE END OF THE EXAMINATION


Hand in BOTH your answer booklet, with any additional sheets firmly attached, and this
question paper.
In addition to this paper you should have available the 2002 edition of the
Formulae and Tables and your own electronic calculator.

ST5 S2005

Faculty of Actuaries
Institute of Actuaries

Define the role of a custodian, and list the services that they might offer in addition to
document safe keeping.
[4]
[Total 4]

(i)

Explain which of the two types of bond portfolio switches are more likely to
be carried out by the following type of investor:
(a)

A liability driven investor, investing in bonds to match liabilities of a


particular duration.

(b)

An unconstrained investor, who is investing in bonds to maximise


returns relative to LIBOR.
[2]

(ii)

Explain why an analysis of reinvestment rates might mean an investor favours


investment in a 10 year bond with a 5% coupon over investing in a 20 year
bond with a 10% coupon, when the yield on the longer bond is 0.5% p.a.
higher.
[2]

(iii)

Set out the processes involved in assessing whether or not there is a potential
yield difference between a 10 year AA rated corporate bond and a 10 year
government bond that can be exploited.
[4]
[Total 8]

(i)

Describe the critical difference between a mortgage backed security and a


regular fixed income security.
[3]

(ii)

A collateralised loan obligation (CLO) is backed by a pool of residential


mortgages and divided into three investment classes W, X and Y. Scheduled
and pre-payment principal repayments are channelled to class W investors
until that group of investors has been completely repaid. Thereafter,
scheduled and pre-payment principal repayments are channelled to class X
investors until that group of investors has been completely repaid. When class
W and class X investors have been fully repaid, all remaining principal
repayments are directed to class Y investors.

ST5 S2005

(a)

State, with reasons, which class of investor bears the most pre-payment
risk.

(b)

Rank in order of increasing duration the three investment classes and


give a reason for your answer.

(c)

Describe, using a simple example, how the par value of the three
classes influences the pre-payment risk of class Y investors.
[5]
[Total 8]

(i)

Describe the main aims of the field of Behavioural Finance as applied to


investment management.

[4]

(ii)

Explain how Behavioral Finance arguments might be used by an equity


investment manager to explain why they hold a persistently underperforming
stock in a portfolio rather than selling out of the position and re-investing in an
alternative stock.
[5]

(iii)

Outline how the irrational actions suggested by Behavioral Finance might be


eliminated.
[3]
[Total 12]

(i)

Describe, with reasons, the types of merger that a UK based retail bank might
consider with a similar sized organisation operating in the UK.
[8]

(ii)

Describe the other factors that would need to be considered if the proposed
merger was to be with an organisation domiciled in another EU country. [6]
[Total 14]

The trustees of a pension fund have appointed an investment manager to invest in


global equities and bonds. The investment manager s performance will be compared
with a benchmark equally divided between two representative broad market indices.
The trustees wish to impose limits on the investment manager s ability to deviate
from the 50/50 proportions to which the benchmark will rebalance on a quarterly
basis.
(i)

Explain why the investment manager might actively deviate from the
benchmark asset allocation on a short-term basis.

[4]

(ii)

List the factors that should be considered by the trustees in setting the limits.
[3]

(iii)

Outline appropriate constraints on these factors.

ST5 S2005

[5]
[Total 12]

PLEASE TURN OVER

You are the investment manager of a life assurance company that has substantial
assets under management and invests a significant proportion of these assets in
alternative investments and derivative-type structures.
A sales person from an investment bank approaches you regarding the purchase of a
complex derivative product.

(i)

List the key questions regarding the derivative product that you would ask the
sales person.
[5]

(ii)

Describe the factors which you would consider in assessing the characteristics
of the derivative in terms of its fit with the life assurance company s
investment portfolios.
[5]
[Total 10]

(i)

Explain why equities are usually analysed in sector or industry groupings. [5]

(ii)

(a)

Discuss the advantages and disadvantages of this sub division.

(b)

Suggest, with reasons, two possible alternative groupings.


[5]

(iii)

State the features that characterise each of the following economic groups:
General Industries
Consumer Goods
Utilities

[6]
[Total 16]

You are an investment consultant to the trustees of a pension scheme. You are given
the following total return data for the fund and the indices included in the benchmark.

UK Equities
Overseas Equities
UK Bonds
Overseas Bonds

Fund Returns
Year 1 Year 2

Year 3

Index Returns
Year 1 Year 2

Year 3

10.1%
14.2%
6.4%
3.2%

17.6%
13.2%
4.1%
1.5%

8.5%
13.5%
7.2%
8.1%

16.0%
1.0%
5.4%
3.1%

2.5%
3.2%
3.1%
1.8%

5.4%
2.1%
4.5%
2.8%

The asset mix of the fund and of the benchmark at the start of year 1 was as follows:

UK Equities
Overseas Equities
UK Bonds
Overseas Bonds

ST5 S2005

Fund

Benchmark

45%
30%
15%
10%

60%
20%
10%
10%

You may assume that there is no rebalancing at any time. No contributions were paid
in and no benefits were paid out during the period.
(i)

Describe the principal sources of deviation between a fund and its benchmark.
[2]

(ii)

Calculate:
(a)
(b)
(c)

the total return for the fund


the benchmark return
the respective contributions from the items in (i)
[10]

(iii)

Comment on the results of your calculations.

END OF PAPER

ST5 S2005

[4]
[Total 16]

Faculty of Actuaries

Institute of Actuaries

EXAMINATION
September 2005

Subject ST5 Finance and Investment


Specialist Technical A
EXAMINERS REPORT

Faculty of Actuaries
Institute of Actuaries

Subject ST5 (Finance and Investment Specialist Technical A)

Sept 2005

Examiners Report

A custodian ensures that financial instruments are housed under a proper system that
permits investment for proper purposes with proper authority.
The following services may also be provided: income collection, tax recovery, cash
management, securities settlement, foreign exchange, stock lending.

(i)

(a)

Anomaly switches, as these are less likely to alter the duration match
of the liabilities.

(b)

Policy switches, as these allow the investor the freedom to attempt to


maximise return based on the investor s view of future changes in the
level and shape of the yield curve.

(ii)

The investor may take the view that despite the 0.5% pa higher yield on the 20
year bond, the need for a greater level of reinvestment during the bond s term
means that the additional return is insufficient compensation for the risk that
reinvestment terms might worsen. This is particularly likely if yields are
currently considered to be high relative to historical levels.

(iii)

First an estimate of the risk premium for the AA rated bond will be needed,
with a view to how this might change over the length of the anomaly switch.
This view will take into account default risk and other factors affecting the
yield such as lack of liquidity, coupon and any tax differences.
This will be used to estimate the additional yield that could be obtained
through changes in the risk premium relative to benchmark government bond
yields.
An allowance should be made for transaction costs at both ends of the switch.
Based on the additional yield, a decision as to whether to proceed with the
switch can be made.

(i)

The borrowers underlying the mortgages in a mortgage backed security have a


right to (but are under no obligation to) repay the mortgage to the lender
at any time during the life of the mortgage at its face value.
This introduces pre-payment risk for the holders of mortgage backed
securities.

Page 2

Subject ST5 (Finance and Investment Specialist Technical A)

(ii)

(a)

Sept 2005

Examiners Report

The W investment class.


All principal repayments go in the first instance to the class W
investors; a rush of prepayments will impact this class of investors the
most.

(i)

(b)

W, X and Y. Class W will be repaid first because of the structure of


the CLO and is likely to have the shortest duration followed in order
by class X and class Y.

(c)

Class Y investors bear very little pre-payment risk if the ratio of the
par value of the securities is 5:2:1 but class Y investors bear significant
pre-payment risk if the ratio of the par values is reversed to read 1:2:5.

BF is based on the idea that a variety of mental biases and decision making
errors affect financial decisions.
Central to BF is the psychology of how and why financial decisions are made.
Analysis within this field is believed to have some predictive qualities and the
findings used to help the proponents in their own decision making.

(ii)

Analysts and investment managers can be unreasonably drawn to a stock


based on understanding and belief in the company s business model and/or
management. This affection can lead them to persistently discount bad
news about the stock.
Investment managers can be highly confident in their own abilities. Over
confidence can lead the practice of managers using good news on a stock to
reinforce their position whilst putting any bad news down to the ignorance of
other market participants.
Managers will be reticent to take responsibility and crystallize a loss having
defended the position to date.

(iii)

Institute a level of discipline in the investment process to reduce bias without


stifling invention and ideas.
Hold periodic peer reviews of the portfolio/decisions.
Create a culture, which actively encourages challenge and debate.
Introduce hard code rules and risk controls to take out some of the human
element.

Page 3

Subject ST5 (Finance and Investment Specialist Technical A)

Sept 2005

Examiners Report

Needs to cover:
(i)

Description of all the types of merger and the reasons why they are instigated
(and why not) as per Unit 8 section 4.

(ii)

Barriers to European integration as per Unit 10, section 9.

(i)

Aim is to outperform benchmark


To outperform you need to be different
Extent of difference will be controlled to minimise risk of underperformance
(and potential termination of mandate)
Justified because:
Markets and asset classes are not perfectly correlated
Asset class and country returns may be predictable because:
Valuations can drift away from fair value
Investors may be slow to incorporate new information
Risk premiums change over time among global markets
Structural barriers exist across global markets
Some market participants (e.g. central banks) may not be motivated by
profits

(ii)

Performance target
Style of management
Correlations between assets
Cash flows and income
Costs of rebalancing
Rebalancing frequency
Scope to use derivatives
Risk Tolerance

(iii)

Not all factors will have limits placed on them as they are not manageable e.g.
cash flows and income but they will be taken into account when considering
other limits i.e. rebalancing to ensure efficient management.
Performance target limit the under performance by more than a set
percentage over a defined period, out performance also needs to be reviewed
(too high risk?).
Style of management

permit only a limited percentage in non-style assets.

Correlations between assets


particular magnitude.

Page 4

monitor for increases or decreases of a

Subject ST5 (Finance and Investment Specialist Technical A)

Costs of rebalancing

Sept 2005

Examiners Report

set a limit on management costs.

Rebalancing frequency place a limit on asset allocation shifts other than


due to cash flow or income.
Scope to use derivatives
of assets.

limit exposure to derivatives to certain percentage

Risk tolerance the funding level, corporate sponsor s financial status and
asset class risk/return expectations will all have different influences at
different times depending on how the risk of each is viewed. e.g. a well
funded scheme with a large surplus may be prepared to consider wider
parameters than one which is less well funded when the company sponsor is
financially sound.

(i)

What market, instrument or other underlying asset drives the performance of


the derivative?
What is the formula for the magnitude of the cash flows likely to arise under
the derivative contract?
What is the timing of those cash flows?
What is the likelihood of receipt or payment of the cash flows?
What is the liquidity of the contract?

(ii)

The factors would be:


return expected what is it going to contribute to overall returns
taxation are there any implications for returns or other aspects of the
portfolio
applicable regulatory valuation rules
what rules may apply and will
they cause complications
accounting treatment of the derivative
is it treated as capital or are
there income aspects that need to be allowed for
integration with the existing portfolio
does it make strategic sense
its effect on risk
what does it do to the overall portfolio s position

(i)

Grouping by industry
To reduce the number of factors that have to be taken into account when
analysing the share.
Quite a lot of industry statistics are available and are usually grouped by
industry.

Page 5

Subject ST5 (Finance and Investment Specialist Technical A)

Sept 2005

Examiners Report

Methods of raising finance are often similar within a group.


Experience similar labour problems, costs of raw materials, consumer demand,
economy will affect them similarly.
Financial statements are prepared in a similar fashion and will use similar
jargon.
No other factor has proven to correlate to shares as closely as industry
statistics.
(ii)

Disadvantages
Tend not to look at companies between sectors, only companies within
sectors.
Some shares don t move with their industry.
Advantages
Can become expert in one sector and understand it very well.
We can decide which factors affect a share price within an industry, then
analyse companies with this in mind.
Could group by:
Large cap/small cap
valuations tend to reflect growth/maturity/financial
strengths differently for large and small cap stocks.
Growth/Value
reflects the different universes and economic/market drivers
operating on these companies.
Exporters/Importers
reflects earnings from domestic or overseas influences
and impact of different regions economic growths and currencies.
Any two sensible suggestions should earn marks.

(iii)

Page 6

See Core reading Unit 3 pages 2 & 3.

Subject ST5 (Finance and Investment Specialist Technical A)

(i)

Sept 2005

Examiners Report

The overall investment performance of a fund can be divided into:


Sector or asset class selection: the extent to which the fund s proportions
in the various sectors will have affected performance.
Stock selection: within any one sector, have the selected stocks performed
better or worse than the sector as a whole?

(ii)

Fund Benchmark Fund Ret Index Ret


UK Eq
O seas Eq
UK Bond
O seas Bond
Total

45%
30%
15%
10%
100%

60%
20%
10%
10%
100%

26.24%
25.14%
14.20%
6.63%
22.14%

19.06%
17.04%
18.07%
14.57%
18.11%

Asset Class
Selection
0.14%
0.11%
0.00%
0.00%
0.25%

Stock
Selection
3.23%
2.43%
0.58%
0.79%
4.29%

Total

3.09%
2.32%
0.58%
0.79%
4.04%

There is a rounding error of 0.01%.


(iii)

The fund has out performed by 4.03%. Asset allocation was poor due to under
weighting UK equities and over weighting overseas equities. Equity stock
selection in both markets was very good whilst bond selection was poor in
both markets. Bond performance was consistently poor for both classes in
each year. UK equity performance was consistently good but overseas equity
performance was volatile with the third year accounting for more than the
stock selection added value in total.

END OF EXAMINERS REPORT

Page 7

Faculty of Actuaries

Institute of Actuaries

EXAMINATION
30 March 2006 (pm)

Subject ST5 Finance and Investment


Specialist Technical A
Time allowed: Three hours
INSTRUCTIONS TO THE CANDIDATE
1.

Enter all the candidate and examination details as requested on the front of your answer
booklet.

2.

You have 15 minutes at the start of the examination in which to read the questions.
You are strongly encouraged to use this time for reading only, but notes may be made.
You then have three hours to complete the paper.

3.

You must not start writing your answers in the booklet until instructed to do so by the
supervisor.

4.

Mark allocations are shown in brackets.

5.

Attempt all 7 questions, beginning your answer to each question on a separate sheet.

6.

Candidates should show calculations where this is appropriate.

AT THE END OF THE EXAMINATION


Hand in BOTH your answer booklet, with any additional sheets firmly attached, and this
question paper.
In addition to this paper you should have available the 2002 edition of the
Formulae and Tables and your own electronic calculator.

ST5 A2006

Faculty of Actuaries
Institute of Actuaries

You are a trustee to a mature but underfunded retirement benefit scheme which has
investments in a wide range of properties. Discuss the appropriateness of passive
management of the property assets within the fund.
[8]

In relation to portfolio construction:


(a)

Define prospective and retrospective tracking errors.

(b)

Discuss the use of prospective and retrospective tracking errors as measures of


risk.

(c)

State why the two measures of risk might differ.


[9]

You are advising the government of a country on how to set up a tax system. The
chief minister wants to tax investment income at a higher rate than earned income,
and to exempt capital gains from tax. His aims are to encourage entrepreneurial
activity and investment and to redistribute wealth from the rich to the poor. Explain
the flaws and unintended consequences of his proposed approach.
[8]

(i)

(ii)

Outline the key principles underlying the relevant legislation relating to


providers of financial services and their clients.

[7]

Describe the points that should be covered by a Statement of Investment


Principles .
[4]
[Total 11]

The following information is provided on a non-UK equity fund.


US
Value at 31.12.04 m.
Value at 31.12.05 m.
Income in period m.
Net New Investment in period m.
Local Index 31.12.04
Local Index 31.12.05
Currency Rate at 31.12.04 v
Currency Rate at 31.12.05 v
Index Yield at 31.12.05 (%)

250
300
5
-10
100
120
1.90
1.75
2.5

Japan
100
140
2
-5
325
400
190
200
1.0

Asset
Europe Asia
100
96
3
10
200
185
1.40
1.50
2.0

50
70
4
0
210
296
1.90
1.75
3.0

Cash

Total

0
5
0
5
100
100
1.00
1.00
4.0

500
611
14
0

The benchmark index is weighted 50% U.S., 30% Europe, 15% Japan and 5% Asia.

ST5 A2006 2

(i)

Evaluate the performance of the fund through a full attribution analysis by


market stating any assumptions that you make.
[15]

(ii)

(a)

Comment on the returns and the results.

(b)

Suggest any investigations that you might undertake to better


understand the results.
[5]
[Total 20]

A fund manager operates an OEIC whose investment strategy is to identify pairs of


shares in the same industry or market sector one of which is forecast to rise in value
while the other is forecast to fall in value over a one-year time period. The manager
buys the shares that he forecasts will rise in value. He borrows the shares that he
expects will fall in value, sells them in the market hoping to buy them back later at a
lower price, and then return the borrowed shares to the lender.
One of the current pairs of shares that the manager has in his portfolio is
from the pharmaceutical sector. He expects PharmaUP to rise in value over the
next year and he expects PharmaDOWN to fall in value over the next year.
(i)

Discuss the risks in this strategy under the following headings:


(a)

The pairing of two shares in the same industry or sector and identifying
one as expected to rise in value while the other as expected to fall in
value.
[6]

(b)

Borrowing shares and selling them in the market hoping to buy them
back at a lower price.
[5]

(c)

The costs of implementing the investment strategy.

(d)

The impact of the majority of investors redeeming their holdings in the


fund on the shareholders who do not wish to redeem their holdings at
some point before the one-year investment time horizon has expired.
[4]

[4]

(ii)

Describe how the manager might implement his pairing strategy to eliminate
some of the risks in (i) (a).
[2]

(iii)

How would the manager reduce the risk to the investors who do not redeem
their holdings in (i) (d).
[1]
[Total 22]

ST5 A2006 3

PLEASE TURN OVER

A mobile phone company requires additional financing and its treasury team has
noticed that it has an asset in the form of future payments by subscribers who are
required to make payments under their monthly contracts prior to the minimum period
ending. The team has approached your bank in order to set up an asset backed
security (ABS) issue via a special purpose vehicle (SPV). The company will transfer
the asset to the SPV in return for a payment of 500m, and the SPV will then issue an
ABS in 3 tranches as shown below, backed by these assets. The company will
purchase the equity tranche from the SPV.
Tranche
Senior
Mezzanine
Equity

Credit rating

Tranche size

AA
BB
n/a

400m
50m
50m

(i)

Explain why the company might wish to use its asset in such a way.

(ii)

Describe the tranched structure, including why there is an equity tranche, and
why the company might wish to purchase it.
[6]

(iii)

Explain the key risks to a purchaser of the mezzanine tranche, and describe
situations when these risks might result in losses.
[5]

(iv)

Comment on how your answer to (iii) would change if the asset value was
based on expected contract receipts for all existing contracts over the next 5
years, including payments after the minimum contract period expires.
[5]
[Total 22]

END OF PAPER

ST5 A2006 4

[6]

Faculty of Actuaries

Institute of Actuaries

EXAMINATION
April 2006

Subject ST5
Finance and Investment
Specialist Technical A
EXAMINERS REPORT
Introduction
The attached subject report has been written by the Principal Examiner with the aim of
helping candidates. The questions and comments are based around Core Reading as the
interpretation of the syllabus to which the examiners are working. They have however given
credit for any alternative approach or interpretation which they consider to be reasonable.

M Flaherty
Chairman of the Board of Examiners
June 2006

Faculty of Actuaries
Institute of Actuaries

Subject ST5 (Finance and Investment Specialist Technical A)

April 2006

Examiners Report

Comments
The solutions should not be taken as comprehensive. There are a number of additional points
that can be made in certain questions and these were awarded appropriate marks. There are
also a number of different solutions that can be derived for question 5 and these were also
awarded appropriate marks despite the fact that it would be unusual to see them in practice.
However alternative solutions tended not to give the same degree of information as those
shown here and consequently marks in subsequent sections of the question were lost.
In general candidates did bookwork well but failed to carry this through in the application
parts of questions and only the better candidates scored anything like reasonable marks in
higher skills parts. Roughly half the paper relates to application and candidates often gave
us bookwork rather than applying their knowledge to the problem in hand. Questions 2, 3, 6
and 7 were good examples of this with candidates on average scoring under 50% of the
marks available. Question 4 was well done by most candidates being bookwork. Question 1,
despite being bookwork, was not as well done as other bookwork sections possibly because of
the context in which it was framed. As has been the case in the past we were disappointed
with answers to question 5 despite the fact that arithmetical errors are not penalised because
we are more concerned that candidates understand the methods and assumptions that they
are using. In particular candidates show too little working, fail to use all the information
that is provided and do not think widely enough. Consequently they did not score well in the
second part of the question.

Page 2

Subject ST5 (Finance and Investment Specialist Technical A)

April 2006

Examiners Report

Passive management aims to track the performance of a specified benchmark index


with no active investment risk.
This approach limits downside risk of poor manager performance.
It also removes scope to benefit from good manager performance.
An under funded scheme may find active management more appealing as a means to
catch up some lost capital.
Fees are generally low however this generalised point is not likely to be true for
property where transaction and tax costs are significant.
Does a suitable index exist covering the wide range of properties held?
Property indices have numerous problems
availability of prices.

non homogeneity, timing, frequency and

It may be possible to gain better matching of an index through a derivative


instrument, but note that derivatives are often based on an investable index.
Active management aims to provide a return in excess of a specified benchmark by
taking investment risk within the portfolio.
It is difficult to correctly identify managers who will consistently outperform.
Fees are generally higher than for passive management.

Solution not split into (a) (c) as answers unlikely to conform to split of question.
Prospective tracking error is an estimation of the scale of volatility of prospective
relative performance given the current portfolio and benchmark.
It is usually expressed in a number of basis points per year.
It provides an estimate of the aggregate amount of investment risk within a portfolio
at the time it is calculated.
It is calculated using a quantitative model and depends on assumptions including: the
likely future volatility of individual stocks and markets relative to the benchmark and
correlations between different stocks and markets.
Tracking errors make no distinction between upside risk and downside risk.
In this regard it may not fit well with the trustees attitude to risk which will likely be
skewed to seek positive performance with a high probability whilst minimising the
probability of a significant negative performance.

Page 3

Subject ST5 (Finance and Investment Specialist Technical A)

April 2006

Examiners Report

Retrospective tracking error is defined as the standard deviation of realised past


annualised relative performance.
It is also usually expressed in a number of basis points per year.
It seeks to provide an ex post summary measure of investment risk based on the
volatility in the monthly (or quarterly) pattern of realised returns.
Prospective and retrospective tracking error will differ because:
Prospective assumes a static and unchanged portfolio and benchmark.
It also relies on many assumptions that are unlikely to be achieved in practice.
Whereas retrospective tracking error is based on the actual portfolio and on the
performance of that portfolio against the benchmark

By exempting capital gains from tax and taxing investment income at a higher rate
than earned income, capital gains will become the most tax efficient method of wealth
creation and investment income will become the least tax efficient method of gaining
wealth.
Potentially this will encourage companies to retain profits rather than distribute them,
leading to higher levels of corporate investment
but it may also reduce the average rate of return for new investment opportunities
as opportunities that would previously have been ignored will now be developed.
The higher level of corporate investment will not necessarily increase the overall
growth rate of the economy as there will be less distributed income reinvested by
companies and individuals.
Individuals will be encouraged to pay themselves in the form of capital gains rather
than income, if they are able to do so, to minimise their tax liability.
This may lead to the creation of schemes that convert income into capital gains, which
across the economy as a whole are unlikely to result in any net wealth creation.
It is worth noting that wealthier individuals will typically be least dependent on earned
income, and will therefore be most able to structure their affairs to minimise their tax
liabilities.
Conversely poorer individuals are unlikely to have such flexibility and will therefore
pay more tax than if they were remunerated through capital gains.
Overseas investors are similarly likely to be able to structure their affairs in a way that
minimises tax.

Page 4

Subject ST5 (Finance and Investment Specialist Technical A)

April 2006

Examiners Report

Attempts to prevent this (e.g. withholding taxes) may inadvertently result in the
country s assets commanding a lower purchase price, impacting on the wealth of the
nation as and when domestic assets are sold to overseas investors.

(i)

The key principles are:


Integrity

high standards and fair dealing.

Skill, care and diligence.


Market practice

observe proper market conduct.

Information about customers


objectives.
Information for customers
dealings.
Conflicts of interest
disclosure to client.
Customer assets
assets.

comprehensive and timely information in all

should be avoided where possible, but otherwise full

proper safeguard, segregation and management of all

Financial resources
risks.

firm must have sufficient assets to meet its business

Internal organisation
keeping.
Relations with regulator
(ii)

details of circumstances and investment

properly trained and supervised staff, proper record

open and co-operative manner.

A Statement of Investment Principles should set out:


Who is taking what decisions and why this structure has been selected.
Fund s investment objective.
Fund s planned asset allocation strategy, asset class projected returns and how
strategy has been arrived at.
What mandates have been given to all advisers and managers.
Nature of fees structures for all advisers and managers, reason behind why set
of structures selected.

Page 5

Subject ST5 (Finance and Investment Specialist Technical A)

(i)

US
Japan
Europe
Asia
Cash
Total
(ii)

April 2006

Examiners Report

Assumption: net new investment occurs in middle of period. Simplification


(1+i/2) allowed. [Whilst candidates were not asked for formulae, it would be
good practice to write out what formulae were used as this shows how the
table of results is arrived at. Since no marks were being awarded for
formulae, the report does not specify them.]
Fund Rtn Bmark Rtn
24.49%
30.29%
46.15%
16.92%
-13.33% -13.67%
40.00%
53.03%
0.00%
0.00%
22.81%
16.23%
(a)

Curr Rtn
8.57%
-5.00%
-6.67%
8.57%
0.00%
1.96%

Alloc
0.00
0.03
2.99
1.84
0.00
4.86

Curr Stock Alt Rtn


0.00 -2.90
0.24
-0.35 5.85
0.45
-0.14
0.86 0.07
0.33 -1.30
0.4
0.00 0.00
0
0.85 1.71 22.20%

Stock
-3.14
5.62
-0.07
-1.30
0.00
1.10

Fund outperformed by 6.58%/5.97%.


Stock selection added 1.71/1. 1%.
Asset allocation added 4.86%.
Currency added 0.85% of the 4.86%.
Under weight Europe and over weight Asia were positive asset
allocation contributions.
Good stock selection for Japan, offset by poor US and Asia stock
selection.

(b)

Income needs investigating as looks like growth stocks in US but value


stocks in rest of world
check style.
Check if any currency hedging in place as Japan return very high vis a
vis benchmark

(i)

(a)

The manager s valuation model may prove incorrect and if it does


investors will lose money on the transaction as the share bought falls in
value while the share shorted rises in value.
The manager takes on a very high degree of stock specific risk with
this strategy.
In particular, PharmaUP may suffer a significant fall in price due to a
factor that is specific to PharmaUP.
For example, an announcement that the Federal Drug Administration
in the US is withdrawing its approval for one of the drugs that
contributes significantly to PharmaUP s profits as a result of the deaths

Page 6

Subject ST5 (Finance and Investment Specialist Technical A)

April 2006

Examiners Report

of several individuals using the drug while the majority of other


pharmaceutical companies may see their share price generally rise.
(b)

There is an assumption in the strategy that the manager will be able to


borrow the shares for a period of one year and this may not be possible
in practice.
If the lender of the shares recalls the loan of the shares the manager
may have to unwind his strategy at a point in time when the share
bought has fallen in value while the share shorted has risen in value
forcing investors to realise a loss

(c)

The strategy will not be profitable unless the difference between the
rise in value of PharmaUP and the fall in value of PharmaDOWN is
sufficient to cover all the following costs:
Bid-Offer spreads
any impact on market depth
stamp duty and other levies
commission/brokerage fees
The manager will have to fund the cost of any dividends arising on the
shares borrowed and this adds to the minimum return necessary to
make the strategy profitable.
These costs will be offset to some extent on the interest income arising
on the cash generated by shorting the shares.

(d)

(ii)

If some investors redeem their shares during the one-year holding


period at a point where the value of PharmaUP is less than its value on
the date the transaction is initiated
or the value of PhamaDOWN is above its value on the date the
transaction is initiated
this may distort the relationship between the pair of stocks and make
the entire trade unprofitable
if the price of PharmaDOWN rises unexpectedly on the forced buying
in operation
and the price of PharmaUP falls unexpected from the forced sale of
shares.

To reduce stock specific risk, the manager might buy in a sector of the market
he forecasts will outperform over the next year and go short in a sector of the
market that will under perform in the same period.
This could be achieved using exchange traded funds.

(iii)

The manager could insist on a minimum investment term of one-year so as not


to interrupt investment strategies with redemptions.

Page 7

Subject ST5 (Finance and Investment Specialist Technical A)

(i)

April 2006

Examiners Report

The asset is being used as collateral to back a loan issue.


The asset is intended to provide higher security than an unsecured loan issued
by the company.
As the ABS is issued through a SPV, purchasers are protected against other
claims made against the company.
These factors will provide lenders (purchasers of the ABS) with more
certainty of repayment than purchasers of an unsecured loan stock, reducing
the company s cost of capital.
The yield on the ABS will therefore be lower than the yield on an unsecured
loan stock of similar term and structure.
Furthermore, it may not be possible for the company to borrow funds in the
market at a realistic rate (e.g. the issuer credit rating is weak).
Using the asset in this way would potentially lead to a more efficient balance
sheet structure, as the asset is illiquid and not receiving any investment return.
However, this must be offset against the transaction costs of the ABS issue,
and the issue may have negative implications for the company s credit
rating as it will reduce asset and income cover for existing unsecured
creditors.

(ii)

The equity tranche has been created to protect purchasers of higher tranches
against the value of the asset being lower than calculated in the prospectus.
Defaults in the customer contracts would first be set against the equity tranche,
then the mezzanine tranche and finally the senior tranche.
The default risks are reflected in the credit ratings, and the equity tranche is
unrated.
The ABS has been structured in this way to reduce the cost of borrowing.
The spread (additional yield) increases steeply as the credit rating declines.
The company may wish to purchase the equity tranche if it is confident that
the asset has been conservatively valued,
and therefore does not want to give up excess returns to other parties.
The nominal return on the equity tranche is likely to be well over 10%, and
therefore higher than the company s average (and possibly marginal) cost of
capital.
In practice it may be difficult to market the equity tranche as contract and
customer retention is to some extent under the control of the company, and

Page 8

Subject ST5 (Finance and Investment Specialist Technical A)

April 2006

Examiners Report

some level of company participation in the issue would be needed to avoid


moral hazard.
(iii)

There is some switching risk, although this should be low as subscribers will
have a penalty if they terminate contracts prior to the minimum period ending.
If the nature of the marketplace changes then this may result in losses.
There is some default risk if subscribers are unable to meet their remaining
contract payments. This risk would be higher if the contract automatically
terminates in certain situations (e.g. redundancy) and losses are likely to
increase in an economic downturn.
Most of the risk is likely to be restricted to the equity tranche.

(iv)

The key factor determining whether a loss will occur is how actual switches
and defaults compare to what has been assumed in the asset valuation.
The switching risk would increase materially as there will be a high expected
number of switches once the minimum contract period ends and the number of
switches is likely to remain at a moderate level for the duration of the 5 year
period
The default risk will also be higher although this is likely to remain broadly
stable over the 5 year period.
A possible exception to this pattern might be if subscribers with lower default
risk were cherry picked by a competitor.
In contrast to the previous scenario, it is likely that the mezzanine tranche will
have significant exposure to these risks, in addition to purchasers of the equity
tranche.
These additional risks would, all other things being equal, result in a higher
spread on the ABS particularly on the mezzanine tranche.

END OF EXAMINERS REPORT

Page 9

Faculty of Actuaries

Institute of Actuaries

EXAMINATION
7 September 2006 (pm)

Subject ST5
Finance and Investment
Specialist Technical A
Time allowed: Three hours
INSTRUCTIONS TO THE CANDIDATE
1.

Enter all the candidate and examination details as requested on the front of your answer
booklet.

2.

You have 15 minutes at the start of the examination in which to read the questions.
You are strongly encouraged to use this time for reading only, but notes may be made.
You then have three hours to complete the paper.

3.

You must not start writing your answers in the booklet until instructed to do so by the
supervisor.

4.

Mark allocations are shown in brackets.

5.

Attempt all 7 questions, beginning your answer to each question on a separate sheet.

6.

Candidates should show calculations where this is appropriate.

AT THE END OF THE EXAMINATION


Hand in BOTH your answer booklet, with any additional sheets firmly attached, and this
question paper.
In addition to this paper you should have available the 2002 edition of the
Formulae and Tables and your own electronic calculator.

ST5 S2006

Faculty of Actuaries
Institute of Actuaries

You are employed by a national government to oversee all aspects of the national debt
management policy.
Discuss how you might use an asset liability model, outlining the inputs, in this task.
[8]

An equity portfolio has a target beta of 1.2 relative to the market index.
(i)

Explain the term beta, and the meaning of a value of 1.2.

[2]

(ii)

Describe the investigations you would make to determine the value added by
the fund manager to the portfolio, assuming full data is available.
[3]

(iii)

List reasons why the performance of the portfolio might differ from that of the
index.
[3]

(iv)

The beta of the portfolio has moved away from the target of 1.2. Explain how
individual stocks can be used to correct the beta.
[2]
[Total 10]

A futures exchange is considering setting up a new European equity futures contract


and there are several existing indices on which the contract could be based.
(i)

Outline the attributes of the relevant indices that you would consider in
deciding which index to use, and name two potentially suitable indices.

[6]

Two years after launch of the new contract, it consistently ranks in the top five
contracts on the exchange. The exchange is however concerned that take-up in
the asset management community has been lower than initially anticipated.
It believes that this reflects the widespread usage of restrictions in mandates for
European equity managers.
(ii)

Describe the types of restrictions that might be made on a European equity


mandate, giving suitable examples.
[4]

The exchange decides to expand its range of contracts by splitting the European
equity contract into constituents that can be combined to re-create the existing
contract.
(iii)

ST5 S2006

List four classifications by which the contract may be sub-divided, and two
other variations on the original contract that might expand its appeal.
[3]
[Total 13]

ABC plc has a share price of 57 pence and paid a dividend of 2 pence per share in the
previous twelve months. It is considering issuing:
a convertible debenture, with a zero coupon, which is convertible into ordinary
shares on the basis of 150 shares per 100 nominal at any time over the next five
years; or
a zero dividend preference share at 100 that will be redeemed in five years time
at 138

(i)

Evaluate the returns that might be achieved from each of these investments
stating any assumptions.
[5]

(ii)

Explain which investors may prefer each investment giving reasons.

[7]
[Total 12]

An investment trust provided the following information to shareholders:


Balance Sheet

Investments
Cash
Total

130
10
140

Income Statement

Dividends
Interest Income

5.0
0.5

Total

5.5

m
Shareholder Funds
Debenture 2016 (nominal value)
Total

110
30
140
m

Interest Expense
Management Expenses
Other Expenses
Dividends Paid
Total

2.4
0.9
0.3
1.9
5.5

The current share price is 170 pence per share and there are 55 million shares in issue.
(i)

Calculate appropriate accounting ratios for the above trust and comment
briefly on them.

[7]

In the recent past many trusts have been paying off their long-term debt.
(ii)

ST5 S2006

Calculate the effect that paying off this trust s debt might have on the results
in (i). You may assume that there are no restrictions to paying off the
investment trust s debt.
[7]
[Total 14]

PLEASE TURN OVER

You are the broker to a UK charitable foundation with assets of 1bn invested in a
diversified portfolio of assets. The foundation is considering making large changes to
its asset allocation and the trustees are particularly concerned about the possibility of
moving market prices (both on sales and purchases of lines of stock).
(i)

List four approaches that might help mitigate this problem, explaining how
these will help.
[4]

(ii)

Describe the assets for which the problem might be particularly acute, and for
which the solutions listed above are unlikely to be effective.
[2]

At a meeting with the trustees, they explained that they would like to implement the
switch in stages, and that their rules currently prohibit them from using derivatives.
For the first stage of the switch they will be investing 100m in Japanese equities and
financing this by selling 50m of UK and 50m of US equities. They wish the switch
to proceed as soon as possible.

(iii)

Describe the practical problems of carrying out such a switch by transferring


physical stock, in addition to the risk of moving market prices.
[10]

(iv)

Explain how the switching process would operate if derivatives were used, and
list the advantages of doing this rather than a physical asset switch.
[5]
[Total 21]

You are an investment manager specialising in equities. You want to set up a fund
that will track the FTSE All-Share index. The fund will match the index weight in
each industry sector, but will not necessarily include every index stock to achieve the
sector weight.
(i)

List the eight economic groups in the FTSE Actuaries Classification and
briefly explain their key characteristics.
[10]

(ii)

Explain in detail why the fund manager will not include every index stock to
achieve the sector weight.
[7]

(iii)

Outline the quantitative investigations that could be made to ensure that the
fund effectively tracks the index after the fund has been set up.
[5]
[Total 22]

END OF PAPER

ST5 S2006

Faculty of Actuaries

Institute of Actuaries

EXAMINATION
September 2006

Subject ST5 Finance and Investment


Specialist Technical A
EXAMINERS REPORT
Introduction
The attached subject report has been written by the Principal Examiner with the aim of
helping candidates. The questions and comments are based around Core Reading as the
interpretation of the syllabus to which the examiners are working. They have however given
credit for any alternative approach or interpretation which they consider to be reasonable.
M A Stocker
Chairman of the Board of Examiners
November 2006

Faculty of Actuaries
Institute of Actuaries

Subject ST5 (Finance and Investment Specialist Technical A) September 2006 Examiners Report

General comments
Overall candidates scored well on the bookwork sections of the paper. However, as in
previous sittings, many fared less well where answers required application (perhaps
reflecting a lack of experience of practical investment problems), especially when numerical
answers were required; as a result questions 4 and 5 proved difficult generally with even
better candidates seeming unable to work their way through demonstrating that they
understood the investments and their key features. This explains why the pass rate was higher
than it has been in some previous sittings but more importantly why there were so many
FA fails.
It is concerning that many candidates appear to have little understanding of the outside
world. An example of this is that hardly any realised that the debenture in Q5 would have to
be repaid at todays price and not par if it was redeemed.
Better candidates were able to apply the bookwork to the situation outlined in the question.
The solutions should not be taken as comprehensive. There were a number of additional
points that could have been made for various questions and these have been rewarded
appropriately.
Comments on individual questions
Q1

In general this question was answered poorly. Many candidates failed to discuss how
asset liability models could be used to oversee national debt management policy,
instead describing the steps involved when carrying out an asset liability study.
Consequently many candidates only picked up the bookwork marks.

Q2

A bookwork question which was answered well by most candidates.

Q3

Overall candidates scored well on this question. In particular Part (ii) was well
answered. Better candidates scored highly on Part (iii).

Q4

This question was answered very poorly with many candidates scoring low marks. It
was clear that candidates struggled to understand the various investment options and
therefore, were unable to calculate the expected returns required in Part (i). The
answers to Part ii were slightly better.

Q5

Another numerical question that was answered poorly. Many candidates failed to
apply the ratios relevant for an Investment Trust. For Part (ii) candidates assumed
that debt could immediately be paid off at face value rather than using a discounted
cashflow approach.

Q6

Candidates answered the bookwork sections of this question well, in particular


Part (i). The range of answers in Part iii distinguished the better candidates as they
were able to articulate the practical issues when implementing an investment switch.
Part iv was generally well answered with candidates showing a good level of
knowledge on derivatives.

Q7

Most candidates scored full marks on Part (i). In general Parts (ii) and (iii) were also
well answered.

Page 2

Subject ST5 (Finance and Investment Specialist Technical A) September 2006 Examiners Report

ALM is an investigation into longer-term issues, involving the projection of assets


and liabilities over periods of several years.
The inputs are the future profile of asset proceeds and liability outgo.
Assets are largely future tax receipts but include future debt issuance and liabilities
are future government spending including redemptions and interest on existing debt.
Future proceeds and outgo will be heavily influenced by the economic and business
cycles together with many politically driven decisions and other government policies.
(Or alternative description of input assumptions.)
The ALM can provide a series of annual differences between income and outgo,
under various simulations of the factors noted above.
Debt issuance (repurchase) is the mechanism by which the government makes up any
shortfall (excess) in the public finances i.e. the pattern of expected amounts of future
issuance is the output of the ALM.
The debt manager will have little or no control over most of the asset or liability items
but can use the ALM study to provide it with information as to the likely pattern of
future issuance requirements.
It will be able to influence the profile of redemptions and therefore have some impact
on the future profile of issuance.

(i)

The Beta of a portfolio is a measure of the portfolios volatility relative to


movements in the whole market.
It is usually defined as the covariance of the return on the portfolio with the
return on the market, divided by the variance of the market return.
A beta of 1.2 means the change in value of the portfolio should be 20% greater
than the change in value of the market.

(ii)

The performance of the portfolio would be compared to the return on the


index.
The portfolios target return should recognise the pre specified level of risk.
Using an index representative of the market the portfolio is invested in, target
returns could be calculated as 1.2x the index return.
(Alternative descriptions using a risk adjusted measure received equivalent
credits.)

Page 3

Subject ST5 (Finance and Investment Specialist Technical A) September 2006 Examiners Report

Quarterly returns for the portfolio could be compared to the quarterly returns
on the target over, say, a five year period.
The excess return would indicate the level of value added by the manager.
Other relevant points.
(iii)

The performance will differ because the portfolio will be unlikely to hold
stocks and sectors in weights that are wholly representative of the index/may
have taken an active position.
The portfolios beta over the period may have varied to levels above or below
1.2 affecting returns.
The portfolio may have other objectives/constraints which affect performance.
The beta is different.
The diversification (or lack of it ) may affect volatility of portfolio returns.
The volume and dealing cost impact of trades in the portfolio.
The effects of cash flow.
The impact of tax.
The effects of expenses.

(iv)

The beta of individual stocks can be assessed by calculating the covariance of


the return for an individual investment with the return on the index.
If the beta of an individual stock was below 1.2 then the stock could be used
as a diversifier for the portfolio as its inclusion would reduce the overall risk
of the portfolio.

(i)

Popularity how widely followed is the index by various classes of investors


(e.g. asset managers, investment banks, hedge funds etc.).
Benchmarking is the index used as a benchmark for European equity
mandates?
Base currency what currency should the futures contract use as a base
currency?
Currency hedging should investment returns in the various local currencies
be assumed to be hedged against the base currency?
Frequency how often is the index published?

Page 4

Subject ST5 (Finance and Investment Specialist Technical A) September 2006 Examiners Report

Total return/withholding taxes would need to be a total return index


(preferably net of withholding taxes).
Method of index construction e.g. based on largest x European stocks by
market capitalisation, or based on a weighted average of underlying country
indices?
Frequency of updates to constituents (or any weightings)
Capitalisation weighted, or based on some other weighting process (equal
weights, capped weights, economic value, etc.)
Whether part of a series of indices (by country, style, sector etc.), so giving
scope for exchange to expand its range of futures contracts in the future if the
demand exists.
Suitable indices FTSE All-World Europe, MSCI Europe, Eurotop, Dow
Jones Eurostoxx.
(ii)

Some funds may have restrictions on the use of derivatives. This would mean
that a particular fund could not invest in the futures contract.
Some stocks may be excluded for ethical or social reasons (or possibly other
reasons in some cases). For example a fund may choose not to invest in
alcohol or tobacco stocks.
A subset of the European equity universe may be excluded in some mandates
as it is reflected elsewhere in an investors portfolio, or the investor has
decided for strategic reasons to exclude this subset. For example a UK
investor may have separate UK equity and European ex-UK mandates.
A fund may have restrictions on self-investment and for very large companies
pension funds this may create operational issues with using the contract.
Examples BP, Novartis.
Funds may have maximum holdings in individual securities or countries
within Europe. For example a 5% maximum investment in any single stock is
a common restriction.

Page 5

Subject ST5 (Finance and Investment Specialist Technical A) September 2006 Examiners Report

(iii)

Split by country.
Split by sector.
Split into large, mid and small cap stocks.
Split by high/medium/low dividend yield.
Split into growth and income stocks.
Variations change from stock returns on an unhedged basis to hedged into a
range of different currencies (or vice versa).
Changing the base currency.
Changing the weighting approach.

(i)

The convertible has a minimum return of 100 per 100 invested assuming
company remains solvent. ZDP has maximum of 138, convertible depends on
share price.
Convertible is ZDP plus call option.
Option price = 100 - 100/138 = 27.5.
Convertible return better than ZDP if share price above 92 pence.
ZDP compounds annually at 6.65% p.a.
Share has dividend yield of 3.5% p.a. assuming no change in rate.
Capital growth of 3.15% required to match ZDP over 5 years.

(ii)

The equity could be attractive to any investor. It has a reasonable yield.


(Or equivalent comment from other investment backgrounds.)
The convertible requires a very strong price rise to be attractive to equity
orientated investors.
Convertible needs 10.0% to achieve same goal.
It might be of interest to a hedge fund wishing to carry out arbitrage
investment.
It could be of interest to an absolute return manager if market volatility was
expected to increase.
The ZDP is of interest to risk averse investors with minimum return criteria.

Page 6

Subject ST5 (Finance and Investment Specialist Technical A) September 2006 Examiners Report

It might be of interest to structured product providers as collateral.


(Other examples of possible investors received equivalent credit.)
It could be of interest to hedge funds in conjunction with the ordinary and
convertible to create an absolute/arbitrage return.

(a)

Yield on investments = 3.85%


Yield on year end cash = 5%
Net asset value per share = 200p, discount = 15%
Interest on debt = 8%
Total expense ratio = 0.86%
Dividend per share = 3.45p, yield = 2.0%
Portfolio has high yield ratio to market
Cash return high need to investigate
Gearing = 140/110 = 27%
Gearing slightly high for conventional trust
(Other relevant ratios plus comment received equivalent credit.)

(b)

Debt coupon = 8%, current 10 year interest rate = 4.5% (say)


Cost of paying off debt = 127.69 * 30m/100 = 38.31m
Assets = 101.69m, nav per share = 184.89
Income, assuming cash and prorate reduction = 3.92m
Management expense ~ 0.65% = 0.66m, other = 0.3m
Available for div = 2.96m = 5.38 pence = 3.2% yield
Might suggest reducing yield ratio

(i)

Selling/purchasing assets in small units


Selling/purchasing assets over a period of time (including use of cashflow)
Using algorithmic trading techniques
Using nominee accounts
Using several banks to carry out the trades
Using derivative contracts
Use cross transactions
All of the above methods will mask the trade to other market participants
and can be used in combination.
Derivatives contracts will be suitable in many cases for changing broad
exposures quickly although the underlying stocks will still need to be
sold/purchased in due course unless the asset allocation change is temporary.

Page 7

Subject ST5 (Finance and Investment Specialist Technical A) September 2006 Examiners Report

(ii)

These approaches are difficult to implement for unmarketable securities where


there is no deep or liquid market.
Assets in this category might include small cap or unquoted equities, certain
high yield bonds or smaller bond issues, and property.

(iii)

Significant coordination is needed by the UK, US and Japanese equity


managers to ensure that the switch goes smoothly.
It may be useful to appoint a transition manager to coordinate the process if
none of the existing asset managers has a specialist team who is able to
manage the project.
There may be several asset managers involved in the process.
The managers may need some time to decide which stocks to buy and sell, and
this may create further delays as the transition occurs.
In practice, it is likely that the sales of UK and US equities will need to be
spread over a period of time, particularly for smaller stocks.
Unless borrowing is permitted (unlikely) the Japanese equity manager can
only buy stock after the UK and US equity managers have sold stock
allowing for settlement periods
This will delay the transition.
This may have an adverse impact due to market movements
or currency movements
However, rushing the sales is likely to impact on the value realised from the
sale.
Tax liabilities may be brought forward and crystallised, particularly on stocks
with large gains since purchase.
Conversely tax losses may be created where stocks are being sold at a loss,
and it is not known when those losses might be offset against future gains in
the UK or USA.
The process will be time intensive and may be expensive in terms of
transaction costs.
It is likely that the fund will suffer both buying and selling fees
although these might be mitigated if the asset managers are able to cross
with transactions for their other clients.

Page 8

Subject ST5 (Finance and Investment Specialist Technical A) September 2006 Examiners Report

The transition programme will need ongoing monitoring in case market


movements mean that the target changes in asset allocation are not achieved,
or exceeded.
(iv)

The UK and US equity managers would sell a number of futures contracts


with exposure equal to the amount of stock to be sold.
The Japanese equity manager would buy a number of futures contracts with
exposure equal to the amount of stock to be purchased.
As the physical sales and purchases are completed the number of futures
outstanding will need to be reduced to ensure that the correct exposure is
maintained.
If the physical sales and purchases are not completed before expiry of the
futures contracts, then any outstanding futures positions will need to be rolled
over.
An operational issue with futures is that initial and variation margins will need
to be deposited as the counterparty will want collateral for any outstanding
positions.
If the various managers do not have sufficient assets to deposit as collateral
then borrowing may be necessary.
Advantages:
Switch can be implemented very quickly.
Futures markets is very liquid, with little risk of any market impact
(particularly as the futures contracts will be based on market indices).
Purchase of futures reduces pressure on asset managers to construct and
execute their buy/sell programs quickly.
If the switch needs to be adjusted the positions can be easily and cheaply
reversed without disturbing the underlying stock portfolios.

Page 9

Subject ST5 (Finance and Investment Specialist Technical A) September 2006 Examiners Report

(i)

Resources
These companies are involved in the extraction and supply of primary
products used throughout the economy. Oil is the most important. Key
characteristics are:

large companies
commodity price dependent
risky
global

Basic industries
This group includes the chemical industry and companies in the building
materials and construction industries, as well as companies producing steel
and other metals, and those engaged in forestry and paper. As such, these
companies are mainly producing intermediate goods.
General industries
General industrial companies are involved in the various stages in the supply
and production of goods. Many of the goods tend to be capital items, i.e.
aircraft, ships, machinery, electronic and electrical equipment. The distinctive
features of both industry groups are:

dependent on the level of investment spending


cyclical
company profits tend to move ahead of the trade cycle
dependent on government spending
volatile profits
high profit margins when conditions are good
low gearing because of volatile profits
possibly exposed to overseas markets and competition

Consumer goods
Companies in the consumer goods groups manufacture consumer durables and
non-durables. Durables include cars, furniture, televisions and white goods
such as washing machines. Non-durables include food and drink,
pharmaceuticals, tobacco, health and household products, beverages and
packaging. Generally the impact of an economic cycle is less severe on nondurable consumer goods companies than on general manufacturers. This is
especially true for companies producing basic necessities. Thus, the consumer
goods group is further divided into cyclical (durable) and non-cyclical (nondurable) sectors. Other key features are:

Page 10

increasingly capital intensive


importance of brand names
increasingly international

Subject ST5 (Finance and Investment Specialist Technical A) September 2006 Examiners Report

moderate to high gearing


low profit margins

Services
These are also now divided into cyclical and non-cyclical sectors. Cyclical
service companies include general retailers, transport, hotel and media
companies, distributors, restaurants and pubs and support services. Noncyclicals include food and drug retailers and telecommunication services.
Once again, the impact of the economic cycle will be greater on the cyclical
group. Other key features are:

labour intensive
the more defensive companies in the group may have high gearing
the domestic market is the most important

Utilities
Utilities are involved in the supply of continuously demanded services to
households and business premises. Examples include electricity, water and gas
distribution. Most UK utilities were formerly owned by the government,
having been privatised during the 1980s. They are vulnerable to some political
risk and to changes in the regulations under which they operate. Demand is
very stable because the services that they provide are essential, or nearly
essential, and because their market share will be stable (often at 100%). Thus,
they are less affected by economic cycles than other groups. Other points are:

they usually require an extensive physical infrastructure. This tends to


make them capital intensive
most utility companies are natural monopolies
they are usually subject to tight government regulation of prices and
vulnerable to other forms of political risk
they generally have low growth prospects; this leads to a high gross
dividend yield
despite their stable demand and large capital requirements, gearing is low
they are largely dependent on the domestic market, although some
companies are diversifying internationally

Financials
The financial group companies are the various industries making up the
financial services industry, e.g. clearing banks, investment banks, general
insurance companies and life assurance companies, investment trusts, real
estate (property) companies. The key distinctive feature of financial group
companies is that they tend to be capital intensive. Otherwise, the features of
companies in this group are quite varied between the different sectors:

banks are highly geared and have volatile profits


general insurers also have volatile profits and virtually no borrowings

Page 11

Subject ST5 (Finance and Investment Specialist Technical A) September 2006 Examiners Report

life insurers have stable profits and low gearing


labour costs are important for many companies in the group
the domestic market is most important but there is increasing
internationalisation

Information Technology
These are the companies involved in the new industries of information
technology hardware, software and the provision of computer services. While
investor demand for such shares has caused share prices to increase
dramatically in the past, many of the companies have yet to make profits or
pay dividends. Dividend yields on these companies are therefore low, and
their assets can be largely intangible.
(ii)

The fund manager only has to track the performance of the index.
So replicating the index is not essential.
Investing in many stocks and so having relatively small individual holdings in
each stock will result in high dealing costs (necessary each time the relative
sector weightings change).
This would reduce the performance of the fund and so cause
underperformance relative to the index.
Research has shown that, after overall market movements have been taken into
consideration, the share price movements of companies within industrial
groupings tends to correlate more closely with each other than with companies
in other industries, so holding a subset may well replicate the performance of
the sector.
The share price movements reflect the changes that have occurred in the
operating environment, and such changes affect companies in the same
industries in similar ways.
A specific sector may only represent a small percentage of the index and
within that sector the small number of stocks the manager proposes to use may
represent a substantial proportion of the total market capitalisation of the
sector.
Stratified sampling of the performance of each sector may have shown that the
performance of the chosen stocks is a very accurate measure of the
performance of the sector as a whole.
Sampling may enable the fund to choose its timing in addressing whether or
when to replicate changes to the underlying index.

(iii)

Page 12

Compare dividend yields, earnings growth and price earnings ratios with the
Index.

Subject ST5 (Finance and Investment Specialist Technical A) September 2006 Examiners Report

For example, within each sector for the fund and the index:

Rank the holdings by increasing yield


Split each sector into an equal number of holdings (e.g. quintiles)
Calculate the weighted average yield of each quintile allowing for the
value of shareholdings as weights

This will help to determine:

Consistency with the portfolio


Identify any style biases (e.g. growth or value)
How risky the portfolios are relative to the index

Historic comparison of the fund performance with the index quarterly over a
period of around three years to determine how well the fund has tracked the
index.
Comparison of risk adjusted performance measures e.g. Sharp or pre-specified
standard deviation.

END OF EXAMINERS REPORT

Page 13

Faculty of Actuaries

Institute of Actuaries

EXAMINATION
13 April 2007 (pm)

Subject ST5 Finance and Investment


Specialist Technical A
Time allowed: Three hours
INSTRUCTIONS TO THE CANDIDATE
1.

Enter all the candidate and examination details as requested on the front of your answer
booklet.

2.

You have 15 minutes at the start of the examination in which to read the questions.
You are strongly encouraged to use this time for reading only, but notes may be made.
You then have three hours to complete the paper.

3.

You must not start writing your answers in the booklet until instructed to do so by the
supervisor.

4.

Mark allocations are shown in brackets.

5.

Attempt all 9 questions, beginning your answer to each question on a separate sheet.

6.

Candidates should show calculations where this is appropriate.

AT THE END OF THE EXAMINATION


Hand in BOTH your answer booklet, with any additional sheets firmly attached, and this
question paper.
In addition to this paper you should have available the 2002 edition of the
Formulae and Tables and your own electronic calculator.

ST5 A2007

Faculty of Actuaries
Institute of Actuaries

(i)

(a)

Define risk-free return.

(b)

List three types of asset that would be expected to provide such a


return.
[2]

(ii)

Write down an equation for the required return on an asset.

(iii)

Explain why it is necessary to estimate the equity risk premium from historical
data.
[2]

(iv)

Give examples of:

(i)

[1]

(a)

the types of distortion that need to be allowed for in any analysis of


historical risk premia.

(b)

the likely impact of allowing for these adjustments.

[6]
[Total 11]

Describe how the economic cycle can impact companies price/earnings ratios.
[5]

A stock market comprises three securities:

Market capitalisation 31/12/2005


Earnings 31/12/200531/12/2006
Share price 31/12/2005
Dividend yield 31/12/2005
Share price 31/12/2006
Dividend yield 31/12/2006
(ii)

Stock A

Stock B

Stock C

$100m
$10m
$20
3%
$24
3.2%

$500m
$25m
$20
2.8%
$25
2.5%

$200m
$25m
$20
4%
$16
4.5%

Describe the characteristics of growth and value stocks.

[2]

A value investor constructs a portfolio at 31 December 2005 comprising 10% stock A,


30% stock B and 60% stock C. In contrast a growth investor has a portfolio of 20%
stock A, 70% stock B and 10% stock C.
(iii)

Explain why the two investors may have constructed their portfolios as they
did.
[3]

(iv)

Calculate the portfolio returns over the period for each of the two investors. [3]

(v)

Describe three other equity styles that other investors may be using.

ST5 A20072

[3]
[Total 16]

(i)

(a)
(b)

Explain what is meant by an agency cost.


Explain how agency costs can arise.
[4]

(ii)

(a)

Explain why conflicts of interest may arise between equity and bond
holders in a company.

(b)

Give three examples of such conflicts of interest including details of


who gains and who loses from the action.
[4]
[Total 8]

A professional fund manager invests in the constituent shares of the FTSE 100 index
and weights the investments of the fund on an arithmetic average basis using the
market capitalisation of the constituents of the FTSE 100 index. The fund manager
adjusts the constituent shares and their weights in line with changes in the weights
used in the construction of the index. To all intents and purposes, the fund attempts to
track the price and yield performance of the FTSE 100 index.
The investors in the fund asked an independent investment consultant to evaluate the
total return performance of the professional fund manager relative to the total return
performance of the FTSE 100 index over the last ten years. The independent
investment consultants report examined the fund managers total return and
concluded that the professional fund manager had under performed the FTSE 100
Total Return Index over the ten year period in question.

(i)

Describe the most likely reasons for the fund managers under performance
relative to the FTSE 100 index over the past 10 years.
[12]

(ii)

Outline two ways of reducing the under performance of the fund manager
relative to the FTSE 100 index.
[2]
[Total 14]

(i)

Sketch, on the same diagram, the payoff at maturity of the following options
on a quoted share assuming that they have the same strike price, same maturity
date and that the ratio of premiums is 2:1 with the call option being the more
expensive option:
(a)
(b)

a put option
a call option
[3]

(ii)

Using the same diagram as in part (i) of this question, sketch the payoff profile
of an equally weighted portfolio consisting of options (i)(a) and (i)(b).
[3]

(iii)

Suggest an investment scenario in which the payoff profile in part (ii) might
be of interest to an investor.
[2]
[Total 8]

ST5 A20073

PLEASE TURN OVER

(i)

Describe three reasons why regulation is considered important in the financial


services industry.
[3]

(ii)

Outline the direct and indirect economic costs of regulation.

(i)

Describe the concept of a trust as it arises in trust law.

(ii)

Suggest the advantages of a debenture trust deed.

[6]
[Total 9]

[5]
[3]
[Total 8]

You are the finance director for a regional Japanese bank and are considering the
issue of a bond. You have the option of issuing the bond in sterling or in local
currency and have available the following information:

Sterling issue
Domestic issue

Spread over government bonds

Government bond yield

+125bps
+175bps

4.50%
2.00%

You should assume the yield curve is flat across all maturities in both the sterling and
the domestic market.
(i)

List the main reasons for issuing a bond.

(ii)

Discuss why the spread over government bonds may differ between the UK
and Japanese markets for the same issuer.
[6]

(iii)

Calculate the return an investor would achieve, in yen terms, by investing and
holding each bond to maturity. State any assumptions made.
[6]

(iv)

Outline the steps you would take to lock into the lowest funding cost.
[2]
[Total 18]

ST5 A20074

[4]

You are a portfolio manager for a high alpha actively managed bond fund. Your
annual performance target is 1.5% p.a. above the return on the all stocks
government treasury bond index. This target is after deduction of your 0.4% annual
management fee.
One of your bond dealer contacts has invited you to subscribe to a new innovative 20
year amortising bond whose return is linked to the mortality experience on a pool of
insured annuitants. Under this bond, the coupon and principal payments are reduced
(or increased) if more (or fewer) annuitants survive than expected, within minimum
and maximum amounts.
The dealer has made a compelling pitch to you that this is the first of a number of
such bond issues and that the spreads on these bonds will narrow considerably over
time as the market gains familiarity with these issues. The Chief Investment Officer
of your investment house requires you to submit a formal analysis of the bond issue
for consideration at the next Investment Committee meeting.
(i)

Outline the issues you would consider in your analysis of the opportunity to
generate outperformance for the bond fund by participating in the offer.
[6]

(ii)

Describe what further investigations and information you would require before
you could make a recommendation on whether to participate and how much to
invest.
[2]
[Total 8]

END OF PAPER

ST5 A20075

Faculty of Actuaries

Institute of Actuaries

EXAMINATION
April 2007

Subject ST5 Finance and Investment


Specialist Technical A
EXAMINERS REPORT

Introduction
The attached subject report has been written by the Principal Examiner with the aim of
helping candidates. The questions and comments are based around Core Reading as the
interpretation of the syllabus to which the examiners are working. They have however given
credit for any alternative approach or interpretation which they consider to be reasonable.

M A Stocker
Chairman of the Board of Examiners
June 2007

Faculty of Actuaries
Institute of Actuaries

Subject ST5 (Finance and Investment Specialist Technical A) April 2007 Examiners Report

Comments
Most candidates scored well on questions 5 and 6 with many achieving full marks. Although
some candidates scored well on questions 2 and 3 also, most candidates attained closer to
half the available marks. Questions 4 and 7 were the worst answered with candidates
achieving typically scores of less than a third of the available marks. For example, few
candidates were able to identify multiple likely reasons for underperformance and most were
unable to describe a debenture trust deed and the advantages thereof. Although it was
pleasing to see the scores achieved by better candidates, it continues to be a source of
frustration and disappointment that candidates appear to ignore valuable information
contained within the question and lose easily achievable marks as a consequence.
In every diet there will be candidates who are very close to the pass mark and yet receive an
FA indeed I suspect candidates would be very surprised to see just how tightly distributed
the marks are; deciding where the pass mark falls will have a material impact on the
numbers of candidates who are successful and the examiners take great care to ensure a
consistency of standard across candidates, subjects and diets. The pass rate for this diet was
very similar to the last session although the pass mark was higher, reflecting the overall
higher scores achieved by candidates on bookwork parts of questions.
All extenuating and mitigating circumstances were considered in awarding grades
coincidentally those candidates who had submitted the most severe mitigating arguments had
in fact achieved sufficiently high marks to justify a Pass grade.
Notwithstanding the high scoring on bookwork elements, candidates should note the bias in
the paper towards recognising higher level skills and practical application this is
intentional and will continue. Likewise the examination system does properly allow for prior
subject knowledge to be assumed. It is not appropriate to repeat all relevant material within
the Core Reading and in the exam creation process, the profession takes great care to ensure
that the paper can be answered by a candidate who has taken a normal route through the
exams - indeed questions have been removed from previous draft papers as a result.
Candidates looking to progress should be aware that the SA series of exams, particularly
investment related, are even less bookwork focussed and require the candidate to
demonstrate a breadth and depth of competency as would be expected from a practising
actuary in a constantly changing discipline. Hence simple regurgitation of bookwork will not
be sufficient to ensure a Pass grade. Candidates should ensure they familiarise themselves
with the current investment issues facing institutional investors in the 18 months preceding a
diet and the solutions being debated by the various stakeholders.

Page 2

Subject ST5 (Finance and Investment Specialist Technical A) April 2007 Examiners Report

(i)

The risk-free rate of return is the rate of return on a security which has no
credit risk. Assets providing such a return include fixed interest government
bonds, inflation-linked government bonds and short-term government bills.

(ii)

Required return = Required risk free real rate of return + expected inflation +
risk premium

(iii)

For an asset with certain cashflows (e.g. a bond) the risk premium can be
estimated from the price since an IRR calculation can be carried out, although
an adjustment needs to be made for default risk.
Conversely, the risk premium from the price of an asset with uncertain future
cashflows (e.g. an equity) cannot be estimated using an IRR calculation.
This leads to a need to analyse historical data, although this may not
necessarily be a good guide as the risk premium is based on expected return,
rather than achieved return.

(iv)

Distortions that need to be corrected for in an analysis of historical risk premia


would include:
Survivorship bias allowance needs to be made for the returns achieved on
securities which have been removed from a sample due to defaults or removal
from a reference index. Allowing for this factor will typically reduce the
measured historical return.
Market valuations/risk preferences allowance should be made for changed
valuations of securities through changes in investor risk preferences. Investors
may be willing to accept a lower/higher risk premium for a given level of
volatility, leading to higher/lower prices currently than in the past.
Achieved dividend growth vs expectations part of the risk premium will
reflect expectations in future dividend growth. If dividends have grown at a
faster or slower rate than originally expected by equity investors, this could
lead to an inaccurate estimate of the risk premium based on historical return
data.

Page 3

Subject ST5 (Finance and Investment Specialist Technical A) April 2007 Examiners Report

(i)

If the economy is moderately buoyant and profits are fairly stable, both
defensive and cyclical companies might be similarly rated in terms of the P/E
ratios.
As the economy starts to move into recession P/E ratios for cyclical companies
are likely to fall while those of defensive companies will remain stable or may
even rise slightly.
At the bottom of the cycle P/E ratios of cyclical companies will probably have
risen from their low point as earnings have fallen, but defensive stocks will
still be more highly rated.
As the economy starts to recover, the P/E ratios of cyclical companies will rise
in anticipation of future earnings growth. P/E ratios of defensive companies
may now be lower than those of cyclical stocks.
As growth continues, the earnings of cyclical companies will catch up with the
share price and P/E ratios will fall back towards their long-term average level.

(ii)

Value stocks typically have low P/E ratios (12 or lower) and higher dividend
yields (4% or more). They tend to be stocks with low expectations of future
earnings growth or are out of favour with investors (reflected in the P/E ratio).
Conversely, growth stocks typically have high P/E ratios (20 or higher) and
lower dividend yields (2.5% or less). These tend to be stocks with higher
expectations of future earnings growth (reflected in the P/E ratio).

(iii)

The neutral weights for the three stocks are 12.5% stock A, 62.5% stock B and
25% stock C. The PE ratios are 10, 20 and 8 respectively, with stock A
yielding close to the average, B having a slightly lower than average yield and
C a high yield. These two measures suggest that stock A has a small value
bias, B has a growth bias and C has a value bias, hence the weightings in the
two investors portfolios.

(iv)

A: 24/20 (1 + 3.2%/2 + 3%/2) - 1 = 23.7%


B: 28.3%
C: -16.6%
Value return = 10% 23.7% + 30% 28.3% + 60% -16.6% = 0.9%
Growth return = 22.9%

(v)

Page 4

Small-cap, large cap, momentum, rotational, contrarian, top-down, bottom-up


(brief description of any three or any other relevant style properly defined)

Subject ST5 (Finance and Investment Specialist Technical A) April 2007 Examiners Report

(i)

Agency costs can arise in an organisation where the owners have delegated
operational decisions.
They become increasingly likely as an organisation grows, although they can
also occur in a smaller organisation where the owners and managers are
separate.
This separation of ownership and management can lead to a divergence of
interests, and such conflicts of interest are called principal-agency conflicts.
Agency costs are defined as the costs of monitoring the agents (managers) and
influencing/incentivising them to act in the interests of the principals
(owners)
and thereby reduce conflicts/create alignment.
Without such monitoring or influencing, management may act in a way that
diverges from the interests of the owners, and this is arguably a consequence
of rational behaviour by the management seeking to exploit their position.

(ii)

Conflicts of interest arise between equity and bond holders since bond holders
have no upside potential beyond return of their capital and interest payments
but are exposed to downside risks.
Conversely equity holders have significant upside potential and typically
exercise day to day control over the company, leading to conflicts.
Possible conflicts can include:

Equity holders underinvesting in a company to increase profits, whilst


reducing long term security for bond holders.

Equity holders influencing management to take excessive risks, increasing


the default risk of the firm (at the expense of bond holders, but not
necessarily for equity holders on an expectation basis).

Equity holders taking excessive dividends from the company, reducing


asset cover of liabilities (including payments due to bond holders).

Page 5

Subject ST5 (Finance and Investment Specialist Technical A) April 2007 Examiners Report

(i)

Fees & Expenses


The investors in the fund must pay investment management fees, custody fees,
audit fees, governance fees and administration fees whereas such fees are not
taken into account in the calculation of returns on FTSE 100 Total Return
Index.
Cost of Rebalancing
The FTSE 100 Total Return Index does not take into account the costs of
rebalancing the index for such activities as new entrants, exits, mergers and
takeovers and changes in the market capitalisation of constituents.
Such costs include stockbrokers commissions, stamp duty and other levies.
Cash Holdings
When the fund manager receives small amounts of dividend income, it may
not be cost effective for her to invest such small amounts across the entire 100
constituents in the correct proportions. The manager will therefore have part
of the portfolio invested in the constituents of the FTSE 100 index and part
invested in cash. The cash holding will cause the manager to under perform
the index in a rising market and out perform the index in a falling market.
This can be a significant issue when there are large inflows or outflows from
the fund, particularly if asset volatility is higher than typical.

(ii)

Futures
The manager may be able to reduce the drag on investment performance
arising from cash holdings by using FTSE 100 futures contracts.
Stock Lending
The manager may be able to reduce the size of her under performance by
engaging in stock lending whereby she will receive a fee for lending stocks to
other institutions (such as the prime brokers of hedge funds) on a collateralised
basis.

Page 6

Subject ST5 (Finance and Investment Specialist Technical A) April 2007 Examiners Report

(i)

Marks should be awarded for the following features:


Same axes and same scales one rather than two diagrams;
Common strike price
2:1 ratio of premiums (suggested on inspection by eye)
General shape of each options hockey stick diagram.

(ii)

Marks should be awarded for the following features:


Same axes and same scales one rather than two diagrams;
Common strike price of the combined option portfolio;
Maximum loss of 3 units (implied by 2:1 ratio of premiums for basic options
in (i)) (suggested on inspection by eye);
V shape of diagram.

(iii)

The portfolio of options pays off at maturity when there is a large move either
way in the price of the underlying stock over the life of the option.
Conversely investors in the portfolio of options will lose out if there is little or
no movement in the price of the underlying stock by the maturity date of the
option.

(i)

Confidence: It is important that there is confidence in the financial system.


Systematic Failure: There is a danger of problems in one area spreading to
other parts of the financial system.
There is believed to be a significant asymmetry of information, negotiating
strength and expertise in the financial markets especially among retail
investors.

(ii)

The direct costs of regulation arise in administering the regulation and in


compliance for the regulated firms.
Some possible indirect costs include:

Consumers behaviour is altered which gives rise to a false sense of


security and a reduced sense of responsibility.

A reduction in the consumer protection measures developed by the market


itself.

Product innovation is reduced.

Competition is reduced.

The sense of professional responsibility amongst intermediaries and


advisors is undermined.

Page 7

Subject ST5 (Finance and Investment Specialist Technical A) April 2007 Examiners Report

(i)

Trusts constitute a relationship between persons in which one person has the
power to manage property and the other person has the privilege of receiving
the benefits from that same property.
The legal owner of the property of a trust is called the trustee and she has the
right to possession of the property.
The beneficiary of the trust is the person who receives all the benefit from the
property.
The divisions between legal and beneficial ownership are normally created by
an express instrument of trust known, usually, as the deed of trust.
The trust deed will also specify the beneficiaries by name or as being persons
who are members of a particular group.
Trustees are required to act in the best interests of the beneficiaries of the trust.
The standard of care required of trustees varies with their level of expertise in
that a higher standard of care is required of those who hold themselves out to
be professional trustees compared with an ordinary man.

(ii)

In a debenture trust deed, the individual interests of the usually substantial


number of holders of a debenture are channelled into the trust.
The trustee holds and protects the aggregate of the personal rights of all
members of the trust.
A debenture trust may be thought of as providing efficient administration.

(i)

Raising new or refinancing capital many possible purposes, acquisition,


investment in the business, fund new projects, financing of pension fund
Restructuring the balance sheet share buy backs, securitising future
cashflows.

(ii)

Spread over government yields reflects the extra premium that the bank must
pay to compensate the investors for extra risks of reduced liquidity and
default. These risks should be similar regardless of the currency of the bond
issue. It is likely that the bank earns its cashflows in local currency so has no
exposure to sterling so one may argue that the sterling denominated issue
has additional currency risks and should have a higher spread to governments
to compensate. The spread is also affected by supply and demand dynamics
the lower spread in reflecting a higher level of demand.

(iii)

Assuming the bond doesnt default


Yen 3.75% p.a.

Page 8

3.25% p.a. (4.5% + 1.25% - 4.5% + 2%)

Subject ST5 (Finance and Investment Specialist Technical A) April 2007 Examiners Report

The government bond used for comparison has a similar maturity/duration to


the Japanese bond being considered.
Currency movements reflect the interest rate differentials between the two
countries.

(iv)

Issue bond in sterling


Implement forward currency transactions to neutralise currency mismatch
risks

(i)

The dealer is probably correct that initially spreads over gilts will be high due
to investors unfamiliarity with such an issue.
Also investors typically (but not always) prefer vanilla bonds without
embedded exposures or options, leading to higher spreads on more complex
issues due to difficulty in assessing if pricing is favourable or not.
However, it is also possible that there is considerable demand in the investor
community for such issues, in which case initial spreads may be lower.
If the market does become more confident about these bonds then subscribing
would offer the prospect of say a 50bps reduction in spreads, which would
amount to 6% increase in value if the duration was 12 years (for example).
Alternatively, if the market did not develop strongly, this could result in the
fund being stuck with a long-term illiquid issue albeit providing a modest
spread over vanilla bonds of a similar term and credit. If the bond had a
shorter term e.g. 5 years, this would be less of a concern.
Such a negative scenario could also occur if the market for mortality-linked
bonds does develop, but with different design features.
Assess impact on duration of the new bond from sensitivity analysis of
expected annuitants survivors. Also important to assess the resultant duration
of the fund from the desired allocation to the new bond, and relative to the
duration of the reference index.

(ii)

Further investigations:

Some sensitivity testing of ultimate proceeds and how mortality


experience might impact on the ultimate return, most likely using
stochastic analysis.

Need to understand how sensitive the return on the bond is to mortality


experience, as opposed to economic factors. In the longer term it may
become important to monitor the sensitivity of the fund to mortality
factors, from a risk control point of view.

Page 9

Subject ST5 (Finance and Investment Specialist Technical A) April 2007 Examiners Report

Investigate size of potential market, from both a supply and a demand


point of view.

Feedback from internal analysts and external market participants, e.g. asset
managers, bond dealers, analysts.

The proposed offer price of the bond is a key bit of information that would
be required.

Specific information on the pool of insured annuitants should be


investigated.

END OF EXAMINERS REPORT

Page 10

Faculty of Actuaries

Institute of Actuaries

EXAMINATION
26 September 2007 (pm)

Subject ST5 Finance and Investment


Specialist Technical A
Time allowed: Three hours
INSTRUCTIONS TO THE CANDIDATE
1.

Enter all the candidate and examination details as requested on the front of your answer
booklet.

2.

You have 15 minutes at the start of the examination in which to read the questions.
You are strongly encouraged to use this time for reading only, but notes may be made.
You then have three hours to complete the paper.

3.

You must not start writing your answers in the booklet until instructed to do so by the
supervisor.

4.

Mark allocations are shown in brackets.

5.

Attempt all 8 questions, beginning your answer to each question on a separate sheet.

6.

Candidates should show calculations where this is appropriate.

AT THE END OF THE EXAMINATION


Hand in BOTH your answer booklet, with any additional sheets firmly attached, and this
question paper.
In addition to this paper you should have available the 2002 edition of the
Formulae and Tables and your own electronic calculator.

ST5 S2007

Faculty of Actuaries
Institute of Actuaries

(i)

State the three main aims of regulation.

(ii)

With any type of regulation there is a cost.

[1]

(a)

Outline the different types of costs involved.

(b)

How do these costs impact the overall objectives of the regulator. [5]
[Total 6]

You are the financial director of a small private software company. Due to a contract
being recently cancelled there is likely to be a short-term liquidity issue where the
company cannot meet its creditors for the next three months, after which the financial
picture looks healthy. The Chairman of the company is opposed to long-term debt.
You have been asked to provide the Chairman with alternative sources of short-term
finance to resolve the impending liquidity issue.

(i)

Describe the financing options available to the company.

[5]

(ii)

Outline the main contract terms that differentiate between the types of
borrowing available.
[4]
[Total 9]

Three companies are intending to raise finance via the debt market.

Company A A government backed company which has a history of strong


profits and low levels of debt.

Company B A public listed company that is well established but has suffered
from decreasing sales and has posted losses for the last two years.

Company C An internet start up company that intends to sell wedding cakes


on-line.

(i)

Explain with reasons the credit rating that is likely to be assigned to each of
the three bond issues.
[2]

(ii)

(a)

Define the term expected default loss.

(b)

Explain how this would vary for each of the three companies.
[3]

All three companies decide to issue zero coupon bonds.


(iii)

ST5 S20072

(a)

Assuming the yield on a comparable treasury stock is 4.25% p.a., state


what yields would be appropriate on a zero coupon bond issued by
each company.

(b)

Calculate the expected default loss for the first year of each bond,
stating any assumptions that you make.

[8]

The Government of the country is concerned that the local currency is appreciating
too fast and decides to introduce immediate controls on transactions by overseas
investors. As a consequence the domestic equity market index has fallen by 15% and
treasury bonds are now yielding 5.25% p.a.
(iv)

Explain how the yields on the bonds issued by the three companies might
change as a consequence of the change in market levels.
[6]
[Total 19]

A friend has decided that they would like to try to increase their wealth by investing
in derivatives. After reading some articles they realise that they do not fully
understand some of the terminology and have approached you for help.
(i)

Explain the main uses of derivatives.

[8]

(ii)

For a derivative contract traded on an exchange:


(a)

Explain the term margin.

(b)

Outline the different types of margin payments that are payable.

(c)

Explain why a clearing house requires these payments.


[3]

(iii)

(a)

Explain the difference between a Put and a Call.

(b)

Explain the term European and American in the context of options.


[2]

The current share price of XYZ is 60p. Your friend has been offered the following
options in XYZ.

(iv)

ST5 S20073

Strike Price

3 Month

6 Month

Call 75p
Put 85p

5p
10p

10p
5p

Given this information, draw the pay-off charts associated with each of these
options clearly identifying the price when the option is in the money.
[6]
[Total 19]

PLEASE TURN OVER

(i)

Describe the main forms of government policy.

[2]

(ii)

Outline the main economic indicators which show whether the policies applied
by the government have been successful.
[2]

To promote growth after a prolonged recession the government has decided to reduce
interest rates from 5% to 3%.
(iii)

Describe how this move is likely to impact:


(a)
(b)
(c)

individuals
businesses
the economy as a whole
[6]
[Total 10]

You are the investment consultant to a 400m pension fund that has a 15% shortfall in
assets compared with the national common funding standard introduced by the newly
created regulator of pension funds. In addition, the new regulator has insisted that all
defined benefit funds have a national common funding level above 105% in seven
years time. One of the trustees has read a newspaper article claiming that more
pension funds are investing in hedge funds as a way of meeting their liabilities and the
requirements of the new regulator.
(i)

(a)

(b)

Outline the main types of hedge fund that the pension fund could
invest in.

[2]

Describe the main investment characteristics of a hedge fund.

[3]

One of the criticisms of hedge funds is the lack of reliable performance data.

(ii)

Explain why there is a lack of credible performance data.

[4]

(iii)

Explain how the fund could invest in hedge funds alongside other assets and
derivatives in order to achieve the national common funding target objectives.
[6]
[Total 15]

An institutional fund management company listed on the Alternative Investment


Market has sought your advice on how it can better manage the operational risks in its
business.
(i)

Outline the risks that you would seek to address.

(ii)

Describe ways in which these risks can be managed.

ST5 S20074

[3]
[9]
[Total 12]

You have recently been appointed as the financial adviser to a private company. The
company wants to provide an initial offering of shares and to be listed on the stock
exchange. You have been asked to provide a valuation of the company.
(i)

Describe the role of a listings authority.

[5]

(ii)

State the information you would wish to see in order to provide a valuation.
[5]
[Total 10]

END OF PAPER

ST5 S20075

Faculty of Actuaries

Institute of Actuaries

EXAMINATION
September 2007

Subject ST5 Finance and Investment


Specialist Technical A
EXAMINERS REPORT
Introduction
The attached subject report has been written by the Principal Examiner with the aim of
helping candidates. The questions and comments are based around Core Reading as the
interpretation of the syllabus to which the examiners are working. They have however given
credit for any alternative approach or interpretation which they consider to be reasonable.

M A Stocker
Chairman of the Board of Examiners
December 2007

Faculty of Actuaries
Institute of Actuaries

Subject ST5 (Finance and Investment Specialist Technical A) September 2007 Examiners Report

Comments
Most candidates scored well on questions 1, 3 and 4 with many achieving full marks.
Although some candidates scored well on questions 2 and 5 also, many candidates attained
closer to half the available marks. Questions 6, 7 and 8 were the worst answered (7 in
particular).
In every diet there will be candidates who are very close to the pass mark and yet receive an
FA indeed I suspect candidates would be very surprised to see just how tightly distributed
the marks are; deciding where the pass mark falls will have a material impact on the
numbers of candidates who are successful and the examiners take great care to ensure a
consistency of standard across candidates, subjects and diets. That said, it was fairly clear
where the hurdle should have been set. The examiners were pleased to see that the pass rate
for this diet was slightly higher than last time even though the pass mark was somewhat
higher. Where candidates scored lower it was typically because although they were able to
reproduce the required bookwork for one or other question, they were unable to apply the
bookwork knowledge appropriately.
Candidates should note the bias in the paper towards recognising higher level skills and
practical application this is intentional and will continue. Likewise the examination system
does properly allow for prior subject knowledge to be assumed. Investment is a necessarily
practical subject and at this level, the examiners expect candidates to demonstrate a breadth
and depth of competency as would be expected from a practising actuary in what is a
frequently evolving discipline. Hence simple regurgitation of bookwork will not be sufficient
to ensure a Pass grade.
Candidates looking to progress should be aware that the SA series of exams, particularly
investment related, are even less bookwork focussed and require the candidate to
demonstrate a breadth and depth of competency as would be expected from a practising
actuary in a constantly changing discipline.
In order to succeed, candidates should ensure they familiarise themselves with the current
investment issues and general market background facing institutional investors in the 18
months preceding a diet and the solutions (and sources of) being debated by the various
stakeholders. A recurring theme in recent years has been a move towards capital market
rather than purely insurance and asset management solutions hence a question regarding
banking and derivative approaches to asset and liability risk management or modern
financial theory and commercial applications should be considered likely scope for
examination.
All extenuating and mitigating circumstances were considered in awarding grades.

Page 2

Subject ST5 (Finance and Investment Specialist Technical A) September 2007 Examiners Report

(i)

To correct market inefficiencies and to promote orderly markets


To protect consumers of financial products
To maintain confidence in the financial system

(ii)

Direct cost

administering the regulation and compliance of firms

Economic cost

An alternation in the behaviour of consumers, who may be given a false


sense of security and a reduced sense of responsibility in their own actions
Undermining of the sense of professional responsibility amongst
intermediaries and advisors
A reduction in consumer protection mechanisms by the market itself
Reduced product innovation
Reduced competition

Money Market Instruments (Unit 5)


(i)

Term loans
Evergreen credit
Revolving credit
Bridging loans
International bank loans
Trade credit

(ii)

Commitment
Maturity
Rate of interest
Security

Page 3

Subject ST5 (Finance and Investment Specialist Technical A) September 2007 Examiners Report

Money Market Instruments (Unit 5)


(i)

Company A, AAA or AA with supporting argument


Company B, AA or A with supporting argument
Company C, BBB/Junk Bond

(ii)

EDL = Value of Treasury Bond Value of Corporate Bond


The expected default loss will go up from A through to C

(iii)

(a)

The following yields or similar would be expected (different yields


would impact subsequent calculations illustrated).
Company
Company A
Company B
Company C
Treasury Bond

(b)

Yield
4.75%
6.00%
8.00%
4.25%

Assume all trade at par.


Treasury Bond = 95.84
Company A = 95.36 loss = 0.48
Company B = 94.18 loss = 1.66
Company C = 92.31 loss = 3.53

(iv)

All else being equal, all bond yields will increase by 100bps to reflect the
change in government bond yields.
However, the equity market has fallen which would imply that there is
concern about the corporate sectors future economic prospects or that
earnings have fallen.
This would suggest a widening of spreads on corporate borrowings relative to
government debt.
The impact on each of the three companies will vary depending on the
sensitivity of their existing and future revenue streams to the factors causing
the economic downturn.
In practice, it is likely that there would be a flight to quality reflecting
reduced liquidity in poorer credits and greater concerns about defaults.
This would mean that demand for higher quality bonds increases and the
demand for lower quality bonds reduces.

Page 4

Subject ST5 (Finance and Investment Specialist Technical A) September 2007 Examiners Report

This would imply that the spreads for companies A/B/C might increase from
50/175/375 bps to 5075/200250/425500bps, before allowing for company
specific factors.

Derivatives (Unit 7)
(i)

The main uses of derivatives are as follows:

Speculation
Exchange-traded derivatives could be used for speculation; effectively
betting on a strong view of a particular market movement. The difference
between speculation using options and speculation using the underlying
asset is that buying the underlying asset requires an initial cash outlay
equal to the total value of what is bought whereas entering into a future
contract or an option contract requires only a fraction of the initial cash
outlay. Thus a much higher level of leverage (gearing) can be achieved.

Arbitrage
Arbitrage involves locking in a riskless profit by simultaneously entering
into two transactions in two or more markets. Using various combinations
of options and the underlying instruments, portfolios with the same return
but with different constituent parts can be created. Arbitrage opportunities
can arise when the prices of these different portfolios get out of alignment
and a riskless profit can be made.
In practice only very small arbitrage opportunities are observed in prices
that are quoted in most financial markets. Also, transaction costs would
probably eliminate the profit for all but the very large investment houses
that face very low transaction costs.

Hedging
Hedging allows a fund manager to reduce a risk that the fund already
faces. Hedging using options, for example, involves taking a long or short
position in a number of options contracts which is the opposite to the
position held in the underlying asset. Conceptually, a loss made in the
underlying asset will be offset by an approximately equal gain on the
options position.
This technique is very useful where say a fund is going to sell its holding
in two or three months and it wishes to avoid a fall in market values.
However, if the market rises there will be a loss on the futures position
approximately equal in value to gain on the underlying equities so the
strategy does close off the opportunity for the fund to participate in any
upward movements in the underlying assets while the hedge is in place.

Page 5

Subject ST5 (Finance and Investment Specialist Technical A) September 2007 Examiners Report

Portfolio management
Options can be used to manage the reallocation of assets from one market
to another. For example, call options on equity indices can be used to gain
exposure to upside movements in the markets; put options can be used to
remove exposure to downside movements in markets. Calls and puts can
be used to change a funds exposure to an asset category or to change a
funds exposure within an asset category.

(ii)

(a)

Margin definition Unit 7 pg1.

(b)

Initial Margin the initial payment put down to cover the risk of the
contract.
Variation Margin the margin which is payable or received on a
daily basis to mark to market.

(iii)

(c)

The clearing house is removing the credit risk and they need some
form of compensation to cover themselves for this risk.

(a)

Put the right to sell an underlying asset for a certain price by a


certain date.
Call the right to buy an underlying asset for a certain price by a
certain date.

(b)

(iv)

American exercised at any date to expiry. European can only be


exercised on expiry date.

Call the loss on the 3 month is 5p to 80p when break even then in the money.
6 month is 10p loss until 85p.
Put 3 month is in profit to 75p then loss of option, for 6 month is in loss at
80p.

Page 6

Subject ST5 (Finance and Investment Specialist Technical A) September 2007 Examiners Report

Government Policy (Unit 11)


(i)

Main forms of government policy


Monetary Policy
Fiscal Policy
National debt management policy
Exchange rate policy
Prices and income policy

(ii)

Main economic indicators


Unemployment
Inflation
Balance of Payments
Economic Growth

(iii)

(a)

Individuals reduce variable rate mortgage and debt payments


(making people better off). Consumers likely to increase spending on
non-essential services and luxury items. Reduced rate of interest will
act as a disincentive for cash savings, although this may be offset by
reduced debt payments for some. Imported goods likely to increase in
cost due to impact on exchange rates.

(b)

Businesses Capital investment and economic growth likely to


increase due to reduced opportunity cost of committing funds.
Increased economic activity likely, offset by lower domestic currency
rates which will increase cost of imports.

(c)

Economy increased inflation and growth expectations (from a low


base), possible recovery and increase in inward foreign investment.
Longer term concerns about uncontrolled inflation may begin to
emerge.

Page 7

Subject ST5 (Finance and Investment Specialist Technical A) September 2007 Examiners Report

Hedge Funds (Unit 5)


(i)

(a)

Global Funds
Event-driven Funds
Market Neutral Funds

(b)

Describe the main investment characteristics of a Hedge Fund


Placing large bets on different asset classes
High level of borrowing
Mix of investments
Willingness to trade in derivatives
Illiquid
High Fees
High risk

(ii)

Partial market coverage


Survivorship bias
Selection bias
Marking to market bias

(iii)

Issues for scheme, trustees and sponsors


Capital management or financing
Pension management

Liability cash flow management


Compensation for inadequacies of bond markets

Deficit reduction
Regulatory, financial and peer group pressures
Finite repair term imposed by regulators scope for absolute return
structures

Surplus control
Short term fix may prove over cautious

Sponsor may be more concerned with volatility of deficit and


earnings/corporate activity impacts rather than size of deficit itself

Page 8

Cash flow hedging coupled with Regulator proposals for Deficit Repair
terms change pension funds from Relative (to yields, inflation) to
Absolute/Targeted Return investors
Alternative assets, including hedge funds, have obvious role in Deficit
Repair

Subject ST5 (Finance and Investment Specialist Technical A) September 2007 Examiners Report

Market neutral hedge funds plus swaps/bonds equal Liabilities Plus fund

Bulk of exposure is through Fund of Funds

Hedge funds have a role as the alpha generator in new products, but could lose
their separate identity

(i)

Not all managers, perhaps even the majority, are skilful

Need a rigorous and different process to identify and invest in persistent


skill

Operational risk is the risk of loss resulting from inadequate or failed internal
processes, people, and systems or from external events.
Operational risk includes IT, legal and compliance risk.
Operational risk differs from market or credit risk as it is typically not directly
taken in return for an expected reward
Operational risk exists in the natural course of corporate activity
Operational risk is more difficult to quantify and measure compare with
market or credit risk.
Operational risk is very important as it seems to have been responsible for
more spectacular corporate losses than the other financial risks.

(ii)

The control of operational risk essentially depends on good management


practices.
To manage operational risk, the investment manager would need to identify,
assess, put together a series of risk mitigation strategies one for each nontrivial risk.
Good management practices include:
Established and documented chains of reporting and responsibility
Separation of duties as between say, front office and back office staff in the
issuing of trading instructions and the confirmation & settlement of such
instructions.
Documented and robust procedure for carrying out essential tasks and for
taking on new activities or developing new products
Ongoing monitoring of risks and their mitigation techniques

Page 9

Subject ST5 (Finance and Investment Specialist Technical A) September 2007 Examiners Report

(i)

A listings authority is responsible for ensuring that any new issue of shares is
conducted in an orderly and fair way, and that the conduct of the company
remains consistent with the listing of the shares after the issue.
A listing authority will ensure that a reasonable amount of financial
information is in the public domain.
Listing authorities are normally concerned with:

(ii)

The production of relevant business and financial information on the issue


of shares.

The process by which shares are offered to potential shareholders and the
price is set for the issue of shares.

Continuing production and dissemination of business and financial


information on a timely basis on companies with listed securities.

The continuing conduct of the market in listed securities with a view to


ensuring that the market is fair to all participants, and that the pricing
process is fair and reasonable.

Rules to ensure that companies with listed securities and connected parties
continue to behave in a manner that does conflict with other objectives of
the listing authority.

The value of the company will be driven by the level and likelihood of future
profits.
General factors

Page 10

Information about the management and an assessment of their ability to


deliver in a public company.
The type and quality of the products sold
Prospects for market growth
How the company fairs against the competition
Details of operating costs
Details of past retained profits
The history of the company

Subject ST5 (Finance and Investment Specialist Technical A) September 2007 Examiners Report

Financial measures

Financial accounts and accounting ratios


Dividend and earnings cover
Profit variability and growth
Level of borrowing
Level of liquidity
Growth in asset values
Comparative figures for other similar companies

END OF EXAMINERS REPORT

Page 11

Faculty of Actuaries

Institute of Actuaries

EXAMINATION
16 April 2008 (pm)

Subject ST5 Finance and Investment


Specialist Technical A
Time allowed: Three hours
INSTRUCTIONS TO THE CANDIDATE
1.

Enter all the candidate and examination details as requested on the front of your answer
booklet.

2.

You have 15 minutes at the start of the examination in which to read the questions.
You are strongly encouraged to use this time for reading only, but notes may be made.
You then have three hours to complete the paper.

3.

You must not start writing your answers in the booklet until instructed to do so by the
supervisor.

4.

Mark allocations are shown in brackets.

5.

Attempt all 7 questions, beginning your answer to each question on a separate sheet.

6.

Candidates should show calculations where this is appropriate.

AT THE END OF THE EXAMINATION


Hand in BOTH your answer booklet, with any additional sheets firmly attached, and this
question paper.
In addition to this paper you should have available the 2002 edition of the Formulae
and Tables and your own electronic calculator from the approved list.

ST5 A2008

Faculty of Actuaries
Institute of Actuaries

(i)

List the types of activity central banks engage in.

[3]

(ii)

Define liquidity risk.

[2]

(iii)

Explain why it might be desirable for a central bank to act as a lender of last
resort to private sector banks, commenting on the nature of banking assets and
liabilities.
[4]

(iv)

Outline the disadvantages of there being a lender of last resort system in place.
[3]

(v)

Describe ways in which these disadvantages could be mitigated.

[2]
[Total 14]

The trustees of a UK pension fund with 800 million in actively managed assets are
looking to restructure the assets in order to more closely match the liabilities. The
current and target structures of the assets are given in the table below. Assets are
managed by three managers currently. All three managers are to be replaced with two
new managers.
Current
Assets
US equities
UK equities
Emerging market equities
Private equity
UK gilts
Total

Current Value
(m)
400
100
100
150
50
800

Target
Assets

Target
Value (m)

UK equities
UK gilts
Overseas bonds

300
400
100

Total

800

The trustees want to move to the target structure immediately but have not yet chosen
the managers for the target structure.
(i)

Describe the biggest mismatches between the current and target portfolios. [2]

(ii)

Outline how the trustees can move the assets towards the target structure
before the new managers for the target portfolio have been appointed.

[4]

Outline the limitations and downsides of the strategies proposed.

[4]

(iii)

The trustees finally decide on the target managers and want to go ahead with the
move to the new structure, however market conditions have changed and liquidity has
decreased and volatility has increased.
(iv)

List the various costs that are incurred when transferring assets.

(v)

Describe how the costs identified in (iv) will be affected by the current market
conditions.
[3]
[Total 15]

ST5 A20082

[2]

You are working for a life office in their investment team and have been presented
with the opportunity to buy for 150m a freehold on an office block that is currently
occupied by a bank. Two years ago, the bank had arranged a 32 year lease with the
current freeholder, as follows:
Term of lease

32 years

Annual rent

First 3 years 9m p.a., payable annually in advance


Thereafter

9m p.a. plus 5 years cumulative inflation,


increasing in line with future inflation, payable
annually in advance

Ground rent

100 p.a. payable annually in advance

Inflation lag

3 months

(i)

Write down an equation for the present value of the remaining rental
payments, expressed in terms of zero coupon interest rates (zt) and inflation
rates (zinf,t)
[3]

When the bank hears that the freehold is in the process of being sold, it offers to set
up an inflation swap to exchange the inflation-linked rental payments for fixed
payments. This would be a separate contract to the lease, and would be subject to
daily collateralisation.

(ii)

Describe the cashflows that would be paid and received under the inflation
swap with the bank.
[4]

(iii)

Explain why the life office might feel the inflation swap makes this transaction
more attractive, despite paying a margin to the bank arranging the swap. [2]

(iv)

Describe the various risks that apply to the life office under the freehold, the
lease and the swap, and explain how they might vary over time and according
to economic factors.
[9]
[Total 18]

(i)

Describe the two principal global equity index series.

(ii)

List five ways in which a large institutional investor can achieve the returns
(gross of costs and tax) of a global equity index.
[3]

(iii)

Explain why this type of index would be more useful for performance
measurement for an overseas investor than the most widely quoted local
equity index.

(iv)

[2]

[2]

Explain why this type of index would be less suitable than the most widely
quoted local equity index as a base for exchange-traded derivative contracts.
[5]
[Total 12]

ST5 A20083

PLEASE TURN OVER

(i)

List the principal aims of financial regulation.

[2]

(ii)

(a)

Define a self-regulatory system.

(b)

Discuss the advantages and disadvantages of self regulation in the


investment management and the securities industries.
[6]
[Total 8]

A risk averse individual coming up to retirement age has around 1% of their


retirement funds invested in the shares of a highly respected bank. The bank has
historically been involved in low risk activities, producing steady returns. Since new
management was put in place 3 years ago the bank has been involved in a number of
high profile risky investments which have gone wrong. As a result there has been a
sharp decline in the share price.
(i)

Outline how the change in management has affected the risk and return profile
of the individuals investment portfolio.
[2]

The individual had the opportunity to sell the stock 6 months ago, but decided to hold
onto the stock. Since then, the share price has fallen further.
(ii)

Outline the various reasons why the stock might not have been sold.

[5]

(iii)

Discuss whether the investment is suitable for the individuals circumstances.


[2]

The investor believes the share price has reached its lowest point and expects it to rise
in the near future. The investor wants to try to make back some of their losses.
(iv)

Describe a technique, using the current share price, that the investor could use
to make a profit on their holding without selling any shares.
[2]

The investor decides to use the technique in (iv).


(v)

Describe the effect on the investors exposure to the bank if the banks share
price rose by 30%.
[2]

(vi)

Describe the effect on the investors exposure to the bank if the banks share
price fell by 30%.
[2]
[Total 15]

ST5 A20084

(i)

(ii)

(iii)

List six factors that are important to take into account when valuing a
company.

[3]

List six sources of information that an analyst may use when valuing a
company.

[3]

Describe how the P/E ratios of the following types of company may change
through an economic cycle:
(a)
(b)
(c)

a house builder
a tobacco company
a retail bank
[9]

Rather than look at P/E ratios an analyst has decided to value the companies within
his sector on a discounted cash flow basis.
(iv)

Discuss the advantages and disadvantages of adopting this methodology. [3]


[Total 18]

END OF PAPER

ST5 A20085

Faculty of Actuaries

Institute of Actuaries

Subject ST5 Finance and Investment


Specialist Technical A
EXAMINERS REPORT
April 2008

Introduction
The attached subject report has been written by the Principal Examiner with the aim of
helping candidates. The questions and comments are based around Core Reading as the
interpretation of the syllabus to which the examiners are working. They have however given
credit for any alternative approach or interpretation which they consider to be reasonable.

M A Stocker
Chairman of the Board of Examiners
June 2008

Faculty of Actuaries
Institute of Actuaries

Subject ST5 (Finance & Investment Specialist Technical A) April 2008 Examiners Report

Comments
Most candidates scored well on questions 5, 7 and to a lesser extent 2, with many achieving
close to full marks. Questions 4, 6 and 3 were the worst answered (3 in particular, with the
average candidate achieving less than a third of the marks available).
In every diet there will be candidates who are very close to the pass mark and yet receive an
FA indeed candidates would be very surprised to see just how tightly distributed the marks
are; deciding where the pass mark falls will have a material impact on the numbers of
candidates who are successful and the examiners take great care to ensure a consistency of
standard across candidates, subjects and diets. That said, it was fairly clear where the
hurdle should have been set with a clear distinction between candidates graded as a Pass
and not. The examiners were disappointed to see that the pass rate for this diet was slightly
lower than last time given the pass mark was lower too. Where candidates scored lower it
was typically because although they were able to reproduce the required bookwork for one or
other question, they were unable to apply the bookwork knowledge appropriately. Few
candidates provided satisfactory answers to calculation questions.
Given the intent of the profession to push out in to wider fields involving the practical
application of actuarial skills in financial risk management and the increasing numbers of
candidates sitting this exam, it continues to be a disappointment that many candidates
achieve such low scores. Indeed, it is most astonishing the numbers who achieve grades of
FC and FD since this would imply very little knowledge and understanding even after a
course of study.
Candidates should note the bias in the paper towards recognising higher level skills and
practical application this is intentional and will continue. Likewise the examination system
does properly allow for prior subject knowledge to be assumed. Investment is a necessarily
practical subject and at this level, the examiners expect candidates to demonstrate a breadth
and depth of competency as would be expected from a practising actuary or senior student in
a frequently evolving discipline, particularly for those looking to progress to SA6. Hence
simple regurgitation of bookwork will never be sufficient to ensure a Pass grade.
As noted before, in order to succeed, candidates must ensure they familiarise themselves with
the prevailing investment issues and the general market background facing institutional
investors in the 18 months preceding a diet, more so the solutions (and sources of) being
debated by the various stakeholders. A recurring theme in recent years has been a move
towards capital market rather than purely insurance and asset management solutions hence
questions regarding corporate finance, banking and derivative approaches to asset and
liability risk management or modern financial theory and commercial applications should be
considered likely scope for examination. Likewise regulation and globalisation are common
issues in many areas.
All extenuating and mitigating circumstances were considered in awarding grades.

Page 2

Subject ST5 (Finance & Investment Specialist Technical A) April 2008 Examiners Report

(i)

determining monetary, interest rate and inflation policy


regulation of banks
implementation of government borrowing
ensuring the performance and integrity of financial markets
intervention in currency markets
printing and minting of notes and coins
taxation
lender of last resort

(ii)

Liquidity risk is the risk that asset owner is unable to recover full value of
asset when sale is desired (or for a borrower, the risk of credit being
unavailable when an maturing loan needs to be refinanced/rolled over)

(iii)

Banks typically hold significant amounts of medium to long dated loan assets
on their balance sheets, which are highly illiquid. In contrast, their liabilities
will typically be shorter-term in nature, including deposits and shorter-term
inter-bank loans.
Without a lender of last resort (LOLR), a bank is exposed to the risk that it
will not be able to maintain payments to its creditors if sufficiently many
deposits are withdrawn or if it is unable to refinance maturing loan payments.
The perception that a bank is nearing such a position can lead to a run on the
bank as deposits are easily withdrawn which can have wider social and
economic impacts.
With a LOLR, a bank is much less likely to end up in such a position. Hence
most developed economies have a LOLR system in place, explicitly or
implicitly.

(iv)

The key disadvantage is moral hazard. The management of banks would have
a weaker incentive to manage liquidity (by term of cashflows under assets and
liabilities) as carefully as if a LOLR was not present. This reflects that in the
latter scenario a bank would become insolvent and either require bailing out
by an acquirer, or creditors to the bank would incur losses (and shareholders
would almost certainly see their capital extinguished, and management would
lose their jobs).
Moral hazard can also be argued to extend to depositors: with a LOLR system
a depositor would not need to assess the credit standing of the bank accepting
the deposit.
Most countries have taken the view that this latter aspect of moral hazard is
acceptable, whereas the former is less acceptable (except where necessary to
prevent contagion to other financial institutions).

Page 3

Subject ST5 (Finance & Investment Specialist Technical A) April 2008 Examiners Report

The other key disadvantage is whether the losses that might accrue to the
central bank under the LOLR system would ultimately be borne by taxpayers.
However in the long run it is not good for the economy for an inefficient
business to receive such support.
(v)

Moral hazard and central bank losses can be reduced by ensuring that banks
borrowing from the LOLR pay a penal rate of interest on loans. In reality this
is unlikely to compensate for the credit risks associated with this type of
lending.
Other measures may include requiring additional collateral for LOLR loans,
nationalisation of a failing bank and ensuring that all other sources of
financing have been explored including acquisitions or other marriage
broking.
Regulation of liquidity management, asset quality, approved persons for
management may also mitigate the disadvantages.

(i)

Asset class mismatch between equities and bonds


Country mismatch between overseas assets and UK assets
No private equity holding going forward

Use of futures/derivatives/swaps to make current assets look like the target


assets
Detailed example e.g.: Short US equity futures by 400m, emerging
market 100m, private equity 150m
Long UK equity 200m, UK gilts 350m, overseas bonds 100m
Ask current managers to restructure their current holdings to replicate asset
classes using physical holdings

(ii)

(iii)

Page 4

Futures might not have suitable futures, especially private equity,


basis risk between active manager and future benchmark,
costs of putting on derivative position,
extra management required to manage future contracts
Physical sell investment costs (commissions, taxes) of moving to interim
strategy and then extra costs of moving to final strategy once selected.
Existing managers might not have capability of managing in other asset
classes.

Subject ST5 (Finance & Investment Specialist Technical A) April 2008 Examiners Report

(iv)

Taxes, commissions, spread costs, market impact costs, FX costs,


administration and custodian costs

Taxes, commissions likely to be unchanged


Spreads widen
Market impact costs increased
Custodian costs likely to increase as larger number of split trades due to
poor liquidity
Probably moving to more liquid/tradable currencies within overseas bond
exposures but will need to sell out of emerging positions

(v)

(i)

Reasonable formulae, with consistent assumptions, such as:

z inf 0.25 29
PV = 9m 1 + 1
1 + ztinf 0.25

Inf
1 0.25 5 t =1

) (1 + z )
t

29
t
+

100

(1 + zt )
t =0

where Inft is the realised inflation index at time t, zt is the zero coupon bond
rate at time t and ztinf 0.25 is the inflation rate at time t (with 3 month lag)
credit also available for assuming that the inflation rate is a series of forwards, in
which case the formula would be as follows:

z inf 0.25 29 t
PV = 9m 1 + 1
1 + ztinf 0.25

Inf
1 0.25 5 t =1 i =1

(ii)

)(1 + z )

29
t
+

100

(1 + zt )
t =0

The first 1 payment (at time t=0) would be excluded from the inflation swap
as there is no inflation linkage. The remaining 29 payments are inflation
linked.
Under these latter payments, the life office would pay out 9m pa plus 5
years known historic inflation (from 5y 3m ago to 3m ago, assuming a 3
month lag in obtaining inflation data) plus actual future inflation (from 3m ago
to the payment date less 3 months) at the time of each annual payment. In
return the life office would receive 9m pa plus 5 years known historic
inflation (from 5y 3m ago to 3m ago, assuming a 3 month lag in obtaining
inflation data) plus expected future inflation under the inflation swap curve
(from 3m ago to the payment date less 3 months).

(iii)

From a regulatory capital perspective the life office may find a fixed nominal
payment more attractive than an inflation-linked payment if its liabilities are
fixed in nature.
From a valuation perspective, the life office may feel that inflation is an asset
that is worth selling if it expects inflation in the future to be lower than the
current breakeven rate of inflation in the swap markets.

Page 5

Subject ST5 (Finance & Investment Specialist Technical A) April 2008 Examiners Report

(iv)

The key sources of risk are:


Freehold

Downturn in the economy can lead to a decline in value as occupancy


levels fall. The long lease provides a degree of protection provided the
bank continues to occupy the building and maintains rental payments.

Oversupply of new office space leads to a decline in value as occupancy


levels fall. The long lease provides a degree of protection.

Location falls out of favour. Reversion value will therefore fall (relative to
similar properties in other locations), and lower rent likely to be realised
on a fresh lease. Long lease provides some protection.

Building design/specification falls out of favour. Reversion value will


therefore fall (relative to other properties in same locations), and lower
rent likely to be realised on a fresh lease. Long lease provides some
protection.

Lease

Tenant cancels lease. This becomes increasingly likely towards the end of
the lease as the tenant will be looking at its needs in the future, and there
may be fewer penalties for cancelling late in the term (assuming a market
rent is being charged, otherwise there would be an incentive to stay or sublet the building).
Earlier in the lease this is a possibility due to restructuring or M&A
activity (or possibly the default of the bank) a new tenant would need to
be found and potentially the inflation swap may no longer match the new
lease.

Tenant renegotiates lease. This could happen at any time during the lease,
and becomes more likely if economic factors are such that rental yields are
falling generally (depending on the terms of the lease). Mismatching issue
if the renegotiated lease breaks the direct inflation link assumed under the
inflation swap (eg move to fixed % increases each year, or rent review
based on rents on comparable properties).

Cost of fulfilling covenants or pursuing leaseholder to fulfil

Void risks. Under either scenario above, there is a risk of void


(incomplete/nil occupancy) which would lead to a loss of rental income.
Void risk increases with time and a downturn in the economy

Inflation swap

Page 6

Changed/broken/new lease. The inflation swap would need to be


cancelled/netted out or run off as the hedge against the inflation linkage in

Subject ST5 (Finance & Investment Specialist Technical A) April 2008 Examiners Report

the rental payments would no longer be valid. This would create further
costs and possible liquidity issues.

(i)

Default risk (if explained well)

FTSE World Indices


The FTSE World Indices cover over 80% of the worlds equity markets in
terms of market capitalisation and are broad market indices that aim to cover
the vast majority of the free float stocks that are available to overseas
investors. Index values are shown for each country, in 5 currencies: Sterling,
US Dollar, Japanese Yen, Euro and the local currency. There are also regional
groupings of countries (weighted by market cap) and an All-World Index.
Morgan Stanley Capital International Indices
This is a series of international equity indices covering both developed and
emerging markets. They are calculated on a market capitalisation weighted
arithmetic basis and total returns are published both gross and net of
withholding tax, and in US Dollar, Euro and local currencies.

(ii)

Replication in tracker portfolio


Sampling
ETF
Buy futures
Buy assets (e.g. cash or stocks) and engage in an OTC swap to pay the return
on the assets and receive equity market return

(iii)

Stocks not available to foreign investors are not included in these indices. This
is not the case for most local indices, so these are often more suitable for
performance measurement purposes than local indices.
Some local indices are weighted averages or total return based
They have a consistent methodology between countries.
They are easier to obtain than some local indices (single data source).
Also, some local indices do not restrict constituent weightings to the free float,
which means they are unsuitable for performance measurement purposes.
Finally, there are index values shown net of withholding taxes and in various
currencies which may be helpful for an overseas investor.

(iv)

The key feature that is needed for an exchange-traded derivative contract is


liquidity. This means that the contract should be based on the most popular
local index to maximise demand from users of the derivative.
This is less of an issue for OTC markets, where client preferences will be
more important.
Liquidity of the underlying stocks is also important to ensure that the
derivative price closely tracks the underlying stocks (as pricing variations can
be arbitraged away rapidly).
This leads to a preference for large-cap stocks over small-cap stocks within the
underlying index.

Page 7

Subject ST5 (Finance & Investment Specialist Technical A) April 2008 Examiners Report

Using a large-cap index as opposed to a broad-market index also means that


the price of the derivative and index value are both available continuously
throughout the day, improving liquidity further.
Where the underlying index is not continuously quoted or the index
constituents are illiquid, it is likely that pricing of the derivatives will include
wider spreads to reflect this illiquidity and uncertainty.

(i)

The principal aims of regulation are:


to correct market inefficiencies and to promote efficient and orderly
markets
to protect consumers of financial products
to maintain confidence in the financial system
to help reduce financial crime

(ii)

A self-regulatory system is organised and operated by the participants in a


particular market without direct government intervention.
Advantages:
The system is implemented by the people with greatest market knowledge.
System is implemented by people who are incentivised to maximise cost
benefit ratio of regulation and ensure system is non-bureaucratic.
Should be more flexible than the alternatives and be able to respond rapidly to
changes in market needs.
Cooperation with a self-regulatory organisation may be more forthcoming
than with a government agency (but not necessarily).
Disadvantages:
Regulator will be perceived to be closer to industry than customers.
This can lead to a lack of confidence from consumers and purchasers
particularly when criticism of industry is high in the wider economy
Self-regulatory organisation likely to have fewer powers to fine and punish
industry members than a government agency established under statute.
Barriers to entry

Page 8

(i)

The bank has become a riskier investment there may be an additional sector
risk if this banks performance impacts or infers a wider contagion.
As the bank increases its risk the expected return investors seek from the
investment should also increase
However, the investment only makes 1% of portfolio so although bank has
increased in risk the impact at the total portfolio level should be minimal.

(ii)

Expect the share price to increase


Regret aversion by maintaining existing arrangements people minimise the
pain associated with feeling of loss
Overconfidence in their ability
Status Quo bias like to keep things the same
Good diversifier in overall portfolio

Subject ST5 (Finance & Investment Specialist Technical A) April 2008 Examiners Report

Could be part of an index tracking portfolio


Tax considerations
Income may be appreciated

(iii)

Individual is close to retirement hence would expect to be in less risky


investments where capital more guaranteed.
However, depends exactly when investor expects to retire.
Small part of overall portfolio so might be suitable if most other holdings are
relatively safe.
If majority of investments are equity then probably not suitable.

(iv)

Write options eg put options for a lower price than the current share price for
which the investor will receive a premium.
The closer to the current price the higher the premium they will receive.

(v)

Using example in (iv) to answer (v) and (vi):


Will keep premium
Current holdings will increase in value by 30%

(vi)

Will make loss on put options and


Would have to buy additional shares at agreed price
Making higher loss and increasing exposure to bank.
Existing holdings decrease in value

(i)

Factors that are important in valuing a company:


Management ability
Quality of products
Prospects for market growth
Competition
Input costs
Retained profits
History

(ii)

Sources of information:
The financial press and other commercial information providers
The trade press
Published accounts
Public statements by the company
The exchange where the securities are listed
Government sources of statutory information that a company has to provide
Visits to the company
Discussions with the companys management
Discussions with competitors
Stockbrokers publications.

Page 9

Subject ST5 (Finance & Investment Specialist Technical A) April 2008 Examiners Report

(iii)

(a)

A house builder is economically sensitive.


The earnings will rise and fall with the economic cycle, they are
volatile.
Investors try to anticipate the future earnings profile.
When the economy is buoyant earnings will be high however investors
will be discounting an economic slowdown and therefore the p/e will
be lower than the average p/e for the company through the cycle.
Similarly when earnings are low the p/e is likely to be high as investors
look forward to an upswing in the economy.

(iv)

(b)

Tobacco companies are economically insensitive.


Therefore the p/e of the tobacco company is relatively insensitive to
the economic cycle.
Investors have no need to anticipate the future earnings profile.

(c)

A retail bank is economically sensitive


The earnings will rise and fall with the economic cycle, they are
volatile.
Investors try to anticipate the future earnings profile.

Discounted cash flow valuations can be very useful in valuing a company.


The result of a discounted cash flow number is an absolute number that can be
compared to other DCF valuations.
DCF valuations can be used where a company generates no profit and pays no
dividend for instance a start up.
It would depend on the sector being analysed, some sectors are more suited to
DCF valuations than others.
DCF valuations are more time consuming.
DCF valuations can be very sensitive to the assumptions made e.g. the
discount rate and the growth rate.
DCF can be difficult to explain to others.

END OF EXAMINERS REPORT

Page 10

Faculty of Actuaries

Institute of Actuaries

EXAMINATION
24 September 2008 (pm)

Subject ST5 Finance and Investment


Specialist Technical A
Time allowed: Three hours
INSTRUCTIONS TO THE CANDIDATE
1.

Enter all the candidate and examination details as requested on the front of your answer
booklet.

2.

You have 15 minutes at the start of the examination in which to read the questions.
You are strongly encouraged to use this time for reading only, but notes may be made.
You then have three hours to complete the paper.

3.

You must not start writing your answers in the booklet until instructed to do so by the
supervisor.

4.

Mark allocations are shown in brackets.

5.

Attempt all 7 questions, beginning your answer to each question on a separate sheet.

6.

Candidates should show calculations where this is appropriate.

AT THE END OF THE EXAMINATION


Hand in BOTH your answer booklet, with any additional sheets firmly attached, and this
question paper.

In addition to this paper you should have available the 2002 edition of the Formulae
and Tables and your own electronic calculator from the approved list.

ST5 S2008

Faculty of Actuaries
Institute of Actuaries

(i)

Set out a formula for calculating the information ratio for a portfolio that
contains two asset classes, defining all terms used.
Asset
Class

1
2
3
Risk Free
(ii)

Tracking
Error (%)

10.0
7.0
5.0
4.0

10.0
5.0
5.0

Asset Class
Correlations
1
2
3
1.00
-0.25
0.25

1.00
0.50

1.00

Using the above data, for each pair of asset classes calculate:
(a)
(b)

Annual
Return (%)

the proportions of each class that satisfy a minimum variance portfolio


the information ratio for each portfolio
[8]

(iii)

Comment on the results in (ii) (b).

(iv)

Explain the effects that gearing and being able to sell short might have on
portfolio returns and the information ratio.
[3]
[Total 16]

[2]

You are the property fund manager at a large life insurance company and have been
approached by a small retail chain that wishes to sell and leaseback the six stores in its
chain.
Discuss the factors that you would consider in determining the sale price.

[3]

[10]

A passive investment manager is planning to launch a high alpha fund with the aim of
outperforming a global index by 3% p.a. The investment manager believes the
outperformance can be generated from three sources:

gaining superior information


processing the information better
eliminating behavioural bias

(i)

(a)

Explain how outperformance might be generated from each of the


three sources.

(b)

Comment on whether the target return can be generated by use of these


sources.

(c)

Comment on whether the 3% p.a. target is reasonable.


[11]

(ii)

ST5 S20082

Discuss the implications the fund launch might have on the structure and
management of the investment management department.
[7]
[Total 18]

A developing country has recently established a stock exchange. There are currently
7 stocks listed, but only one pays a dividend. Dividends are currently paid to
investors free of tax. The company that operates the exchange wishes to create an
index to measure the performance of the listed companies.
(i)

Give the formula that could be used to calculate the index value.

[2]

It has been decided that the initial value of the index will be 10,000 and details of the
7 companies are:
Initial Market
Cap
Initial Price
Company
(EDUm)
(C)
A
B
C
D
E
F
G

500
300
200
700
800
900
1,000

200
150
50
350
400
100
500

Price at end
of first day
(C)

Price at end
of second day
(C)

205
151
50
345
402
102
505

205
155
52
348
380
103
503

(Initial Market Capitalisation is measured in millions of Equivalent Dollar Units


EDU and Prices are in Cents where 100 Cents = 1 EDU)
Company E had declared a dividend of 25 and the stock went ex-dividend at the
beginning of the second day.

(ii)

Calculate the value of the index at the end of day 1 and day 2.

[4]

(iii)

Calculate the total return produced by the index over the two days.

[2]

(iv)

Outline the practical problems that would be encountered by an overseas


pension fund investment manager in using this index as a basis for indextracking management.
[9]
[Total 17]

You are an investment manager working for the investment arm of a small life
insurance company that has used exchange-traded options as part of its equity
portfolio management.
(i)

List the uses of equity market indices.

(ii)

List the main features and characteristics of the main equity indices of UK,
USA, Japan, Germany and France.
[5]

(iii)

State the main uses of exchange-traded options.

(iv)

Outline the appropriateness of using exchange-traded options for your


company.
[4]
[Total 15]

ST5 S20083

[4]

[2]

PLEASE TURN OVER

A wealthy UK investor is considering buying shares in a luxury car producer based in


an emerging country. The unlisted car producer has been running for less than 12
months, but has decided to float its equity on the local stock exchange in 6 weeks
time. The investor has been offered shares at a price which is at a 10% discount to the
expected flotation price.
(i)

Outline the investigations that the investor should carry out prior to deciding
whether to invest in the new company.
[5]

(ii)

Outline the difficulties the investor might encounter when trying to research,
analyse and place a valuation on the company.
[2]

The investor expects the global economy to slow in the next 12 months and enter into
a recession.
(iii)

Discuss how the company and the quoted shares might be affected if the
investors predictions are correct.

[3]

The investor decides not to invest in the stock directly, but takes out a 3 month call
option on the stock at a price which is 5% below the expected flotation price of 50p.
The company floats later in the month. On the first day the stock increases by 3%
over the actual flotation price of 45p.

(iv)

Define the term out of the money for both a put option and a call option
giving a brief example of each.
[2]

(v)

Outline how the price of the call option is likely to have changed in the six
weeks since purchase.
[3]
[Total 15]

The government of an emerging country is encouraging local companies to set up


pension funds for their employees. Historically the government has provided
retirement provision and, therefore, there has been a lack of regulation of institutional
investments. You are a financial advisor to the government and have been asked to
develop a regulatory framework for the new pension funds.
(i)

State why it is important for there to be regulation of institutional investments.


[3]

(ii)

Outline the principles you would recommend adopting under legislation for
institutional investment.
[6]
[Total 9]

END OF PAPER

ST5 S20084

Faculty of Actuaries

Institute of Actuaries

Subject ST5 Finance and Investment


Specialist Technical A
EXAMINERS REPORT
September 2008

Introduction
The attached subject report has been written by the Principal Examiner with the aim of
helping candidates. The questions and comments are based around Core Reading as the
interpretation of the syllabus to which the examiners are working. They have however given
credit for any alternative approach or interpretation which they consider to be reasonable.
R D Muckart
Chairman of the Board of Examiners
December 2008

Faculty of Actuaries
Institute of Actuaries
30/12/2008

Subject ST5 (Finance and Investment Specialist Technical A) September 2008 Examiners Report

General comments
Generally a poorly answered paper than previous diets; although the pass rate was
consistent with recent diets, this equated to a disappointingly lower pass mark. Candidates
typically answered Questions 5 and 6 better than the others, with Questions 1 and 3
attracting the worst responses.
Many candidates seemed to understand the key issues being examined and so appreciated the
general content of solutions that the examiners were looking for however those that were
unsuccessful will find their solutions lacked sufficient (and often the most basic) detail and
scored lower accordingly. Worse, some candidates deviated from the topic and included
irrelevant material although candidates will not be explicitly penalised for this, it gives an
impression of a lack of understanding and, more importantly, wastes valuable time. Where
candidates made relevant points in other parts of their solutions, the examiners have used
their discretion as to whether to recognise these answers or not.
Again there were many candidates close to the pass mark whom were awarded an FA most
candidates would be very surprised to see just how tightly distributed the marks are; deciding
where the pass mark falls will have a material impact on the numbers of candidates who are
successful and the examiners take great care to ensure a consistency of standard across
candidates, subjects and diets. It was fairly clear where the hurdle should have been set; as
a result, the pass rate for this diet was similar to last time. However the pass mark remains
much lower than the examiners feel ought to achievable by candidates, many of whom are
likely to be working as advisers or asset managers in this most practical of fields. Several
candidates were awarded an FD in this diet and the examiners remain concerned by the
numbers of candidates still achieving only an FC grade, since this too would imply little
preparation or, worse, knowledge and understanding.
Candidates are reminded of a bias in the paper towards recognising higher level skills and
practical application this is intentional and will continue. Likewise the examination system
does properly allow for prior subject knowledge to be assumed. Investment is a necessarily
practical subject and, at this level, the examiners expect candidates to demonstrate a breadth
and depth of competency as would be expected from a senior student in a frequently evolving
discipline. Hence simple regurgitation of bookwork will never be sufficient to ensure a Pass
grade and this was evident from the dispersion of candidates responses in the more
differentiating questions.
As noted before, in order to succeed, candidates must ensure they familiarise themselves with
the prevailing investment issues and the general market background facing institutional
investors in the 18 months preceding a diet, more so the solutions (and sources of) being
debated by the various stakeholders. A recurring theme in recent years has been a move
towards capital market rather than purely insurance and asset management solutions hence
questions regarding banking and derivative approaches to asset and liability risk
management or modern financial theory and commercial applications should be considered
likely scope for examination. New asset classes and ways of investment will themselves
generate new types of risk and so the need for new regulation and ways of monitoring and
management.
All extenuating and mitigating circumstances were considered in awarding grades.

Page 2
30/12/2008

Subject ST5 (Finance and Investment Specialist Technical A) September 2008 Examiners Report

(i)

IR = [ri + (1 - )rj rf]/[2i2 + (1 - )2j2 +2(1 - )ijij]


where = proportion in asset class i
ri, rj = annual return for asset class i, j i = 1, 2 i < j 3
rf = risk free return
i, j = tracking error asset class i, j
ij = correlation between asset classes i and j.

(ii)

To satisfy minimum variance differentiate denominator of IR with respect to


and set to zero. This gives a solution of
= [j2 ijij] / [i2 + j2 - 2ijij]
Asset Pair

r (%)

(%)

IR

1,2
1,3
2,3

0.25
0.125
0.5

3.75
1.625
2.00

3.95
4.84
4.33

0.95
0.34
0.46

(iii)

Only the combination of assets 1 and 2 gives a better IR than asset1 or asset 2
on their own. The minimum variance portfolio does not give the best IR for
each combination. This outlines the problem of using minimum variance
portfolios in building a portfolio.

(iv)

Gearing will give additional returns provided the cost of gearing is less than
the assets return. The debt is usually at a fixed rate of interest and so has no
variance. Thus although higher returns are achieved they will be at the cost of
higher risk (tracking error) unless the cost of debt is less than the risk free rate
(unlikely). Thus the information ratio will fall.
Selling short an asset that has a lower expected return and re-investing in a
higher returning asset will increase the return but will significantly increase
the risk. However if the assets are moderately to highly correlated and the
asset being bought has a sufficiently high risk adjusted return over the asset
being sold it will be possible to have an improved IR. However such
situations tend to be arbitraged away very quickly.

The most important consideration is the retailers quality and financial strength as the
rental income from all the properties in the portfolio are dependent on these factors.
It is therefore important to assess the financial position of the company both pre and
post the sale and leaseback.
Why is the company looking to sell the properties?
What will it be doing with the money it raises?
How long has the company been trading?

Page 3
30/12/2008

Subject ST5 (Finance and Investment Specialist Technical A) September 2008 Examiners Report

How long has the current management team been running the chain, do they have any
plans for retirement/succession?
What future plans do the management have for the chain?
Has it been through more than one economic cycle? If so how did it perform during a
recession? If not does it sell necessities or luxury items?
The properties themselves are also important.
Their location both in terms of which towns and the location within the town.
Age and the state of repair.
Could other retailers easily move into the space?
The lease details would need to be determined.
How long are the leases?
Are there any break clauses?
Are the leases on upward only rent reviews?
How often are rents reviewed?
What rent will be paid initially?
Finally details of the yields on similar properties would need to be ascertained.

(i)

(a)+(b) Alpha is the difference between a funds expected returns based on its
beta and its actual returns. Alpha is sometimes called the value that a
portfolio manager adds to the performance. If a fund returns more than
what you'd expect given its beta, it has a positive alpha. If a fund
returns less than its beta predicts, it has a negative alpha.
Looking at the three strands separately:

Gaining superior information

This should not be confused with inside information.


In practice this is probably the most difficult of the three areas to gain a
competitive edge.
Information can be gained from a number of sources the company
itself, their competitors, their customers, their suppliers, the press etc.

Page 4
30/12/2008

Subject ST5 (Finance and Investment Specialist Technical A) September 2008 Examiners Report

In order to gain superior information an analyst will probably need to


spend more time researching the company and its industry.
This has implications for the number of stocks an analyst can cover
and therefore for the total number of analysts required.

Processing the information better

This is an area that has received a lot of attention in the recent past
with the advent of quantitative models.
These models may allow investors to better identity anomalies and
thereby make better decisions.
Given the large amount of data available a system that processes this
information better may well lead to improved or more rapid decision
making.
Given the wide variety of information available any processing system
will have to be very flexible.
Better models of companies and sectors can also be developed to better
predict the future profitability and cash flows of a company or sector.
Again given the diversity of companies and sectors it is difficult to
devise a financial model that will be applicable to all companies.

Eliminating behavioural bias

This is easier said than done.


There are a number of behavioural biases.
Namely: Anchoring, loss aversion, framing, over confidence and
mental accounting.
Elimination of all these sources of bias would take considerable
expertise in the field of behavioural finance.
In order to effectively eliminate these biases some form of mechanistic
investment process may be required.
If it were possible to eliminate behavioural bias this would not
necessarily lead to the generation of alpha as if other investors were
still influenced by behavioural bias these biases could influence asset
prices.
Therefore the elimination of behavioural bias may lead to better long
term performance but it may have a detrimental impact on short term
performance.
Page 5
30/12/2008

Subject ST5 (Finance and Investment Specialist Technical A) September 2008 Examiners Report

(c)

Given/depending on the arguments advanced in (b) it would seem that


the 3% target is challenging, especially allowing for fees of
management. It is possible that such a performance is achievable in
the short term (and this would be borne out by historic manager return
statistics), but may be much harder to sustain a long term competitive
advantage, especially as other managers may come to adopt similar
styles and so arbitrage out the scope for added value.
Credit will be given for other reasoned arguments.

(ii)

In order to implement such a strategy the department would need:

A large number of analysts unless the aim was only to cover a limited part
of the market.

A quantitative team.

Experts in behavioural finance.

A data entry function to input the large amounts of data.

All these functions would need co-ordination and there would need a person or
people to reconcile the different recommendations and actually construct a
portfolio.
It is possible that an investment process with all these inputs may become
unwieldly.
There is also the possibility that the various input streams may produce
conflicting recommendations.
There is a danger that a portfolio constructed using these ideas could exhibit
abnormally high or low tracking errors. This would need careful explanation
to potential clients.
The attribution and explanation of performance would also be very complex.
The cost of such an operation may mean that this approach is only open to
fund managers with significant funds under management.

(i)

The formula is:


Index level at time t = K

Where

i W ( Pit / Pi 0 )
i Wi

Pit is the price of stock i at time t.


Pi0 is the price of stock i at the base date.
Wi is the weight applied to stock i.

Page 6
30/12/2008

Subject ST5 (Finance and Investment Specialist Technical A) September 2008 Examiners Report

K is a constant related to the starting value of the index at the base


date.
The weights used are usually the market capitalisations at the base
date.
The initial value if the index can be set at any number other than zero,
however it is usually a round number e.g. 1,000.
(ii)

From the equation above the value of K in this case = 10,000.


The index value at end of day 1 =
10,000*(512.5+302+200+690+804+918+1010)/4,400 = 10,082.95
The index value at the end of day 2 =
10,000*(512.5+310+208+696+760+927+1,006)/4,400
= 10,044.32

(iii)

In calculating a total return it is usually assumed that dividends are reinvested


back into the index at the ex-dividend date. In this case there was one dividend
of 25 and stock E went ex-dividend at the start of day 2.
Total return = ((10082.95+50/0.44)*10044.32/10082.95)/10,000
= 1.01575
i.e. 1.58%

(iv)

The investors in the fund must pay investment management fees, custody fees,
audit fees, governance fees and administration fees whereas such fees are not
taken into account in the calculation of returns on the Index.
The Index includes the reinvestment of gross dividends paid by its constituent
companies whereas the investment manager will only receive such dividends
net of withholding tax.
The Index does not take into account the costs of rebalancing the index for
such activities as new entrants, exits, mergers and takeovers and changes in
the market capitalisation of constituents.
Such costs include stockbrokers commissions, stamp duty and other levies.
When the fund manager receives small amounts of dividend income, it may
not be cost effective for her to invest such small amounts across the
constituents in the correct proportions.
The manager will therefore have part of the portfolio invested in the
constituents of the index and part invested in cash.
The cash holding will cause the manager to under perform the index in a rising
market and out perform the index in a falling market.

Page 7
30/12/2008

Subject ST5 (Finance and Investment Specialist Technical A) September 2008 Examiners Report

Needs to reflect situation of non-domestic investor and the ability to replicate


or otherwise track individual markets.
There may be limited derivatives available to develop synthetic approaches.

Problem with definition of emerging market. This will vary between investors
and index providers.
Lack of homogeneity means alternatives for stock/sector exposures may not be
closely correlated.
At individual market level and relevant weights, there may be foreign
ownership restrictions, different share classes and different definitions of
capitalisation according to free float.
Some markets may be very concentrated with associated liquidity issues. This
could have implications for investors with caps on exposures to particular
companies.
Marketability and availability of stock will vary and political instability can
cause capital control issues and so grounds for inclusion/exclusion within
index with limited notice of change.
For total return, income adjustment should reflect investor circumstances in
terms of reinvestment (actual receipt may be long after dividend declaration)
and taxation e.g. unrecoverable taxes.
Pricing and valuation information may be poor and untimely which will affect
dealing and monitoring of tracking.
Costs of dealing may be higher and may need to be reflected in judging
tracking success.
Restrictions on investment in certain countries imposed either by trustees or
regulation may render index less appropriate.
May have undue sector or stock biases versus total portfolio.
Research, administration, custody and dealing costs may be disproportionate
or difficult to facilitate.
Taxation will be a particular problem especially capital gains tax.
If making direct investment, unlikely to have portfolio similar to index.
Other practical management and monitoring issues.

Page 8
30/12/2008

Subject ST5 (Finance and Investment Specialist Technical A) September 2008 Examiners Report

(i)

Measure of short term market movements


Providing a history of market movements
Tool for estimating future movements in market given past trends
Benchmark for assessing portfolio performance
Valuing notional portfolio
Analysing sub-sectors of the market
As a basis for index funds to track a particular market
To provide basis for the creation of derivative instruments

UK -FTSE 100- largest 100 companies by market cap. Account for 80% of
total market. Weighted arithmetic average basis. Free float.
USA Dow, 30 shares. Unweighted arithmetic average.
S&P 500, weighted arithmetic index
Japan Nikkei 225 companies, unweighted arithmetic average
Topix 1100 shares, market cap weighted arithmetic
Germany DAX 30 shares, total return index
France CAC, 250 shares free float, market cap

(ii)

(iii)

Efficient portfolio or transition management i.e. Asset allocation,


equitising cash holdings
Long term and/or short term risk reduction i.e. Hedging strategic
exposures
Creating structured products with bespoke payoff profiles
Speculation

(iv)

Options are financial instruments that convey the right, but not the
obligation, to engage in a future transaction on some underlying security,
or in a futures contract.
Exchange traded options have standardized contracts, and are settled
through a clearing house with fulfillment guaranteed by the credit of the
exchange. Since the contracts are standardized, accurate pricing models
are often available.
Trading options entails the risk of the option's value changing over time.
However, unlike traditional securities, the return from holding an option
varies non-linearly with the value of the underlier and other factors.
A further, often ignored, risk in derivatives such as options is counterparty
risk. In an option contract this risk is that the seller won't sell or buy the
underlying asset as agreed. However exchange trading enables
independent parties to engage in price discovery and execute transactions.
As an intermediary to both sides of the transaction, the exchange provides:
- fulfillment of the contract is backed by the credit of the exchange,
which typically has the highest rating (AAA),
- counterparties remain anonymous,

Page 9
30/12/2008

Subject ST5 (Finance and Investment Specialist Technical A) September 2008 Examiners Report

enforcement of market regulation to ensure fairness and transparency,


and
- maintenance of orderly markets, especially during fast trading
conditions.
Basic options are to buy/sell puts/calls (and combinations thereof)
depending on investors view of the markets
As a small life office, ETOs offer low administration/efficient portfolio
management and the ability to hedge risks to comply with regulatory or
statutory requirements, particularly in volatile markets when fund
cashflows are uncertain. Similarly they can be combined with other
investments to create more attractive pay-off profiles with minimal
counter-party risk.
However like all standardised contracts, there will be inherent basis risk
between the option and the offices underlying holdings and there may be
cashflow or other trading risks if options are exercised.
Buying options involves paying a premium which may be subsequently
proven to have been wasted and so impact returns (and potentially
competitive positioning).
-

(i)

What investigations
Management ability
Quality of the cars/products
Prospects for market growth, market research and outlook for future
economy
Competition, who else makes the same type of cars. What is their business
model like
Input costs
R&D costs
Likely Profit
Marketing and sales strategy
The accounting ratios
Predicted level of borrowing

(ii)

Describe the difficulties.


Lack of publicly available information to analyse the company.
Lack of company history for profit and cost analysis.
Upfront costs of company setting up be higher than on-going.
Difficult to predict demand for new brand.
Lack of luxury car companies in same period of development for which to
benchmark.

(iii)

Describe how the company and the quoted shares might be affected.
Car company would be defined as consumer good, durable, cyclical.
As enter into recession PER will fall, share price likely to be depressed
Sales of new cars likely to fall to less demand.
Profits likely to decrease due to reduced demand and potential reduction in
price. Input costs likely to remain unchanged.

Page 10
30/12/2008

Subject ST5 (Finance and Investment Specialist Technical A) September 2008 Examiners Report

(iv)

Define the term out of the money for both a call option and a put option
giving a brief example for each option.
Out of money for a call option means the current share price is less than
the strike price attached to the option. Example if strike price is 250 and
current price is 200, out of the money.
Out of money on put means that the current price of share is higher than
the strike price attached to the option. Example current share price is 300
and the strike price attached to put option is 250.

(v)

Marks were given for reasoned arguments reflecting points such as:
There is a price to pay for the option which once added to current price
means out of the money initially.
The actual floatation price was lower than expected.
Price reflects volatility and time values.

(i)

Describe why it is important.


To protect the ultimate beneficiaries from gross incompetence or
mismanagement by fund managers.
To encourage confidence in investment schemes and the benefits they
secure.
To promote the accumulation of investible funds.

(ii)

The principles you would recommend.


Effective decision making decisions only taken by people with skill,
information etc.
Clear objectives Setting investment objectives which represent best
judgement on funds liabilities.
Take account of attitude to risk
Focus on asset allocation should set strategic asset allocation to be in line
with required risk return characteristics of individual fund circumstances
Expert advice need for expert advice for actuarial and investment advice
Explicit mandates Agree fund manager objectives, benchmark, risk
parameters. Understand the managers approach in attempting to achieve
objective.
Clear time scales of measurement and evaluation.
Activism Managers should have explicit strategy on activism
Appropriate Benchmarks explicit and benchmarks appropriate to assets
being invested in.
Default fund options.
Performance Measurement Have formal performance measurement
agreements including the period which reviewed.
Transparency Statement of investment principles on who is taking
decisions, the funds investment objective, planned asset allocation,
mandates given to managers, fee structures.
Regular Reporting members should be sent annual results of monitoring
of fund managers.

END OF EXAMINERS REPORT


Page 11
30/12/2008

Faculty of Actuaries

Institute of Actuaries

EXAMINATION
22 April 2009 (pm)

Subject ST5 Finance and Investment


Specialist Technical A
Time allowed: Three hours
INSTRUCTIONS TO THE CANDIDATE
1.

Enter all the candidate and examination details as requested on the front of your answer
booklet.

2.

You have 15 minutes before the start of the examination in which to read the
questions. You are strongly encouraged to use this time for reading only, but notes
may be made. You then have three hours to complete the paper.

3.

You must not start writing your answers in the booklet until instructed to do so by the
supervisor.

4.

Mark allocations are shown in brackets.

5.

Attempt all six questions, beginning your answer to each question on a separate sheet.

6.

Candidates should show calculations where this is appropriate.

Graph paper is required for this paper.

AT THE END OF THE EXAMINATION


Hand in BOTH your answer booklet, with any additional sheets firmly attached, and this
question paper.

In addition to this paper you should have available the 2002 edition of the Formulae
and Tables and your own electronic calculator from the approved list.

ST5 A2009

Faculty of Actuaries
Institute of Actuaries

(i)

Describe the uses of performance measurement for an investment portfolio.


[6]

(ii)

[8]

(iii)

Describe the key reasons why hedge fund index returns are likely to overstate
actual returns and understate volatility for a typical hedge fund investor. [4]

(iv)

State the formula for the Sharpe ratio, defining any terms you use.

[2]

(v)

Explain why hedge funds highlight the Sharpe ratio in their promotional
material, rather than the Treynor or Jensen ratios.

[2]

(vi)

Discuss the limitations and disadvantages associated with portfolio


performance measurement.

Describe the key limitations of the Sharpe ratio as a measure of the skill of a
hedge funds managers.
[5]
[Total 27]

You are the portfolio manager for a global equity pooled fund and have received a
quarterly analysis of companies in the European telecoms sector.
(i)

(ii)

(a)

Describe what you would expect to see in a high quality piece of


fundamental research of this nature.

(b)

List the factors that you might expect to see included in the numerical
analysis.
[6]

Outline the additional commentary that you would expect to see for a
company in the sector that is more highly leveraged than average, with a
significant amount of debt due to be repaid in the next two years.

[3]

Your company is considering launching two new global equity pooled funds, the
Global Equity (Higher Leverage) Fund and the Global Equity (Lower Leverage)
Fund. The intention is for a combined investment in the two new funds to broadly
correspond to an investment in the existing fund. The two funds will invest in the
same universe of underlying companies and can make the same buy/sell decisions as
the existing global equity fund. However, companies that are highly geared can only
be invested in by the Higher Leverage Fund.
(iii)

Discuss why a potential investor might find the choice of two new funds more
attractive than the existing global equity fund.
[7]

(iv)

Explain why an investor should not expect investments of $1m in each of the
two new funds to perform precisely in line with a single $2m investment in the
existing global equity fund.
[4]
[Total 20]

ST5 A20092

The trustees of a pension fund decide to purchase a three year swap contract under
which the pension fund will receive a fixed rate payment stream. The pension fund is
required to pay a floating rate payment stream in return. The pension fund receives
the following information about the swap and the likely payments:

Term 3 years
Notional value of swap 50m
Payments are made in arrears semi-annually
The swap year calculations assume there are 360 days in a year
Period

Number of days in period

Annual Forward Interest


Rate

1
2
3
4
5
6

183
181
182
182
181
183

4.00%
4.25%
4.5%
4.75%
5.0%
5.25%

(i)

Define the term puttable swap.

[1]

(ii)

Explain why a pension fund may wish to purchase a puttable swap.

[2]

(iii)

State the type of swap that the trustees have purchased.

[1]

(iv)

Using the information above calculate:


(a)
(b)

Present value of the floating rate payments.


The fixed rate of the swap.

[4]

The pension fund trustees proceed with the proposed contract for the payments
described above and the fixed rate of the swap is set at 4.75% pa.
(v)

Calculate the profit or loss to the pension fund at the end of the swap contract.
[4]

(vi)

Explain what difference there would have been to the profit/loss on the swap if
interest rates had risen during the duration of the swap contract.
[2]
[Total 14]

ST5 A20093

PLEASE TURN OVER

(i)

(a)

Define the term Warrant.

(b)

State two differences between a Warrant and an Option.


[2]

(ii)

(a)

Define the terms American call and European call.

(b)

Explain which one is likely to attract a higher premium.


[2]

(iii)

Draw a diagram for each of the following strategies and explain why an
investor may wish to undertake such strategies.
(a)

Long one call at a strike price of (X a)


Short two calls at a strike price of X
Long one call at a strike price of (X + a)
All three have same expiry date

(b)

Buying one call and one put with the same expiry and strike price

(c)

Buying call options of a certain strike price and selling the same
number of call options at a lower strike price (in the money) with the
same expiry date.
[6]

As part of an investors portfolio there are 100 call options that have been written
with an exercise price of 1.50 and an expiry date of November. The option premium
received was 0.50 per option.
(iv)

State the payoff for the investor.

[1]

(v)

Draw the payoff chart for the entire holding.

[2]

(vi)

Calculate the profit or loss to the investor if the price of the share at expiry is:
(a)
(b)
(c)

0.75
1.50
2.15

State any assumptions made.

[2]

The derivatives exchange where the call options are traded requires an initial margin
of 20% of the premium received. In addition a variation margin has to be paid equal to
100% of the option price movement. The value of the premium at the end of
September was 0.55.
(vii)

Calculate the total margin the investor has had to post to the exchange at the
end of September.
[2]
[Total 17]

ST5 A20094

(i)

Define the par yield of a bond.

[1]

(ii)

Write down the equation of value that needs to be satisfied by the par yield,
C2, of a two year bond (interest paid annually in arrears), in terms of the zero
coupon yield, ZCt, at time t.
[2]

(iii)

Calculate the zero coupon yields at times 1, 2 and 3 from the following par
yield curve, assuming coupons are paid annually in arrears:
Term

Par Yield

1
2
3
4

5.50%
5.40%
5.35%
5.30%
[6]

(iv)

Describe three techniques that can be used to identify bond anomaly switching
opportunities.
[6]
[Total 15]

(i)

(a)

Describe the primary purpose of an investment manager agreement.

(b)

List typical restrictions that might be included within an investment


manager agreement for a global bonds portfolio for a charity fund.
[3]

(ii)

Explain why agreements for active managers generally adopt a relatively


prescriptive approach, rather than giving the manager complete discretion in
how they achieve the target return.
[4]
[Total 7]

END OF PAPER

ST5 A20095

Faculty of Actuaries

Institute of Actuaries

Subject ST5 Finance and Investment


Specialist Technical A
EXAMINERS REPORT
April 2009

Introduction
The attached subject report has been written by the Principal Examiner with the aim of
helping candidates. The questions and comments are based around Core Reading as the
interpretation of the syllabus to which the examiners are working. They have however given
credit for any alternative approach or interpretation which they consider to be reasonable.
R D Muckart
Chairman of the Board of Examiners
July 2009

Faculty of Actuaries
Institute of Actuaries

Subject ST5 (Finance and Investment Specialist Technical A) April 2009 Examiners Report

Comments
Pleasingly, a better answered paper than previous diets leading to a higher pass rate even with a
higher pass mark. Candidates typically answered Questions 1, 4 and 5 much better than the others,
with Questions 2 and 3 attracting the worst responses, considerably so. This is not surprising given
that Questions 2 and 3 represented the opportunity to demonstrate higher level skills in terms of nonstandard/practical application of theory to current issues in investment hence candidates who wish
to progress to SA6 will need to improve their understanding of and approach to this type of question.
That said, most candidates seemed to identify and understand the key issues being examined and so
appreciated the general content of solutions that the examiners were looking for however those that
were unsuccessful will find their solutions lacked sufficient (and often the most basic) detail and
scored lower accordingly (this was most evident in Question 6). Many candidates still deviate from
the topic and include irrelevant material or over emphasise minor points although candidates will
not be explicitly penalised for this, it gives an impression of a lack of understanding and, more
importantly, wastes limited time. Time and priority management are key skills actuaries need to have.
Where candidates made relevant points in other parts of their solutions, the examiners have used their
discretion as to whether to recognise these answers or not. Likewise the examiners share and agree
alternative possible solutions to questions during the marking process.
Again there were many candidates close to the pass mark whom were awarded an FA most
candidates would be very surprised to see just how tightly distributed the marks are; deciding where
the pass mark falls will have a material impact on the numbers of candidates who are successful and
the examiners take great care to ensure a consistency of standard across candidates, subjects and
diets. Several candidates were awarded an FD in this diet and the examiners remain concerned by
the numbers of candidates still achieving only an FC grade, since this too would imply little
preparation or, worse, knowledge and understanding.
Candidates are reminded of a bias in the paper towards recognising higher level skills and practical
application this is intentional and will continue. Likewise the examination system does properly
allow for prior subject knowledge to be assumed. Investment is a necessarily practical subject and, at
this level, the examiners expect candidates to demonstrate a breadth and depth of competency as
would be expected from a senior student in a frequently evolving discipline. Hence simple
regurgitation of bookwork will never be sufficient to ensure a Pass grade and this was evident from
the dispersion of candidates responses in the more differentiating questions.
As noted before, in order to succeed, candidates must ensure they familiarise themselves with the
prevailing investment issues and the general market background facing institutional investors in the
18 months preceding a diet, more so the solutions (and sources of) being debated by the various
stakeholders. A recurring theme in recent years has been a move towards capital market rather than
purely insurance and asset management solutions hence questions regarding banking and
derivative approaches to asset and liability risk management or modern financial theory and
commercial applications should be considered likely scope for examination. New asset classes and
ways of structuring investment will themselves generate new types of risk (such as operations,
liquidity, credit and counterparty), so the need for new ways of monitoring and management.
All extenuating and mitigating circumstances were considered in awarding grades and, where there
was a genuine cause, credit given.

Page 2

Subject ST5 (Finance and Investment Specialist Technical A) April 2009 Examiners Report

(i)

There are various reasons why the performance of an investment portfolio will
be measured:
1. To improve future performance. First, data collected during performance
monitoring can form the inputs for planning future strategy. Secondly, if
fund managers know that their performance is being measured, it might
give them an extra incentive to maximise the returns of the funds they
manage.
2. Comparison of the rate achieved against a target rate. Many funds will
have one or more target rates of return. For example, the trustees of a
pension fund will want to know the rate of return achieved on the
investments compared with the rate of return assumed in the actuarial
valuation.
3. Comparison against the performance of other portfolios, an index and/or a
benchmark portfolio. Those responsible for the funds will want to know
how the performance of the portfolio compares with other portfolios. On
the basis of this information, they are able to make decisions regarding the
future investment of the assets, e.g. should a new fund manager be hired?
Also, by analysing the performance against a notional portfolio, it may be
possible to identify some relative strengths and/or weaknesses of
individual fund managers (e.g. in sector or stock selection).
Other reasons could include the assessment of performance related fees or
more generic assessments of success/failure of the portfolio.

(ii)

There are several limitations and disadvantages of portfolio performance


measurement.
Projection of past results: the fact that a particular result was attained in the
past does not mean that it will occur in the future. There is a random element
in investment returns and it may be difficult to determine how much a fund
managers results are due to method and how much to luck. Furthermore a
technique that proved successful in a particular set of circumstances may not
work so well in changed circumstances in the future.
Risk: in the long term we would expect a riskier strategy to produce higher
average returns. The measurement of relative performance should therefore
take account of the degree of risk taken on by a fund manager.
Timescale: determining the frequency of performance measurement
calculations requires a delicate balance between assessing performance
frequently enough so that problems can be spotted and corrected and avoiding
spurious conclusions based on too short a measurement period.
Differing fund objectives: different funds may have different objectives and
constraints. Comparisons between such funds may not be valid.

Page 3

Subject ST5 (Finance and Investment Specialist Technical A) April 2009 Examiners Report

Impact on fund manager behaviour: knowledge of how, and how often he will
be assessed is likely to influence the investment strategy of a manager. This
may not be in the funds best interests. For example, frequent monitoring can
encourage a short term approach to investment.
Cost: users of performance measurement services must balance the value of
the service against the cost. Also, for a number of assets (e.g. property),
valuation is difficult, time-consuming and very subjective. Detailed, frequent
calculations based on subjective valuations are inappropriate.
(iii)

(iv)

Index statistics will be influenced by:

Survivorship bias unlike in other asset classes, it is difficult to obtain


data on hedge fund failures when backfilling a history at the time an
index is launched by an index provider. This will create an upward bias.

Selection bias funds with a good history are more likely to apply for
inclusion at the time of reviewing index constituents. Similarly, it is not
always possible to obtain accurate performance information from a failing
hedge fund so the provider may only be able to exclude the fund rather
than report its full losses. Both of these factors will create an upward bias.

Marking to market bias where underlying securities are illiquid, funds


may use stale prices or mark asset values to a valuation model. Whilst
this does not necessarily result in a bias to the return data, it is likely to
result in lower volatility figures than would be the case otherwise.

S=

Rp r
p

where Rp is the return on the portfolio


r is the risk-free rate of return (usually taken to be 3 month LIBOR)
p is the standard deviation of portfolio returns
(v)

Page 4

The Sharpe ratio is widely understood by investors.


The Sharpe ratio can be calculated without reference to the beta of a
portfolio...
...making it more practical to compare strategies across different asset
classes...
...including asset classes where there may not be an obvious market portfolio
that can be used as a reference point.
Both the Jensen and the Treynor ratios rely on beta being a proxy for the level
of active risk taken by the asset manager.
[Credit was awarded for any other reasonable points made]

Subject ST5 (Finance and Investment Specialist Technical A) April 2009 Examiners Report

(vi)

The key limitations of the Sharpe ratio with regard to hedge fund returns are as
follows:

Mismatch with utility function the Sharpe ratio falls as downside or


upside volatility increase. In practice, investors will specifically be
concerned to avoid downside risks within their utility function.

Sampling issues historic calculations will be limited by the length of


available history. With hedge fund strategies, it is possible to employ a
strategy that appears low risk for an extended period but has a large tail
risk (e.g. selling volatility by writing out-of-the-money put options), and
the Sharpe ratio will not capture information about low frequency high
severity events.

Non-normal return distributions the Sharpe ratio will give a consistent


measure of excess return to risk for normal distributions, however standard
deviation is not a useful proxy for risk for all return distributions. This is
particularly the case for many hedge fund strategies.

All of these sources of distortion can result in sub-optimal investment


decisions.

(i)

(a)

The analysis should identify and analyse the key factors affecting the
future profitability of companies within the sector...
...and offer an outlook for the sector as a whole.
The analysis should enable the portfolio manager to form a view on the
attractiveness of the sector relative to other sectors...
...and also form a view on the relative attractiveness of individual
companies within the sector.
The analysis should also comment on the timescale over which
differences between perceived value and market prices might converge
(or if not, why they might persist)...
...and the recommendations should be justified by a combination of
numerical analysis and qualitative research...

(b)

The analysis should include historical statistics and forward-looking


estimates for several of the following factors to enable a picture of the
financial position of the companies to be built up:

Revenues
Operating profit
Pre-tax profit
Earnings per share
Price/earnings ratio
Price/book value
Dividend yield
Outstanding debt

Page 5

Subject ST5 (Finance and Investment Specialist Technical A) April 2009 Examiners Report

Where appropriate, the numerical analysis may need to be


supplemented by qualitative commentary to justify the
recommendations for each company.
(ii)

There should be additional commentary/analysis on the following:

(iii)

details of maturities of the companys existing bonds and loans


trends in the companys balance sheet over the next 5 years
trends in revenues and operating expenses [P&L alternatively] over the
next 5 years
the likelihood of the existing borrowings being refinanced in the current
climate without other compensating corporate actions
potential corporate actions that may need to be carried out to facilitate a
refinancing (e.g. disposals or rights issues to raise cash)
the likely change in financing costs as a result of the refinancing, based on
analysis of recent loan spreads for the sector and the economy
impact on credit rating
investor appetite for bonds/syndicated loans from this issuer

A new investor will not necessarily gravitate to the established global equity
fund, depending on his/her requirements.
Particular reasons why the new funds might offer a better fit for the investors
requirements could include one or more of the following factors:
Alpha based view
The investor believes that the fund manager has greater potential to deliver
alpha in the Lower Leverage or Higher Leverage niches than in the main fund
that invests in both categories of company.
This may reflect that the investor believes that a global approach to equity
investment is better suited to, for example, the Lower Leverage category,
where there are fewer country-specific factors that need to be accounted for in
selecting stocks.
Beta-based views

Page 6

Long term: the investor wishes to have a long-term bias towards Lower
Leverage or Higher Leverage companies.
Short-term: the investor believes that at the current point in the economic
cycle, Lower Leverage or Higher Leverage companies have better
prospects.
The above may reflect a risk-based view (e.g. more highly leveraged
companies have higher volatility) or a return-based view (e.g. more highly
leveraged companies will underperform at times of high interest rates /
high credit spreads).

Subject ST5 (Finance and Investment Specialist Technical A) April 2009 Examiners Report

Alternatively the above may reflect a preference to overweight particular


sectors (e.g. mining companies and oil companies tend to employ less
leverage than say financial companies) within the portfolio.
Where one of the two niche funds is being chosen based on beta-based
view (whether long-term or short-term), the particular product would only
be appropriate if the investor believes that the manager is capable of
delivering alpha in the chosen category (otherwise a passive strategy
would better suit the investor).

Portfolio construction/style diversification


The investor may be working around other equity styles within the existing
portfolio, and one of the two niche funds may be more appropriate (e.g. the
investor wishes to invest in a style neutral manner but feels that the Lower
Leverage fund will better complement an investment in a Global Equity
Growth fund offered by another manager).
The investor may also have adopted a risk-budgeting approach and one of the
two niche funds may be a more suitable addition to the portfolio from the
earmarked funds for investment. This would assume that the investor does not
have strong alpha and beta views, that would naturally lead to additional
adjustments elsewhere in the investors portfolio.

(iv)

At the launch date of the two niche funds it would appear that equal
investments in the two funds would in aggregate equal the existing fund.
However, the two approaches would begin to diverge almost immediately,
although not greatly as they are based on broadly the same stock/sector
selection decisions.
Differences would arise due to inflows and outflows from investors into the
different pooled funds, resulting in varying cash weightings and transaction
costs which would impact on the relevant fund.
Further differences will arise if a stock was reclassified as moving from the
Lower Leverage category to the Higher Leverage category (or vice versa).
This reflects that for the existing fund this would not lead to a buy/sell
decision (in the absence of other factors), whereas for the two niche funds one
would need to sell the stock and one would need to buy the stock (in the
absence of other factors). Crossing trades will mitigate against market impact
and transaction costs to the extent that the Lower Leverage and Higher
Leverage funds are making equivalent but opposite changes in a particular
stock.

(i)

A swap agreement in which the fixed rate receiver has the right to terminate
the swap on one or more dates prior to its scheduled maturity. This early
termination provision is designed to protect a party from adverse effects of
large changes in fixed rates.

(ii)

The pension fund wants to enter into swaps to reduce risk but the actual
liabilities are subject to refinement which might mean swaps adjustment.

Page 7

Subject ST5 (Finance and Investment Specialist Technical A) April 2009 Examiners Report

There is a yield pick up on the swap and therefore, is being held for tactical
reasons and not as a long term investment.
(iii)

Interest rate swap.

(iv)
Period

Number of days
in period

1
2
3
4
5
6

183
181
182
182
181
183

Annual
Forward
Interest
4.00%
4.25%
4.50%
4.75%
5.00%
5.25%

1/2 year
interest rate
2.03%
2.14%
2.28%
2.40%
2.51%
2.67%

value
term
payment
days

50000000
3
semi-annual arrears
360

(a)
PV
1
1/[1+(days/360*Interest)]
2 1/[1+(days/360*Interest) 2 periods]
3
etc.
4
etc.
5
etc.
6
etc.
Total

Discount
0.9801
0.9596
0.9382
0.9162
0.8938
0.8705
5.5584

PV of payments
996406
1025205
1067229
1100102
1123398
1161602
6473942

Notional
24910160
24122468
23716197
23160035
22467962
22125746
140502569

Candidates were given credit for rounded solutions rather than the
level of detail shown. Full marks were not available if candidates
assumed a half year rather than a specific day count.
(b)
PV of notional
PV of floating rate
Theoretical swap rate
(v)

140502569
6473942
4.61%

PV of fixed rate = 4.75 / 4.61 6473942 = 6670520


Profit = 6670520 6473942 = 196,578 (or the rounded equivalent)
The fixed rate is higher than the theoretical swap rate so assuming the
payments reflect the assumed then the pension fund will be in the money, i.e.
calculation should show a profit, as they have been paid a higher amount from
the bank (via fixed) than paid out.

Page 8

Subject ST5 (Finance and Investment Specialist Technical A) April 2009 Examiners Report

(vi)

The higher interest rates would mean the pension fund would be paying out
more than assumed and therefore, the profit assumed would be reduced or
turned into a loss (out of the money)

(i)

(a)

An option issued by a company. The holder has the right to purchase


shares at a specified price at specified times in the future.

(b)

Options can have more flexible exercise dates than warrants.


Warrants are new shares issued and therefore dilute share capital,
options are right to buy existing share capital.
Warrants are OTC where as options tend to be exchange traded.
Warrants have longer expiry than options.

(ii)

(a)

European right to purchase at set price at set date in future.


American right to purchase at set price at any point before expiry
date.

(b)
(iii)

American as it has the added flexibility.

The charts illustrate the basic shape of the payoff and credit was given for
similar, suitably annotated graphs
(a)

Butterfly spread

Investor does not believe a stock will rise or fall much before expiry
thinks volatility will be low. Wants limited risk strategy but also
limits profit.

Page 9

Subject ST5 (Finance and Investment Specialist Technical A) April 2009 Examiners Report

(b)

Straddle
Investor believes the underlying price will change significantly but
does not know which way it will go. Profit if volatility is high.

(c)

Bear spreads
A bear call spread is a limited profit, limited risk options trading
strategy that can be used when the options trader is moderately bearish
on the underlying security. Thinks the share price will fall.

(iv)

O St +K where St is greater than K, otherwise O. St = price of stock,


K = exercise price, O is price of option.

(v)

Chart would show 50 profit when share price starts at 0 until exercise price
1.50. The investor would then start to decrease the profit. At 2.00 exercise
the investor profit would be 0. At 2.50 the loss would be 50

Page 10

Subject ST5 (Finance and Investment Specialist Technical A) April 2009 Examiners Report

(vi)

0.75 = 50 profit,
1.50 = 50 profit assuming can buy stocks in market at zero cost
2.15 = 15 loss assuming can buy stocks at zero cost. Loss on purchase of
shares is 65 and profit from premium 50

(vii)

Initial margin is 0.2 50p 100 shares = 10. Then have to post 100% of the
movement which is 5p. The additional margin is then 5 for the 100 shares so
the total margin = 15.

(i)

The par yield is the coupon rate that would be required for a coupon-paying
bond to be valued at par under the current interest rate curve.

(ii)

1 = C2 (1 + zc1 )

+ (1 + C2 )(1 + zc2 )

(iii)
Term
1
2
3
4
(iv)

Par Yield
5.50%
5.40%
5.35%
5.30%

V(Bond)
100.000%
100.000%
100.000%
100.000%

V(Bond exc last)


94.882%
90.114%
85.679%

ZC Yield
5.500%
5.397%
5.346%
5.290%

Any three from:


Yield differences: in considering possible anomaly switches, yield differences
are widely used to identify individual bonds which seem cheap or dear, in
relation to other bonds. However, because of the fact that high coupon bonds
are likely to have higher gross yields than low coupons, a high gross yield
does not in itself indicate that a bond is cheap. The investor must examine
whether the yield difference is greater or less than it has been in the past.
A problem with the evaluation of individual bonds in relation to a fitted yield
curve has been the stability of the method used to fit the curve. It is now more
usual to review a computer generated history of yield spreads between pairs of
actual bonds.
Price ratios: these can be monitored as well as yield differences. Ideally, a
switch under consideration will look attractive, in relation to both yield and
price histories. A practical problem in using price ratios is that they do not
allow for the fact that the two bonds may have different coupons; they will
have different prices but will both be redeemed at 100. So the ratio of the two
prices will display a trend. This history of price ratios may be adjusted by this
trend to produce what are often known as stabilised price ratios.
Price models: some bond analysts have devised price models which try to
assess the correct price for a stock, given the key variables. A stocks price
is considered anomalous if the actual price differs from the price derived from
the model.
Page 11

Subject ST5 (Finance and Investment Specialist Technical A) April 2009 Examiners Report

Yield models: rather than compare a bonds yield with a redemption yield
curve it can be compared with one of the alternatives such as a yield surface or
par yield curve.

(i)

(a)

The primary purpose of the agreement is to act as a business contract


between the investment manager and the investor/client. As a
minimum it would set out the services to be carried out by the manager
for the investor and the agreed fees.

(b)

Typical restrictions would include:

(ii)

limitations on permitted asset classes for manager to invest in


limitations on leverage (explicit and implicit, through derivative
contracts)
maximum and minimum ranges for holdings in particular asset
classes
maximum ranges for holdings in a single company or single
industry sector
prohibitions on particular stocks for ethical/SRI reasons (e.g.
cluster bomb manufacturers)
prohibitions on self-investment in the investors own securities

A fund will often have a number of different managers and mandates


managing its assets. A prescriptive approach within manager agreements
allows the asset allocation to be controlled and managed at a global level,
whereas if the asset managers had complete discretion in security selection it
is unlikely that the actual assets would closely resemble the target asset
allocation.
Additionally, many funds will wish to place restrictions on permitted
investments and leverage as part of their overall risk controls and wider social
responsibilities.
Finally, a relatively prescribed agreement enables the investor to challenge the
manager more easily in the event of the assets not being invested in line with
the investors wishes, or in the event of mismanagement taking place.

END OF EXAMINERS REPORT

Page 12

Faculty of Actuaries

Institute of Actuaries

EXAMINATION
1 October 2009 (pm)

Subject ST5 Finance and Investment


Specialist Technical A
Time allowed: Three hours
INSTRUCTIONS TO THE CANDIDATE
1.

Enter all the candidate and examination details as requested on the front of your answer
booklet.

2.

You have 15 minutes before the start of the examination in which to read the
questions. You are strongly encouraged to use this time for reading only, but notes
may be made. You then have three hours to complete the paper.

3.

You must not start writing your answers in the booklet until instructed to do so by the
supervisor.

4.

Mark allocations are shown in brackets.

5.

Attempt all seven questions, beginning your answer to each question on a separate
sheet.

6.

Candidates should show calculations where this is appropriate.

AT THE END OF THE EXAMINATION


Hand in BOTH your answer booklet, with any additional sheets firmly attached, and this
question paper.

In addition to this paper you should have available the 2002 edition of the Formulae
and Tables and your own electronic calculator from the approved list.

ST5 S2009

Faculty of Actuaries
Institute of Actuaries

Describe the following terms:


(a)
(b)
(c)
(d)

split capital investment trust.


rights issue.
scrip issue.
share split.
[6]

(i)

Explain the relationship between forward and futures prices.

[5]

(ii)

(a)

Define the term basis risk.

(b)

Explain the reasons why basis risk may arise when a futures contract is
used to hedge a position in the cash market.
[3]

(iii)

State the formula for the optimal hedge ratio, defining the terms used.

(iv)

Outline why fixed income derivatives are more difficult to value than equity
derivatives.
[4]

(v)

Determine the price of a 10-month European call option on a 9.75 year bond
with a face value of 1,000. Assume that:

[2]

the current cash bond price is 1269


the strike price is 1300
the 10-month risk free interest rate is 2.6% p.a., and
the volatility of the forward bond price in 10 months is 9% p.a.

The bond pays a semi-annual coupon of 6% and coupon payments of 30 are


expected in 3 months and 9 months. Assume that the 3-month and 9-month
risk-free interest rates are 2% and 2.5% p.a. respectively.
[8]
[Total 22]

ST5 S20092

An investment consultant advises two pension funds that are both long-term investors
in separate global equity portfolios managed by Makeoff Global Investment
Company. Over the 12 months to 31 December 2008 the returns for the two clients
have been materially different. On further investigation, the investment consultant
obtains the following information:

Beta of portfolio
Holding in Banks
Investment Style
(i)

(ii)

Pension Fund A

Pension Fund B

0.8
4% underweight to benchmark
Value

1.2
10% overweight to benchmark
Growth

(a)

Define the term Beta.

(b)

Describe how the Betas quoted above will have impacted performance
over the period under review.
[3]
Explain what is meant by the terms Value and Growth.

(a)
(b)

Give an example of the type of shares that Value and Growth style
investors would invest in.
[4]

(iii)

Explain, using the information in the table above, which pension fund would
have been expected to have performed better during the period under review.
[3]

Another long-term investor follows the same investment strategy as Pension Fund B.
However, during the 12 months to 31 December 2008 they have experienced different
performance to Pension Fund B.

(iv)

State two reasons why the performances might be different.

(i)

Discuss the key factors to be considered in monitoring and controlling credit


risk.
[4]

(ii)

List the principal questions that a credit rating agency will ask in assessing and
ascribing an issuer rating for a company that issues debt.
[4]

(iii)

Explain why a bond issued by a company might have a higher or lower credit
rating than the company itself.
[2]
[Total 10]

ST5 S20093

[2]
[Total 12]

PLEASE TURN OVER

(i)

Describe the problems with, and the possible solutions to, the investment
technique known as liability hedging.

[8]

The trustees of a pension scheme with two sections (Section A and Section B) wish to
reduce the impact of interest rate changes on the amount of the difference between the
present value of the assets and the present value of the liabilities.
The table below shows the payments that are due to be paid out from each scheme
section and also those from a bond the trustees are thinking of purchasing to achieve
their investment objective.
Bond
Year (t) Cashflows
1
2
3
4
5

10
10
10
10
100

Section A
Liabilities
11
0
5
32
93

Section B
Liabilities
5
10
13
27
85

Assumptions

All payments are made annually in arrears.


Interest rate is 4.75% per annum.
All final calculations are rounded to nearest whole number.

(ii)

ST5 S20094

Assuming no other investments, state with reasons for which Section the bond
is better suited to achieve the trustees objectives. Show all your calculations.
[5]
[Total 13]

(i)

Describe the key features of a Real Estate Investment Trust (REIT).

[3]

REITs are relatively high-yield investments and a REIT must pay out at least 90% of
its taxable profit as a dividend to shareholders.
(ii)

Explain how you would expect the share price of a REIT to change with a rise
in interest rates.
[3]

You have been asked to assess the value of a possible REIT investment, Equity in
Property, which has a current market capitalisation of $8bn. You have been given the
following accounting information:

Rental income
Fee and asset management
Total Revenues

2008
2007
1,808,925 1,799,581
14,373
9,582
1,823,298 1,809,163

Property maintenance
498,608
464,981
Taxes and insurance
196,987
181,890
Property management
68,058
72,416
Fee and asset management
7,819
7,885
Depreciation
444,339
419,039
General and administration
38,810
46,492
Other costs
1,162
18,284
Total Expenses
1,255,783 1,210,987

(iii)

Operating Income
Net earnings

567,515
543,847

598,176
421,313

Capital Expenditures

181,948

156,776

Explain why traditional equity valuation metrics like the earnings-per-share


(EPS) ratio, earnings growth, and the price-to-earnings (P/E) multiple do not
apply.
[2]

You have proposed basing your valuation on a measure of Funds from Operations
(FFO), which excludes depreciation and the gains on sales of depreciable property.
(iv)

Calculate and reconcile FFO for each of the two years with net earnings.

[2]

Shareholders real estate holdings must be maintained (apartments must be regularly


redecorated, for example), so FFO is not quite the true residual cash flow remaining
after all expenses and expenditures.
(v)

Calculate an Adjusted FFO (AFFO) for each year as a better measure of


distributable income.
[1]

(vi)

Explain how you would use FFO and AFFO to value the proposed investment
in Equity in Property in order to recommend a purchase or not.
[6]
[Total 17]

ST5 S20095

PLEASE TURN OVER

Two investors have the same time horizon to complete the following trades.

Investor A trading 100m of equities.


Investor B trading 1bn of equities.

(i)

(a)

List four types of transaction costs.

(b)

Explain how these will differ between the two investors.


[4]

Another investor with 500m invested in equities believes equity markets will fall by
35% over the next 12 months. The general market consensus is markets will rise by
5% over the next 12 months.
(ii)

Set out three strategies that the investor could adopt to protect themselves
from a fall in equity markets.
[6]

(iii)

Explain the residual risks that remain with each strategy.

(iv)

Describe the effect adopting each strategy would have on the investors
investment performance if the equity markets increased by 5% over the next
12 months.
[6]
[Total 20]

END OF PAPER

ST5 S20096

[4]

Faculty of Actuaries

Institute of Actuaries

Subject ST5 Finance and Investment


Specialist Technical A
September 2009 examinations

EXAMINERS REPORT

Introduction
The attached subject report has been written by the Principal Examiner with the aim of
helping candidates. The questions and comments are based around Core Reading as the
interpretation of the syllabus to which the examiners are working. They have however given
credit for any alternative approach or interpretation which they consider to be reasonable.
R D Muckart
Chairman of the Board of Examiners
December 2009
Comments for individual questions are given with the solutions that follow.

Faculty of Actuaries
Institute of Actuaries

Subject ST5 (Finance and Investment Specialist Technical A) September 2009 Examiners
Report

1
a. An investment trust where the ordinary share capital consists of income
and capital shares. Holders of income get distributed income, holders of
capital little or no income but get residual value of assets after income
shares have been redeemed at fixed value.
b. Issue of further shares at a given price to existing shareholders in
proportion to their existing shareholdings. The purpose is for the issuing
company to raise more money.
c. Sometime called capitalisation or bonus issue is a further issue of new
shares (with the original nominal value) to existing shareholders in
proportion to their holdings. Reserves are capitalised to provide the
additional shareholders' equity.
d. Existing shares are split into two shares of half the original nominal
value. No new capital is raised and no reserves are capitalised.
[6]

2
(i) The main difference between (OTC) forwards and (exchange-traded) futures is
that, for a forward, there is no cash flow until the maturity. For a future, there
are daily marking-to-market and settlement of margin requirements.
If interest rates are constant then the values of the cash flows are equal and,
hence, the prices must also be equal. When interest rates vary unpredictably,
forward and futures prices are no longer the same because of the daily cash
flows from settlement and the interest earned on cash received (or paid on
borrowing). When the price of the underlying asset is strongly positively
correlated with interest rates, a long futures contract will be more attractive
than a similar long forward contract and futures prices will tend to be higher
than forward prices. The reverse holds true when the asset price is strongly
negatively correlated with interest rates.
The theoretical differences between forward and futures prices for contracts
that last only a few months are, in most circumstances, sufficiently small to be
ignored. However, for long-term futures contracts, the differences between
forward and futures rates are likely to become significant. To convert futures
to forward interest rates, a convexity adjustment is applied:
Forward rate = Futures rate

2t1t2

where t1 is the time to maturity of the futures contract, t2 is the time to


maturity of the rate underlying the futures contract and is the standard
deviation of the change in the short-term interest rate in one year. (A typical
value for is 1.2%). [Note that the forward and futures rates in this
expression are expressed in continuously compounded form .]
(ii)
a. Basis risk can be defined as the residual risk that results when the two
sides of a hedge do not move exactly together.

Page 2

Subject ST5 (Finance and Investment Specialist Technical A) September 2009 Examiners
Report

b. It may arise if:

The asset whose price is to be hedged is not exactly the same as the
asset underlying the futures contract

The hedger is uncertain as to the exact date when the asset will be
bought or sold.

The hedge requires the futures contract to be closed out well before
its expiration date.

(iii) The optimal hedge ratio, h, (ratio of the size of the position taken in futures
contracts to the size of the exposure) is given by:
h=

where
F

is the standard deviation of S, the change in spot prices

is the standard deviation in F, the change in futures price

and

is the correlation coefficient between S and F.

(iv) Fixed income derivative payoffs will be dependent in some way on the level of
interest rates. They are therefore more difficult to value than equity
derivatives, since:

The behaviour of an individual interest rate is more complicated than


that of a stock price.

For the valuation of many products, it is necessary to develop a model


describing the behaviour of the entire yield curve.

The volatilities of different points on the yield curve are different.

Interest rates are used for discounting as well as for determining


payoffs from the derivative.

(v) Assuming that the bond prices at the maturity of the option are log-normally
distributed, the value of the call option c is given by
c = P(0,t) [F0
where

(d1) X (d2)]

(x) is the standard cumulative Normal distribution function,

d1 = (ln (F0 / X) + ( 2T / 2) /

and

d2 = (ln (F0 / X) ( 2T / 2) /
F0 (the forward bond price) = (B0 I) / P(0,T)
where B0 is the bond price at time zero and
I is the present value of the coupons that would be paid during the life of the
option.
In this case, I = 30 e

0.25

0.02

+ 30e

Thus F0 = (1269 59.293) e0.8333

Page 3

0.75

0.026

0.025

= 59.293

= 1236.203

Subject ST5 (Finance and Investment Specialist Technical A) September 2009 Examiners
Report

Then d1

= (ln (1236.203 / 1300) + 0.092

10/24) / (0.09 (10/12) )

= ( 0.0503197 + 0.003375) / 0.0821583


= 0.57139
and d2

= ( 0. 0503197 0.003375) / 0.0821583


= 0.65355
0.8333

Hence, c = e

0.026

1236.203

= 0.97857 [(1236.203

0.57139) 1300

0.2839) (1300

0.65355)]

0.2567)]

= 16.83

3
(i)
a. Beta is a measure of a stock's volatility relative to movements in the
whole of the market and is thus a measure of systematic risk. It is usually
defined as the covariance of the return on the stock with the return on the
market, divided by the variance of the market return.
b. Pension Fund A would have been less volatile than the market, Pension
Fund B would have shown more volatility.
(ii)
a. Value investing is a style of investing based on picking shares that have
low valuations relative to their current profits, cash flows and dividend
yield. Value factors commonly analysed include:
Low

Book to Price

Earnings
Sales

Yield

to Price

Growth shares are shares with high price to book values. The expectation
is that earnings and profits will grow above average. Other factors
analysed include:
Sales

Growth

Return

on Equity

Earnings

Revisions

b. Growth internet/tech, clean tech


Value utilities, consumer staples
(iii) Performance over 12 months has been negative

Page 4

Subject ST5 (Finance and Investment Specialist Technical A) September 2009 Examiners
Report

Low beta expected to perform better as less volatile than high beta (everything
being equal)
Financials underperformed market in general so being underweight would be
better
Growth stocks tend to underperform value when markets are falling
Overall we would expect Pension A to perform better
(iv) Cashflows
Tax differences
Management Fee structure
Performance calculation in different base currencies
Credit was given for other sensible reasons

4
(i) The key factors in managing credit risk are:
the

creditworthiness of the counterparties with which an institution deals

the

total exposure to each counterparty

Creditworthiness of counterparties can be controlled by placing limits on the


credit ratings (as published by the major rating agencies) with which an
institution may deal. It can be also controlled in derivatives transactions by
dealing on a recognised exchange with a central clearing house which stands
as counterparty to all deals, rather than over-the-counter. The clearing house
will seek to protect itself by requiring the counterparties to deposit margin
with it. These margin payments are a particular example of the use of
collateral provided by a counterparty as a tool against credit risk.
It is important to monitor and place limits on the credit exposure to any single
counterparty. This will involve aggregating exposures in different areas. For
example a pension fund may hold both equity and debt issued by a bank as
well as having cash on deposit with the same bank and having them as a
counterparty to a derivatives deal. It will also be necessary to be aware of the
particular relationships between different companies within the same group.
Credit risk can be controlled by the use of Credit Default Swaps and other
credit derivatives.
(ii) Ratings agencies will seek to understand the following issues:
fundamental
competitive

Page 5

risks of the companys industry

position (relative to peers)

Subject ST5 (Finance and Investment Specialist Technical A) September 2009 Examiners
Report
downside risk
quality of
cash

vs. upside potential

profitability vs. EPS growth

flow generation vs. book profitability

forward

looking analysis

strategy,

management track record and risk appetite

capital

structure and financial flexibility

Specifically:
Purpose
What does the company do and why do they need to borrow? Possible reasons
for seeking finance include:
organic

growth

acquisition
investment
capital

in an associated company

expenditure

dividend

/ share buy-back

Payback
What is the expected source of repayment? Is there a secondary source? Issues
to consider include:
cash

flow / profit profile (over time)

possible

sale of assets and / or businesses

refinancing

Risks
What risks (quantitative and qualitative) could jeopardise debt servicing in
future? Factors to consider include:
macro

considerations (industry analysis and competitive trends,


regulatory environment, sovereign macro-economic analysis)

company specific issues

market position)
Structure

Page 6

(qualitative analysis, financial performance,

Subject ST5 (Finance and Investment Specialist Technical A) September 2009 Examiners
Report

Does the bond structure reflect the risks and protect investors interests?
(Structure, Status, Safeguards, Pricing)
(iii) A higher rating would apply where the bond has additional security relative to
an unsecured creditor of the issuer (e.g. a fixed or floating charge, or seniority
due to some other factor). [1; 1/2 if no example]
A lower rating would apply where the bond has weaker security relative to an
unsecured creditor of the issuer (e.g. the bond is subordinate to unsecured
creditors). [1; 1/2 if no example]

5
(i) Liability hedging is where the assets are chosen in such a way as to perform in
the same way as the liabilities. A specific example of this is the familiar
concept of immunisation, where assets are matched to liabilities by term in
order to hedge interest rate risk (to some degree). Other familiar forms of
hedging would include matching by currency and the consideration of the real
or nominal nature of liabilities when determining the choice of assets.
However, these examples relate only to specific characteristics of the
liabilities, whereas liability hedging aims to select assets which perform
exactly like the liabilities in all states.
The most familiar example would therefore be the choice of assets to hold in
order to hedge unit-linked liabilities.
In most cases the problem is solved by establishing a portfolio of assets,
determining a unit price by reference to the value of the asset portfolio, and
then using this price to value units held, allocated or realised.
However, even this simple approach can generate many practical problems
use of historic prices for transactions, moving between bid and offer pricing
bases, delays in notification of new money / withdrawals / units allocated or
realised.
A particular problem may arise when intermediaries are given delegated
authority to switch clients holdings between funds, which may result in
extreme volatility of movements for myriad small holdings.
A potentially greater problem arises when the assets held are not the same as
those underlying the value of the liabilities.
Thus, if units are allocated and realised by reference to some external fund,
then it is likely that the internal investment manager will not know what assets
are held by the external manager at any given point in time.
Alternatively, the requisite information may only be available after some
delay, by which time the assets actually held by the external manager are
likely to have changed.
An extreme example of this problem is where the value of liabilities is linked
to some external index (for example, guaranteed contracts where the

Page 7

Subject ST5 (Finance and Investment Specialist Technical A) September 2009 Examiners
Report

movement of market indices determines the value of the contract in some


way). In order to hedge such liabilities, use is often made of over-the-counter
derivatives purchased from an investment bank, thereby avoiding the
uncertainty (and expense) of rolling-over short term exchange traded
derivatives over the lifetime of the underlying contract.
Credit was given for other sensible issues discussed
(ii)

Year (t) Interest rate


1
1.0475
2
3
4
5

Year (t)
1
2
3
4
5

Year (t)
1
2
3
4
5

Bond
10
10
10
10
100

1st
condition
10
9
9
8
79
115

2nd
condition 3rd condition
10
10
18
36
26
78
33
133
396
1982
484
2240

Interest
rate
1.0475

1st
Liability A condition
11
11
0
0
5
4
32
27
93
74
115

2nd
condition
11
0
13
106
369
499

Interest
rate
1.0475

1st
Liability B condition
5
5
10
9
13
11
27
22
85
67
115

2nd
condition 3rd condition
5
5
18
36
34
102
90
359
337
1685
484
2187

Liability A fails at the second test of immunisation, Liability B matches all


three conditions.

6
(i) REITs work much like closed-end pooled funds, but instead of owning a
portfolio of securities, the REIT owns a portfolio of real estate properties
and/or mortgages.

Page 8

Subject ST5 (Finance and Investment Specialist Technical A) September 2009 Examiners
Report

REITs are registered securities and trade in the secondary market, like stocks.
As a result, investors get the benefit of diversification (since most REITs own
a large number of properties) and liquidity.
Unlike other pooled funds, REITs are permitted to use leverage the income
from the properties within the REIT is then used to pay the costs of any loans
involved.
There are two main types of REITs:
Equity REITs these invest mainly in actual real estate properties, such as
office buildings, residential property eg apartments, warehouses and shopping
centres. Equity REITs are usually not highly leveraged.
Mortgage REITs these invest mainly in mortgages and construction loans
for commercial properties and tend to use leverage to a greater degree than
equity REITs.
(ii) Total return from REIT is dividends plus price appreciation. Unlike other
quoted equities, most of the expected return of a REIT comes not from price
appreciation but from dividends.
On average, about two thirds of a REIT's return comes from dividends.
As a high-yield investment, a REIT can be expected to exhibit sensitivity to
interest rate changes.
Typically there is a strong inverse relationship between REIT prices and
interest rates.
On average, it would be safe to assume that interest rate increases are likely to
be met by REIT price declines although the actual change will vary by sector.
For example, some argue that in the case of residential and office REITs rising
interest rates would drive up REIT prices because increasing rates correspond
to economic growth and more demand.
However individual REITS may perform differently depending on their
underlying property exposures and degree of leverage.
(iii) From 2007 to 2008, Equity in Property's net income, or earnings grew by
almost 30% (+$122,500 to $543,847).
These net income numbers, however, include depreciation expenses, which are
significant line items.
For most businesses, depreciation is an acceptable non-cash charge that
allocates the cost of an investment made in a prior period.
But real estate is different than most fixed-plant or equipment investments in
that property rarely 'depreciates' in value (in the short term) as the result of
physical wear.
Net income, a measure reduced by depreciation, is therefore an inferior gauge
of performance and so valuation measures based on earnings are equally
flawed.

Page 9

Subject ST5 (Finance and Investment Specialist Technical A) September 2009 Examiners
Report

(iv) The general calculation involves adding depreciation back to net earnings
(since depreciation is not a real use of cash) and subtracting the gains on the
sales of depreciable property.
These gains are subtracted because we assume that they are not recurring and
therefore do not contribute to the sustainable dividend-paying capacity of the
REIT.
Hence the calculation and reconciliation of net income to FFO for EiP is:

Net earnings
Plus Depreciation
Gain on Depreciable Property
Sales
Other miscellaneous Depreciation
items and gains
FFO

2008
543847
444339
(300426)

2007
421313
419039
(102614)

69838

100651

757598

838389

Credit was given for appropriate description of the calculation, since the requisite
data was not provided in the question.
(v) FFO does not deduct for capital expenditures required to maintain the existing
portfolio of properties, hence the most important adjustment made to calculate
AFFO is the subtraction of capital expenditures.
FFO
Minus Capital Expenditures
AFFO

757598
(181948)
575650

838389
(156776)
681613

This number can be taken directly from the accounts as an estimate of the cash
required to maintain existing properties, although you could make a better
estimate by looking at the specific properties in the REIT.
(vi) Once we have the FFO and the AFFO, we can try to estimate the value of the
REIT.
The key assumption here is the expected growth in FFO or AFFO.
This involves analysing the underlying prospects of the REIT and its sector
exposure, considering:
Prospects

for rent increases

Prospects

to improve/maintain occupancy rates

specific plan to upgrade/upscale properties A popular and successful


tactic is to acquire low-end properties and upgrade them to attract a
higher quality tenant. Often a virtuous cycle ensues. Better tenants lead
to higher occupancy rates (fewer evictions) and higher rents.
growth prospects Many REITs favour fostering FFO growth
through acquisition, but it's easier said than done. An REIT must

External

Page 10

Subject ST5 (Finance and Investment Specialist Technical A) September 2009 Examiners
Report

distribute most of its profits and therefore does not have a lot of excess
capital to deploy. Many REITs, however, successfully prune their
portfolios: they sell underperforming properties to finance the
acquisition of undervalued properties.
The total return on a REIT investment comes from two sources: (1) dividends
paid and (2) price appreciation.
Expected price appreciation comprises two components:
1. Growth in FFO/AFFO
2. Expansion in the price-to-FFO or price-to-AFFO multiple
Given a market capitalisation of $8 billion, then:
Price/FFO = 8000/758 = 10.55x
Price/AFFO = 8000/575.7 = 13.9x
Interpreting price-to-FFO or price-to-AFFO multiples is not an exact science,
and the multiples will vary with market conditions and specific REIT subsectors (for example, apartments, offices, industrial).
Want to avoid buying into a multiple that is too high.
If you are looking at a REIT with favourable FFO/AFFO growth prospects,
then consider both sources together.
If FFO grows at 10%, for example, and the multiple of 10.55x is maintained,
then the price will grow 10%. But if the multiple expands about 5% to 11x,
then price appreciation will be approximately 15% (10% FFO growth + 5%
multiple expansion) making the current market valuation more attractive.
Debt is ignored by assuming that Equity in Property's debt burden is modest
and in line with the industry peers.
If EiPs leverage (debt-to-equity or debt-to-total capital) were above average,
we would need to consider the extra risk implied by the additional debt and
adjust the valuation accordingly.

7
(i)
a. Bid/offer spreads
Taxes
Market impact costs
Commission costs
Opportunity costs
There may be rebates payable if a Multilateral Trading Facility (MTF) is
used.
b. Trades are relatively small compared to market and you would have to
establish the names they are trading in, as larger trade might be highly
liquid where as small trade might be small cap

Page 11

Subject ST5 (Finance and Investment Specialist Technical A) September 2009 Examiners
Report

Everything being equal (timing and stocks traded), bid/offer slightly


higher on 1bn trade
Taxes the same (proportionally)
Market impact higher on 1bn trade
Commission depends but might be lower on larger trade
Opportunity costs will depend on trading time etc. Would be
proportionally equal if traded together
(ii) Sell the equities
Short equity futures
Buy puts set at a level investor is willing to see market value decrease
Use Total Return swaps.
(iii) Sell equities market risk, if equities rise then miss out on the upside, risks
sell at the wrong time
Short equity futures investment performance risk, basis risk
Buy puts investment performance risk, might not be able to buy puts for all
shares in portfolio.
Swaps investment performance risk.
For all derivative-based strategies, counterparty / default risk is a further issue
when over the counter approaches are used.
(iv) Sell equities Would be in cash. Mismatch performance between cash and the
equity markets. Lose out on dividends.
Short equity futures would lose out on market performance would still pick
up alpha from mismatch of futures and underlying portfolio. Could under or
out perform depending on alpha.
Buy puts depends the levels that they are set at. There would be the negative
drag on paying for puts. However, for some shares in the portfolio that fall in
value then could positively impact overall performance as cap losses on those
securities.
Swaps Mismatch performance between the equity market and the other side
of the swap.

END OF EXAMINERS REPORT

Page 12

Faculty of Actuaries

Institute of Actuaries

EXAMINATION
28 April 2010 (pm)

Subject ST5 Finance and Investment


Specialist Technical A
Time allowed: Three hours

INSTRUCTIONS TO THE CANDIDATE


1.

Enter all the candidate and examination details as requested on the front of your answer
booklet.

2.

You have 15 minutes before the start of the examination in which to read the
questions. You are strongly encouraged to use this time for reading only, but notes
may be made. You then have three hours to complete the paper.

3.

You must not start writing your answers in the booklet until instructed to do so by the
supervisor.

4.

Mark allocations are shown in brackets.

5.

Attempt all six questions, beginning your answer to each question on a separate sheet.

6.

Candidates should show calculations where this is appropriate.

AT THE END OF THE EXAMINATION


Hand in BOTH your answer booklet, with any additional sheets firmly attached, and this
question paper.
In addition to this paper you should have available the 2002 edition of the Formulae
and Tables and your own electronic calculator from the approved list.

ST5 A2010

Faculty of Actuaries
Institute of Actuaries

(i)

Describe the key characteristics needed in a commodities index before an


exchange can use the index as a basis for launching a suite of derivatives
contracts.
[5]

(ii)

Explain, by reference to hedging activity, why it would be reasonable to


expect that derivatives based on a commodities index constructed using a
broad range of basic commodities would be more liquid at longer maturities
(e.g. over 1 year) than derivatives based on the individual commodities.
[4]
[Total 9]

(i)

State six factors that need to be considered when analysing the taxation of
investment returns.
[3]

A tax authority is considering introducing Capital Gains taxation of unrealised gains.


(ii)

Discuss the potential advantages and disadvantages of this proposal.

[8]
[Total 11]

The following data has been extracted from the annual report of a UK trading
company.
Income statement

000s
Revenue
Cost of sales
Gross profit
Other expenses
Operating profit
Finance income
Finance costs
Net profit before tax
Tax
Profit after tax

5121.5
(3458.5)
1663.0
(1099.3)
563.7
16.3
(37.9)
542.1
185.1
357.0

Balance sheet
Assets
Non current assets
Current assets
Inventories
Trade receivables
Other current assets
Cash

2355.5

364.4
192.6
13.9
88.2
659.1

Total assets

ST5 A20102

3014.6

Equity and liabilities


Share capital
Other reserves
Retained earnings

673.7
491.2
757.8
1992.7

Non current liabilities


Long term borrowings

348.8

Current liabilities
Trade and other payables
Current tax payable

548.4
194.7
743.1

Total liabilities

1091.9

Total equity and liabilities

3014.6

(i)

Calculate the main ratios for assessing performance that are likely to be of
interest to shareholders and managers, including ratios on profitability and
liquidity.
[8]

(ii)

Comment on the financial position of the business as demonstrated by these


figures.
[7]
[Total 15]

A wealthy investors current asset allocation is 90% equities and 10% cash. The
allocation to equities is mainly invested domestically, with a small allocation
overseas. The domestic equity holdings represent about 5% of the total market
capitalisation of the newly established exchange. The investor wants to change the
allocation to 40% cash, 60% domestic bonds and has approached a transition manager
to implement the new strategy.
(i)

Suggest reasons why the investor might want to change the current asset
allocation.

[3]

(ii)

Outline the costs the transition manager should highlight prior to executing a
trade on behalf of the investor.
[3]

(iii)

(a)

Describe the problems that could be incurred with implementing the


new asset allocation.

(b)

Suggest potential solutions to the problems identified in (a) and their


limitations.
[12]
[Total 18]

ST5 A20103

PLEASE TURN OVER

Following recent economic turmoil in the credit markets, a consortium of


governments has proposed launching a new global currency unit, the GCU, that will
be freely convertible and provide greater security and independence from each
governments fiscal actions than the currencies currently in existence.
(i)

Describe the types of investor who will be attracted to GCU-denominated


investments at the time of the launch of the GCU.
[5]

(ii)

Compare the risk-free overnight interest rate that is likely to be available in the
GCU with that for other major currencies such as the Dollar, Euro, Yen and
Sterling.
[4]

(iii)

Explain how you would expect the term structure of the GCU swap curve to
compare to the US Dollar swap curve shortly after the GCU is successfully
launched.
[9]

(iv)

Explain how your answer to (iii) would differ for the Euro swap curve.
[2]
[Total 20]

A charity and an insurance company invest their assets with the same investment
manager. The finance directors of the two institutions, who know each other well, are
discussing the performance of the investment manager over dinner. They are
surprised that the returns of their respective portfolios have differed considerably over
the last year.
(i)

Suggest reasons why the performance of the two portfolios might differ.

[3]

The following portfolio performance and benchmark performance results have been
provided by the insurance company.
The strategic asset allocation is 10% domestic equities, 25% overseas equities, 15%
cash and 50% bonds.
The portfolio at the start of the year was 2.6m with 1.02m invested in equities in
line with the equity benchmark split. The cash value at start of the year is 410,000.

ST5 A20104

Value at the end of


Domestic Equities value

Quarter 1
314.7

Quarter 2
335.2

Quarter 3 Quarter 4
266.2
270.2

Benchmark return
Overseas Equities value

7.4%
801.5

6.5%
777.4

7.0%
612.7

2.0%
704.6

Benchmark return
Cash value

10%
418.2

4.2%
426.6

11.0%
439.4

12.0%
443.8

Benchmark return
Bonds value

1.5%
1216.8

2.0%
1277.6

5.0%
1366.1

1.0%
1420.8

Benchmark return

6.0%

7.0%

2.0%

4.0%

(all figures in 000s)


The portfolio is rebalanced at the end of the second quarter back to the strategic
benchmark.
(ii)

(a)

Calculate the quarterly and the yearly total portfolio returns.

(b)

Calculate the quarterly and yearly benchmark returns.

(c)

State the under or outperformance of the fund relative to the


benchmark for each quarter and for the year.

State any assumptions you make.


(iii)

Calculate, for each quarter, the returns attribution from:

Stock performance for each asset class


Stock performance at the total fund level
Asset class performance at the total fund level

showing whether the effect had a positive or negative impact on the


performance of the portfolio relative to the benchmark.
(iv)

[8]

[8]

Explain the limitations and disadvantages associated with measuring the


performance of the investment manager.
[8]
[Total 27]

END OF PAPER

ST5 A20105

Faculty of Actuaries

Institute of Actuaries

EXAMINERS REPORT
April 2010 examinations

Subject ST5 Finance and Investment


Specialist Technical A

Introduction
The attached subject report has been written by the Principal Examiner with the aim of
helping candidates. The questions and comments are based around Core Reading as the
interpretation of the syllabus to which the examiners are working. They have however given
credit for any alternative approach or interpretation which they consider to be reasonable.

R D Muckart
Chairman of the Board of Examiners
July 2010

Faculty of Actuaries
Institute of Actuaries

Subject ST5 (Finance and Investment Specialist Technical A) April 2010 Examiners Report

General comments
Candidates typically answered Questions 2, 4 and 6 much better than the others, with
Question 3 and 5 attracting the worst responses. Question 5 represented the opportunity to
demonstrate higher level skills in terms of non-standard/practical application of theory to
current issues in investment. Question 3 required the manipulation of accounts and core
financial information arguably a key skill in any exam looking at financial and investment
matters.
Most candidates seemed to identify and understand the key issues being examined and so
appreciated the general content of solutions that the examiners were looking for however
those that were unsuccessful will find their solutions lacked sufficient detail or application of
knowledge and scored lower accordingly (this was most evident in Questions 1 and 2 where
the first parts were well answered, the latter part less so).
Candidates are reminded of a bias in the paper towards recognising higher level skills and
practical application. Likewise the examination system does properly allow for prior subject
knowledge to be assumed. Investment is a necessarily practical subject and, at this level, the
examiners expect candidates to demonstrate a breadth and depth of competency as would be
expected from a senior student in a frequently evolving discipline. Hence simple
regurgitation of bookwork will never be sufficient to ensure a Pass grade.
In order to succeed, candidates must ensure they familiarise themselves with the prevailing
investment issues and the general market background facing institutional investors in the 12
18 months preceding a diet, more so the solutions (and sources of) being debated by the
various stakeholders. A recurring theme in recent years has been a move towards capital
market rather than purely insurance and asset management solutions hence questions
regarding banking and derivative approaches to asset and liability risk management or
modern financial theory and commercial applications should be considered likely scope for
examination. Against a background of the credit crisis, new asset classes and ways of
structuring investments will themselves generate new types of risk (such as operations,
liquidity, credit and counterparty), so the need for new ways of regulation, monitoring and
management. Finally the examiners encourage candidates to recognise there are different
types of investor beyond purely pension funds and different taxation, time line and cost
considerations will apply.

Page 2

Subject ST5 (Finance and Investment Specialist Technical A) April 2010 Examiners Report

(i)

Interest the exchange would want to ensure that there was sufficient interest
in the index from investors, speculators and hedgers, who are the three main
categories of derivatives users.
Measurement frequency the index would need to be calculated frequently
(e.g. daily) to ensure consistency between the contract values and the
underlying assets.
Calculation process the methodology for construction of the index needs to
be transparent and well understood. This would need to extend to the
weightings and replacement of different commodity assets and clear criteria
about the quality/purity of the commodities being referenced, their location
and delivery dates.
Without the above criteria being satisfied, volumes of contracts will be modest
and the consequential liquidity of the contracts will be relatively low.
Whilst a niche contract may be considered worthwhile for an investment bank,
for an exchange it would generally be considered to be a failure if volumes
remained weak.

(ii)

At short maturities, the individual commodity derivatives may be more liquid


due to hedging activity by producers and customers of the commodity.
However, for longer maturities, there is diminishing interest in such hedging
activity as producers and customers are less able to predict their production
level and input requirement respectively. Also, there may be greater scope to
pass on price changes to end-users, reducing the need to hedge.
Therefore at longer maturities, demand is likely to arise from the activities of
investors and speculators. A broad commodities index would appeal to such
investors who wish to express a positive or negative view on aggregate levels
of demand (based on economic activity). In this scenario, derivative contracts
based on an index would be far more liquid than those based on a single asset.

(i)

Factors that need to be considered are:

the total rate of tax on an investment including consideration of


withholding tax
how the tax is split between different components of the investment return
the timing of tax payments
whether the tax is deducted at source or has to be paid subsequently
the extent to which tax deducted at source can be reclaimed by the
investor
to what extent losses or gains can be aggregated between different
investments or over different time periods for tax purposes

(ii)

Lack of cash to fund tax bill


Treatment of unlisted assets

Page 3

Subject ST5 (Finance and Investment Specialist Technical A) April 2010 Examiners Report

(i)

Subsequent losses
Practical workloads
Rate of tax?
Impact on dividend policy
Allowances?
Impact on investor behaviour?
International comparisons
(with some words of explanation)

Profitability ratios

ROCE

Netprofit before tax and interest


Sharecapital + reserves + long term debt

563.7 + 16.3
1922.7 + 348.8

580
2271.5

= 25.5%
or

Net profit before tax


Sharecapital + reserves

542.1
1922.7

= 28.2%

Asset utilisation ratio


Revenue
Sharecapital + reserves + long term debt

5121.5
2271.5

= 225%

Profit margin

Net profit before tax and interest


Revenue

Page 4

580
5121.5

= 11.3%

Subject ST5 (Finance and Investment Specialist Technical A) April 2010 Examiners Report

Gross profit margin

Gross profit
Revenue

1663.0
5121.5

567.3
5121.5

659.1
743.1

= 32.5%
Operating profit margin

Operating profit
Revenue
= 11.1%
Liquidity ratios
Current ratio

Current assets
Current liabilities
= 0.9 : 1
Quick ratio

Current assets inventories


Current liabilities
=

659.1 364.4
743.1

= 0.4 : 1

Asset gearing
Borrowings
Equity

348.8
1922.7

348.8
348.8 + 1922.7

= 18%
or
Borrowings
Borrowings + Equity

= 15.4%

Page 5

Subject ST5 (Finance and Investment Specialist Technical A) April 2010 Examiners Report

(ii)

High asset utilisation (225%) but relatively low profit margin (11.3%)
suggests a pile em high / sell em cheap strategy. Gross profit 32.5%, net
profit 11% suggests that Other expenses are significant. Liquidity ratios
look inadequate by conventional standards?
But the company is a retail operation with high levels of stock turnover and
low levels of Trade receivables (since most sales are for cash). This is
reinforced by the high levels of current liabilities in the form of Trade
payables since there are typically long delays in paying suppliers for goods.
This explains the anomalous liquidity ratios. Clearly, the ratios need to be
compared with competitors, sector averages and historic equivalents to assess
their adequacy.

The low level of gearing is due to the relatively low fixed assets especially
freehold property). Instead, sale and leaseback is used to generate cash.

Page 6

(i)

Their personal circumstances/investment objective have changed


Need for liquidity going forward
They might be expecting a fall in equity markets
They might expect equities to underperform relative to other asset classes over
the short/medium term
Tax benefits might have changed
Any other sensible suggestion

(ii)

Any taxes associated with buying or selling assets


Commission costs payable on sales and purchases
Bid-offer spreads on both purchases and sales
Price impact of selling or purchasing assets in the market
Any charges by the transition manager or administration charges.
Any foreign exchange costs that might be incurred.
Any rebates that might be achieved by using MTFs
Potential under / out performance due to the timings of equity sales

(iii)

(a)

Owning such a large percentage of the equity market mean sales will
have an impact on prices.
The potential lack of liquidity due to size of current holdings i.e.
finding buyers (equities) or sellers (bonds).
But some equities might be quoted on other exchanges (which might
provide extra liquidity)
Information leakage to the market about what is happening which will
hinder ability to sell assets/purchase assets.
The dealing costs involved
The time needed to implement the change given the size of the
holdings
The possibility of tax, purchase tax or capital gains taxes

(b)

Potential solutions are to:


Sell equity futures and purchase bond futures to change exposure
without physical transaction.

Subject ST5 (Finance and Investment Specialist Technical A) April 2010 Examiners Report

Limitations Suitable futures? Liquidity of futures? Basis risk


Spread sales over a period of time
Limitations The investor might be expecting a market fall and
therefore wants to dispose of assets quickly
Information leakage if in the market too long or too many times
Look for crossing opportunities with other investors to reduce
dealing/spreads costs.
Might not be people looking to cross
Information leakage
Use any cash flows into/out of portfolio to move closer towards the
desired portfolio.
Any other sensible suggestion

(i)

Most liability-focussed investors will typically be interested in receiving an


investment return within a domestic currency. This reflects that they are
investing to meet a liability that comprises an obligation in an existing
currency. Such investors would include insurance companies and pension
funds, and the majority of retail investors who are investing to meet longer
term domestic liabilities (e.g. retirement savings, debts etc.).
The GCU will have a strong appeal to investors who are more multinational in
their outlook, and this group would include retail and high net worth investors
with savings in excess of their domestic liabilities, and multinational investors
such as corporates, sovereign wealth funds, governments, supranational
institutions...
...who are currency-neutral for some or all of their assets.

(ii)

The interest rate is likely to be lower than that in other major currencies as the
risk of devaluation through a weak fiscal policy will be removed.
There is still some devaluation risk if weak fiscal policies in several countries
leads to an expansion of credit and asset values rise, however this is a
secondary risk factor.
Conversely, if the GCU is less liquid or less widely used than the major
currencies, this will lead to slightly higher interest rates to reflect this
illiquidity and higher transaction costs.

(iii)

Soon after the GCU is launched, there will not have been many loans issued
that are GCU-denominated. Therefore there will be relative little supply of
GCU fixed rates.
Any supply of fixed rates (payers) would arise from assets in other currencies
being swapped to GCU interest rates. This activity would typically arise from
the activities of borrowers in other currencies, or investors who wish to take a
view on financing costs in the GCU being lower than in other currencies.
Such a view might arise from a belief that the GCU will not appreciate relative
to other currencies (allowing for the initial difference in swap rates).

Page 7

Subject ST5 (Finance and Investment Specialist Technical A) April 2010 Examiners Report

Demand for fixed GCU rates (receivers) would come from investors who wish
to take a view on financing costs in the GCU being higher than in other
currencies.
There may also be a degree of hedging activity in either direction from
recipients of relatively certain overseas cashflows who wish to pay fixed GCU
rates, or payers of relatively certain overseas cashflows who wish to receive
fixed GCU rates. This type of hedge would be more appropriate where the
mix of overseas currencies was somewhat unstable, and less appropriate in
other cases due to the additional basis risk relative to hedges carried out in
currency pairs.
On the assumption that supply and demand are broadly in balance, one would
expect the GCU swap curve to be lower than the US Dollar swap curve, with
the GCU-US Dollar curve being downward sloping. This reflects that the
expected loss due to currency depreciation will be much lower in the GCU
than the US Dollar, due to lower/nil impact of fiscal policy on the currency.
Reasons why the above might not be the case in the short term might be that
the GCU is not as liquid as the US Dollar. This difference in liquidity may
itself have a term structure, complicating the comparison.

Page 8

(iv)

The difference between the Euro and GCU swap curves should have a flatter
term structure than the difference between the US Dollar and GCU swap
curves. This reflects the weaker impact of a single countrys fiscal policy on
the value of the Euro compared to the US Dollar, although there should still be
a downward slope as there is a risk of concerted policy actions at times of
deflationary pressure.

(i)

Invested in different strategies, different portfolio manager


Invested in different asset classes
Different investment restrictions or tracking error limits
Frequency of rebalancing might differ
Investment fees charged by the manager might differ
Cashflows during the year can alter performance
Different tax status

Subject ST5 (Finance and Investment Specialist Technical A) April 2010 Examiners Report

(ii)

Portfolio returns are net of investment manager fees and taxes

Start value
Domestic Equities
Overseas Equities
Cash
Bonds

Domestic Equities value


Domestic Equities return
Benchmark return
Overseas Equities value
Overseas Equities return
Benchmark return
Cash value

2600 benchmark
291.4
728.6
410.0
1170.0

100
10
25
15
50

Quarter 1 Quarter 2
Rebalance Quarter 3 Quarter 4
314.7
335.2
281.7
266.2
270.2
8.0%
6.5%
5.5%
1.5%
7.4%
6.5%
7.0%
2.0%
801.5
777.4
704.2
612.7
704.6
10%
3%
13%
15%
10%
4.2%
11.0%
12.0%
418.2
426.6
422.5
439.4
443.8

Cash return
Benchmark return
Bonds value

2.0%

2.0%

4.0%

1.0%

1.5%
1216.8

2.0%
1277.6

5.0%
1366.1

1.0%
1420.8

4.0%

5.0%

3.0%

4.0%

6.0%
2751.2

7.0%
2816.8

2816.8

2.0%
2684.4

4.0%
283
9.3

2862.2

2816.8
2862.2

2813.5

2964.1

Bonds return
Benchmark return
Total portfolio

Benchmark

2768.1

1408.4

Quarter and Yearly Outperformance answers


Calculate quarterly
and total year out
performance

Q1

Q2

Q3

Q4

5.8%

2.4%

4.7%

5.8%

Domestic Equities

0.1

Benchmark
Outperformance

6.5%
0.7%

3.4%
1.0%

1.7%
3.0%

5.4%
0.4%

Overseas Equities
Cash
Bonds

0.25
0.15
0.5

Year Fund
Year Benchmark
Year outperformance

9.2%
14.0%
4.8%

Fund

Benchmark

Benchmark end value


Return

2964
14.0%

Page 9

Subject ST5 (Finance and Investment Specialist Technical A) April 2010 Examiners Report

(iii)

Stock attribution answers

Stock attribution
Domestic equity return
Benchmark
Stock attribution

Q1
8.0%
7.4%
0.6%

Q2
6.5%
6.5%
0.0%

Q3
5.5%
7.0%
1.5%

Q4
1.5%
2.0%
-0.5%

Overseas equity return


Benchmark
Stock attribution

10.0%
10.0%
0.0%

3.0%
4.2%
1.2%

13.0%
11.0%
2.0%

15%
12%
3.0%

Cash return
Benchmark
Stock attribution

2.0%
1.5%
0.5%

2.0%
2.0%
0.0%

4.0%
5.0%
1.0%

1.0%
1.0%
0.0%

Bond return
Benchmark
Stock attribution

4.0%
6.0%
2.0%

5.0%
7.0%
2.0%

3.0%
2.0%
5.0%

4.0%
4.0%
0.0%

Stock attribution total


Benchmark total
Overall Stock attribution

5.8%
6.6%
0.8%

2.4%
2.9%
0.5%

4.7%
1.7%
3.0%

5.8%
5.1%
0.6%

Asset class attribution answers


Asset class attribution
Fund
Benchmark
Asset class attribution

(iv)

Q1
6.6%
6.5%
0.1%

Q2
2.9%
3.4%
0.5%

0.0%

Q3
1.7%
1.7%
0.0%

Projection of past results too much reliance on past results which are no
guide to the future performance.
Timescale balancing too frequent, which requires additional administration,
to very infrequent which limits the possibility of detecting any performance
issues. Skill versus luck can be blurred over short-term.
Differing fund objectives might not have a suitable benchmark or peer group
to measure against
Impact on investment manager behaviour knowledge of how being assessed
could influence behaviour of manager to focus too much on measure and not
using their skill.
Costs costs of associated of monitoring and putting together reports etc.

END OF EXAMINERS REPORT

Page 10

Q4
5.1%
5.4%
0.2%

Faculty of Actuaries

Institute of Actuaries

EXAMINATION
8 October 2010 (pm)

Subject ST5 Finance and Investment


Specialist Technical A
Time allowed: Three hours

INSTRUCTIONS TO THE CANDIDATE


1.

Enter all the candidate and examination details as requested on the front of your answer
booklet.

2.

You have 15 minutes before the start of the examination in which to read the
questions. You are strongly encouraged to use this time for reading only, but notes
may be made. You then have three hours to complete the paper.

3.

You must not start writing your answers in the booklet until instructed to do so by the
supervisor.

4.

Mark allocations are shown in brackets.

5.

Attempt all nine questions, beginning your answer to each question on a separate
sheet.

6.

Candidates should show calculations where this is appropriate.

AT THE END OF THE EXAMINATION


Hand in BOTH your answer booklet, with any additional sheets firmly attached, and this
question paper.
In addition to this paper you should have available the 2002 edition of the Formulae
and Tables and your own electronic calculator from the approved list.

ST5 S2010

Faculty of Actuaries
Institute of Actuaries

Outline the main stages in an Asset-Liability Modelling exercise.

[6]

Outline the issues associated with the use of Net Asset Value as a measure of
investment performance.

[7]

You have recently taken over as the pension manager for a declining manufacturing
company which has had a defined benefit pension scheme for more than 35 years.
The fund accounts for the pension scheme for the last two years are set out below.
2009

2010
000s

Contributions and benefits


Contributions received
2,621
Transfers in
400
Benefits payable
(11,000)
Administration expense
(385)
Return on Investment
Investment Income
Return on investments
Investment fees
Balance brought forward
Total

1,265
5,500
(300)
88,655
86,756

2,778
408
(12,320)
(400)

1,151
5,005
(330)
86,756
83,048

Comment on what the fund accounts for 2009 and 2010 indicate about the pension
scheme.
[4]

(i)

Define the term Value at Risk.

(ii)

Calculate the daily Value at Risk with a 95% confidence level for a stock
holding with a current value of 20,000. Assume that log-returns on the stock
show a mean of zero and an annualized volatility of 15%.
[2]

(iii)

Outline the deficiencies associated with using Value at Risk to monitor and
control market risk for a portfolio.
[6]
[Total 9]

ST5 S20102

[1]

The government of a small island population, whose livelihood mainly depends on


fishing, has become increasingly concerned at the lack of private provision for
retirement. In order to ensure islanders are provided for in retirement the government
plans to introduce a compulsory saving scheme. The government has no prior
financial experience of such saving schemes.
(i)

Discuss the different objectives of the islanders that should be considered


when designing the new scheme.
[4]

A young adventurous fisherman with no dependents and who earns a below average
salary is considering his investment options for retirement.
(ii)

Discuss, for each of the objectives identified in (i), which asset type(s) could
be suitable to meet the objectives of the young adventurous fisherman.
[7]
[Total 11]

You are employed as a retail analyst at a well respected investment bank. A very
successful national pizza delivery company is planning to expand into a new overseas
market and is looking to raise additional capital. The company has approached your
bank for financial assistance. You have been asked to prepare a report for the
investment bank on whether they should provide the requested finance.
(i)

List the factors that you would consider when evaluating the pizza delivery
company.
[3]

(ii)

Outline the areas you would investigate to form a view on the factors in (i).
[3]

(iii)

ST5 S20103

Comment on any potential issues the management of the pizza delivery


company should consider when entering an overseas market.
[3]
[Total 9]

PLEASE TURN OVER

A washing machine manufacturer, WashCo, is looking to purchase two rival


businesses, CleanCo and SpinCo, which are located in two different overseas
countries from WashCo. As at the end of the last financial year the three companies
posted the following profits:
WashCo
Gross Profits
Company Tax
Dividend paid
Tax rate paid by shareholders
on distributed profits

CleanCo

SpinCo

100m
30m

75m
25m

50m
10m

40m

30m

20m

30%

10%

0%

SpinCo pays an extra 10m tax on the dividends paid. In all three countries companies
are required to distribute 40% of gross profits as dividends.
(i)

(a)

Identify the tax system in place for each company.

(b)

Outline the structure of each tax system in (a).


[3]

(ii)

(a)

Calculate the total tax payable on each companys profits and


dividends.

(b)

Calculate the tax payable by each company as a percentage of gross


profits.
[3]

(iii)

(a)

Express the tax payable by each company as a percentage of gross


profits.

(b)

Comment on the differences between the results in parts (ii)(b) and


(iii)(a).
[5]

If WashCo is successful in purchasing its two rivals, the board of directors will locate
the company in one of the three locations of the current companies. The board have
asked you, as an independent tax advisor, to help them make the decision.
(iv)

Explain which country you would recommend the board moves the company
to.
[2]

(v)

Suggest other taxation factors that the board of directors should consider when
making a final decision.
[2]
[Total 15]

ST5 S20104

(i)

Explain, using the principles of behavioural finance, the types of biases that
can affect an investors view of the probability of an event occurring that is
outside their control, such as an equity market crash.
[4]

(ii)

Describe the additional forms of bias that are introduced when an active
investor has the discretion to improve an outcome compared with a passive
(index) investor.
[3]

In a developing country, house prices have been rising rapidly for several years
fuelled by loans of up to 100% of the value of the house. The government has
introduced rules that restrict the amount of borrowing to 50% of the value of the
house. Following the restriction in lending, house prices have started to fall and are
expected to do so for the next couple of years.
(iii)

Explain the behaviours exhibited by the following people:


(a)

An investor who has been successful in the past buying residential


property to profit from capital appreciation, who has decided to
continue to buy residential property over the next couple of years.

(b)

An economist who has written a recent article commenting that house


prices were always expected to fall. The economist uses historic data
to prove this point. All news articles written prior to the borrowing
restriction only indicated that prices would continue to rise.
[5]

A couple have taken advantage of the falling house prices and purchased a new house.
As part of the move they wish to review their house insurance options. They are
deciding whether to stay with their current insurance provider or to move to a new
entrant into the market.
Details of the two policies are as follows:
ExistingCo

NewCo

Premium

650

500

Excess payable

100 per claim

80120 depending on
profitability of company

Policy schedule

Detailed booklet setting out all


terms and conditions

Limited information provided


on terms and conditions

The couple have not made a claim in the past and it is unlikely that they will make a
claim over the coming year. The couple decide to take out the policy with
ExistingCo.
(iv)

ST5 S20105

Outline, with reasons, three possible behaviours which explain why the couple
have stayed with ExistingCo.
[5]
[Total 17]

PLEASE TURN OVER

A large investment fund currently has $500 million invested in US equities. As a


result of a recent asset allocation meeting, the decision has been taken to invest a
further $400 million in US equities by selling the corresponding amount of UK
equities. The asset allocation team has emphasised the need for the switch to go
ahead as soon as possible.
(i)

Describe the practical problems of carrying out such a switch without the use
of derivatives.
[6]

(ii)

Explain how this switching process can be made easier by the use of
derivatives.

[4]

You decide to buy US equity exposure by buying the S&P500 December futures
contract. You know that you have sufficient cash to cover the margin position. The
unit of trading is $500 per index point and you have been quoted a price of $800 for
the December contract.
(iii)

Calculate the number of contracts you would need to buy to gain the required
US equity exposure.
[2]

Prior to this additional US equity investment, the fund had no holdings in the
telecommunications sector. Telecommunications stocks constitute 18% of the
S&P500 index.
(iv)

Calculate the exposure of the fund to telecommunications after the purchase of


the futures.
[1]

(v)

Discuss how you might attempt to eliminate the telecommunications


weighting.

[6]

The asset allocators of the fund are bullish on US stocks but they are worried about
the level of the dollar, which they think may depreciate in the short-term against
sterling. As a result, they would like only one half of the fund exposed to the dollar.
(vi)

Explain how you might achieve this reduced exposure, including in your
answer details of the problems that would have to be overcome.
[3]
[Total 22]

END OF PAPER

ST5 S20106

INSTITUTE AND FACULTY OF ACTUARIES

EXAMINERS REPORT
September 2010 examinations

Subject ST5 Finance and Investment


Specialist Technical A

Introduction
The attached subject report has been written by the Principal Examiner with the aim of
helping candidates. The questions and comments are based around Core Reading as the
interpretation of the syllabus to which the examiners are working. They have however given
credit for any alternative approach or interpretation which they consider to be reasonable.
T J Birse
Chairman of the Board of Examiners
January 2010

Institute and Faculty of Actuaries

Subject ST5 (Finance and Investment Specialist Technical A) September 2010 Examiners Report

General comments
Pleasingly, this diet reversed the trend and was a much better answered paper than previous diets
resulting in a higher pass rate even with a higher pass mark. Candidates typically answered
Questions 1 and 6 much better than the others (albeit foregoing a lot of marks), with Question 2 and 4
attracting the worst responses, considerably so, with average scores of less than 30% of the available
marks and given the fairly basic subject matter, this was something of a surprise. Indeed questions 3
and 9 were little better answered and in question 3, candidates continued to demonstrate difficulties
with accounts based questions, a fairly fundamental area of investment analysis. Questions 5 and 8
represented opportunities to demonstrate higher level skills in terms of non-standard/practical
application of theory to current or unusual issues in investment hence candidates who wish to
progress to SA6 will need to improve their understanding of and approach to this type of question.
That said, most candidates seemed to identify and understand the key issues being examined and so
appreciated the general content of solutions that the examiners were looking for however those that
were unsuccessful will find their solutions lacked sufficient (and often the most basic) detail or
application of knowledge and scored lower accordingly. Many candidates still deviate from the topic
and include irrelevant material or over emphasise minor points although candidates will not be
explicitly penalised for this, it gives an impression of a lack of understanding and, more importantly,
wastes limited time. Time and priority management are key skills actuaries need to have. Where
candidates made relevant points in other parts of their solutions, the examiners have used their
discretion as to whether to recognise these answers or not. Likewise the examiners share and agree
alternative possible solutions to questions during the marking process including a meeting convened
to review a common "test batch".
Some candidates believed that parts of Q7 were asking for the same information twice - this was not
the intent of the examiners but in order to treat all candidates fairly, the second part was question was
discounted, although all relevant points in either section were given credit for and the overall marks
scaled up accordingly.
Candidates are reminded of a bias in the paper towards recognising higher level skills and practical
application this is intentional and will continue. Likewise the examination system does properly
allow for prior subject knowledge to be assumed. Investment is a necessarily practical subject and, at
this level, the examiners expect candidates to demonstrate a breadth and depth of competency as
would be expected from a senior student in a frequently evolving discipline. Hence simple
regurgitation of bookwork will never be sufficient to ensure a Pass grade and this was evident from
the dispersion of candidates responses in the more differentiating questions.
As noted in previous reports, in order to succeed, candidates must ensure they familiarise themselves
with the prevailing investment issues and the general market background facing institutional investors
in the 1218 months preceding a diet, more so the solutions (and sources of) being debated by the
various stakeholders. A recurring theme in recent years has been a move towards capital market and
corporate finance rather than purely insurance and asset management solutions hence questions
regarding banking and derivative approaches to asset and liability risk management or modern
financial theory and commercial applications should be considered likely scope for examination.
Against a background of the credit crisis, new asset classes and ways of structuring investments will
themselves generate new types of risk (such as operations, liquidity, credit and counterparty), so the
need for new ways of regulation, monitoring and management. Finally the examiners encourage
candidates to recognise there are different types of investor beyond purely pension funds and different
taxation, time line and cost considerations will apply.

Page 2

Subject ST5 (Finance and Investment Specialist Technical A) September 2010 Examiners Report

ALM is increasingly associated with stochastic modelling (although deterministic


methodologies can be used).
The main stages in an ALM exercise are usually as follows:
1. The key objectives that investment and funding policy should aim to achieve need
to be clarified. These involve objectives such as:
future on-going funding levels
future solvency levels
future company contribution rates
2. Suitable assumptions to use in the study need to be agreed.
3. Data needs to be collected to carry out the projections.
4. The overall nature of the liabilities is considered a broad-brush analysis of
current funding level, maturity and cash flow.
5. An analysis would be carried out to identify how the scheme might progress in
the future if different investment strategies were adopted. It would also be
appropriate to test the sensitivity of the results to different parameter assumptions.
6. Different asset mixes would then be analysed in more detail to assess the risks
(relative to the liabilities) and the rewards of each alternative under consideration.
7. The results would be summarised and presented.
ALM will often be used in a control cycle context that involves monitoring the
experience and revisiting earlier stages of the process.

The net asset value of a company, or the net asset value per share, is clearly only one
component of overall value. So, if other things are equal, a share with a higher
proportion of its share price represented by net asset value should be cheaper than a
share that has less asset backing.
However other things are unlikely to be equal as the market will, in both cases, be
attaching a full value to all future cash flow, including that resulting from holding all
existing assets and liabilities.
The net asset value is, in the end, an accounting number, so it is important to
understand how it has arisen and to make appropriate adjustments. For example,
property or other non-quoted assets may be difficult to value.
For example, a company that has expanded by acquisition will have acquired
goodwill on its balance sheet which will form part of its net asset value. A similar
company that has only grown organically will appear to have a lower net asset value

Page 3

Subject ST5 (Finance and Investment Specialist Technical A) September 2010 Examiners Report

per share. Generally, goodwill will have to be evaluated for relevance and removed if
appropriate in order to make valid comparisons.
Some businesses require more assets than do others. For example a manufacturing
business will generally require plant, premises and stock whereas a service business
will typically require less assets. Comparisons of net asset value between companies
in different sectors will inform only about the difference between the sectors. Some
assets are more intangible/harder to value. Human capital i.e. an asset of service
companies is rarely included in NAV.
Net assets surplus to those required to run the business may not attract full value. It is
usually regarded as inefficient for a company to hold surplus assets, and also it is
harder to maintain management discipline when there is a substantial asset cushion.
Net Asset Value will not reflect risk
Points in favour include the fact that NAV is

readily available
objective
independent
auditable
relevant for break-up

The main observation is that the benefits payable are greater than the investments and
other income. All outgoings are increasing, all incomings (except Contributions) are
falling. This suggests that either one or more of the following:

The scheme is very mature and/or the scheme is being run-off/wound down. It
will be necessary to consider the extent of any existing overfunding / surplus and
how this is being drawn down.

The schemes assets are underperforming to expectations by a large percentage.

The scheme is underfunded and liabilities are significantly in excess of the assets.
It will be necessary to consider the degree of employer covenant and how the
underfunding might be made good.

Credit was given for other appropriate comments.

Page 4

Subject ST5 (Finance and Investment Specialist Technical A) September 2010 Examiners Report

(i)

Value at Risk is a measure of the maximum loss that might be suffered on a


portfolio or a holding in a specified timescale and with a specified probability.
As such, it is a measure of Market Risk the risk relating to changes in the
value of a portfolio due to movements in the market value of the assets held.

(ii)

Given an annualized volatility of 15%, the daily standard deviation is 0.15/


250 (assuming 250 trading days in the year). Thus the VaR is given by
20000 1.6449 0.15/ 250 = 312.10
(Alternatively, if we assume 365 days in the year, the daily standard deviation
is 0.15/ 365 and the VaR is
20000 1.6449 0.15/ 365 = 258.29
(Credit was given for any sensible number of working days given international
variations.)

(iii)

VaR calculated using the assumption that stock log returns are distributed
normally is found to be an underestimate in practice. This is because short
period stock returns show a more fat-tailed distribution.
The assumption of normality also implies that stock returns are symmetrically
distributed. In practise, return distributions may be skewed.
The calculation assumes that the past experience will be maintained, whereas
the future probability distribution may well be different (particularly regarding
volatility).
The VaR figure, once calculated, does not say anything about how bad losses
could actually be (and definitely does not specify the worst possible loss).
As a risk measure, VaR has poor aggregation qualities. The VaR of a merged
portfolio may exceed the sum of the VaRs of the individual portfolios
depending on correlation/(lack of) diversification consideratons.
VaR ignores any problems relating to market liquidity.
There is no single optimal choice for the time horizon and confidence level at
which to calculate VaR.
VaR is often not well understood and so applied (especially on the retail side).

(i)

Attitude to risk of the islanders, particularly the investment risk of investing in


different asset classes.
Time preference and consumption needs of the islanders. Preferred retirement
age.

Page 5

Subject ST5 (Finance and Investment Specialist Technical A) September 2010 Examiners Report

Existing assets of islanders; and disposable income available for savings.


Need for diversification
The islanders needs/preferences between the need for income or capital
growth. Inflation / growth protection requirements
Tax position of the islanders
The level of provision the islanders need/want in retirement
Flexibility of contributions.
(ii)

The fisherman is adventurous so he is likely to have a higher than normal


acceptance of risk so he is likely to invest in higher risk asset classes such as
equities, hedge funds (or any other sensible suggestion)
He is young so has a long way to retirement so is more than likely to be happy
to have assets invested for a long time and therefore can accept risk, again
equity type exposure is probably appropriate.
He earns a low salary so is unlikely to have any existing assets. If he wants to
build a level of assets quickly he will invest in higher return seeking assets
such as equities. However, if he is concerned that he wants to maintain the
minimal assets he has then he is more likely to invest in more stable assets
such as bonds or cash.
Tax position he will want to try and minimise tax and therefore look for
assets that have lower tax implications. The asset class will be dependant on
the tax rules for the various asset classes.
If we assume the fisherman wants as much as possible in retirement then he
will opt for higher growth assets such as equities, however if he wants more
certainty over the level of benefits in retirement then would invest in less risky
assets such as cash and bonds.
Within each asset class, it will be appropriate to diversify the holdings.
(Credit was given for other relevant comments)

Page 6

(i)

Management ability
Management experience in running businesses overseas
Quality of products
Prospects for growth, especially in overseas market
Competition both in domestic and overseas market
Input costs
Retained Profits

Subject ST5 (Finance and Investment Specialist Technical A) September 2010 Examiners Report

History of company
Existing borrowing/capital structure

The financial accounts and accounting ratios


Dividends and the earnings cover
Profit variability and growth
Level of borrowing
Level of Liquidity
Growth in asset values
Market / market potential
Logistics required
Personnel availability (employment levels, etc.)
Competitor analysis i.e. comparative figures for other similar companies

Is there demand for the pizza?


Are there local differences in preferred pizzas
Lack of brand recognition
Language barriers?
Any legal or regulatory constraints?
Logistics including where will ingredients/personnel be sourced from
How will new overseas department be structured
Financial management e.g. currency
Any other sensible suggestions

(ii)

(iii)

(i)

WashCo Classical tax system where profits are taxed twice once in the
hands of the company and once in the hands of the investor.
CleanCo Split-Rate tax system where retained and distributed profits are
taxed twice but at a different rate.
SpinCo Imputation tax system where the company has to deduct some of the
tax payable by investors on distributions and pay it direct to the government.

(ii)

WashCo 30m in company tax and 12m for dividends = total 42m, 42%
CleanCo 25m in company tax and 3m for dividends = total 28m, 37%
SpinCo 10m in company profits and 10m in dividends = 20m, 40%

Page 7

Subject ST5 (Finance and Investment Specialist Technical A) September 2010 Examiners Report

(iii)

WashCo = 30%
CleanCo = 33%
SpinCo = 40%
WashCo profits have the highest overall tax burden but the company burden is
the lowest of the three companies.
The highest tax percentage is for WashCo, however the total tax payable is
split at the same rate between the company and shareholders. Spin Co has the
lowest rate of company tax but when combined the amount payable on
distributed profits the tax is the highest of all three, and as a company the tax
payable is the highest of all three.

(iv)

The new entity would want to be in the region where they pay the lowest tax
rate. Although WashCo profits have the highest total tax, the burden on the
company is the lowest so you would recommend to the board of directors that
the new entity remained in the original country. However, if shareholders
interests were to predominate, the company should locate where CleanCo is
located as the total tax paid on profits and dividends is the lowest.

(v)

How stable are the taxation rates, are they likely to change in the future?
Are there any tax reliefs available in the countries that would reduce the tax
bill, i.e. carry over previous losses.
The timing and frequency of the tax payment, does it all need to be paid at
once?
Their own holdings in the company which will effect their tax payments.
The tax treatment of international transactions i.e. double taxation
agreements or the application of transfer pricing policies

(i)

Anchoring this is the term used to refer to the fact that people base their
views of the likelihood of an event on recent experience.
Dislike of negative events the degree to which an outcome is considered
negative or positive has a significant influence on an estimate of its likelihood.
In general, people are optimists and overestimate the likelihood of positive
events.
Representative heuristics people find more probable that which they find
easier to imagine.
Availability people are influenced by the ease with which something can be
brought to mind. This can lead to biased judgements when examples of one
event are inherently more difficult to imagine than examples of another.

Page 8

Subject ST5 (Finance and Investment Specialist Technical A) September 2010 Examiners Report

(ii)

Individuals are typically overconfident about their own skills and insights.
Overconfidence arises from:
Hindsight bias events that happen will be thought of as having been
predictable prior to the event, events that do not happen will be thought of as
having been unlikely prior to the event.
Confirmation bias people will tend to look for evidence that confirms their
point of view (and will tend to dismiss evidence that does not justify it).
The discrepancy between confidence and accuracy increases as an individuals
expert knowledge increases (even where accuracy improves with knowledge,
confidence increases by more).
Option issues such as regret aversion may also be relevant.

(iii)

(a)

Anchoring although the data shows that capital appreciation is


unlikely to continue, the investor is anchored that history will repeat
itself and therefore experts opinion is not given due consideration.
Overconfidence past evidence of the investor's ability to make
money means could have become overconfident in his own abilities.

(b)

(iv)

Hindsight bias Events that happen will be thought of as having been


predictable prior to the event, i.e. re-evaluating past evidence to prove
the scenario was always likely to occur.

Regret aversion by retaining the existing arrangements, people minimise the


possibility of regret that new policy is not as good as current one.
Ambiguity aversion people pay premium for rules. Here the policy schedule
is unclear for the lower premium and therefore pay premium to receive more
detailed rules of existing co.
Status Quo bias people have a preference to keep things as they are.
Representative heuristics as the amount of detail increases, its apparent
likelihood may increase.

(i)

Considerable teamwork required by UK and US fund managers to ensure that


the switch goes smoothly. Very time intensive.
US fund manager can only buy as much as UK fund manager can sell at any
one time.
May take some time for both managers to formulate their buy and sell
programs.

Page 9

Subject ST5 (Finance and Investment Specialist Technical A) September 2010 Examiners Report

May take time for UK to physically sell some of their stocks. Trying to sell
stocks aggressively may result in poor prices being obtained.
Similarly aggressive buying by the US manager may well result in poor prices
being obtained.
These problems are particularly acute when unmarketable securities are
involved or where the normal market size for deal in the securities is small.
Costly two sets of commissions and two spreads to be paid.
No mention if switch is strategic or tactical if tactical may well have to
reverse position in the near future.
The possibility of the crystallisation of capital gains leading to a tax liability.
(ii)

UK fund manager simply sells a number of futures contracts whose exposure


is equivalent to the amount of stock needed to be sold and US fund manager
buys a number of futures whose exposure is equivalent to the amount of stock
needed to be bought.
Decision is implemented immediately.
Futures markets very liquid little risk of price drag.
Purchase of futures gives fund managers time to construct and execute their
buy/sell programs. As more stock is bought (sold) so the requisite number of
futures can be sold (bought back).
If decision is tactical the positions can be more easily and cheaply reversed out
of without harming the underlying stock portfolios.

(iii)

Value of one contract is 500 * 800 = $400,000


No of contracts 400,000,000 / 400,000 = 1000

(iv)

Telecommunications exposure = (500 * 0 + 400 * 18%) / 900


= 8% or $72 million

(v)

Look at main constituents of S&P500 Telecommunications sector. To


completely negate telecommunications exposure:
Sell stocks constituting the sector short (if allowed to) equivalent to the new
exposure of the $900m fund.
Buy put options on the stocks in the sector again equivalent to the exposure.
Could just do either of the above with a representative group of stocks to
reduce the complexity of the transaction.

Page 10

Subject ST5 (Finance and Investment Specialist Technical A) September 2010 Examiners Report

Sell futures on traded Telecommunications indices this would introduce


some element of mismatching risk.
Enter into an OTC swap transaction to exchange Telecommunications returns
for some other suitable sector. Introduces counterparty risk.
(vi)

If the investment is via futures, it may be that only the margin is exposed to
the dollar anyway. If have full exposure, then sell forward currency contracts
equivalent to half of the funds value.
Such contracts are short term and need to be rolled over (though not a problem
here).
To precisely keep one half of the fund exposed to the dollar, need to know the
value of the fund on the expiry/sale of the contract. This is unlikely, so some
estimate of funds future value required.
Hedging small amounts such as dividend receipts will be costly. However,
would have to ensure that exposure to US market is still maintained after
reducing telecommunications exposure.

END OF EXAMINERS REPORT

Page 11

INSTITUTE AND FACULTY OF ACTUARIES

EXAMINATION
20 April 2011 (pm)
Subject ST5 Finance and Investment
Specialist Technical A
Time allowed: Three hours
INSTRUCTIONS TO THE CANDIDATE
1.

Enter all the candidate and examination details as requested on the front of your answer
booklet.

2.

You have 15 minutes before the start of the examination in which to read the
questions. You are strongly encouraged to use this time for reading only, but notes
may be made. You then have three hours to complete the paper.

3.

You must not start writing your answers in the booklet until instructed to do so by the
supervisor.

4.

Mark allocations are shown in brackets.

5.

Attempt all eight questions, beginning your answer to each question on a separate
sheet.

6.

Candidates should show calculations where this is appropriate.

AT THE END OF THE EXAMINATION


Hand in BOTH your answer booklet, with any additional sheets firmly attached, and this
question paper.
In addition to this paper you should have available the 2002 edition of the Formulae
and Tables and your own electronic calculator from the approved list.

ST5 A2011

Institute and Faculty of Actuaries

Explain what is meant by stress testing a portfolio.

(i)

Outline the main use of swaps in portfolio management by a company seeking


to match assets and liabilities.
[3]

(ii)

Discuss the main advantages that interest rate swaps have over conventional
fixed interest securities in portfolio management by a company seeking to
match its assets and liabilities.
[3]
[Total 6]

[4]

(i)

Outline the different types of infrastructure projects.

[1]

(ii)

Describe the key characteristics of infrastructure investment.

[5]

(iii)

Comment on the differences between infrastructure and private equity


investment.
[3]
[Total 9]

A portfolio consists of 0.5m in asset X and 0.3m in asset Y. Assume that the daily
volatilities of both assets are 0.6%, that the correlation between their returns is 0.4 and
that returns on the assets are normally distributed.
(i)

Calculate the 10-day 98% Value at Risk of the portfolio.

(ii)

Calculate the saving in VaR from holding a diversified portfolio rather than
holding two separate assets.
[3]

(iii)

Discuss the practical limitations and difficulties associated with diversifying a


portfolio.
[5]
[Total 11]

ST5 A20112

[3]

A risk-seeking wealthy investor has decided to invest a small portion of his assets
with an investment manager for his proposed retirement in five years. The investor is
deciding between two investment managers, High Return Investment Company and
Strong Growth Investment Company. High Return specialises in equity investment,
whereas Strong Growth invests mainly in Government Bonds. Both managers are
large international companies with substantial assets under management.
(i)

Discuss which investment manager is likely to be most suitable for the


investor.

[5]

One of the investors advisors suggests a different investment manager, FBNX


Investors, a small start up company with five employees investing mainly in futures
and credit default swaps.
(ii)

(iii)

List the financial risks faced by an investor investing with an investment


manager.

[2]

Discuss how the financial risks are likely to differ between investing with
FBNX Investors and investing with Strong Growth Investment Company. [5]
[Total 12]

A small listed company specialising in sourcing, processing and supplying restaurants


with beef and pork has recently agreed in principle to buy a chain of restaurants. The
terms of the agreement are still being finalised.
(i)

State with reasons what sort of acquisition the takeover is.

[2]

(ii)

Discuss why the company would be interested in buying the chain of


restaurants.

[3]

Whilst the takeover is still being finalised a series of market reports are published that
suggest the country is heading for a recession and that the cost of livestock is likely to
increase by 30% over the next 12 months.
(iii)

Discuss the impact on the business for both the company and the chain of
restaurants of:
(a)
(b)

the recession
the predicted increase in livestock costs
[6]

(iv)

Describe the actions regarding the takeover that the company's Board of
Directors might take in the light of the market reports.
[5]
[Total 16]

ST5 A20113

PLEASE TURN OVER

A risk averse investor has historically invested her equity portfolio with an index
tracking manager, but has recently decided to change to an active equity manager.
The investor has four managers to choose from and has been provided with the
following information on performance over the last four quarters:

Q1

Q2

Q3

Q4

Manager A
Manager B
Manager C
Manager D
Index benchmark return

3.5%
2.0%
5.0%
4.0%
2.0%

2.0%
2.0%
2.0%
3.0%
2.0%

5.0%
3.0%
6.0%
4.0%
3.0%

8.5%
4.0%
11.0%
6.0%
4.0%

Standard deviation of index


Risk free rate

0.6
7% p.a.

(i)

(ii)

Covariance
with Index
0.6
0.2
0.67
0.4

(a)

Calculate the outperformance of the investment managers relative to


the benchmark.

(b)

Rank the investment managers from 1 to 4, with 1 being the highest


outperformer.
[6]

(a)

Calculate the risk adjusted performance of the investment managers


using the Jensen measure.

(b)

Rank the investment managers from 1 to 4, with 1 being the highest


performer.
[6]

(iii)

Discuss which manager is likely to be the most suitable for the investor.

(iv)

Outline the circumstances in which standard deviation is a more appropriate


measure of risk for an investor than beta.
[2]
[Total 18]

ST5 A20114

[4]

A listed supermarket chain (Best Supermarket) based in a small country wishes to


expand internationally over a three year period. The Board of Directors is considering
the different alternatives for raising capital to implement their growth plans.
(i)

Outline the key differences between equity and debt financing.

(ii)

(a)

List the different debt instruments Best Supermarket could issue to


raise the required finance.

(b)

List the main features that differentiate the types of debt described in
(a).
[5]

(iii)

[3]

Discuss how the features listed in (ii) (b) are likely to differ between Best
Supermarket and a large international supermarket chain looking to raise
additional finance.
[5]

Best Supermarket has produced strong gross profits over the last two years while the
country has been in recession.
(iv)

Suggest reasons why the company needs to raise finance even though its gross
profits have been strong.
[5]

(v)

Discuss how Best Supermarkets share price is likely to have performed over
the last two years compared to the price of shares in a chain of luxury handbag
stores.
[6]
[Total 24]

END OF PAPER

ST5 A20115

INSTITUTE AND FACULTY OF ACTUARIES

EXAMINERS REPORT
April 2011 examinations

Subject ST5 Finance and Investment


Specialist Technical A

Institute and Faculty of Actuaries

Subject ST5 (Finance and Investment Specialist Technical A) April 201, Examiners Report

General comments
Pleasingly, this diet continued the trend from last October and was a much better answered
paper than in recent years resulting in a higher pass rate. Candidates typically answered
Questions5, 6 and 7 much better than the others (albeit still foregoing 30-40% or more of
marks available), with Question 4 attracting the worst responses, considerably so, with
average scores of around a quarter of the available marks. Indeed question 1 was little
better answered. Questions 1 and 4 go to the heart of modelling and understanding risk and
a lack of understanding in these areas has been cited as a key factor in the "Credit Crisis"
when risk management models failed. This is an area where actuaries could reasonably feel
they could offer relevant skills and knowledge so it is important candidates demonstrate this.
Likewise understanding of the theory and practicalities of portfolio diversification are core
tenets of investment. Questions 5, 6 and 7 represented opportunities to demonstrate higher
level skills in terms of non-standard/practical application of theory to current or unusual
issues in investment hence candidates who wish to progress to SA6 will need to improve
their understanding of and approach to this type of question. The examiners were pleased to
see progress in the scores being achieved as well as better data handling.
Most candidates seemed to identify and understand the key issues being examined and so
appreciated the general content of solutions that the examiners were looking for however
those that were unsuccessful will find their solutions lacked sufficient (and often the most
basic) detail or application of knowledge and scored lower accordingly. Many candidates
still deviate from the topic and include irrelevant material or over emphasise minor points
although candidates will not be explicitly penalised for this, it gives an impression of a lack
of understanding and, more importantly, wastes limited time. Time and priority management
are key skills actuaries need to have. Where candidates made relevant points in other parts
of their solutions, the examiners have used their discretion as to whether to recognise these
answers or not. Likewise the examiners share and agree alternative possible solutions to
questions alongside the approach outlined below.

Candidates are reminded of a bias in the paper towards recognising higher level skills and
practical application this is intentional and will continue. Likewise the examination system
does properly allow for prior subject knowledge to be assumed. Investment is a necessarily
practical subject and, at this level, the examiners expect candidates to demonstrate a breadth
and depth of competency as would be expected from a senior student in a frequently evolving
discipline. Hence simple regurgitation of bookwork will never be sufficient to ensure a Pass
grade and this was evident from the dispersion of candidates responses in the more
differentiating questions.
In order to succeed, candidates must ensure they familiarise themselves with the prevailing
investment issues and the general market background facing institutional investors in the 12
18 months preceding a diet, more so the solutions (and sources of) being debated by the
various stakeholders. Hence questions regarding banking and derivative approaches, as well
as asset management and insurance solutions, to asset and liability risk management
(including model risk) or modern financial theory and commercial applications should be
considered likely scope for examination. Against a background of the credit crisis, new asset
classes and ways of structuring investments will themselves generate new types of risk (such
as operations, liquidity, credit and counterparty), so the need for new ways of regulation,
monitoring and management. This paper also looked at the cost of capital, a major

Page 2

Subject ST5 (Finance and Investment Specialist Technical A) April 201, Examiners Report

consideration for clients since capital can no longer be assumed to be freely available or low
cost. Finally the examiners encourage candidates to recognise there are different types of
investor beyond purely pension funds and different taxation, time line and cost considerations
will apply - it would seem that candidates have taken this on board.
Whilst the examiners will tolerate bullet point style responses, some candidates handwriting
was too poor to assess and they will have lost marks. Likewise "text speak" abbreviations
will not be accepted.

Page 3

Subject ST5 (Finance and Investment Specialist Technical A) April 201, Examiners Report

The risks that are incurred by extreme market events can be identified and
investigated by the process of financial stress testing. This involves subjecting a
portfolio to extreme market moves by radically changing the underlying portfolio
assumptions and characteristics, in order to gain insight into portfolio sensitivities to
predefined risk factors. This pertains in particular to asset correlations and
volatilities. There are two types of stress test:

to identify weak areas in the portfolio and investigate the effects of localised
stress situations by looking at the effect of different combinations of correlations
and volatilities

to gauge the impact of major market turmoil affecting all model parameters, while
ensuring consistency between correlations while they are stressed

A major part of establishing a comprehensive stress testing framework should


therefore focus on constructing stress test scenarios that apply to the specific portfolio
(i.e. a deterministic approach). These scenarios should be tailored to reveal
weaknesses in the portfolio structure in terms of risk exposure and sensitivity, and
should thus focus on the risk factors that the portfolio is most exposed to.
(Candidates who suggest a stochastic approach needed to justify this to be given
credit.)

(i)

A company can use swaps to reduce risk by matching its assets and liabilities.
For example a company which has short term liabilities linked to floating
interest rates but long term fixed rate assets can use interest rates swaps to
achieve a more matched position. Currency swaps would be used by a
company with liabilities in one currency and assets in another.
An inflation swap allows a receiver of inflation-linked payments to pay these
to a counterparty in return for receiving a fixed payment. Typical payers of
inflation under inflation swaps will include holders of loans with inflationlinked payments or leaseholders who receive inflation-linked rental income.
Institutional investors such as pension funds, with inflation-linked liabilities,
can use inflation swaps to receive inflation and thereby hedge the market risk
from uncertain future inflation within their liabilities.
(Equivalent marks were awarded for any other example developed in
equivalent detail.)
Swaps might be also used as a short-term transition management tool.

(ii)

Page 4

Flexibility. IRSs are OTC so can be made to measure for the companys
portfolio.
Dealing costs. With the exception of the most liquid bonds, IRSs can have
lower dealing costs.
Complexity. Assuming that the company has the appropriate expertise and
systems, an IRS hedge can be less complex than putting together a portfolio of
bonds and attempting immunisation.

Subject ST5 (Finance and Investment Specialist Technical A) April 201, Examiners Report

A suitable swaps deal might be quicker to implement than transactions in


physical assets.
(Other sensible comments were also awarded marks.)

(i)

Infrastructure tends to be separated into two broad subsets economic and


social. Economic infrastructure includes highways, water and sewerage
facilities, energy distribution and telecommunication networks whereas social
infrastructure encompasses schools, universities, hospitals, public housing and
prisons.

(ii)

Infrastructure assets are generally characterised by high development costs


(high barriers to entry) and long lives. They are generally managed and
financed on a long-term basis. Other features include Large unit size, Illiquid
and Inflation hedging properties.
Infrastructure assets display a number of characteristics that distinguish them
from more traditional equity or debt investments. The assets themselves tend
to be single purpose in nature, such as gas pipelines, toll roads or hospitals.
The private investors participation in the asset is often for a finite period.
This is generally a function of the agreement the investor has made with the
government authority, or a function of the natural useful life of the asset. In
either case, infrastructure assets are characterised by their long lives. In fact
the capital invested in these projects is often referred to as patient capital, in
that the initial development involves high upfront capital costs with payback
occurring over the assets generally lengthy life.
One of the key characteristics of infrastructure assets, and what can make
them particularly attractive as investments, is that they tend to be, or exhibit
the characteristics of, natural monopolies. Under a natural monopoly,
economies of scale are such that the unit cost of a product will only be
minimised if a single firm produces the entire industry output. This
environment has the potential to weaken market forces, so there may be a need
for tighter regulation, particularly when there are few, if any, alternative
suppliers of the infrastructure. In this case, firms operating in a natural
monopoly, protected from new competitors by the high barriers to entry, may
be able to earn abnormal profits by charging higher prices.

(iii)

Infrastructure is generally specific events (i.e. building a new school), private


equity is normal buying or supplying capital for private companies (i.e. buying
a pizza restaurant chain).
Private equity tend to have shorter investment durations than infrastructure
projects but this is not always the case and some private equity deals have a
long lead time.
Private equity is only privately financed, whereas infrastructure projects can
have an element of public finance. Infrastructure assets and companies can be
listed.
Page 5

Subject ST5 (Finance and Investment Specialist Technical A) April 201, Examiners Report

Infrastructure tends to have bond like investment returns, although some


infrastructure projects have equity like returns. Private Equity tends to be more
equity like returns associated with higher levels of risk.

(i)

Daily volatilities are: Asset X


Asset Y

3000
1800

)
)

portfolio = (30002 + 18002 + 2 0.4 3000 1800)


= (16,560,000)
= 4069.398
The 10-day 98% VaR is then
10 2.0537 4069.398
(ii)

= 26428.17

The 10-day 98% VaRs for the individual assets are:


Asset X
10 2.0537 3000

= 19483.11

10 2.0537 1800 []

= 11689.97

Asset Y

Total:

31172.98

Thus the benefit of diversification is 31172.98 26428.17 = 4744.81


(Marks were awarded for suggesting suitable approaches, even if the results
were incorrectly evaluated).
(iii)

There are significant costs associated with the process of diversification


(dealing, administration, research and the loss of economies of scale).
The stability of the parameters and may not survive periods of market
stress. In extreme conditions all correlations tend to 1.
Diversification only guards against specific risk. Systematic risk is still an
issue.
There may not be suitable diversifying assets available at an acceptable cost.
There may be statutory restrictions or mandate restrictions on the assets
available.
The paramount need to match liabilities may restrict the assets available.

Page 6

Subject ST5 (Finance and Investment Specialist Technical A) April 201, Examiners Report

The need to apply diversification to intermediate cash flows (e.g. dividends) is


a further practical problem.

(i)

It depends what the investor wants to use the money for: on-going cash
requirements or to fund an annuity in retirement.

If the investor wants it for retirement funds he only has a short time to
retirement so is probably concerned with capital preservation on his assets,
so would probably favour Strong Growth as the asset class has lower
volatility.

However, the investor is risk seeking which means he might prefer the
High Returns Company as equities have higher return.

The assets are only a small portion of the investors total assets. It depends
what the investors other assets are invested in. If the rest of the assets are
in higher risk asset classes then he might prefer Strong Growth to dampen
down volatility at the total portfolio level. If the rest of assets are in low
returns then he might want a portion of assets in higher return asset classes
so would be High Return manager.

Overall the actual manager chosen will depend on the underlying


motivations and other assets held.

Other issues to consider include currency risk, the tax position of the
investor and the level of management charges levied.

Market Risk
Credit Risk
Operational Risk
Liquidity Risk
Relative performance risk

Market risk Market risk reflects the risk of changes in the value of
portfolio due to market movements. Strong Growth is invested in
Government bonds which tend to be low volatility. FBNX Investors are
invested in futures and CDS which tend to exhibit more volatility and the
market risk is likely to be higher with FBNX Investors.

Credit risk Risk that a counterparty will be unable to fulfil their


obligations. Government bonds tend to be issued directly through the
government (or government agency) or via market makers of large
institutions. Credit risk tends to be relatively low. FBNX Investors are
invested in futures which are exchange traded instruments and carry lower
risk than OTC contracts. CDS are OTC instruments and can be issued by a

(ii)

(iii)

Page 7

Subject ST5 (Finance and Investment Specialist Technical A) April 201, Examiners Report

wide variety of counterparties with different levels of credit quality.


Overall, would expect FBNX to have higher credit risk.

(i)

Operational risk fraud or mismanagement within the investment


manager. Operational risk tends to be higher where there is not appropriate
segregation of duties, enough staff or lack of technology which means
more human intervention (and potential errors). In a large investment
manager that has been established for a long time most operational issues
should be addressed. FBNX only has 5 employees which means that
employees will complete several different roles (poor segregation of
duties). Also, likely that not have the IT budget to have the required
infrastructure. FBNX is likely to have higher operational risk than Strong
Growth.

Liquidity risk is the risk of not having cash needs due to liquidity of
portfolio. Government bonds tend to be highly liquid even in times of
market distress when there is often a flight to quality, so Strong Return
should not suffer too many liquidity issues. Depending on the futures
FBNX are invested in, they could be highly liquid or not. CDS tend to
have good liquidity in normal market conditions but liquidity can dry up in
terms of market distress (as seen following Lehmans collapse). FBNX is
likely to suffer higher liquidity risk than Strong Growth.

Relative performance risk risk of underperforming relative to peer group.


This depends on the quality of investment staff and the investment process.
It is impossible to say how this will impact either manager. Past
performance can be used but is not a good guide to the future.

It is a vertical takeover as the restaurant is part of the overall supply chain the
processing company is involved in. It is an upward vertical acquisition.

(ii)

Page 8

Diversify returns within the company

Has money to invest and restaurant offers good rate of return relative to
other investments

The restaurant chain is being offered at a good price which makes it


attractive

Utilisation of unused tax benefits

Protection against threat of takeover


Enhance EPS potentially higher margin business

Low financing costs


Improved co-ordination and administration of the supply chain
Access to complementary resources

Subject ST5 (Finance and Investment Specialist Technical A) April 201, Examiners Report

(iii)

Response to similar actions by competitors

Recession
Supplier Likely that some sources of business, e.g. restaurants will reduce
orders as people reduce discretionary spend. Its other sources of business
(general public) is likely to be unaffected as people still need to eat and pork
and beef are relative affordable meats. If they supply meats to other parties
such as hotels these are likely to be impacted by the recession and supplies
will be reduced. Overall, the supplier is likely to see business reduce.
Restaurants During recession people reduce discretionary spend and
restaurants are affected. There is likely to be reduced customers reducing the
profits. In recession consumers look to restaurants to have special offers
putting further pressure on profits.
Livestock costs
Supplier It is uncertain what the impact is likely to be on the level of supply.
As a supplier they are passing on the product after processing and as long as
the purchasers can pass on the cost then profits are likely to be maintained. If
the rise changes consumer behaviour and they substitute another product for
pork and beef then demand is likely to decrease which will impact costs.
Restaurants The cost will impact the prices it charges its customers. When
prices rise rapidly over a short period of time, the restaurant is unlikely to pass
on the total price rise immediately as people take time to adjust to the new
cost. Over time as people accept the new price then price rises can be passed
on. Therefore, profits will be affected in short term but unlikely to have an
impact over a longer period of time.

(iv)

Pull out of deal


Look to reduce their offer to reflect market uncertainty
Do nothing
Delay the deal until market conditions become more certain
Look to have the final price contingent on certain targets being hit
Seek an alternative investment strategy

7
Calculations

Rb

Portfolio returns

Manager A
Manager B
Manager C
Manager D

14.41%
9.47%
15.28%
11.94%

20.3%
11.4%
21.1%
18.1%
11.4%

(About half the available marks were awarded for the approach adopted, with the balance for
the correct evaluations.)

Page 9

Subject ST5 (Finance and Investment Specialist Technical A) April 201, Examiners Report

(i)
Manager A
Manager B
Manager C
Manager D

Outperformance

Rank

8.8%
0.0%
9.6%
6.6%

2
4
1
3

(ii)
Risk Adjusted
Manager A
Manager B
Manager C
Manager D
(iii)

5.9%
2.0%
5.8%
6.1%

2
4
3
1

The investor is risk adverse so would prefer a lower risk manager. The
manager with the lowest risk is manager B. However, the investor is looking
to add returns from active manager, and manager B looks like it has very
index like returns so might prefer one of the other managers, such as manager
A. Other issues to consider would include the tax position of the investor,
whether the figures quoted are net of management charges and the need to
consider more than one year's results.
(Alternate conclusions reached were given equivalent credit if adequately
argued.)

(iv)

Standard deviation is appropriate where the whole of investors wealth is


being considered. If only a subset of assets then beta is the most appropriate
method.

(i)

Ownership equity dilutes ownership interest, debt has no impact on


ownership

Cashflows equity does not require cashflows but can pay optional
dividends. Debt requires regular payments for coupon

Voting rights equity often carries voting rights, debt does not

Priority of payout in default Debt has higher priority than equity in the
event of liquidation of a company
Impact on financial ratios Debt and equity have different impact on
financial ratios
Additional security or restrictive covenants may be required for debt
financing.
Tax treatment of the returns on the capital raised

Page 10

Subject ST5 (Finance and Investment Specialist Technical A) April 201, Examiners Report

(ii)

Debt securities are usually issued with a specified redemption date.


Equities are usually irredeemable.

(a)

Corporate Bonds longer term debt, issued with various coupon


rates and maturities

Commercial paper Short term issued directly by company

Term Loans A loan from a bank for a specific amount that has a
specified repayment schedule and a floating interest rate.

Evergreen Credit Can borrow up to a specified limit with no


fixed maturity

Revolving Credit Like evergreen but with fixed maturity,


normally up to three years

Bridging loans advances to be repaid from specified income

International bank loans

Commitment
Maturity
Rate of interest
Security

(b)

(iii)

Commitment This is where the borrower pays a fee to have advanced


funds where required. It depends on the cashflow requirements of the two
organisations so impossible to say how these are likely to differ.

Maturity It is likely that a large international chain will be given longer


terms than Best Supermarket as the lenders are likely to have greater
confidence the established supermarket can pay back loans over a period
of time.

Rate of interest lenders charge higher rates of interest to higher risk


borrowers. Therefore, Best Supermarket is likely to pay a higher rate of
interest as it will be seen as higher risk.

Security Again as Best Supermarket will not be seen as good a credit risk
as the established supermarket it is likely to be asked for greater security in
return for the loan.

(Appropriate credit was given for discussion of alternative classification of the


features in part (ii) (b).)

Page 11

Subject ST5 (Finance and Investment Specialist Technical A) April 201, Examiners Report

(iv)

(v)

Short term cashflow issues and therefore loan will assist paying short term
debts.

Gross profits might not have been retained and therefore has no additional
capital to expand. This might be due to the purchase of fixed assets or the
level of dividends paid.

Potentially high taxes meant net profits were low.

The expansion plans cost more than the retained profits.

Predicting lower gross profits in their core market due to recession which
will reduce self finance.

Best Supermarket sells essential goods and demand is relative inelastic. It is a


non-cyclical (defensive) business. On the other hand luxury handbag market
has elastic demand and in time of recession, people have less money,
discretionary spend is reduced and therefore, a reduced number of handbags
are being sold. It is a cyclical business.
As the country entered recession the luxury handbag share price would have
reduced in value to a greater extent than Best Supermarket. Following the
initial falls and during the recession the two share prices are likely to have
performed similarly. As the country starts to leave recession and confidence
return the luxury handbag share is likely to outperform Best Supermarket.
However, the actual performance of the handbag store will depend on the
demand for their product relative to other handbag companies. Best
Supermarket will probably lag the market as the country leaves the recession
which is typical for a defensive stock.

END OF EXAMINERS REPORT

Page 12

INSTITUTE AND FACULTY OF ACTUARIES

EXAMINATION
28 September 2011 (pm)
Subject ST5 Finance and Investment
Specialist Technical A
Time allowed: Three hours
INSTRUCTIONS TO THE CANDIDATE
1.

Enter all the candidate and examination details as requested on the front of your answer
booklet.

2.

You have 15 minutes before the start of the examination in which to read the
questions. You are strongly encouraged to use this time for reading only, but notes
may be made. You then have three hours to complete the paper.

3.

You must not start writing your answers in the booklet until instructed to do so by the
supervisor.

4.

Mark allocations are shown in brackets.

5.

Attempt all nine questions, beginning your answer to each question on a separate
sheet.

6.

Candidates should show calculations where this is appropriate.

AT THE END OF THE EXAMINATION


Hand in BOTH your answer booklet, with any additional sheets firmly attached, and this
question paper.
In addition to this paper you should have available the 2002 edition of the Formulae
and Tables and your own electronic calculator from the approved list.

ST5 S2011

Institute and Faculty of Actuaries

(i)

Outline the principal characteristics of credit default swaps.

(ii)

Outline the characteristics of structured products.

(i)

Describe the process used for pricing forward currency deals.

[3]
[8]
[Total 11]

[2]

The two year zero coupon interest rates in Countries A and B are currently 2% and
3% respectively. The spot exchange rate between the currencies is 1.15 units of
Country As currency for one unit of Country Bs currency.

(ii)

Calculate the two-year forward exchange rate.

(iii)

State how forward currency rates are usually presented.

(i)

Describe the differences between Exchange Traded Funds and Index Funds.
[3]

(ii)

Outline why an Exchange Traded Fund will not exactly replicate an index
benchmark.
[3]
[Total 6]

(i)

State the assumptions about investor behaviour that underpin mean-variance


portfolio theory.
[2]

(ii)

Determine the minimum variance portfolio made up of two assets A and B,


given the following asset characteristics:

ST5 S20112

[1]
[Total 5]

Asset

Expected return

Standard deviation of return

A
B

5%
9%

5%
8%

The correlation between the two assets (AB) is +0.3.


(iii)

[2]

[3]

Determine the expected return and the standard deviation of return of the
minimum variance portfolio.
[2]
[Total 7]

A UK pension fund has assets in excess of 4bn, managed across multiple asset
classes using a number of investment managers. Following the disposal by the
sponsoring company of one of its subsidiaries, the pension fund is required to pay a
bulk transfer of approximately 200m to another pension fund in six weeks time.
The amount to be paid will be determined as at the date of sale and then adjusted to
the date of payment by the total return for the FTSE All Share Index. Under the terms
of the sale agreement, the pension fund is obliged to settle the payment by transferring
stock representing a reasonable cross-section of the fund.
(i)

Explain the investment risk for the pension fund which is introduced by the
liability for this payment, illustrating your answer with a simple example. [3]

(ii)

(a)
(b)

Explain how derivatives may be used to mitigate this risk.


List the problems that may occur.
[5]
[Total 8]

(i)

Outline the principal characteristics of fundamental share analysis.

[2]

A private equity firm that specialises in turning around failing firms has recently
raised $500m. One potential investment is a bed manufacturer (HotSleep) whose
main market is supplying independent hotel chains in its local country. HotSleep is a
family owned partnership which has been running for over 100 years. The company
has not made a profit for the last three years.
(ii)

List the financial and qualitative factors the private equity firm will have
considered to establish the valuation of HotSleep.

[5]

(iii)

Discuss what changes the private equity firm could make to improve the
financial position of HotSleep.
[5]
[Total 12]

(i)

Discuss the advantages and disadvantages of the four main methods used by
passive fund managers to match the investment performance of a benchmark.
[7]

(ii)

Suggest reasons why investors might prefer a passive rather than an active
approach to bond fund management.
[6]
[Total 13]

ST5 S20113

PLEASE TURN OVER

A portfolio manager who worked at Only Investment Management had a successful


track record over a number of years. A highly successful rival, QTRM Investment
Managers, persuaded the portfolio manager to work for them instead two years ago.
Since moving employment, the portfolio manager has underperformed the benchmark
index by 3% each year.
(i)

Suggest reasons for the portfolio manager underperforming in the last two
years.
[4]

QTRM are interested in understanding more about what drives performance in their
portfolios and hire a specialist firm to look at each portfolio managers track record.
The specialist firm finds the following:
(1)

On average, portfolio managers hold onto underperforming stocks for six


months longer than they should.

(2)

Portfolio managers often ignore investment analysts reports when


choosing which stocks to buy and sell.

(3)

The underlying stock analysis is often changed by portfolio managers to


justify holding onto stocks.

(4)

Some portfolio managers sell stocks as soon as any bad news on the
stock is published in the market.

(5)

When given a choice of several stock recommendations, portfolio


managers tend to choose the first recommendation on the list.

(ii)

Explain the type of behaviour exhibited by the portfolio managers for each of
the five points above.
[7]

(iii)

Suggest actions that could be taken to stop some of the behaviours identified
in (ii).
[5]
[Total 16]

ST5 S20114

You are the investment advisor to the trustees of a pension scheme. The trustees
terminated the mandate of one of the schemes equity investment managers (Super
Return Inc) and replaced them with a new equity investment manager (Thinking
Portfolio Managers) over the course of the year. At the end of the year the trustees
review the schemes performance and have questioned the actual scheme return
relative to the benchmark. The trustees have asked you to produce a report on the
performance.
Period 1
Domestic Equities value
Domestic Equities return
Benchmark return
Overseas Equities value
Overseas Equities return
Benchmark return
Small Cap Equities
Small Cap Equities return
Benchmark return
Cash
Cash return
Benchmark return

3,500,000
9.0%
11.0%
4,000,000
4.0%
5.0%
2,000,000
3.0%
3.0%
500,000
1.0%
1.0%

Period 2

8.0%
7.0%
7.0%
7.0%
6.0%
6.0%
1.5%
1.0%

Transition
Period
3,000,000
7.0%
7.0%
3,000,000
6.0%
5.0%
1,000,000
6.0%
8.0%
3,000,000
2.0%
2.0%

Period 4

Period 5

6.0%
6.0%

8.0%
5.0%

5.0%
5.0%

7.0%
8.0%

4.0%
4.0%

5.0%
3.0%

1.0%
1.0%

3.0%
2.0%

The values in the table above represent the values at the start of the period. The
trustees adjusted the allocation to equities and cash to $10 million at the start of the
transition as shown in the table above.
Both managers were measured against the same benchmark:

Domestic Equities
Overseas Equities
Small Cap Equities
Cash

50%
30%
15%
5%

Assets are rebalanced at the discretion of the investment manager. Following the
transition, Thinking Portfolio Managers asset allocation was in line with the
benchmark. No other rebalancing took place apart from during the transition period.
(i)

Calculate the benchmark return and the portfolio return for the following:
(a)
(b)
(c)

(ii)

ST5 S20115

Super Return Inc (Periods 1 and 2 separately)


Assets during transition (Period 3)
Thinking Portfolio Managers (Periods 4 and 5 separately)

[7]

Calculate:
(a)

The cash value taken out of the scheme at the start of the transition
period.

(b)

Total scheme portfolio return and benchmark return over the entire
period.
[3]

PLEASE TURN OVER

(iii)

Calculate for each period the returns attribution from:

Stock performance for each asset class


Stock performance at the total fund level
Sector performance at the total fund level

showing whether the effect had a positive or negative impact on the


performance of the portfolio relative to the benchmark.
(iv)

Discuss how transition management could have been used during the portfolio
switch to improve performance.
[4]
[Total 22]

END OF PAPER

ST5 S20116

[8]

INSTITUTE AND FACULTY OF ACTUARIES

EXAMINERS REPORT
September 2011 examinations

Subject ST5 Finance and Investment


Specialist Technical A
Purpose of Examiners Reports
The Examiners Report is written by the Principal Examiner with the aim of helping
candidates, both those who are sitting the examination for the first time and who are using
past papers as a revision aid, and also those who have previously failed the subject. The
Examiners are charged by Council with examining the published syllabus. Although
Examiners have access to the Core Reading, which is designed to interpret the syllabus, the
Examiners are not required to examine the content of Core Reading. Notwithstanding that,
the questions set, and the following comments, will generally be based on Core Reading.
For numerical questions the Examiners preferred approach to the solution is reproduced in
this report. Other valid approaches are always given appropriate credit; where there is a
commonly used alternative approach, this is also noted in the report. For essay-style
questions, and particularly the open-ended questions in the later subjects, this report contains
all the points for which the Examiners awarded marks. This is much more than a model
solution it would be impossible to write down all the points in the report in the time allowed
for the question.
T J Birse
Chairman of the Board of Examiners
December 2011

Institute and Faculty of Actuaries

Subject ST5 (Finance and Investment Specialist Technical A) Examiners Report, September 2011

General comments
Candidates are reminded of a bias in the paper towards recognising higher level skills and
practical application this is intentional and will continue. Likewise the examination system
does properly allow for prior subject knowledge to be assumed. Investment is a necessarily
practical subject and, at this level, the examiners expect candidates to demonstrate a breadth
and depth of competency as would be expected from a senior student in a frequently evolving
discipline. Hence simple regurgitation of bookwork will never be sufficient to ensure a Pass
grade and this was evident from the dispersion of candidates responses in the more
differentiating questions.
In order to succeed, candidates must ensure they familiarise themselves with the prevailing
investment issues and the general market background facing institutional investors in the 12
18 months preceding a diet, more so the solutions (and sources of) being debated by the
various stakeholders. Hence questions regarding banking and derivative approaches, as well
as active and passive asset management and insurance solutions, to asset and liability risk
management (including model risk) or modern financial theory and commercial applications
should be considered likely scope for examination. Against a background of the credit crisis,
new asset classes and ways of structuring investments will themselves generate new types of
risk (such as operations, liquidity, credit and counterparty), so the need for new ways of
regulation, monitoring and management. Finally the examiners encourage candidates to
recognise there are different types of investor beyond purely pension funds and different
taxation, time line and cost considerations will apply it would seem that candidates have
taken this on board.
Whilst the examiners will tolerate bullet point style responses, some candidates handwriting
was too poor to assess and they will have lost marks. Likewise text speak abbreviations
will not be accepted.
Specific comments on September 2011 paper
This paper had a similar pass mark to the April diet which resulted in a lower pass rate, albeit
comparable with previous years. Candidates typically answered Questions 4 and 6 much
better than the others (albeit still foregoing 3040% or more of marks available), with
Questions 2 and 9 attracting the worst responses, considerably so, with average scores of
around a third of the available marks. These latter two questions were two of the calculation
biased ones and, notwithstanding the performances in questions 4, it is disappointing to see a
lack of skill demonstrated in this area given recent improvements in data handling in previous
exams.
Question 1 dealt with some of the core products cited as a key factor in the Credit Crisis
when risk management failed. Actuaries could reasonably feel they could offer relevant
skills and knowledge in this area, so it is important candidates demonstrate understanding.
Questions 3, 5 and 7 focussed on the practical aspects of investment as distinct from theory.
Question 8 took this further in looking at behavioural aspects markets and funds are
susceptible to such biases and it is important for candidates to be able to recognise and
identify such distortions; although one of the better answered questions, a lot of marks were
missed and candidates should expect further examination in this area. Many questions
represented opportunities to demonstrate higher level skills in terms of nonstandard/practical application of theory to current or unusual issues in investment hence

Page 2

Subject ST5 (Finance and Investment Specialist Technical A) Examiners Report, September 2011

candidates who wish to progress to SA6 will need to improve their understanding of and
approach to such questions.
Most candidates seemed to identify and understand the key issues being examined and so
appreciated the general content of solutions that the examiners were looking for however
those that were unsuccessful will find their solutions lacked sufficient (and often the most
basic) detail or application of knowledge and scored lower accordingly. Many candidates
still deviate from the topic and include irrelevant material or over emphasise minor points
although candidates will not be explicitly penalised for this, it gives an impression of a lack
of understanding and, more importantly, wastes limited time. Time and priority management
are key skills actuaries need to have. Where candidates made relevant points in other parts of
their solutions, the examiners have used their discretion as to whether to recognise these
answers or not. Likewise the examiners share and agree alternative possible solutions to
questions alongside the approach outlined below.

Page 3

Subject ST5 (Finance and Investment Specialist Technical A) Examiners Report, September 2011

(i)

A credit default swap is a contract that provides a payment if a particular event


occurs. The party that buys the protection pays a fee to the party that sells the
protection. If the credit event occurs within the term of the contract a payment
is made from the seller to the buyer. If the credit event does not occur within
the term of the contract, the buyer receives no monetary payment but has
benefited from the protection during the tenure of the contract.
There are two ways to settle a claim under a credit default swap:

(ii)

A pure cash payment, representing the fall in the market price of the
defaulted security. However, the market value may be difficult to
determine.

The exchange of both cash and a security (physical settlement). The


protection seller pays the buyer the full notional amount and receives, in
return, the defaulted security.

A structured product is a pre-packaged investment strategy in the form of a


single investment.
A typical structured product will consist of two components:

A Note essentially a zero-coupon debt security that provides capital


protection, i.e. guarantees a return of all or part of the initial investment at
maturity

A derivative component that provides exposure to one or several


underlying assets such as equities, commodities, FX or interest rates.

Returns from the derivative can be paid out in the form of coupons during the
lifetime of the product, or added to proceeds at maturity.
In some, more complex, structured products, the amount invest in the zerocoupon debt security can vary dynamically over time depending on a predetermined view. And in others, the capital protection may itself be dependent
on the performance of the underlying assets.
Structured products are also typically provided in a packaged format that
provides advantages to investors over investing directly in the underlying
derivatives for example:

Page 4

Practical investors may be unable to invest themselves in the underlying


derivatives. The structured product also provides a pre-packaged
investment strategy that does not require active intervention by the
investor.

Subject ST5 (Finance and Investment Specialist Technical A) Examiners Report, September 2011

Legal the format may be designed to satisfy legal or regulatory


requirements as to accessible investments for retail or institutional
investors.

Tax the tax treatment from the structured product may be more
favourable than direct investment.

Accounting for example, the product may be structured so as to avoid


income statement volatility from the underlying derivatives.

Structured products may lack transparency, making them hard for the investor
to assess. There may be cost advantages to the investor (compared to using
the underlying instruments. Alternatively, the structured product may cost
more.

(i)

The forward rate is the guaranteed price agreed today at which the buyer will
take delivery of the currency on a specific future date. Pricing of forward
contracts is known as Covered interest parity (CIP) and involves the spot
rate and money market interest rates in the two countries. If rd and rf are the
interest rates in the domestic and foreign markets and F and S are the forward
and spot rates, then
F = S (1 + rd) / (1 + rf) [] for a one year contract.

(ii)

Forward rate

= Spot rate (1 + rA)2 / (1 + rB)2


= 1.15 (1.02)2 / (1.03)2
= 1.1278

(or Forward rate = Spot rate exp 2(rA rB)


= 1.15 e0.02
= 1.1272 )
(iii)

In practice, outright forward rates do not usually appear on a dealers screens.


Instead, forward points are quoted where forward points = F S. Thus the
outright forward rate is the spot rate plus forward points.

(i)

Index Fund is open ended fund, exchange traded fund is closed ended.

Index fund redeem at NAV (sometimes with spread). Exchange traded


funds not always priced at NAV

Index Fund tend to replicate main indices, exchange traded funds can be
much more focussed on a sector

Page 5

Subject ST5 (Finance and Investment Specialist Technical A) Examiners Report, September 2011

(ii)

(i)

(ii)

Tracking error not holding all securities in benchmark


Management fees
Expenses
Handling of dividends
Treatment of tax taken at source

Mean-variance portfolio theory relies on some strong and limiting


assumptions about investor behaviour. It is assumed that:

(i) investors select their portfolios on the basis of the expected return and
the variance of that return over a single time horizon.

(ii) investors are never satiated. At given level of risk, they will always
prefer a portfolio with a higher return to one with a lower return.

(iii) investors dislike risk. For a given level of return they will always
prefer a portfolio with lower variance to one with higher variance.

The covariance for the two assets CAB is given by ABAB = .0012
Minimum variance occurs when the proportion of asset A(xA)
=

VB C AB
VA 2C AB + VB

0.0064 0.0012
0.0025 0.0024 + 0.0064

0.0052
= 0.8
0.0065

xB = (1 xA) = 0.2

(iii)

E = xAEA + xBEB = 0.8 5% + 0.2 9% = 5.8%


V = xA2VA + xB2VB + 2xAxB CAB

= .00224
So standard deviation = 4.7%

Page 6

Subject ST5 (Finance and Investment Specialist Technical A) Examiners Report, September 2011

(i)

The amount of the bulk transfer is linked solely to the level of the market for
UK equities.
The fund is invested across a range of asset classes in accordance with a set
benchmark distribution.
Unless the proportion of the benchmark allocated to UK equities explicitly
allows for this bulk transfer, part of the bulk transfer liability is effectively
mismatched by asset class.
If, for example, the portfolio is 50% invested in UK equities and 50% in
overseas equities, bonds, properties etc., then circa 100m of the bulk transfer
is matched by asset class. However, there is still a circa 100m liability linked
to UK equities which will be settled by transferring securities to the requisite
value from the overseas equities, bonds, properties etc. classes i.e. this 100m
is mismatched by asset class.

(ii)

You may be able to reduce your exposure to these other asset classes and
increase your exposure to UK equities by using financial futures.
In total, you need to change your exposure for approx. 100m i.e. sell 100m
worth of futures on these other asset classes and buy 100m worth of UK
equity futures.
A variety of futures would be sold related to the markets in the other asset
classes and in proportion to the distribution of the assets amongst these
markets.
At the time of payment the futures position would be unwound.
The principal problem is that there may not be appropriate derivative contracts
for some of these other asset classes e.g. property, venture capital etc. Other
main problems are:

(i)

The basis risk associated with futures contacts


The need to take account of foreign exchange hedging in relation to
overseas asset classes
The trustees may be restricted in their use of derivatives
Margin payments may need to be funded
The investment managers may lack the requisite expertise,
which may result in additional costs

Fundamental analysts use a variety of techniques to try to determine whether a


share is over- or under-valued by the market. Most methods involve an
attempt to obtain a better estimate of future earnings or dividends, either by
the use of a superior model or by the use of information which hasnt been
taken into account by the market. The process can be considered as consisting
of two stages. The first is the construction of a model of the company which

Page 7

Subject ST5 (Finance and Investment Specialist Technical A) Examiners Report, September 2011

allows future cash flows and earnings to be estimated. The second involves
the use of the output from the first stage to determine whether the companys
securities are over- or under-valued by the market. In practice, a wide range
of techniques is used and the degree of sophistication employed varies greatly.
(ii)

The analyst will consider the following:

In order to consider the above they will undertake the following analysis:

(iii)

Management ability
Quality of products
Prospects for market growth
Competition
Input costs
Retained profits
History

Financial accounts and accounting ratios


Dividend and earnings cover
Profit variability and growth
Level of borrowing
Level of liquidity
Growth in asset value
Expenditure
Performance relative to other similar companies

Reduce Costs

Reduce expenses this could be done by:


Reducing size of work force
Reduce salaries of firm or cut benefits
Reduce costs of manufacturing by sourcing cheaper materials
Improving efficiency of current process or outsourcing
activities/processes to cheaper location
Reduce/eliminate non-essential expenses
If firm has debt, maybe able to restructure debt to lower borrowing
costs
Move headquarters to more tax beneficial location

Page 8

Subject ST5 (Finance and Investment Specialist Technical A) Examiners Report, September 2011

Increase Revenues

(i)

Increase prices of beds


Expand product range available to market
Expand beyond hotel market and offer to public as well
Expand into overseas markets
Stop producing any loss making products

A discussion of the four main methods is provided below.


Full replication involves holding every stock in the chosen index in the index
proportions. Thus the index is fully matched.

This can be an expensive approach to matching the investment performance of


the index if there were a lot of stocks in the index.
Imagine the dealing costs involved in full replication of the FTSE All Share
Index as stocks move in and out of the index and dividend income is
reinvested in the correct proportions across the index.
The need to be a forced buyer or seller as index constituents change may
result in trades being made at unattractive prices.
Stratified sampling entails purchasing a sample of the stocks in the index so
that the proportions of the fund in the specified industry categories matches
that of the index. Some mismatching is inevitable.

Fewer stocks are required compared with full replication and this should lower
transactions costs.
The method requires a significant statistical analysis to find the sample that
best matches the performance of the index.
There will be less 'forced' buying and selling involved.
Optimisation entails constructing a portfolio that matches the index in certain
specified fundamental factors (e.g. price earnings ratio, capitalisation, and
beta) that are known to affect performance.

Choosing the fundamental factors in the first place is a problem that requires
high level analytical skills choosing appropriate stocks thereafter is yet
another problem. This approach requires ongoing analysis and computing
power to carry out the optimisation.
Again the method has the advantage of requiring less stocks than full
replication and hence lower transaction costs. However some mismatching is
inevitable.

Page 9

Subject ST5 (Finance and Investment Specialist Technical A) Examiners Report, September 2011

Synthetic funds are constructed using derivatives on the underlying assets


rather than holding the assets themselves.

For example, a passive fund manager could hold cash (perhaps in the form of
T-bills) and futures on the index. Provided suitable derivatives exist/are used,
the index is fully matched, at least in capital terms.
If futures are underpriced relative to fair value the manager may outperform an
index fund that holds stocks directly and vice versa.
The necessity to roll over the index futures every few months can give rise to
basis risk [] which may cause the fund to outperform/under perform a
portfolio holding the stocks directly.
The costs associated with constructing the synthetic fund may be significant.
(ii)

Matching an index may be problematical due to the lack of fungibility i.e. you
can only buy what bonds others want to sell. Hence matching an index may be
quite easy in gilts but very hard in corporate space. Moreover a single issuer
can have multiple stocks available with different terms and features, some
rated some not, so you could get a yield pick up for the same underlying credit
risk simply because, say, there is less demand for unrated issues.
Where the index being matched is a very broad market index, indexation
exposes the investor to market risk whereas active management which aims to
beat the performance of the index exposes the investor to both market and
stock specific risk.
Index portfolio managers tend to deliver a narrower range of returns compared
to active managers targeting the same benchmark. This has led some
observers to argue that the average active manager provides a poor risk-return
trade-off relative to the average index manager targeting the same benchmark
index.
Index funds tend to beat the average active manager which aims to beat the
same benchmark index.
Markets are relatively efficient information is disseminated quickly and
simultaneously to all major market participants who take the correct action,
which is quickly reflected in stock prices and any outperformance by
generated by an active manager may not justify the extra dealing and fund
management costs.
The problem with active management is that while some active managers do
indeed produce returns well in excess of the benchmark the question is can
they be identified in advance and can they consistently outperform the index.
Where a bond fund is subject to taxation, active fund management means an
earlier incidence of capital gains tax. Index funds have a very low level of

Page 10

Subject ST5 (Finance and Investment Specialist Technical A) Examiners Report, September 2011

turnover, so until all or a part of the fund is disposed of it (and ultimately the
investor) pays minimal capital gains tax.
The need to match liabilities may make the use of passive management
preferable.

(i)

Portfolio manager picked underperforming securities (i.e was unlucky)

Portfolio manager might have been given a different brief (asset class) to
manage than they are use to managing before

Portfolio restrictions might be too restrictive limiting ability to add value

Portfolio managers style might be out of favour (i.e. growth manager and
market conditions have been value markets)

Quality of analyst recommendations have been poor


The skill of the supporting team or systems may be less developed

1 regret aversion retaining current arrangements minimise possibility


of regret. Also, anchoring and adjustment to keep convincing themselves
the new price is correct

2 Overconfidence in own abilities and believes own judgement is best

3 Overconfidence confirmation bias, looks to change views to suit their


own views

4 Dislike of negative events - influence of the 'valence' of the outcome

5 Primary effect like to pick first option on list

Have strong sell rules (stop losses) to stop portfolio managers holding onto
underperforming securities too long

Have all sell decisions reviewed by second person to stop securities being
sold too quickly

Have a model portfolio which is decided by a group of portfolio managers


rather than stock selection influenced by a single manager

Remunerate portfolio managers on their stock selection could increase


focus on performance

(ii)

(iii)

Page 11

Subject ST5 (Finance and Investment Specialist Technical A) Examiners Report, September 2011

Introduce rules that all stock selection must have an analyst


recommendation

Change way stock recommendations are presented to stop stock at the top
of list being selected

9
Start value
Domestic Equities
Overseas Equities
Small Cap Equities
Cash

10000000
3500000
4000000
2000000
500000

Benchmark

1
50%
30%
15%
5%

Period 1

Period 2

Period 4

Period 5

Domestic Equities value

3500000

3815000

Transition
Period
3000000

5255000

5570300

Domestic Equities return

9.0%

8.0%

7.0%

6.0%

8.0%

11.0%

7.0%

7.0%

6.0%

5.0%

Overseas Equities value

4000000

4160000

3000000

3153000

3310650

Overseas Equities return

4.0%

7.0%

6.0%

5.0%

7.0%

Benchmark return

5.0%

7.0%

5.0%

5.0%

8.0%

2000000

2060000

1000000

1576500

1639560

Small Cap Equities return

3.0%

6.0%

6.0%

4.0%

5.0%

Benchmark return

3.0%

6.0%

8.0%

4.0%

3.0%

500000

505000

3000000

525500

530755

Cash return

1.0%

1.5%

2.0%

1.0%

3.0%

Benchmark return

1.0%

1.0%

2.0%

1.0%

2.0%

10000000.0

10540000.0
6.6%

10510000.0
10510000.0
5.1%

11051265.0

7.5%

10000000.0
10540000.0
6.3%

Domestic
4120200

Overseas
4451200

Small Cap
2183600

Cash
512575

Total return
12.7%
14.5%

Benchmark return

Small Cap Equities

Cash

Total portfolio
Benchmark

5.5%

Answers
(i)
(a)

Super Return
Benchmark Return

(b)

Transition Period
Benchmark Return

3210000

3180000

1060000

3060000

5.1%
6.3%

(c)

Think Return
Benchmark Return

6015924

3542396

1721538

546678

12.5%
10.9%

Page 12

Subject ST5 (Finance and Investment Specialist Technical A) Examiners Report, September 2011

(ii)
(a)

Cash taken out

1267575

(b)

Total scheme
Total benchmark

33.3%
35.0%

(iii)

Stock attribution

Period 1

Period 2

Period 4

Period 5

8.00%
7.00%

Transition
Period
7.00%
7.00%

Domestic equity return


Benchmark

9.00%
11.00%

6.00%
6.00%

8.00%
5.00%

Stock attribution

2.00%

1.00%

0.00%

0.00%

3.00%

4.00%
5.00%

7.00%
7.00%

6.00%
5.00%

5.00%
5.00%

7.00%
8.00%

Stock attribution

1.00%

0.00%

1.00%

0.00%

1.00%

Small Cap Equity


Benchmark

3.00%
3.00%

6.00%
6.00%

6.00%
8.00%

4.00%
4.00%

5.00%
3.00%

Stock attribution

0.00%

0.00%

2.00%

0.00%

2.00%

Cash Benchmark
Benchmark

1.00%
1.00%

1.50%
1.00%

2.00%
2.00%

1.00%
1.00%

3.00%
2.00%

Stock attribution

0.00%

0.50%

0.00%

0.00%

1.00%

Actual alloc, benchmark


perf
Actual / actual
Overall Stock attribution

6.50%

6.52%

5.00%

5.15%

5.46%

5.40%
1.10%

6.90%
0.39%

5.10%
0.10%

5.15%
0.00%

7.02%
1.56%

Q1
6.50%

Q2
6.52%

5.00%

Q3
5.15%

Q4
5.46%

7.50%
1.00%

6.55%
0.03%

6.30%
1.30%

5.15%
0.00%

5.45%
0.01%

6.20%

7.08%

6.30%

5.15%

7.00%

5.40%
0.80%

6.90%
0.17%

5.10%
1.20%

5.15%
0.00%

7.02%
0.02%

6.20%

7.08%

6.30%

5.15%

7.00%

7.50%
1.30%

6.55%
0.52%

6.30%
0.00%

5.15%
0.00%

5.45%
1.55%

Overseas equity return


Benchmark

Sector attribution answers


Sector attribution
Actual alloc, benchmark
perf
Benchmark / benchmark
Sector attribution

Alternative approach
Benchmark alloc, actual
perf
Actual / actual
Overall Stock attribution
Sector attribution answers
Sector attribution
Benchmark alloc, actual
perf
Benchmark / benchmark
Sector attribution

Page 13

Subject ST5 (Finance and Investment Specialist Technical A) Examiners Report, September 2011

(iv)

Transition managers have performance benchmark, so assets would have


been managed to the target asset allocation which would have improved
performance

Transition manager could have used derivatives to manage against target


benchmark whilst underlying securities are bought/sold

Transition manager has skills in trading securities which might be


beneficial compared with the investment manager trading the portfolio

END OF EXAMINERS REPORT

Page 14

INSTITUTE AND FACULTY OF ACTUARIES

EXAMINATION
25 April 2012 (pm)
Subject ST5 Finance and Investment
Specialist Technical A
Time allowed: Three hours
INSTRUCTIONS TO THE CANDIDATE
1.

Enter all the candidate and examination details as requested on the front of your answer
booklet.

2.

You have 15 minutes before the start of the examination in which to read the
questions. You are strongly encouraged to use this time for reading only, but notes
may be made. You then have three hours to complete the paper.

3.

You must not start writing your answers in the booklet until instructed to do so by the
supervisor.

4.

Mark allocations are shown in brackets.

5.

Attempt all seven questions, beginning your answer to each question on a separate
sheet.

6.

Candidates should show calculations where this is appropriate.

AT THE END OF THE EXAMINATION


Hand in BOTH your answer booklet, with any additional sheets firmly attached, and this
question paper.
In addition to this paper you should have available the 2002 edition of the Formulae
and Tables and your own electronic calculator from the approved list.

ST5 A2012

Institute and Faculty of Actuaries

(i)

Outline the characteristics of debentures.

(ii)

Explain the attraction of debentures for:


(a)
(b)

[3]

issuers
investors
[7]
[Total 10]

(i)

State the differences between forward and future interest rate contracts.

[2]

(ii)

State the formula for converting a futures rate to a forward rate, defining any
terms that you use.
[2]

A Eurodollar futures quote is for a three-month contract and is expressed with


quarterly compounding.

(iii)

Calculate the forward rate consistent with a six-year Eurodollar futures price
quote of 96, where the volatility of short-term interest rates is 1% p.a.
[3]

(iv)

State the procedure for valuing a swap using forward rate agreements.
[3]
[Total 10]

(i)

List the key industrial classification features of banks.

(ii)

Describe how domestic bank shares are expected to perform relative to


consumer non-durable goods shares during:
(a)
(b)

[2]

low economic growth


high economic growth
[4]

A retired professional is deciding whether to invest part of his portfolio in a number


of domestic bank shares. His requirement is for capital growth over a short horizon of
one to two years.
Over the past year bank shares have fallen in value by an average of 60% due to
concerns about global economic growth. The current forecast is for an economic
recovery in three to four years time.
(iii)

Explain why the proposed investment might not be suitable for the individual.
[5]

(iv)

List other ways in which this investor could gain exposure to banks without
investing directly in the underlying shares.
[2]
[Total 13]

Explain why declines in property prices may create losses in other parts of the
economy.

ST5 A20122

[14]

(i)

Outline three ways of minimising an investor's capital gains tax liability.

[3]

A wealthy investor with multi-residency status is deciding which country to


declare her personal wealth in to minimise taxes payable. Currently the
investors portfolio consists of:
(All values in 000s)
Asset

Start
Year 1

End
Year 1

End
Year 2

End
Year 3

Property Main residence


Property Investment
portfolio
Share portfolio
Income from shares
Automobile collection
Art collection

2,000
3,750

2,000
4,000

2,500
5,000

2,500
4,500

2,800
1,000
150

3,000
300
1,000
250

6,000
500
1,200
300

5,000
500
1,500
0*

* The art collection was sold at the end of Year 2.


The investor is choosing between the tax regimes in two countries as set out
below:
Tax

Country A

Country B

Income tax

50%

10%

Capital
gains tax

30%, payable each year


based on appreciation of
asset value during the
year

15%, payable only once every 3


years based on value of assets at end
of 3rd year. Assets disposed of
before 3 year period not subject to
tax

Allowances

Main residence is
exempt from tax

Depreciating assets (includes


automobiles) can offset 25% of
value from any tax payable

Capital loss

20%, claim back tax on


depreciation of asset
value during the year

No allowance made for capital


losses

(ii)

Calculate the total tax that would be payable in each of the three years and in
total, in each country, split between income and capital tax. State any
assumptions you make.
[7]

(iii)

Describe two ways the investor might be able to lower her future total tax
burden, assuming the above tax regimes remain unchanged.
[2]

(iv)

Suggest reasons why an investor might decide to declare residency in a


country where the capital gains tax rates are higher than in another country. [2]
[Total 14]

ST5 A20123

PLEASE TURN OVER

(i)

Describe absolute and relative approaches to pricing assets.

[4]

(ii)

Explain the function of beta in the Capital Asset Pricing Model (CAPM).

[3]

In a market where the CAPM holds, the following parameters are known:
Risk-free rate of interest = 5% p.a.
Expected market rate of return = 9% p.a.
Standard deviation of an efficient portfolios returns = 0.10
Standard deviation of market returns = 0.2

(iii)

Calculate the expected return on the portfolio.

(iv)

Outline reasons why the CAPM may not hold true.

(i)

State how the MSCI style indices distinguish between value and growth
stocks.

(ii)

[2]
[5]
[Total 14]

[2]

List factors which identify:


(a)
(b)

growth stocks
value stocks
[4]

(iii)

Describe other equity investment styles an investor might use.

(iv)

Assess whether the following stocks would be classified as value or growth


stocks.
Company

Products/Services

Big Bang Theory Ltd

Develops computer software for mobile /


cell phones

Power2u

Provides electricity to the northern region of a


country

Classic Wooden
Furniture Ltd

Has produced chairs and tables for


schools for the last 100 years

Superfluid

An energy drinks company which has recently


expanded into countries where energy drinks
are new to the market

In the sticks

A regional supermarket with plans to introduce


stores nationally

[3]

GiveMeSomeCredit.com A bank with a stable client base


[6]

ST5 A20124

All these stocks have been included in an equally weighted portfolio (with no
rebalancing).
You have been given the following investment performance results:
Company
Big Bang Theory Ltd
Power2u
Classic Wooden Furniture Ltd
Superfluid
In the sticks
GiveMeSomeCredit.com
Total Benchmark return
Growth Benchmark return
Value Benchmark return
(v)

Year 1

Year 2

Year 3

12%
7%
4%
21%
5%
9%
11%
14%
7%

5%
8%
7%
2%
11%
8%
5%
5%
7%

2%
8%
9%
6%
1%
9%
4%
1%
8%

Calculate the following:


(a)

Return of the total portfolio compared with the total benchmark return

(b)

Return of the growth portfolio compared with the growth benchmark


return and total benchmark return.

(c)

Return of the value portfolio compared with the value benchmark


return and total benchmark return.
[8]

(vi)

Assess whether the returns in (v) indicate that the economy was growing or
contracting during the period calculated. (You may assume that the portfolio
is representative of the entire economy.)
[2]
[Total 25]

END OF PAPER

ST5 A20125

INSTITUTE AND FACULTY OF ACTUARIES

EXAMINERS REPORT
April 2012 examinations

Subject ST5 Finance and Investment


Specialist Technical A
Purpose of Examiners Reports
The Examiners Report is written by the Principal Examiner with the aim of helping
candidates, both those who are sitting the examination for the first time and who are using
past papers as a revision aid, and also those who have previously failed the subject. The
Examiners are charged by Council with examining the published syllabus. Although
Examiners have access to the Core Reading, which is designed to interpret the syllabus, the
Examiners are not required to examine the content of Core Reading. Notwithstanding that,
the questions set, and the following comments, will generally be based on Core Reading.
For numerical questions the Examiners preferred approach to the solution is reproduced in
this report. Other valid approaches are always given appropriate credit; where there is a
commonly used alternative approach, this is also noted in the report. For essay-style
questions, and particularly the open-ended questions in the later subjects, this report contains
all the points for which the Examiners awarded marks. This is much more than a model
solution it would be impossible to write down all the points in the report in the time allowed
for the question.

T J Birse
Chairman of the Board of Examiners
July 2012

Institute and Faculty of Actuaries

Subject ST5 (Finance and Investment Specialist Technical A) Examiners Report April 2012

General comments
Most candidates seemed to identify and understand the key issues being examined and so
appreciated the general content of solutions that the examiners were looking for however
those that were unsuccessful will find their solutions lacked sufficient (and often the most
basic) detail or application of knowledge and scored lower accordingly. Whilst some
candidates are too narrow in their responses, a greater number still deviate from the topic and
include irrelevant material or over emphasise minor points although candidates will not be
explicitly penalised for this, it gives an impression of a lack of understanding and, more
importantly, wastes limited time. Time and priority management are key skills actuaries need
to have. Where candidates made relevant points in other parts of their solutions, the
examiners have used their discretion as to whether to recognise these answers or not.
Likewise the examiners share and agree alternative possible solutions to questions alongside
the approach outlined below.
Candidates are reminded of a bias in the paper towards recognising higher level skills and
practical application this is intentional and will continue. Likewise the examination system
does properly allow for prior subject knowledge to be assumed. Investment is a necessarily
practical subject and, at this level, the examiners expect candidates to demonstrate a breadth
and depth of competency as would be expected from a senior student in a frequently evolving
discipline. Hence simple regurgitation of bookwork will never be sufficient to ensure a Pass
grade and this was evident from the dispersion of candidates responses in the more
differentiating questions.
Given the greater volatility in recent years and globalisation/integration of markets and
economies, delivering an acceptable return from a long term strategy against an increasing
short term focus and political/regulatory backdrop has become increasing challenging for
investors. In order to succeed, candidates must ensure they familiarise themselves with the
prevailing investment issues and the general market background facing institutional investors
in the 1218 months preceding a diet, more so the solutions (and sources of) being debated
by the various stakeholders. Hence questions regarding banking and derivative approaches,
as well as active and passive asset management and insurance solutions, to asset and liability
risk management (including model risk) or modern financial theory and commercial
applications should be considered likely scope for examination. Against a background of the
credit crisis, new asset classes and ways of structuring investments will themselves generate
new types of risk (such as operations, liquidity, credit and counterparty), so the need for new
ways of regulation, monitoring and management. Finally the examiners encourage
candidates to recognise there are different types of investor beyond solely pension funds, and
that different taxation, time line and cost considerations will apply to each type of investor
it would seem that candidates have taken this on board.
Whilst the examiners will tolerate bullet point style responses, some candidates handwriting
was too poor to assess and they will have lost marks. "Text speak" abbreviations will not be
accepted.

Page 2

Subject ST5 (Finance and Investment Specialist Technical A) Examiners Report April 2012

Specific comments on April 2012 paper


This paper had a similar pass mark to the September diet and resulted in a higher pass rate,
which was a pleasing result. Candidates typically answered Question 7 much better than the
others (albeit still foregoing a third of marks available), with Questions 4 and 1 attracting the
worst responses, considerably so, with average scores of between a quarter and a third of the
available marks. Question 1 was predominantly bookwork knowledge so the scores are
disappointing.
Conversely, Questions 5 and 7 were two of the more calculation biased ones and it was
pleasing to see more skill demonstrated in this area.
Question 2 focussed on different derivative contracts and pricing, an increasing area of focus
for actuaries generally as liability driven investment grows in popularity. Questions 3 and 5
focussed on the practical aspects of investment as distinct from theory, highlighting that the
client could more likely be an individual than a institution requiring different considerations.
Many questions represented opportunities to demonstrate higher level skills in terms of nonstandard/practical application of theory to current or unusual issues in investment hence
candidates who wish to progress to SA6 will need to improve their understanding of and
approach to such questions.

Page 3

Subject ST5 (Finance and Investment Specialist Technical A) Examiners Report April 2012

(i)

A debenture is a loan made to a company which is secured against the assets


of the company. Debentures usually have a floating charge over the assets of
the company, so that debenture holders rank above other creditors should the
company be wound up. Alternatively, mortgage debentures have fixed
charges they are secured against specific assets set out in the legal
documentation.

(ii)

Issuers
Issuers are likely to be able to obtain funding via the debt capital markets at
lower borrowing spreads if they issue debentures than if they borrow on an
unsecured basis.
Funding may be available at longer maturities, or in larger size, than if
borrowing on an unsecured basis.
At times of market stress it is desirable to have multiple sources of funding to
maximise the likelihood of being able to source new funds, therefore issuers
often want to ensure they have a presence in the debenture markets.
Financial institutions often have significant illiquid assets (e.g. mortgages) that
are available for use as collateral cover, therefore it is efficient to use them in
this way to reduce funding costs.
Debentures are issued with a fixed redemption date and carry a fixed rate of
interest, so the issuer has a known debt servicing commitment.
Interest payments are tax deductible.
Debenture holders have no right to interfere with the running of the company.
Investors
Investors may be willing to invest in debentures to a greater extent than
unsecured bonds with some issuers due to concerns about credit risk.
Debentures are likely to have a higher credit rating than unsecured bonds, so
may fit elsewhere in a portfolio for a given issuer.
Due to the different credit characteristics, a debenture will provide some
diversification compared to an unsecured bond, as it is exposed to different
risk premia (both the issuers credit risk, and that of the collateral assets).
For a hold to maturity investor, debentures have very low risk as they are
likely to offer very high recovery rates due to the collateral pool.
Credit will be given for relevant points from part (ii) that were included in
part (i)

Page 4

Subject ST5 (Finance and Investment Specialist Technical A) Examiners Report April 2012

(i)

Forwards are over the counter instruments. Futures are exchange-traded.


Forwards are bespoke contracts. Futures are standardised.
For a forward, there is no cash flow until the maturity. For a future, there is
daily marking-to-market and settlement of margin requirements.

(ii)

To convert futures to forward interest rates, a convexity adjustment is applied:


Forward rate = Futures rate 2t1t2
where t1 is the time to maturity of the futures contract t2 is the time to maturity
of the rate underlying the futures contract and is the standard deviation of
the change in the short-term interest rate in one year.

(iii)

t1 = 6, t2 = 6.25, = 0.01 so the convexity adjustment is .001875 (or 0.1875%)


The (nominal) futures rate i(4) is 4%, so the equivalent annual rate is 4.0604%.
Thus the forward rate is 4.0604 0.1875 = 3.8729%
Using a continuous compounding then the alternative answer in 3.8655%
which will be given similar credit.

(iv)

The procedure is:


1.

Calculate forward rates for each of the floating rates that will
determine swap cash flows.

2.

Calculate swap cash flows on the assumption that the floating rates
will equal the forward rates.

3.

Set the swap value equal to the present value of these cash flows.

(i)

Capital intensive
Highly geared
Volatile profits
Labour costs important
Domestic market is most important
Highly regulated

(ii)

In low economic growth a consumer non-durable stock is less affected by the


economy than an organisation with volatile profits (banks) which are expected
to decrease. Conversely in high economic growth you would expect banks to
make higher profits. Therefore, in low economic growth you would expect the
durables to outperform the bank stock, whereas in higher growth bank stocks
are expected to outperform the consumer durables.

Page 5

Subject ST5 (Finance and Investment Specialist Technical A) Examiners Report April 2012

Variations in demand for the shares will exacerbate these effects.


(iii)

Because the individual is retired it is likely they are concerned with capital
preservation and equities being volatile are not usually suitable for risk averse
individuals.
On this occasion the individual wants capital growth so equities might be
suitable. However, given their retired status their time horizon might be
shorter than economic recovery period when bank stocks are expected to
underperform.
Also, the economy might make it difficult for banks to produce profits and the
bank stocks are not at the bottom of their valuation cycle.
The investor might wish to diversify away from one sector.

(iv)

A pooled fund holding bank stocks


An ETF invested in bank stocks
Derivatives
Bond holdings

Property losses can rapidly result in losses being transmitted to other parts
of the economy as property assets are often purchased using borrowed funds,
rather than purchased on an outright basis.
This means that if a property falls in value, the owner (borrower) will see an
increase in their loan to value (LTV), and hence represent a greater credit risk
for the lender. If the funds have been borrowed from a regulated financial
institution (e.g. a bank) then they will need to hold more capital against the
loan, creating balance sheet strains, and potentially the bank may need to sell
long-term assets such as loans to restore capital levels.
If a number of lenders are impacted simultaneously, as would often happen if
property prices fall, then they will all face capital strains. This will reduce
their ability to advance funds to new borrowers, and create shareholder losses.
They will also become less creditworthy themselves, resulting in funding
strains or higher borrowing costs.
Householders would experience problems when selling properties in the
depressed market. As the property value was now lower than their mortgage
borrowing, they would experience negative equity. This would in turn limit
their ability to fund a new property purchase and thus cause the property
market to stagnate. Where the move was needed due to a planned change in
employment, this would then influence the labour market. A regional
differential in property price movements, so that prices were falling in some
parts of the country but remaining stable elsewhere, could exacerbate this
problem.

Page 6

Subject ST5 (Finance and Investment Specialist Technical A) Examiners Report April 2012

Where property loans had been used as collateral in asset-backed


securitisations such as mortgage-backed securities or collateralised debt
obligations, the fall in property prices would reduce the demand for such
assets by investors. Equally, lenders would find that short-term funding, e.g.
on the interbank market, would become more expensive (or might even
become unavailable). This problem would be particularly acute if the lender's
activity concentrated on property loans.
The fall in property values and transactions might reduce government tax
revenues. Business supporting the property market, such as estate agents and
conveyancing solicitors, would also be adversely affected. Conversely, there
might be increased demand for property refurbishments or improvements from
homeowners unable to move. However the availability of second mortgages
and further loans would be limited due to the reduced collateral available and
exacerbated by additional caution from lenders.
Similar effects would be experienced in the commercial property sector if real
estate prices fell. The use of property values to support debenture loans or
sale and leaseback transactions could be affected. The property development
sector (builders, architects, material suppliers, etc.) would also be adversely
affected, particularly if the decline in prices was expected to persist for a
substantial period.
The fall in capital values will be reflected in an increase in rental yields.
Indeed the lack of turnover in the property market and the increase in required
LTVs may generate excess demand for rental property leading to a further
increase in rents. This will effect an income transfer from tenants to landlords
and the resulting change in disposable income may have wider economic
impacts and consequent changes in asset capital values according to relative
consumption patterns.

(i)

Capital gains tax is usually payable on disposal of an asset. This can lead to
investors attempting to defer tax liabilities by avoiding the crystallisation of a
capital gain.
Using capital losses to offset capital gains in the same year.
Derivatives can be used to reduce exposure to an asset rather than selling the
asset itself.
The existence of an annual tax-free allowance can also lead to investors selling
and repurchasing assets to crystallise a gain in order to take advantage of their
annual allowance.

Page 7

Subject ST5 (Finance and Investment Specialist Technical A) Examiners Report April 2012

(ii)

(iii)

Country A
Income tax
Capital gains tax
Total

Year 1
150000
165000
315000

Year 2
250000
1275000
1525000

Year 3
250000
210000
40000

Total
650000
1230000
1880000

Country B
Income tax
Capital gains tax
Total

Year 1
30000

Year 2
50000

Year 3
50000
1968750
2018750

Total
130000
1968750
2098750

Declare income tax in Country B and capital gains in Country A.


Have main residence in Country A.
Sell all assets in Country B every 3 years to exempt from capital gains tax.

(iv)

Overall tax paid in country of residence might be lower (i.e. lower income
tax).
Might not have any assets which are subject to capital gains.
Has some capital losses which are more advantageous to be written off in
country of residence.
If tax is only paid on capital gains (compared to a country where rates are
lower but tax is on the value of the assets).
Non-financial reasons as described.

(i)

Absolute pricing prices assets by reference to exposure to fundamental sources


of macroeconomic risk. The consumption-based and general equilibrium
models (such as CAPM) are examples of this approach, which can be used to
predict how prices might change if policy or economic structures change.
Relative pricing, as exemplified by Black-Scholes option pricing and
Arbitrage Pricing Theory, considers the value of an asset given the price of
some other assets. Here we use as little information about fundamental risk
factors as possible, and we do not ask where the prices of the other assets
came from.

(ii)

Page 8

Beta is a measure of a stocks volatility relative to movements in the whole


market. Usually defined as the covariance of the return on the stock with the
return on the market, divided by the variance of the market return. It is a
measure of the stocks exposure to non-diversifiable systematic risk.
Beta is useful because it allows the expected return of any security to be
expressed as a linear function of the securitys covariance with the market as a
whole.

Subject ST5 (Finance and Investment Specialist Technical A) Examiners Report April 2012

(iii)

Expected return = 5% + (0.1 / 0.2) (9 5) = 7%

(iv)

Empirical evidence suggests that the line relating return to beta has been too
flat in recent years, and that while return has not risen with beta it has been
related to other measures such as market capitalisation or book-to-market
ratio. CAPM predicts that beta is the only reason that expected returns differ
The risk free rate is not truly attainable, due to factors such as default,
inflation and currency risk.
CAPM assumes that investors can borrow money at the same rate of interest
as at which they can lend. In practice, borrowing rates are higher than lending
rates.
Markets are not perfect (with information freely and instantly available to
all investors). Similarly, investors will not share the same estimates of
expected returns, standard deviations and covariances of securities.

(i)

The MSCI style indices take the universe of a standard index and ranks the
securities according to Price-to-Book values. The top half stocks with low
Price-to-Book values is associated with the Value style and the bottom half
stocks with high Price-to-Book values is associated with the Growth style.

(ii)

Five Growth factors are:

Sales Growth
Earnings Growth
Forecast Earnings Growth
Return on Equity
Earnings Revisions

and five Value factors:

(iii)

Book to Price
Dividend Yield
Earnings Yield
Cash Flow Yield
Sales to Price

Other equity investment styles include:

Momentum purchasing (selling) those stocks which have recently risen


(fallen) significantly in price on the belief that they will continue to rise
(fall) owing to an upward (downward) shift in their demand curves.

Contrarian doing just the opposite to what most other investors are
doing in the market in the belief that investors tend to overreact to news.

Page 9

Subject ST5 (Finance and Investment Specialist Technical A) Examiners Report April 2012

(iv)

Rotational moving between Value and Growth depending on which style


is believed to be attractive at any particular point in time.

Big Bang Tech stock in market which is expanding growth stock


Power 2u Utility which is non-cyclical stock classic value stock
Classic Wooden Furniture mature, stable market value
Superfluid Expansion into markets that are growing growth
In the sticks expansion plans growth
GiveMeSomeCredit.com stable client base, seems to be value

(v)
Company

Year 1

Year 2

Year 3

Big Bang Theory Ltd


Power2u
Classic Wooden Furniture Ltd
Superfluid
In the sticks
GiveMeSomeCredit.com
Total Benchmark return
Growth Benchmark return
Value Benchmark return

12%
7%
4%
21%
5%
9%
11%
14%
7%

5%
8%
7%
2%
11%
8%
5%
5%
7%

2%
8%
9%
6%
1%
9%
4%
1%
8%

Total
return
19.95%
24.80%
21.30%
16.01%
17.72%
28.31%
21.21%
18.50%
23.65%

Total portfolio return


Growth portfolio return
Value portfolio return

9.7%
12.7%
6.7%

6.7%
5.8%
7.7%

3.7%
1.1%
8.7%

21.35%
17.89%
24.80%

Growth
Value
Value
Growth
Growth
Value

Workings
Company
Big Bang Theory Ltd
Power2u
Classic Wooden
Furniture Ltd
Superfluid
In the sticks
GiveMeSomeCredit.com
Portfolio return
Growth portfolio return
Value portfolio return

Page 10

100.00
100.00
100.00
100.00
100.00
100.00
600.00
300.00
300.00

Year 1
12.0% 112.00
7.0% 107.00
4.0% 104.00
21.0%
5.0%
9.0%
9.7%
12.7%
6.7%

121.00
105.00
109.00
658.00
338.00
320.00

Year 2
5.0% 117.60
8.0% 115.56
7.0% 111.28
2.0%
11.0%
8.0%
6.7%
5.8%
7.7%

123.42
116.55
117.72
702.13
357.57
344.56

Year 3
2%
8%
9%
6%
1%
9%
3.7%
1.1%
8.7%

Total Return
119.952
124.805
121.295
116.015
117.715
128.315
21.35%
17.89%
24.80%

Subject ST5 (Finance and Investment Specialist Technical A) Examiners Report April 2012

Answer

(vi)

Total portfolio return


Total benchmark return

Calc
21.35%
21.21%

Answer

Total Growth portolio return


Growth benchmark return
Total benchmark return

17.89%
18.50%
21.21%

0.61%
3.32%

Value portfolio return


Value benchmark return
Total benchmark return

24.80%
23.65%
21.21%

1.16%
3.59%

0.14%

From the calculations it shows that value stocks have outperformed growth
stocks. When an economy is contracting, value stocks tend to outperform so it
is likely the economy was contracting.

END OF EXAMINERS REPORT

Page 11

INSTITUTE AND FACULTY OF ACTUARIES

EXAMINATION
2 October 2012 (pm)
Subject ST5 Finance and Investment
Specialist Technical A
Time allowed: Three hours
INSTRUCTIONS TO THE CANDIDATE
1.

Enter all the candidate and examination details as requested on the front of your answer
booklet.

2.

You have 15 minutes before the start of the examination in which to read the
questions. You are strongly encouraged to use this time for reading only, but notes
may be made. You then have three hours to complete the paper.

3.

You must not start writing your answers in the booklet until instructed to do so by the
supervisor.

4.

Mark allocations are shown in brackets.

5.

Attempt all seven questions, beginning your answer to each question on a separate
sheet.

6.

Candidates should show calculations where this is appropriate.

AT THE END OF THE EXAMINATION


Hand in BOTH your answer booklet, with any additional sheets firmly attached, and this
question paper.
In addition to this paper you should have available the 2002 edition of the Formulae
and Tables and your own electronic calculator from the approved list.

ST5 S2012

Institute and Faculty of Actuaries

Outline the process of asset / liability mismatch reserving.

[6]

(i)

Define arbitrage.

[1]

(ii)

Explain how the concept of arbitrage can be applied to derive the price of a
forward contract.
[4]

(iii)

Explain why, in practice, it would not be possible to generate unlimited profits


from a legitimate arbitrage opportunity.
[3]
[Total 8]

(i)

Define behavioural finance.

(ii)

Describe the behaviours exhibited in each of the following scenarios:

ST5 S20122

[2]

(a)

A pension scheme is looking to award a new global equity mandate.


The trustees of the scheme have invited five investment managers to
each present their proposed strategy, with the intention of selecting one
of them. The investment manager who has been asked to present third
during the day has subsequently requested to present last.

(b)

A risk averse individual has sold their portfolio of Government bonds


and invested the proceeds in tulip (a flower) futures. Tulip futures
have been the best performing asset class over two years and are
increasing in value each month. Many financial commentators predict
they will continue to rise in value.

(c)

A holiday company, which ran a competition for a free holiday, has


sent the 10 winners a brochure with 300 different choices of holiday all
worth the same amount. Seven of the 10 winners pick the first holiday
in the brochure.

(d)

A pension fund manager is having operational problems with the


pension funds custodian. The custodian has recently received a lot of
negative publicity and service standards have been slipping. The
funds investment consultant recommends a search for a new custodian
is undertaken, but the pension manager is very resistant to the idea.
[8]
[Total 10]

A government has decided to introduce regulations to strengthen the capital position


of domestic companies. The additional capital required by a company will depend on
the historic volatility of profits for that company's sector.
(i)

Describe the impact the capital requirements will have on:


(a)
(b)
(c)
(d)

Utility companies
Consumer goods
Industrials
Banks
[8]

(ii)

Outline the reasons why the government would want to introduce the new
regulations.
[3]

(iii)

Suggest possible negative effects of the introduction of the regulations.


[5]
[Total 16]

(i)

Outline the main financial risks faced by institutional investors.

(ii)

For each of the following scenarios, describe the financial risks present:

(iii)

[4]

(a)

An options trader who is responsible for valuing and settling his own
trades has recently been found to have incorrectly represented a
number of trades over the last year. This has resulted in a large loss for
an investment bank.

(b)

In a small country the standard asset allocation for a pension scheme is


a balanced mandate based on the allocation of the average pension
scheme. One of the larger pension schemes has now decided to change
to a scheme specific benchmark, which has resulted in a higher
allocation to equities.

(c)

A large bank which specialises in uncollateralized derivatives has


recently gone bankrupt. This has caused banks in other countries to
restrict their lending in the affected market.

(d)

An island with very few regulatory rules has been successful in


attracting hedge funds to be registered in their country. Due to a
shortage of financial professionals, each professional is responsible for
about 200 funds on average.
[8]

Suggest possible actions to reduce the risks identified in (ii).

[5]

[Total 17]

ST5 S20123

PLEASE TURN OVER

(i)

Outline the process of liability hedging.

(ii)

Discuss the difficulties that may arise in constructing an investible portfolio of


hedge assets.
[6]

(iii)

Outline the Liability Driven Investment approach to setting investment


strategy.
[7]
[Total 17]

(i)

Describe what is meant by the terms:


(a)
(b)
(c)

[4]

active management
passive management
core and satellite portfolios
[7]

A small pension fund has recently decided to invest in US large capitalisation equities
and an emerging markets equity portfolio. As their investment consultant you have
been asked to comment on whether the US large capitalisation equities strategy
should be managed on an active or passive basis.
(ii)

Set out, with reasons, what you would advise the trustees.

[4]

For the emerging markets equity portfolio, the trustees have decided to follow an
active investment strategy with a manager called Emergaine Capital Markets. One of
the trustees recently met a friend who operates a company, Commertoze, which
manages emerging market equity portfolios on a passive basis. As a result of the
conversation the trustee is suggesting that part of the emerging markets equity
portfolio is managed on a passive basis.
(iii)

Comment on the financial reasons why the trustee has put forward the
proposed structure.

[2]

The trustees have decided to investigate whether a proportion of the emerging markets
equity portfolio should be managed on a passive basis. They have been presented
with the following information:
Manager
Emergaine Capital Markets
Commertoze
Benchmark

Year 1

Year 2

Year 3

33%
14%
14%

18%
9%
9%

7%
15%
13%

Emergaine Capital Markets applies an annual management fee of 0.75% applied to


the gross end of year value. In addition it applies a fee of 10% of the gross
outperformance of the benchmark each year.
Commertoze applies a fee of 0.15% of the average asset value during the year.
All charges are deducted at the end of each year.

ST5 S20124

(iv)

Calculate:
(a)

The gross of fees returns for both investment managers and the
benchmark for the entire three year period.

(b)

The net (of fees) value of both the Emergaine and Commertoze
portfolios at the end of each of the three years, assuming an initial
investment of 10 million dollars in each.

(c)

The net of fees returns for both investment managers at the end of
three years.
[10]

(v)

Comment on the trustees' decision to invest in an active strategy rather than a


passive strategy based on the results in (iv).
[2]

(vi)

Suggest how the trustees might be able to achieve a better net of fee return
with the active manager.
[1]
[Total 26]

END OF PAPER

ST5 S20125

INSTITUTE AND FACULTY OF ACTUARIES

EXAMINERS REPORT
September 2012 examinations

Subject ST5 Finance and Investment


Specialist Technical A

Introduction
The Examiners Report is written by the Principal Examiner with the aim of helping
candidates, both those who are sitting the examination for the first time and using past papers
as a revision aid and also those who have previously failed the subject.
The Examiners are charged by Council with examining the published syllabus. The
Examiners have access to the Core Reading, which is designed to interpret the syllabus, and
will generally base questions around it but are not required to examine the content of Core
Reading specifically or exclusively.
For numerical questions the Examiners preferred approach to the solution is reproduced in
this report; other valid approaches are given appropriate credit. For essay-style questions,
particularly the open-ended questions in the later subjects, the report may contain more points
than the Examiners will expect from a solution that scores full marks.
D C Bowie
Chairman of the Board of Examiners
December 2012

Institute and Faculty of Actuaries

Subject ST5 (Finance and Investment Specialist Technical A) Examiners Report September 2012

General comments
Investment is a practical subject and, at this level, the examiners expect candidates to
demonstrate a breadth and depth of competency as would be expected from a senior student
in a frequently evolving discipline. Hence simple regurgitation of bookwork will never be
sufficient to ensure a Pass grade and this was evident from the dispersion of candidates
responses in the more differentiating questions.
Most candidates seemed to identify and understand the key issues being examined and so
appreciated the general content of solutions that the examiners were looking for. Candidates
are reminded to avoid being too narrow in their responses to questions, but ensure that
responses remain relevant and do not labour minor points. Candidates will not be explicitly
penalised for this last activity, but it gives an impression of a lack of understanding and, more
importantly, wastes limited time. The examiners have used their discretion as to whether or
not to recognise valid points for one part of a question made in another. Likewise the
examiners share and agree alternative possible solutions to questions alongside the approach
outlined below.
Investment is a fast evolving subject driven by the greater volatility and globalisation/
integration of markets and economies alongside the challenges of delivering an acceptable
return for a long term strategy in the context of a focus and political/regulatory backdrop that
is increasingly short term. In order to succeed, candidates must ensure they familiarise
themselves with the prevailing investment issues and the general market background facing
institutional investors in the 1218 months preceding a diet, more so the solutions (and
sources thereof) being debated by the various stakeholders. Hence questions regarding
banking and derivative approaches, as well as active and passive asset management and
insurance solutions, to asset and liability risk management (including model risk) or modern
financial theory and commercial applications should be considered likely scope for
examination.
Against a background of the credit crisis, new asset classes and ways of structuring
investments will themselves generate new types of risk (such as operations, liquidity, credit
and counterparty) and so the need for new ways of regulation, monitoring and management.
Finally the examiners encourage candidates to recognise there are different types of investor
beyond purely pension funds so that different taxation, time line and cost considerations will
apply - it would seem that candidates have taken this on board.
Whilst the examiners will tolerate bullet point style responses, some candidates handwriting
made assessment difficult and they may have lost marks. Likewise "text speak"
abbreviations will not be accepted.

Page 2

Subject ST5 (Finance and Investment Specialist Technical A) Examiners Report September 2012

Specific comments on September 2012 paper


This paper resulted in a relatively high pass rate. It was of a comparable standard to previous
exams and so the examiners were pleased to note candidates were generally better prepared,
providing better content in quality and not just quantity. The examiners are hopeful that this
trend will continue.
Candidates typically answered Question 5 much better than the others (albeit still foregoing a
third of marks available), with Questions 1 and 6 attracting the worst responses, considerably
so, with average scores of around a third of the available marks. Question 1 was
predominantly bookwork knowledge so the scores are disappointing. Question 6 focuses on
Liability Driven Investing, one of the most topical issues facing many institutional investors
today and so candidates might reasonably have been expected to be aware of the key issues.
Candidates generally scored very well on the risk and management aspects of Question 5 and
2.
Questions 4 and 7 focussed on the practical aspects of investment as distinct from theory,
right down to stock level and candidates generally gave a good account of the detailed
considerations required. Many questions represented opportunities to demonstrate higher
level skills in terms of non-standard/practical application of theory to current or unusual
issues in investment candidates who wish to progress to SA6 will need to improve their
understanding of, and approach to, such questions.

Page 3

Subject ST5 (Finance and Investment Specialist Technical A) Examiners Report September 2012

Asset / liability mismatch reserving is an example of the use of modelling in actuarial


work. The emerging asset and liability position is projected under a range of possible
conditions (economic, environmental, etc.) in order to establish the extent to which
assets and liabilities are mismatched. Appropriate supplementary reserves can then be
set up to cover the possible levels of shortfall identified.
The modelling can, as usual, be carried out using either deterministic or stochastic
methodologies. In a deterministic framework, it is up to the modeller to decide the
nature and extent of the scenarios to be tested for the purpose of setting the reserves.
At its simplest, the investigation may be restricted to the current portfolio of assets
and liabilities only, and consider the impact of an immediate change in conditions,
rather than involve projections of the emerging state of the fund. Such an approach is
often referred to as resilience testing. However, with modern computer modelling
power readily available, more dynamic approaches are typically adopted.
These would include the use of stochastic techniques, where multiple projections
would be made in order to generate many possible future scenarios. Most often, the
stochastic element of the projections would apply to the asset portfolio and investment
returns, in order to assess exposure to systematic risk. Given that a finite number of
projections must be performed, assessment of the results is often carried out in the
form of ruin probability, that is, the outcomes are ranked in terms of a target measure
(such as the shortfall of assets relative to liabilities at a specified future date).
Additional reserves are then set up at a level sufficient to cover all but a specified
proportion of such shortfalls.

(i)

The simultaneous buying and selling of two economically equivalent but


differentially priced portfolios so as to make a risk free profit.

(ii)

We can apply the concept of arbitrage to derive the price, F0, of a forward
contract in terms of the spot price S0. No arbitrage requires that
F0 = S0erT where T is the time when the forward contract matures and r is the
risk-free rate of interest (for an investment maturing at time T). If this equality
did not hold, arbitrage possibilities would exist. If F0 < S0erT the investor can
sell the asset short at the current spot price S0, invest the sale proceeds riskfree (to accumulate a sum S0erT), and, at the same time, enter into a long
forward contract to buy the asset at time T at price F0. This will generate a
risk-free profit of S0erT F0 for no initial outlay.
Similarly, if F0 > S0erT unlimited profit can be made from a strategy of
borrowing S0 now to buy the asset and entering into a short forward contract to
sell the asset at time T for F0. At that time the loan and accumulated interest
of S0erT will be repayable, leaving the investor with a risk-free profit of F0
S0erT . The only price for the forward, F0, that eliminates the arbitrage
opportunities is S0erT.

Page 4

Subject ST5 (Finance and Investment Specialist Technical A) Examiners Report September 2012

(iii)

Cannot get access to unlimited borrowing.


Arbitrage opportunities only exist for a finite time frame before other investors
recognise the arbitrage and the opportunity is closed. The use of modern
technology increasingly limits such arbitrage opportunities.
Frictional / transaction costs, taxes, etc. The loan interest rate may be higher
than the risk free rate. Operational and credit risk may be involved.
Basis risk e.g. the funds borrowed are recalled before the investment period
expires.
Limits on the use of short-selling.

(i)

The field of behavioural finance looks at how a variety of mental biases and
decision-making errors affect financial decisions. It relates to the psychology
that underlies and drives financial decision-making behaviour.

(ii)

(a)

Recency effect in some instances the final option (presentation) is


preferred.

(b)

Anchoring and adjustment people base future perceptions based on


past experience and expert opinion. The valuation is then amended for
the scenario to fit the assumptions.
Could also mention myopic loss aversion investors less risk averse
when faced with a multiple period of gambles.

(c)

Primary effect people are more likely to choose the first option
presented.
Effect of options a greater range of options tends to discourage
decision making.

(d)

Status Quo bias people like to keep things the way they are.
Regret aversion by retaining the existing arrangements people
minimise the possibility of regret.

Other behaviours cited were given credit if fully described so that their applicability to the
scenario was clearly demonstrated.

Page 5

Subject ST5 (Finance and Investment Specialist Technical A) Examiners Report September 2012

(i)

Utility stable profits so likely to have little additional capital requirements.


Consumer goods durables tend to have volatile profits so will have
additional requirements. Non-durables have more stable profits so capital
requirements will tend to be lower.
Industrials In general volatile profits which means that industrials are likely
to be hit hard by capital requirements.
Banks have volatile profits and therefore banks are likely to be hit hard by the
introduction of the regulations.
If banks are required to hold extra capital then this is likely to impact how
cash is invested as capital reserves tend to be held in low risk assets. As a
result the bank profits are likely to be lower which will negatively impact the
share price.

(ii)

To lower the risk of firms becoming insolvent in times of poor economic


climate.
To increase investor/consumer confidence in the market and protect
consumers.
To bring them in line with regulations in other countries.
To restrict new entrants to well funded organisations.

(iii)

Lower profits as capital being used to meet reserves plus the cost of
monitoring. Companies may respond by charging higher prices or reducing the
workforce.
Higher barriers to entry as new companies might not have enough capital to
meet the requirements.
Companies that dont have enough capital might be forced into bankruptcy.
Could make companies less competitive relative to international peers.
Capital requirements will mean less cash to spend on growing business and
product development. Hence lower international competitiveness.
Companies might restructure to get into a less volatile sector.
Companies might relocate to other countries.
Tax revenues might be reduced.

Page 6

Subject ST5 (Finance and Investment Specialist Technical A) Examiners Report September 2012

(i)

Market risk the risk relating to changes in the value of the portfolio due to
movements in the market value of the assets held.
Credit risk the risk that a counterparty to an agreement will be unable or
unwilling to fulfil their obligations.
Operational risk the risk of loss due to fraud or mismanagement within the
fund management organisation itself.
Liquidity risk the risk of not having sufficient cash to meet operational needs
at all times.
Relative performance risk the risk of underperforming comparable
institutional investors.

(ii)

(a)

Operational risk The trader has the opportunity to influence the price
of the trades he makes and therefore the risk of fraud is increased.
Liquidity risk the loss might have caused the bank to have liquidity
problems due to loss.
Credit risk risk that other banks might not lend to affected bank as
perceived higher credit risk.

(b)

Market risk Equities can be volatile so this could result in capital


losses and therefore the risk that portfolio value decreases.
Relative performance risk if the pension scheme is holding a
different asset allocation compared to other schemes then there is the
risk that the allocation underperforms relative to the other schemes.
This can be exaggerated by holding greater allocation to equities which
can be volatile.

(c)

Credit risk restricted lending as fear that other banks in country with
bankrupt bank could be affected and therefore other countries reduce
credit risk exposure.
Also liquidity risk.

(iii)

(d)

Operational risk the professionals have too many funds to look after
and regulation is light which increases the risk of fraud and errors due
to lack of oversight.

(a)

Risk could be reduced by separating trading function from the


operations department so that a trader cannot settle own trades, and
someone in operations is responsible for settling trades. This is referred
to as segregation of duties.

(b)

Return asset allocation to the peer group to reduce relative


performance risk.

Page 7

Subject ST5 (Finance and Investment Specialist Technical A) Examiners Report September 2012

The pension fund could look to offset increased volatility in the equity
portfolio by investing remaining assets in less volatile assets (such as
cash) to reduce risk of portfolio declining in value.
(c)

The banks lending could insist on derivative deals being completed on


a collateralised basis.
Banks could ask for Government guarantee if the banks in the country
fail. This will allow lending to take place again.

(d)

(i)

Limit the number of funds that a professional could represent.


Increase regulation.

Liability hedging is where the assets are chosen in such a way as to perform in
the same way as the liabilities (that is to change in value by the same
proportion). A specific example of this is the concept of immunisation, where
assets are matched to liabilities by term in order to reduce interest rate
sensitivity (to parallel movements in the yield curve). Other forms of hedging
would include matching by currency and the consideration of the real or
nominal nature of liabilities when determining the choice of assets. However,
these examples relate only to specific characteristics of the liabilities, whereas
liability hedging aims to select assets which perform exactly like the liabilities
in all states.
The most familiar example would be for an investor to hold a portfolio of
government bonds (in the appropriate currency) until maturity to meet a prespecified stream of future fixed payments. Provided the future payments do
not change in amount or timing, the coupon and principal proceeds from the
bond portfolio can be used to meet the obligation to make the payments.

(ii)

Difficulties with this approach arise for the following reasons:

Such an approach requires a bond asset to be held that is equal in present


value to the future payments discounted at bond yields (using the full yield
curve). Therefore, only a partial hedge is only possible if asset cover is
less than 100%.

If the latter payments are payable after the principal payment of the longest
available government bond then it will not be possible to hedge these
payments at present (until longer maturity bonds become available, i.e.
creating reinvestment risk).

Due to gaps between bond maturities (particularly at longer durations),


there may be a need to reinvest or disinvest bonds prior to maturity, and
the hedge may therefore be imperfect.

The use of government bonds gives risk to a (small) degree of credit risk
that may not necessarily be reflected in the liability. If other bonds are
used, they are more risky.

Page 8

Subject ST5 (Finance and Investment Specialist Technical A) Examiners Report September 2012

(iii)

If the tax status of the government bonds worsens, this will mean the assets
are likely to be insufficient to meet the liability payments.

Due to the above factors, there may be some mark to market risks between
the asset value of the bond portfolio and the present value of the liability
payments discounted using the bond yield curve. In some cases this may
be a material risk factor, but in other cases this will be much smaller than
uncertainties in the liability payments themselves or other portfolio risks.

Liability Driven Investment (LDI) is the terminology used to describe an


investment decision where the asset allocation is determined in whole or in
part to a specific set of liabilities.
LDI is not a strategy or a type of product available in the market but an
approach to setting investment strategy that controls asset-liability
mismatches. Thus, while liability hedging seeks to address specific features of
the liability structure, LDI is a more holistic approach to developing an
investment strategy.
An LDI approach would typically aim to achieve a close match of the
following liability features:

The interest rate sensitivity (duration) of the liabilities. Investments that


are used to match the duration of liabilities include fixed rate bonds and
interest rate swaps.

The inflation-linkage of the liabilities. Investments used to match the


inflation exposure of liabilities include inflation linked bonds and inflation
swaps.

The shape of the liabilities. The shape of the liabilities will depend on when
the cashflows are expected to be paid. Although it is possible to construct a
bond portfolio where bond payments match the projected liability payments
for a pension fund it is often more difficult to match longer duration payments
(4050 years) due to the limited issuance or non-availability of bonds. This
presents particular challenges for long-dated liabilities, especially inflation
linked liabilities. In order to purchase assets that match the shape of cashflows
at longer durations, investors rely on using swaps to hedge both interest rate
and inflation risks.
However, non-investment risks such as longevity tend to remain, although
products are being developed to manage non-investment risks and are gaining
in popularity.
Examples relating to asset classes other than bonds were given equivalent credit.

Page 9

Subject ST5 (Finance and Investment Specialist Technical A) Examiners Report September 2012

(i)

(ii)

(a)

Active investment managers apply various types of judgement to the


selection of portfolios with the objective of outperforming a
benchmark. Active management offers the prospect of large returns
(in excess of fees paid) and the limitation of peer group risk.
However, successful selection of active investment managers is hard to
achieve and timing the changes to the line-up of active managers is
also very difficult.

(b)

Passive investment managers are, typically, index-trackers. They


manage assets without taking active investment decisions. Instead,
their objective is to track closely the performance of a specified index.
This offers the advantages of lower cost and volatility, but with the
loss of upside potential and the implicit restriction to markets and asset
classes where a suitable benchmark exists.

(c)

An increasingly popular fund management structure is to manage the


majority of the fund (the core portfolio) on a passive, low-cost basis.
Specialist satellite managers are then employed to provide increased
performance (in excess of fees paid) in respect of the balance of the
fund. This may extend to employing a number of competing managers
in respect of the specialist asset classes, if the size of the overall fund
warrants this. Increasingly, the satellite managers will include hedge
fund and private equity specialists.

The trustees have to decide how much return will be derived from beta
(exposure to systematic risk) and how much from alpha (asset selection to
exploit market inefficiencies). For the alpha allocation (manager
outperformance) the trustees need to decide which is the most efficient way to
generate the alpha.
The alpha can be generated from either the emerging markets portfolio or the
US equities portfolio. US large cap is a highly efficient market and therefore
difficult to generate alpha. Emerging markets is less efficient and should be
easier to generate alpha. Based on the choices recommend to invest US
equities on passive basis.

(iii)

To reduce the risk of underperforming the benchmark by applying a core and


satellite approach to the emerging market portfolio.
Has concerns that Emergaine are not the most suitable manager and therefore,
wants to decrease allocation.
Trustee wants to spend alpha budget on another bit of the portfolio rather than
the emerging markets.
To cut costs.

Page 10

Subject ST5 (Finance and Investment Specialist Technical A) Examiners Report September 2012

(iv)

Gross of Fees
Manager
Emergaine
Capital
Markets
Commertoze
Benchmark

Value 1

Return 1 Value 2

10000000 33%
10000000 14%
10000000 14%

13300000
11400000
11400000

Return 2 Value 3

18%
9%
9%

Return 3

15694000 7%
12426000 15%
12426000 13%

Value 4

Total
Return

14595420 46.0%
14289900 42.9%
14041380 40.4%

Net of Fees
Manager
Emergaine
Capital
Markets
Commertoze
Benchmark

Value 1

Return 1 Value 2(net) Return 2 Value 3

10000000 33%
10000000 14%
10000000 14%

13010250
11383950
11400000

18%
9%
9%

Value 3
(net)

15352095 15119862
12408506 12390661
12426000

Value 4
(net)

Return 3

Value 4

7%
15%
13%

14061471 13956010
14249260 14229280
14041380

Total
Return (net)

39.6%
42.3%
40.4%

Note:
Candidates may choose to apply the returns obtained to the year-end fund values in calculating the performance fee for Emergaine. Credit should
be given for this alternative approach the appropriate figures are:

Value 1
Emergaine
Capital
Markets

Return 1 Value 2 (net) Return 2 Value 3

10000000 33%

12947550

18%

Value 3
(net)

15278109 15026020

Value 4
(net)

Return 4

Value 4

7%

13974199 13869392

Total
Return (net)

38.7%

Page 11

Subject ST5 (Finance and Investment Specialist Technical A) Examiners Report September 2012

(v)

Based on results there is very little to choose between active and passive
managers on a net of fees basis. The passive manager has slightly
outperformed which suggests that up to all the portfolio could be invested on a
passive basis. However, past performance is no guarantee for future and active
manager might offer outperformance in the future.

(vi)

Negotiate lower management fees with the active manager.


Have a high watermark applied to performance fee to reduce performance fee
payable.
Increase risk that manager is allowed to take.

END OF EXAMINERS REPORT

Page 12

INSTITUTE AND FACULTY OF ACTUARIES

EXAMINATION
17 April 2013 (am)
Subject ST5 Finance and Investment
Specialist Technical A
Time allowed: Three hours
INSTRUCTIONS TO THE CANDIDATE
1.

Enter all the candidate and examination details as requested on the front of your answer
booklet.

2.

You have 15 minutes before the start of the examination in which to read the
questions. You are strongly encouraged to use this time for reading only, but notes
may be made. You then have three hours to complete the paper.

3.

You must not start writing your answers in the booklet until instructed to do so by the
supervisor.

4.

Mark allocations are shown in brackets.

5.

Attempt all eight questions, beginning your answer to each question on a separate
sheet.

6.

Candidates should show calculations where this is appropriate.

AT THE END OF THE EXAMINATION


Hand in BOTH your answer booklet, with any additional sheets firmly attached, and this
question paper.
In addition to this paper you should have available the 2002 edition of the Formulae
and Tables and your own electronic calculator from the approved list.

ST5 A2013

Institute and Faculty of Actuaries

(i)

Outline the two key investment objectives for a fund holding


assets below 100% of its liabilities, and the conflict between them.

[5]

(ii)

Describe two measures of the risk of the portfolio in (i) relative to its
benchmark, including details of the data used and any assumptions made. [6]

(iii)

Suggest how the two measures in (ii) can be adapted to measure asset risks
relative to liabilities.
[3]
[Total 14]

An investment manager who previously managed property funds has decided to offer
a guaranteed capital return type product which will offer the client the higher of the
following returns each quarter:
1.

fixed 4% per quarter

2.

the rate of increase/decrease in the average value of football players in a certain


Football league (Footy)

(i)

Discuss the difficulties of calculating the Footy index return.

[5]

(ii)

Describe changes that could be made to the Footy index to reduce some of the
difficulties highlighted.
[4]

You have been given the following information in respect of the Footy index.
End of quarter
Footy value

1
105

2
108

3
106

4
112

5
117

6
119

7
124

8
127

The start value was 100.


At the beginning of period 4, one player valued at 4 was removed and one valued at 6
was added.
(iii)

Calculate the overall return for the 2 year period:


(a)
(b)

of the Footy
of the fund
[4]
[Total 13]

ST5 A20132

M-Loans is a specialist firm offering finance to small companies. Prior to any


funding being agreed the company must undergo a rating agency assessment.
(i)

List the areas a rating agency would focus on in its analysis.

[4]

(ii)

Discuss the potential problems M-Loans may face during a recessionary


period.

[4]

(iii)

Suggest how these problems could be mitigated.

(i)

Describe the key characteristics of each of six bond-like assets which could be
used as alternatives to government bonds.
[8]

(ii)

Describe the circumstances in which these asset classes might exhibit


different returns to government bonds.
[5]
[Total 13]

(i)

Define what is meant by a contango market.

(ii)

Explain what is likely to happen to prices of the following commodities during


a global recession:
(a)
(b)
(c)

[3]
[Total 11]

[2]

crude oil
gold
pork bellies
[6]

(iii)

Explain the principal benefits of investing in alternative asset classes such as


commodities.
[3]

(iv)

State the ways in which an investor could invest in commodities.

(v)

Outline the benefits and disadvantages of each method in (iv).

(i)

State the formula for calculating a total return index, defining


all the terms used.

[3]

Given the following data, calculate the value of the total return
index for Day 2.

[2]

(ii)

(iii)

ST5 A20133

Day

Capital index

Total return index

XD adjustment

1
2

5857.52
5774.20

4080.63

168.82
170.85

Calculate the dividend yield over the day.

[3]
[6]
[Total 20]

[1]
[Total 6]

PLEASE TURN OVER

You are a new provider of an individual defined contribution pensions savings


product with a simplified investment approach sold via the internet. The product
charges a total management fee of 50bps p.a. (all assets are directly managed) and
there are only two fund choices:

a growth fund, and

an inflation-linked annuity matching fund (at retirement it is mandatory to use


pension savings to buy an inflation-linked annuity)

The growth fund invests in a diversified range of asset classes with a benchmark asset
allocation of:
40% equity;
20% alternative asset classes (property, private equity, infrastructure);
20% investment grade bonds;
20% high yield bonds and asset backed credit.
Individual contributors can either select their chosen mix of growth fund and annuity
fund units, or opt for a lifestyling approach that phases from the growth fund into the
annuity fund based on the chosen retirement date. Individuals are permitted to
transfer to another provider with no penalty.
You are confident that the low fee scale, the simplified approach and the attractions of
diversified investment strategies will permit the rapid growth of market share and
assets under management.
(i)

(ii)

(i)

Describe the advantages and disadvantages of such an approach from


the investors perspective.
Describe the specific challenges for the provider in managing the
underlying investments as assets under management grow.

[8]

[8]
[Total 16]

State the formula for calculating a forward interest rate, defining all terms
used.
[3]

Assume that the 3-month Libor rate is 5% p.a. and the six-month rate is 5.5% p.a.
(with continuous compounding). A forward rate agreement has been set up where we
will receive a rate of 7% p.a., measured with quarterly compounding, on a principal of
$1m, between the end of months 3 and 6.
(ii)

Calculate the value of the forward rate agreement.

END OF PAPER

ST5 A20134

[4]
[Total 7]

INSTITUTE AND FACULTY OF ACTUARIES

EXAMINERS REPORT
April 2013 examinations

Subject ST5 Finance and Investment


Specialist Technical A

Introduction
The Examiners Report is written by the Principal Examiner with the aim of helping
candidates, both those who are sitting the examination for the first time and using past papers
as a revision aid and also those who have previously failed the subject.
The Examiners are charged by Council with examining the published syllabus. The
Examiners have access to the Core Reading, which is designed to interpret the syllabus, and
will generally base questions around it but are not required to examine the content of Core
Reading specifically or exclusively.
For numerical questions the Examiners preferred approach to the solution is reproduced in
this report; other valid approaches are given appropriate credit. For essay-style questions,
particularly the open-ended questions in the later subjects, the report may contain more points
than the Examiners will expect from a solution that scores full marks.
The report is written based on the legislative and regulatory context pertaining to the date that
the examination was set. Candidates should take into account the possibility that
circumstances may have changed if using these reports for revision.
D C Bowie
Chairman of the Board of Examiners
July 2013

Institute and Faculty of Actuaries

Subject ST5 (Finance and Investment Specialist Technical A) April 2013 Examiners Report

General comments on Subject ST5


Candidates are reminded of a bias in the paper towards recognising higher level skills and
practical application this is intentional and will continue. Likewise the examination system
does properly allow for prior subject knowledge to be assumed. Investment is a necessarily
practical subject and, at this level, the examiners expect candidates to demonstrate a breadth
and depth of competency as would be expected from a senior student in a frequently evolving
discipline. Hence simple regurgitation of bookwork will never be sufficient to ensure a Pass
grade and this was evident from the dispersion of candidates responses in the more
differentiating questions.
Given the greater volatility in recent years and globalisation/integration of markets and
economies, delivering an acceptable return from a long term strategy against an increasingly
short term focus, disclosure regime and political/regulatory backdrop has become
increasingly challenging for investors. However, the challenge can be viewed as just a more
complex variant of the traditional risk/reward trade-off where the return-free risk is
becoming increasingly and unfortunately common. Investors generally only have assets
because they have liabilities and it is the latter that will drive strategy. In order to succeed,
candidates must ensure they familiarise themselves with the prevailing investment issues and
the general market background facing institutional investors in the 1218 months preceding a
diet. This particularly includes the solutions being debated by the various stakeholders.
Hence questions regarding banking and derivative approaches, as well as active and passive
asset management and insurance solutions, to asset and liability risk management or modern
financial theory should be considered likely scope for examination. Against a background of
the credit crisis, new asset classes and ways of structuring investments will themselves
generate new types of risk (such as benchmark, model, operations, liquidity, credit and
counterparty) and also the need for new ways of regulation, monitoring and management.
Finally the examiners encourage candidates to recognise there are different types of investor
and stakeholders beyond purely pension funds where different taxation, time line and cost
considerations will apply it would seem that candidates have taken this on board.
Whilst the examiners will tolerate bullet point style responses, some candidates handwriting
was too poor to assess and they will have lost marks. Likewise text speak abbreviations
will not be accepted.
Comments on the April 2013 paper
This paper had a lower pass mark than the previous diet although perhaps more consistent
with prior years given the pass rate. The examiners believed that the paper was of a
comparable standard to previous exams but the examiners are looking for candidates to be
better prepared, providing better content in quality and not just quantity so that both an
increase in pass mark and rate is witnessed.
Candidates typically answered Question 5 and 6 much better than the others (albeit still
foregoing 2540% of the marks available), with Question 1 attracting the worst responses,
considerably so, with average scores of around a third of the available marks. That said,
some candidates did achieve nearly full marks on this question (and some achieved full marks
on questions 6 and 8). Question 1 highlights a real challenge facing many institutions today
and the need for pragmatism as well as theory so the average scores are disappointing.

Page 2

Subject ST5 (Finance and Investment Specialist Technical A) April 2013 Examiners Report

Question 2 also looked at benchmarks and construction issues benchmark risk, data
management and the unintentional consequences they may have for successful investing is a
challenge that investors are only really starting to appreciate properly. Question 3 looks at a
particularly topical issue given the public scrutiny on lending to businesses and the role of
credit agencies in the financial crisis and now, likewise Question 4 as many investors search
for yield.
Questions 5 and 7 required a good knowledge of bookwork and its application, and so
probably were the questions to differentiate candidates, whereas Questions 6 and 8 were
fairly standard numerical calculations and high scores were to be expected.
Many questions represented opportunities to demonstrate higher level skills in terms of nonstandard/practical application of theory to current or unusual issues in investment
candidates who wish to progress to SA6 will need to improve their understanding of and
approach to such questions.
Most candidates seemed to identify and understand the key issues being examined and so
appreciated the general content of solutions that the examiners were looking for however
those that were unsuccessful will find their solutions lacked sufficient (and often the most
basic) detail or application of knowledge and scored lower accordingly. Whilst some
candidates are too narrow in their responses, a greater number still deviate from the topic and
include irrelevant material or over emphasise minor points although candidates will not be
explicitly penalised for this, it gives an impression of a lack of understanding and, more
importantly, wastes limited time. Time and priority management are key skills actuaries need
to have. Where candidates made relevant points in other parts of their solutions, the
examiners have used their discretion as to whether to recognise these answers or not.
Likewise the examiners share and agree alternative possible solutions to questions alongside
the approach outlined below.

Page 3

Subject ST5 (Finance and Investment Specialist Technical A) April 2013 Examiners Report

(i)

Ensure security by maintaining solvency coverage (in some cases allowing for
planned future contributions to the fund)
Aim to generate long-term investment returns above the risk-free
rate so as to reduce the size of required future contributions
The first of these objectives will encourage hedging of liabilities and holding
low risk assets, whereas the second of these objectives will encourage holding
more risky assets in pursuit of excess returns (and reducing hedging activity
where there is a cost to hedging).

(ii)

Retrospective tracking error annualised standard deviation of portfolio return


relative to benchmark return. This is based on historic / observed returns
It is often useful to differentiate between downward and upward semi-standard
deviation (by separating returns above and below the benchmark respectively).
Prospective tracking error a forward looking measure of the risk of the
portfolio relative to the benchmark based on a quantitative model which
assumes that the current structure remains unaltered and makes assumptions
about the volatility of stocks or asset classes and correlations between them .
Different models will have different levels of granularity.
Other measures that might be used include the Jensen and pre-specified
standard deviation risk-adjusted performance measures. The information
ratio also makes use of the retrospective tracking error.

(iii)

Instead of using an asset-based benchmark, the benchmark can be set to be the


total return on a pool of assets that are a close match for the liabilities, i.e. a
liability proxy.
Then the retrospective tracking error would measure the magnitude of overall
asset liability mismatches on a historic basis, and the prospective tracking
error would measure the same on a forward looking (model) basis.

(i)

Each football player is unique so no defined price


The market value of a footballer is only known when a trade takes place
Estimation of player value is very subjective
Valuations will be carried out at different points in time
The price at which the price is agreed between buyer and seller is often
confidential.
Large number of players to assess

Page 4

Subject ST5 (Finance and Investment Specialist Technical A) April 2013 Examiners Report

(ii)

Use a subset of players to reduce the amount of calculation required


Use valuation points where most transfers take place to get better idea of price
inflation
Use a proxy increase rate where values are not known, such as inflation
Devise a pricing methodology based on some factors such as division play in,
age etc.

(iii)
Footy return
Fund return

(i)

1.029
1.04

0.981
1.04

1.037
1.04

1.045
1.045

1.017
1.04

1.042
1.042

1.024
1.04

24.6%
39.1%

The fundamentals of the rating agencies approach to rating companies


will focus on:

(ii)

1.050
1.050

fundamental risks of the companys industry


competitive position (relative to peers)
downside risk vs. upside potential
quality of profitability vs. EPS growth
cash flow generation vs. book profitability
forward looking analysis
strategy, management track record and risk appetite
capital structure and financial flexibility

Increase in defaults from companies taking loans


Likely to be poorer quality companies looking for funding
Maybe fewer loans being taken as companies decide to exit business or not
finance expansion like previously
Quality of data from rating agency. In fast changing credit situation then
ratings could be behind cycle.

(iii)

Ask for additional security in event of default


Only lend to sectors that expect to do well or not decline significantly during
recession. Thus diversification would be sensible.
Use different analysis to rate clients rather than using credit rating agency
May need to offer more competitive rates to take higher proportion of a
smaller market.
Use credit derivatives to mitigate risk.
Securitise the loans (to transfer risk to other investors).
Page 5

Subject ST5 (Finance and Investment Specialist Technical A) April 2013 Examiners Report

(i)

(ii)

Alternative bond-like assets include:

Agency bonds bonds issued by governmentsponsored agencies e.g.


nationalised industries and local authorities (in the UK) or Federal
Mortgage Associations, states and counties and school boards (in the US).

Investment Grade Corporate bonds, preference shares and debentures


bonds rated at least Baa (by Moodys) or BBB (by Standard and Poors)

High Yield bonds bonds rated below investment grade

Convertible bonds bonds that may be converted into equity at a later date

Distressed debt securities of companies or government entities that are


either already in default, under bankruptcy protection or in distress and
heading for bankruptcy

Event linked bonds bonds with coupons and redemption payments


conditional on the nonoccurrence of a defined event (such as an
earthquake)

Interest rate and inflation swaps swapping fixed payments on a notional


principal for payments linked to market interest rates such as LIBOR or an
inflation index

Credit default swaps contracts that provide a payment if a specified


credit event occurs

Mortgage Backed Securities (MBS) and Asset Backed Securities (ABS)


(Securitisations) bonds that are serviced and repaid exclusively out of
a defined element of future cash flow from a bundle of assets owned by the
issuer

Infrastructure assets debt serviced from the cash flow generated by an


infrastructure project

The above assets could exhibit gains or losses relative to government


bonds due to one of the following factors:

Divergent yields between low risk assets (e.g. high quality government
bonds) and risky assets (e.g. credit, equities), e.g. flight to quality or a
risk assets rally

Asset class specific factors (e.g. general rerating of a single asset class)

Issuer or security level factors (e.g. the downgrade or default of an issuer)


Particular risk factors for the above asset classes include:

Page 6

Subject ST5 (Finance and Investment Specialist Technical A) April 2013 Examiners Report

Prepayment risk MBS, ABS, Agency bonds have incentives for


borrowers to bring forward payments on the underlying loans (e.g. falling
interest rates and ability to refinance at a lower rate)

Changing views on probability of default can impact at an issuer, sector


or asset class level

Changing views on recovery after default (or loss given default) can
impact at an issuer, sector or asset class level

Illiquidity risk (if the terms of the bonds are different to government
bonds)

(i)

A contango market is a commodity futures market where the futures price


normally exceeds the spot price. This is usually because the cost of carry (the
financing cost of holding the underlying commodity, plus storage costs ) is
positive and future price = spot price + cost of carry.

(ii)

Crude oil in recession there is less demand for oil which reduces output.
Reduced output and weakened demand have negative impact on price
therefore, decreasing cost of commodity
Gold during a recession gold is often regarded as a safe haven for investors.
Therefore, the demand for gold usually increases, resulting in the value of gold
commodities to rise.
Pork bellies during a recession then people start to switch from expensive
foods to cheaper foods. Pork bellies are a cheaper meat and it is likely that
people will switch from more expensive meats therefore, leading to a
moderate rise in pork belly prices

(iii)

(iv)

The principal benefits of alternative investments are:

Potential for higher returns, possibly from increased beta, market


inefficiencies, pricing anomalies or the skill in selection / management of
the investor.

Diversification due to a lack of correlation with existing assets or by


exposure to underlying risks that are uncorrelated so reducing the overall
portfolio risk.

Should an investor wish to gain exposure to commodity price movements it


can do so in three ways:

Invest in the underlying commodity (or basket of commodities)


Commodity derivatives (on either single commodities or an index) which
are widely traded on major exchanges such as Euronext.liffe and the
Chicago Mercantile Exchange.

Page 7

Subject ST5 (Finance and Investment Specialist Technical A) April 2013 Examiners Report

(v)

Invest in companies whose share price is influenced by commodity prices


(such as oil and mining companies).

Invest in a commodity fund offered by an investment manager.

Comment on management costs and skills, minimum bargain size, scope for
diversification, basis risk with derivatives, volatility, liquidity and physical
settlement / storage / shipping / transportation.
Holding individual contracts introduces risk of being delivered against .
Disadvantages if companies are used as a proxy for commodity investment:

It is unlikely that there will be exposure to just one commodity.

The companys management may alter the exposure via acquisitions or


disposals or by hedging its position .

The companys share price may be influenced by other factors.

The company will incur various operating expenses which will dilute
overall return.

Use of commodity shares (mining, exploration companies) gives less


diversification from equity market than physical would.

Investment management commodity fund


Advantage: access to specialist management, liquid, small universe, etc.
Disadvantage: High management fee, excessive sector concentration.

(i)

The formula to obtain a total return index at time t is:


TRI ( t ) = TRI ( t 1)

I (t )
I ( t 1) [ XD ( t ) XD ( t 1)]

where TRI(t) is the total return index, I(t) is the capital index at time t,
XD(t) is the value of the accumulated XD adjustment at time t.

Page 8

(ii)

TRI (Day 2) = 4080.63 5774.20 / (5857.52 [170.85 168.82])


= 4023.98

(iii)

Dividend yield = (170.85 168.82) / 5857.52


= 0.0347% per day, or 13.5% p.a.

Subject ST5 (Finance and Investment Specialist Technical A) April 2013 Examiners Report

(i)

Advantages:
The simplified approach reduces the likelihood of contributors making poor
savings choices. This is particularly the case for less sophisticated
contributors, who might be overwhelmed by the large ranges of funds offered
by competitors.
There are significant cost savings under such an approach, due to low
distribution costs and by eliminating third party asset management (e.g.
specialist funds, ETFs, hedge funds etc.). The transparent charging structure
is likely to be attractive as is the absence of any penalty if funds are
subsequently transferred to another provider.
It is likely that investors will need less advice to manage their pension plan.
The diversified growth fund permits access to expert investors who can
allocate to new asset classes, apply tactical asset allocation skills and stock /
sector selection skills.
Not all of these asset classes may be available on competitors platforms.
Disadvantages:
Some potential customers will prefer to invest with a more traditional provider
who operates a number of different investment funds that they can choose to
invest in.
There is a high level of trust required that the provider will invest assets
appropriately, and have strong investment capability in all asset classes, and
be able to add value through access to expert investment views (TAA, alpha,
new investment strategies, etc.), since it is not possible to select an alternative
asset allocation.
The provider has no track record for these funds in their current form.
The customer will need to be confident that the low fee scale will be
maintained over time and that other expenses and costs will be managed
appropriately.
Customers will need to be confident that the providers approach will scale,
otherwise active returns may weaken as assets under management grow.

(ii)

So long as the provider has a good understanding of liquidity requirements,


inflows and outflows, a diversified approach is straightforward to manage.
Significant outflows can cause particular difficulties due to 40% of the assets
being highly illiquid, with corporate bonds having poorer liquidity than
equities.

Page 9

Subject ST5 (Finance and Investment Specialist Technical A) April 2013 Examiners Report

In the event of significant disinvestments this is likely to lead to the asset


allocation moving above its desired allocation to illiquid assets.
A further complication is that with customers able to invest and disinvest
according to their own choice, it may be difficult to fairly allocate returns on
long-dated investments (e.g. private equity and infrastructure) to customers.
Sudden inflows may mean that assets are held in cash pending identification of
suitable investment opportunities. This is unlikely for equities and investment
grade corporate bonds but is a possibility for alternative assets, high yield and
asset backed credit.
Finally, the provider will need to ensure that its investment process scales
appropriates as assets under management grow. This is particularly the case
with regards to identification of alpha opportunities, new asset classes and
tactical asset allocation shifts.

(i)

If R1 and R2 are the zero-coupon rates for maturities T1 and T2


respectively, and RF is the forward interest rate for the period between
T1 and T2, then
RF =

(ii)

R 2T 2 R1T1
T 2 T1

1 = 0.05, 2 = 0.055, T1 = 0.25, T2 = 0.5


RF = 0.06
The quarterly- compounded rate is then given by 4[e0.06 0.25 1] = 0.06045
The value, V, of a FRA where it is specified that an interest rate RK will
be earned for the period of time between T1 and T2 on a specified principal of
L can be evaluated as:
V = L(RK RF) (T2 T1) e R 2T 2
So V

= 1,000,000 (0.07 0.06045) 0.25 e0.055 0.5


= $ 2322.74

END OF EXAMINERS REPORT

Page 10

INSTITUTE AND FACULTY OF ACTUARIES

EXAMINATION
25 September 2013 (am)
Subject ST5 Finance and Investment
Specialist Technical A
Time allowed: Three hours
INSTRUCTIONS TO THE CANDIDATE
1.

Enter all the candidate and examination details as requested on the front of your answer
booklet.

2.

You have 15 minutes before the start of the examination in which to read the
questions. You are strongly encouraged to use this time for reading only, but notes
may be made. You then have three hours to complete the paper.

3.

You must not start writing your answers in the booklet until instructed to do so by the
supervisor.

4.

Mark allocations are shown in brackets.

5.

Attempt all eight questions, beginning your answer to each question on a separate
sheet.

6.

Candidates should show calculations where this is appropriate.


AT THE END OF THE EXAMINATION

Hand in BOTH your answer booklet, with any additional booklets firmly attached, and this
question paper.
In addition to this paper you should have available the 2002 edition of the Formulae
and Tables and your own electronic calculator from the approved list.

ST5 S2013

Institute and Faculty of Actuaries

11

(i)

Outline the characteristics of the following asset classes:


(a)
(b)
(c)
(d)
(e)

Bond repo
RPI swap
Currency Coupon swap
Dividend swap
Volatility swap
[8]

12

(ii)

Discuss the risks that arise from investing in RPI swaps and how these can be
mitigated.
[5]
[Total 13]

(i)

State the principal aims of financial regulation.

[2]

Following a financial crisis in Actuaria, a significant proportion of the blame for the
crisis was attributed to overfriendly regulation by the Financial Regulator. In
response to this the regulatory regime has been made significantly stricter and less
friendly.

13

(ii)

Discuss the possible implications of this change in regime for the financial
markets in Actuaria.
[10]
[Total 12]

(i)

Compare the relative advantages and disadvantages of top down and


bottom up management techniques.

[6]

A wealthy individual has decided to set up an investment management company


BigDosh Management. The company will try to attract investors seeking alternative
sources of return.
You have been employed as a strategic advisor to BigDosh. As part of your role you
have been asked to advise on the following investment strategies:
(a)
(b)
(c)
(d)
(ii)

ST5 S20132

fund focused on mining stocks


fund investing in classic cars
fund exploiting anomalies in exchange rates
fund investing in fitness centre shares

Discuss which of the management techniques outlined in part (i)


would be most suited to each investment strategy.

[12]
[Total 18]

14

15

(i)

Describe the role and functions of a custodian.

[5]

(ii)

Outline the advantages of a Central Securities Depository.

[3]

(iii)

Describe some of the issues that might be encountered in moving from


an environment where the use of a regulated custodian is voluntary to
one where it is mandatory for all collective investments.
[4]
[Total 12]

The authorities of an emerging country are to allow its citizens to invest in non-cash
assets for the first time. However, the authorities are concerned by the lack of
investment knowledge amongst its citizens and therefore want to impose restrictions
on the type of investments that can be held.
Suggest the type of restrictions the authorities might impose.

16

(i)

[6]

State the formula for calculating the total return on a gilt index for an investor
subject to income tax, defining all the symbols used.
[3]

You have been given the following data:

(ii)

ST5 S20133

Day

Capital
index

Total return
index

XD adjustment

Accrued
interest

1
2

172.52
171.86

2797.01

168.82
170.85

1.892
1.904

Calculate the value of the total return index for Day 2. Assume that the rate of
tax is 20%.
[2]
[Total 5]

PLEASE TURN OVER

17

(i)

Explain the difference between a clean and a dirty bond price.

[2]

You have been given the following data in respect of a bond:


Coupon rate p.a.
No. of coupon payments p.a.
Redemption date
Redemption value
Settlement date
Net redemption yield
(ii)

5%
2
31 December 2014
102
30 October 2013
6% p.a. nominal

Calculate the dirty price of the bond for a taxpayer subject to 30% tax on
income.

[6]

The same issuer has another bond with similar liquidity and the same maturity date,
but where the issuer has the option 1 month before 31 December 2014 to extend the
term of the bond by a further 5 years at the same coupon rate.

18

(iii)

Explain how the price of this second bond would differ from the price of the
first bond.
[2]

(iv)

Set out the circumstances in which the price of the second bond will be
volatile relative to the price of the first bond near to expiry.
[4]
[Total 14]

(i)

State the reasons for performance measurement.

[3]

A pension fund has employed a single investment manager, WLM, to manage


assets on their behalf. WLM have split the portfolio between two portfolio managers.
One portfolio manager is focused on generating returns from sector decisions, the
other portfolio manager maintains sector neutrality, and generates returns through
stock selection.
WLM is assessed against the following benchmark, which is rebalanced at the end of
each period:
Industrials
Utilities
Financials

ST5 S20134

35%
25%
40%

Sector
Manager
Start
Period 1
Industrials
Stock A
Stock B
Sector benchmark

25%

Utilities
Stock C
Stock D
Sector benchmark

25%

Financials
Stock E
Stock F
Sector benchmark

50%

Stock
Manager
(Value
$m)

Return
Period 1

Sector
Manager
Start
Period 2

Return
Period 2

30%
$15
$20

10%
15%
12%

$10
$15

12%
6%
10%

$20
$20

20%
2%
15%

10%
20%
15%
30%
10%
15%
10%
40%
2%
2%
2%

Assumptions

The sector manager achieves the benchmark return in each sector.

The stock selection manager invests in line with the sector allocation benchmark
and rebalancing takes place at the start of each period.

Both managers are given $100 million to invest at the start of the period.

(ii)

Calculate and show outperformance or underperformance for the following:


(a)

total portfolio return compared to total benchmark return, for period 1,


period 2 and the two periods combined

(b)

portfolio return compared to benchmark return for period 1 only:

Industrials sector
Utilities sector
Financials sector

(c)

attribution from stock performance at the total fund level for period 1,
period 2 and the two periods combined

(d)

attribution from sector performance at the total fund level for period 1,
period 2 and the two periods combined

(Show all workings.)


(iii)

Comment on the relative performance of the two strategies.

END OF PAPER
ST5 S20135

[15]
[2]
[Total 20]

INSTITUTE AND FACULTY OF ACTUARIES

EXAMINERS REPORT
September 2013 examinations

Subject ST5 Finance and Investment


Specialist Technical A

Introduction
The Examiners Report is written by the Principal Examiner with the aim of helping
candidates, both those who are sitting the examination for the first time and using past papers
as a revision aid and also those who have previously failed the subject.
The Examiners are charged by Council with examining the published syllabus. The
Examiners have access to the Core Reading, which is designed to interpret the syllabus, and
will generally base questions around it but are not required to examine the content of Core
Reading specifically or exclusively.
For numerical questions the Examiners preferred approach to the solution is reproduced in
this report; other valid approaches are given appropriate credit. For essay-style questions,
particularly the open-ended questions in the later subjects, the report may contain more points
than the Examiners will expect from a solution that scores full marks.
The report is written based on the legislative and regulatory context pertaining to the date that
the examination was set. Candidates should take into account the possibility that
circumstances may have changed if using these reports for revision.
D C Bowie
Chairman of the Board of Examiners
January 2014

Institute and Faculty of Actuaries

Subject ST5 (Finance and Investment Specialist Technical A) Examiners Report, September 2013

General comments on Subject ST5


Candidates are reminded of a bias in the paper towards recognising higher level skills and
practical application this is intentional and will continue. Likewise the examination system
does properly allow for prior subject knowledge to be assumed. Investment is a necessarily
practical subject and, at this level, the examiners expect candidates to demonstrate a breadth
and depth of competency as would be expected from a senior student in a frequently evolving
discipline. Hence simple regurgitation of bookwork will never be sufficient to ensure a Pass
grade and this was evident from the dispersion of candidates responses in the more
differentiating questions.
Whilst the examiners will tolerate bullet point style responses, handwriting that is too poor to
assess will lose marks. Likewise text speak abbreviations will not be accepted.
Comments on the September 2013 paper
This paper showed a welcome improvement in the quality of answers, with a corresponding
higher success rate than in recent diets. It is hoped that this will continue in the future.
Comments on individual questions are incorporated in the solutions below.
Many questions represented opportunities to demonstrate higher level skills in terms of nonstandard/practical application of theory to current or unusual issues in investment. Most
candidates seemed to identify and understand the key issues being examined and so
appreciated the general content of solutions that the examiners were looking for however
those that were unsuccessful will find their solutions lacked sufficient (and often the most
basic) detail or application of knowledge and scored lower accordingly. Thus, weaker
candidates found difficulties with the later parts of Questions 11, 12, 13, 14 and 17.
Whilst some candidates are too narrow in their responses, a greater number still deviate from
the topic and include irrelevant material or over emphasise minor points although
candidates will not be explicitly penalised for this, it gives an impression of a lack of
understanding and, more importantly, wastes limited time. Time and priority management
are key skills actuaries need to have.
Where candidates made relevant points in other parts of their solutions, the examiners have
used their discretion as to whether to recognise these answers or not. Likewise the examiners
share and agree alternative possible solutions to questions alongside the approach outlined
below.

Page 2

Subject ST5 (Finance and Investment Specialist Technical A) Examiners Report, September 2013

11

(i)

(a)

A repo (or repurchase agreement) is a form of secured lending


whereby an investor buys stock from a dealer who, in turn, agrees to
buy the stock back again at a later date at an agreed price. Such deals
are of a short-term nature. The repo market is very liquid.

(b)

RPI swaps (swapping fixed rate for index return).

(c)

Currency coupon swaps. (Exchanging a fixed interest rate in one


currency for a floating interest rate in another currency. This is a
combination of an interest-rate swap and a currency swap.)

(d)

Dividend swaps (exchanging the dividends received on a reference


pool of equities in return for a fixed rate).

(e)

Volatility swaps (exchanging a fixed rate in return for the experienced


volatility of price changes of a reference asset).

General characteristics of swaps were also credited (once) under (b)(e):

illiquid
principal not exchanged
OTC deals that introduce counterparty risk

While most candidates demonstrated a reasonable knowledge of these asset classes, many did
not recognise that one leg of the swap contracts provides a fixed return.
(ii)

Risks
Losses due to the counterparty defaulting
Liquidity risks when trying to disinvest
Rapid changes in interest rates leaving investors out of the money and unable
to make collateral call
Cross-hedging risk
Basis risk
Risk of changes to the (external) index used as the basis for the swap
Mitigation
Only deal with high quality counterparties
Regular credit review of counterparty exposure
Diversification of counterparties
Regular collateral call to make sure that cover any in-the-money amounts
Invest only in more liquid end of RPI swaps curve
Ensure adequate modelling carried out to understand maximum collateral
requirements.

Some candidates suggested incorrectly that RPI swaps could be exchange-traded, with
clearing house credit risk cover. Similarly, any suggestion that credit default swaps could be
arranged were not awarded marks.

Page 3

Subject ST5 (Finance and Investment Specialist Technical A) Examiners Report, September 2013

12

(i)

The principal aims of regulation are:

to correct market inefficiencies and to promote efficient and orderly


markets
to protect consumers of financial products
to maintain confidence in the financial system
to help reduce financial crime

Well answered by most candidates


(ii)

The financial markets will probably benefit from tighter regulation but the
extra regulation will incur additional cost.
The new regulation is likely to reduce the size or at least the growth of the
financial industry due to financial services companies moving to more
friendly countries.
The tighter regulation could be considered to be prudent but all virtues taken
to extremes become vices.
The new regulation is likely to hinder new entrants and probably reduce levels
of competition.
The new regulation may result in some forms of moral hazard effect on the
financial industry.
The new regime may inhibit the creation of new financial products.
The new regime may create a competitive disadvantage for local companies
selling on international or foreign markets.
Improvement in reputation for prudence of local financial industry.
The regime may possibly reduce any information asymmetry in the market.
It may include a switch from Principles based regulation to Rules based
currently most financial markets have a significant emphasis on principles
based regulation, more rules based regulation may moderate this somewhat.
One of the main rules in political economics is not to shock the system a
significant change from friendly to not very friendly regulation could be
considered to be a shock.

The question asked for possible implications of this change in regime. Many candidates
wasted time by exploring potential details of the new regulations.

Page 4

Subject ST5 (Finance and Investment Specialist Technical A) Examiners Report, September 2013

13

(i)

Advantages of top down


Top down approach is generally better for controlling risk of portfolio (e.g. by
the use of load ratios or load differences) as it offers a more balanced
diversified portfolio than bottom up.
Can be argued that more return comes from differences in asset allocation
rather than stock selection.
Top down offers a big picture analysis of markets rather than getting lost in
individual stocks as bottom up does.
Top down strategies can be implemented using cheap instruments such as
futures rather than owning physical securities compared to bottom up.
Top down better for ensuring that liabilities are matched.
Advantages of bottom up
Bottom up focuses on individual stocks so seek companies with stronger
returns rather than just holding a selection of stocks in a portfolio.
Bottom up allows more focus on absolute return investing rather than
following more market returns in a top down approach.
Generally more sustainable than top down, less influenced by external factors
that are hard to predict.

Generally well answered.


(ii)

(a)

Mining stocks
Investment strategy focussed on one sector although the mines could
be focussed on different areas, such as consumables such as coal or
iron or areas such as gold and diamonds.
The best investment strategy would involve both a top down and
bottom up strategy. It is important to focus on economic factors that
will drive demand for the various mined products, i.e. is there likely to
be higher demand for gold.
Having used top down to look at mining sectors that look most
favourable then use bottom up to select the best stocks in each sector.

(b)

Investing in classic cars


With an area as bespoke as classic cars the real return on the portfolio
will be the cars chosen as an investor would gain very little benefit
from analysis on economic conditions as this will impact the market as
a whole.
Page 5

Subject ST5 (Finance and Investment Specialist Technical A) Examiners Report, September 2013

Therefore, bottom up approach seems most appropriate to choose the


right cars that will generate the return.
(c)

Fund exploiting differences in exchange rates


The factors that drive the differential in interest rates are economic
factors not company specific factors.
Therefore, top down analysis would be used to determine pricing
differentials and potential returns.

Many candidates discussed the efficiency of foreign exchange markets and the fact that
anomalies would quickly disappear, but still recommended a bottom up approach despite
this.
(d)

Fund investing in fitness centre shares


This is a specific sector of the market so it would be expected that the
focus would be on stock picking so bottom up analysis.
Although it would be a strongly bottom up approach there will be a top
down element to it as the fund manager will need to decide which
countries will offer the higher returns in the fitness sector industry.
Share prices of fitness centres are influenced by the amount of
disposable income of people and also the perception of fitness in each
country (both of which are top down factors).

Generally well answered.

14

(i)

The principal function of custody is to ensure that financial instruments are


housed under a proper system that permits investment for proper purposes
with proper authority.
The custodian is thereby able to account independently for any financial
transactions.
Custodians are usually banks or other regulated institutions. Fund management
firms, who would at one time have included custody within their function, are
increasingly outsourcing such activities. Custodians offer not only custody of
documents but also a range of services such as:

Page 6

income collection
tax recovery
cash management
securities settlement
foreign exchange
stock lending

Subject ST5 (Finance and Investment Specialist Technical A) Examiners Report, September 2013

Also, the custodian will often exercise voting rights on behalf of the manager
or trustees. However, the custodian has no duty to investigate the propriety of
instructions which appear to be in order (unless a specific monitoring function
has been agreed).
Administration of investment activities in overseas markets is often a vital
element of the custodians role. Although the basic process of acquiring and
disposing of an asset is essentially the same the world over, processes are very
different in different markets. The infrastructure, payments system, clearing
house and banking / settlement arrangements in many markets place the
investor at varying degrees of risk.
Generally well answered.
(ii)

Eliminates one link in the settlement chain making process more efficient.
Eliminates the potential for sub-custodian error.
Provides securities lending and borrowing.
Carry out repo settlement.
Reduces administration.

Many candidates struggled to identify relevant points.


(iii)

It is likely that prior to the requirement that many investors, particularly larger
institutions, will maintain their own custody and record keeping functions as
internal departments.
Smaller investors will use third party providers as they lack the scale and
internal expertise to operate such functions in-house.
To move to the new environment will mean that internal teams will need to
seek regulatory approval, or that the activity they carry out will need to move
to external firms.
This will be disruptive in the short-term during the transitional period and
result in setup costs for investors, and potentially a higher ongoing cost.
It is likely that both the regulator and institutions will need a considerable
period to achieve a smooth transition.
The introduction of regulation may result in moral hazard (e.g. lack of
scrutiny).
There may not be sufficient numbers of regulated custodians available.

Again, many candidates found difficulty in applying their knowledge to this scenario.

Page 7

Subject ST5 (Finance and Investment Specialist Technical A) Examiners Report, September 2013

15

Maximum that could be held in non cash assets


Maximum that could be held in non domestic assets
They might prohibit investment in a particular asset class completely
Might impose the maximum that can be held in any one security or asset class/sector
Prohibit the use of some asset classes, such as using derivatives for speculation rather
than hedging or the use of private or unquoted equities.
Impose various liquidity constraints
Ethical or social limitations
Compulsory investment in registered investment schemes (which themselves have
strict guidelines such as well diversified portfolios and risk controls).
Require minimum levels of wealth or cash before non-cash assets can be used.
Restrict short selling.
Require minimum holding / investment period.

Generally well answered, although some candidates suggested standard points such as
requiring the use of financial advisers or using tax systems to encourage specific investments.
The question specifically asked for restrictions on the type of investments that can be held,
so these answers were not given credit.

16

(i)

The return over a given period for an investor subject to income tax is:

I 2 I1 (1 T )( XD2 XD1 ) T ( ACC2 ACC1 )


I1
where I1, XD1 and ACC1 are the index number, the ex-dividend adjustment to
date and the accrued interest respectively at the beginning of the period.
Similarly I2, XD2 and ACC2 are the respective figures at the end of the period,
T is the rate of tax.
The formula for the equity total return index
TRI (t ) TRI (t 1)

I (t )
I (t 1) [ XD(t ) XD (t 1)]

was awarded half marks (since the question referred specifically to a gilt index).

Page 8

Subject ST5 (Finance and Investment Specialist Technical A) Examiners Report, September 2013

(ii)

Return on day 2

= [(171.86 172.52) + 0.8 (170.85 - 168.82)


0.2 (1.904 1.892)] / 172.52
= 0.005574

TRI(2)

17

(i)

= 2797.01 (1 + 0.005574) = 2812.60

The clean price is the price of a bond excluding any interest that has accrued
(since issue or the most recent coupon payment).
The dirty price is the price of a bond including the accrued interest.
Thus clean price = dirty price accrued interest.

The treatment of accrued interest must be explicitly specified.


Generally well answered.
(ii)

The dirty price of bond per 100 face value is found by discounting the net
coupons and the redemption amount. No adjustment is made to exclude
accrued interest.
So, dirty price = 102 / 1.0715 + 0.7(2.5 / 1.0100 + 2.5 / 1.0403
+ 2.5 / 1.0715)
= 95.1919 + 0.7 (2.4752 + 2.4031 + 2.3331)

= 100.24
Any suitable day count was acceptable in determining the discount factors.
Some candidates struggled with this basic evaluation. Errors encountered included
calculating the coupon using the redemption value, omitting to net down coupons for tax,
applying the income tax charge to the redemption value and failing to allow for the frequency
of coupons.
(iii)

The investor has sold an option to the issuer.


Investors should assume that the issuer will exercise their right if this is
beneficial to the issuer.
Since the bond is identical in all other respects, this bond will have a lower
market value than the original bond.

Many candidates failed to recognise that the issuer had the option to extend.
(iv)

The difference in price between the two bonds will reflect the likelihood of the
option being exercised (driven by interest rate volatility and time to expiry)

Page 9

Subject ST5 (Finance and Investment Specialist Technical A) Examiners Report, September 2013

and the value to the issuer in exercising the option (driven by the forward
interest rate relative to 5%)
Therefore near to expiry the main reason why the price differential would be
volatile is due to high interest rate volatility, and the forward interest rate
being close to 5% (the interest rate applying to the extension).
Supply / demand for each bond.
Threat of changes in future tax treatment (in the extension period).
Some candidates had difficulty applying the basic Black-Scholes assessment to the
scenario given.

18

(i)

To improve future performance


Comparison of the rate achieved relative to a target rate of return
Comparison of performance against other portfolios or index

Well answered.
(ii)

Calculate and show outperformance or underperformance for:


(a)

total portfolio return compared to total benchmark return

Benchmark return
Total portfolio return
outperformance
(b)

Combined
22.34%
22.00%
0.34%

Period 1
12.5%
12.0%
0.5%
9.2%
10.0%
0.8%
13.2%
15.0%
1.8%

attribution from stock performance at the total fund level


Total Fund stock

Page 10

Period 2
8.55%
8.93%
0.38%

portfolio return for each sector compared to benchmark sector return


Sector return
Industrials
Benchmark
outperformance
Utilities
Benchmark
outperformance
Financials
Benchmark
outperformance

(c)

Period 1
12.70%
12.00%
0.70%

Period 1
0.85%
12.85%

Period 2
0.50%
8.42%

Combined
0.35%

Subject ST5 (Finance and Investment Specialist Technical A) Examiners Report, September 2013

(d)

attribution from sector performance at the total fund level


Total Fund sector

Period 1
0.15%

Period 2
0.13%

Combined
0.02%

Some candidates failed to recognise that both managers contributed to stock performance.
The details of the calculations are set out in the attached schedule.

Page 11

Subject ST5 (Finance and Investment Specialist Technical A) Examiners Report, September 2013

(a)

Portfolio return
Sector portfolio
0
1

Stock portfolio

100

Total

100

200

(15 1.1) + (20 1.15) + (10 1.12)


+ (15 x 1.06) + (20 x 1.2) + (20 x 1.02) = 111.0

100 (0.25 1.12 + 0.25 1.10 + 0.50 1.15)


= 113.0

111.0 (0.15 1.1 + 0.2 1.2 + 0.1 1.1


+ 0.15 1.15 + 0.2 0.02 + 0.2 0.02) = 121.6

113 (0.3 1.15 + 0.3 1.1 + 0.4 1.02)


= 122.4

+12.0%

224.0
+ 8.93%

244.0
+ 22.0%

Benchmark return
Period 1
0.35 0.12 + 0.25 0.1 + 0.4 0.15 = 12.7%
Outperformance

(b)

Period 2
0.35 0.15 + 0.25 0.1 + 0.4 0.02 = 8.55%

Period 1
12.0% 12.7% = 0.7%

Period 2
8.93% 8.55% = 0.38%

Period 1
Benchmark

Diff

Industrials

(15 1.1 + 20 1.15 + 25 1.2) /60


= 67.5 / 60 = 12.5%

12.0%

+0.5%

Utilities

(10 1.12 + 15 1.06 + 25 1.1) / 50


= 54.6 / 50 = 9.2%

10%

0.8%

Financials

(20 1.2 + 20 1.02 + 50 1.15) / 90


= 101.9 / 90 = 13.2%

15%

1.8%

Stock attribution
Page 12

= +22.34%

+0.34%

Subject ST5 (Finance and Investment Specialist Technical A) Examiners Report, September 2013

Average sector weighting sector benchmark performance


Period 1
((25 + 35)/200 1.12 + (25 + 25)/200 1.1
+ (50 + 40) / 200 1.15) 1 = 12.85%

Period 2
((30 + 35) / 200 1.15 + (30 + 25)/200 1.1
= (40 + 40) / 200 1.02) 1 = 8.43%

Attribution from stock performance 12% 12.85% = -0.85%

(d)

Sector attribution balance of difference


0.7% (0.85%) = +0.15%

8.93% 8.43% = +0.5%

0.38% 0.50% = 0.13%

Page 13

Total

0.35%

+0.02%

Subject ST5 (Finance and Investment Specialist Technical A) Examiners Report, September 2013

(iii)

In period one the portfolio underperformed the benchmark. This was driven by
stock underperformance, especially Stock F. Sector performance provided a
positive contribution in period one but was not enough to make up for
underperformance of stock selection.
In period two the stock performance was positive whilst the sector
performance was negative. Overall, the stock performance outweighed the
sector underperformance which lead to positive return to benchmark.
For the total period the sector manager was in line with benchmark
performance whereas the stock manager underperformed the benchmark.
Overall, the sector manager was more successful than the stock manager over
the period, although the fund would have performed better in a passive
strategy.

Credit was given for any relevant comments based on the candidate's answer to part (ii).

END OF EXAMINERS REPORT

Page 14

INSTITUTE AND FACULTY OF ACTUARIES

EXAMINATION
24 April 2014 (am)

Subject ST5 Finance and Investment


Specialist Technical A
Time allowed: Three hours
INSTRUCTIONS TO THE CANDIDATE
1.

Enter all the candidate and examination details as requested on the front of your answer
booklet.

2.

You have 15 minutes before the start of the examination in which to read the
questions. You are strongly encouraged to use this time for reading only, but notes
may be made. You then have three hours to complete the paper.

3.

You must not start writing your answers in the booklet until instructed to do so by the
supervisor.

4.

Mark allocations are shown in brackets.

5.

Attempt all eight questions, beginning your answer to each question on a new page.

6.

Candidates should show calculations where this is appropriate.


AT THE END OF THE EXAMINATION

Hand in BOTH your answer booklet, with any additional sheets firmly attached, and this
question paper.
In addition to this paper you should have available the 2002 edition of the Formulae
and Tables and your own electronic calculator from the approved list.

ST5 A2014

Institute and Faculty of Actuaries

An investor has an actively managed bond portfolio currently comprising 50% US


government bonds and 50% US investment grade corporate bonds.
The portfolio has specified duration targets for each type of bond and there are
various other restrictions on the bonds that can be held.
The bond manager is benchmarked against a broad market index comprising all US
denominated investment grade bonds (government, supranational, agency, corporate),
and is targeted with outperforming this index by 1% p.a. net of fees.
The manager charges a fee of 0.20% p.a. plus a performance fee of 10% of returns in
excess of the 1% target.
Describe the implications of this mandate.

(i)

(a)

Define the beta of a portfolio.

(b)

State the performance implications of a portfolio with a beta of 2.

[8]

[2]
An equity portfolio aims to have a beta of 1.5.
(ii)

Outline the investigations that could be performed to determine the value


added by the fund manager to the portfolio, assuming that full data is
available.

[3]

(iii)

List six reasons why the performance of the portfolio might differ from that of
the benchmark index.
[3]
[Total 8]

(i)

List the factors that investors should consider in determining the impact of tax
on an investment.
[3]

A major economy has decided to simplify its tax system by taxing all income (earned,
unearned or gifts) and realised gains and losses at a single rate of 15%. Each citizen
will have an annual personal tax-free allowance of $10,000. Interest payments on
personal borrowings may be offset against income, but all other allowances will
disappear. Activities and investment schemes that previously enjoyed favourable tax
treatment will now be taxed at standard rates.
(ii)

ST5 A20142

Describe the potential impacts on the personal investment marketplace.


[7]
[Total 10]

The following information relates to the performance of two investment trusts and
their equivalent benchmark index over a three year period. The annual risk-free rate
of return over this period was 4% per annum.
Trust A
Annual return (% p.a.)
Standard deviation (% p.a.)
Correlation coefficient with index

9.0
13.5
0.36

Trust B

Index

8.0
9.5

7.0
6.5

0.75

1.00

(i)

Calculate four different risk adjusted performance measures for each trust. [8]

(ii)

Comment on the results from part (i), stating any limitations that apply to
them.
[4]
[Total 12]

The manager of a global inflation-linked bond fund valued at US$5 billion wishes to
alter the country allocation, switching all US$2 billion that is currently invested in the
UK market to the US market.
(i)

Describe the problems (and the costs) that would be encountered in a switch of
this size.
[6]

After further analysis, it is the expectation that the allocation will be partially reversed
in three to six months time to reallocate US$1.5 billion to the UK.
(ii)

ST5 A20143

Discuss the advantages and disadvantages of using total return swaps rather
than a physical switch for this combined asset allocation change (i.e. the
immediate switch and the planned future reallocation).
[7]
[Total 13]

PLEASE TURN OVER

(i)

State the main factors affecting equity prices.

[2]

An individual has recently received an inheritance from a family member and has
decided to invest in the equity market for the first time. They have approached a
financial adviser to provide guidance on the individual equities in which they should
invest. The individual wishes to have exposure in their portfolio to companies which:

are large.
are risky.
have a high dividend yield.
have a well-known brand name.
operate globally.
move ahead of the trade cycle.
have volatile profits.
have high gearing.

(ii)

(i)

(a)

Identify five equity sectors which would ensure that all of the
characteristics above are covered.

(b)

State, for each of these sectors, the characteristics from the above list
which are satisfied.

(c)

State any additional characteristics that the holdings in each sector


might have.
[11]
[Total 13]

State the main aims of financial regulation.

[2]

In the middle of a deep recession and with the financial services sector in distress,
Bank A acquired Building Society B and performed due diligence on the transaction
using its own internal team of financial analysts. The statutory regulator subsequently
discovered that Building Society B held a large amount of bad debt on its books
which the bank was unaware of. This resulted in the need for a significant capital
injection into Bank A in order for it to remain solvent.
(ii)

ST5 A20144

Show how diligent application of the key principles underlying the financial
services legislative framework can help to avoid this type of problem.
[12]
[Total 14]

An intern at an asset management company has been given a project to write an


investment report on a fast growing on-line fashion company which was founded less
than 12 months ago.
(i)

Outline the research that the intern could undertake including:

the factors on which he should focus.


the data he would use.
[6]

(ii)

Discuss issues that he might face when completing the investment report and
any potential solutions to these issues.
[10]

(iii)

Compare the similarities and differences in the approach if the analysis was
for a credit rating agency rather than an asset management company.
[6]
[Total 22]

END OF PAPER

ST5 A20145

INSTITUTE AND FACULTY OF ACTUARIES

EXAMINERS REPORT
April 2014 examinations

Subject ST5 Finance and Investment


Specialist Technical A
Introduction
The Examiners Report is written by the Principal Examiner with the aim of helping
candidates, both those who are sitting the examination for the first time and using past papers
as a revision aid and also those who have previously failed the subject.
The Examiners are charged by Council with examining the published syllabus. The
Examiners have access to the Core Reading, which is designed to interpret the syllabus, and
will generally base questions around it but are not required to examine the content of Core
Reading specifically or exclusively.
For numerical questions the Examiners preferred approach to the solution is reproduced in
this report; other valid approaches are given appropriate credit. For essay-style questions,
particularly the open-ended questions in the later subjects, the report may contain more points
than the Examiners will expect from a solution that scores full marks.
The report is written based on the legislative and regulatory context pertaining to the date that
the examination was set. Candidates should take into account the possibility that
circumstances may have changed if using these reports for revision.
D C Bowie
Chairman of the Board of Examiners
July 2014

Institute and Faculty of Actuaries

Subject ST5 (Finance and Investment Specialist Technical A) April 2014 Examiners Report

General comments on Subject ST5


Candidates are reminded of a bias in the paper towards recognising higher level skills and
practical application this is intentional and will continue. Likewise the examination system
does properly allow for prior subject knowledge to be assumed. Investment is a necessarily
practical subject and, at this level, the examiners expect candidates to demonstrate a breadth
and depth of competency as would be expected from a senior student in a frequently evolving
discipline. Hence simple regurgitation of bookwork will never be sufficient to ensure a Pass
grade and this was evident from the dispersion of candidates responses in the more
differentiating questions.
Whilst the examiners will tolerate bullet point style responses, handwriting that is too poor to
assess will lose marks. Likewise text speak abbreviations will not be accepted.
Specific comments on the April 2014 paper
Comments on individual questions are incorporated in the solutions below.
Many questions represented opportunities to demonstrate higher level skills in terms of nonstandard/practical application of theory to current or unusual issues in investment. Most
candidates seemed to identify and understand the key issues being examined and so
appreciated the general content of solutions that the examiners were looking for however
those that were unsuccessful will find their solutions lacked sufficient (and often the most
basic) detail or application of knowledge and scored lower accordingly. Thus, weaker
candidates found difficulties with Question 1 and the later parts of Questions 3, 7 and 8.
Whilst some candidates are too narrow in their responses, a greater number still deviate from
the topic and include irrelevant material or over emphasise minor points. Although
candidates will not be explicitly penalised for this, it gives an impression of a lack of
understanding and, more importantly, wastes limited time. Time and priority management
are key skills actuaries need to have.
Weaker candidates often fail to respond to the specific issues included in the question.
Instead, they regurgitate a generic answer based on the syllabus topic. More care needs to be
given to crafting answers that directly address the points raised in the question.
Where candidates made relevant points in other parts of their solutions, the examiners have
used their discretion as to whether to recognise these answers or not. Likewise the examiners
share and agree alternative possible solutions to questions alongside the approach outlined
below.

Page 2

Subject ST5 (Finance and Investment Specialist Technical A) April 2014 Examiners Report

There is a mismatch between the index used for measuring returns, and the investment
guidelines given to the manager.
This leads to an incentive for the manager to align the investment portfolio to the
index rather than the investors investment guidelines. This may not be the investors
expectation.
In some circumstances there could be significant deviations between the managers
target portfolio and the actual portfolio, since the restrictions will constrain the
manager. The manager will be uncomfortable with this as their active management
process will be constrained.
Particular examples of situations where the deviations could be significant include:

Composition of index differs markedly from a 50% government bond / 50%


corporate bond mix

In particular, the index includes supranational and agency bonds

Duration of index differs markedly from a 50% government bond / 50% corporate
bond mix

Different segments of the bond markets diverge in their returns (e.g. flight to
quality scenario benefiting government bonds, or a dash for trash benefiting
lower grade bonds)

The size of the portfolio will also influence ability to gain access to corporate bond
issues.
The performance fee strongly incentivises the manager to minimise risk relative to the
aggregate index, rather than the investors expected portfolio. Indeed, the need to
outperform the index by more than 1% to earn the performance fee may incentivise
the manager to take excess risk. This may be exacerbated by the relatively low
fixed fee.
For all these reasons, it would be preferable for the benchmark to be aligned more
closely to the investment guidelines or if this is not possible, to restructure the fee to
remove the performance fee.
Many candidates needed to give more attention to the impact of the fee structure on the
managers actions.

Page 3

Subject ST5 (Finance and Investment Specialist Technical A) April 2014 Examiners Report

(i)

(ii)

(a)

The Beta of a portfolio is a measure of the portfolios volatility relative


to movements in the whole market. It is usually defined as the
covariance of the return on the portfolio with the return on the market,
divided by the variance of the market return.

(b)

A beta of 2 means the change in value of the portfolio should be 100%


greater than the change in value of the market.

The performance of the portfolio would be compared to the return on the


index. The portfolios target return should recognise the pre specified level of
risk. Using an index representative of the market the portfolio is invested in,
target returns could be calculated as 1.5x the index return. Quarterly returns
for the portfolio could be compared to the quarterly returns on the target over,
say, a five year period. The excess return would indicate the level of value
added by the manager.
Explanations using the Jensen measure or Attribution analysis are valid
alternatives provided they reference a beta of 1.5.

(iii)

The performance will differ because the portfolio will be unlikely to hold
stocks and sectors in weights which are wholly representative of the index.

The portfolios beta over the period may have varied to levels significantly
above or below 1.5 affecting returns

The portfolio may have other objectives/constraints which effect


performance.

The diversification (or lack of it ) may affect volatility of portfolio returns;

The volume and dealing cost impact of trades in the portfolio

The effects of other expenses

The effects of cash flow

The impact of tax

Some securities in the benchmark are unmarketable, and cannot be held.

The benchmark may not have been available for a long enough period

This question was generally well answered.

Page 4

Subject ST5 (Finance and Investment Specialist Technical A) April 2014 Examiners Report

(i)

The total rate of tax on an investment.


How the tax is split between different components of the investment return.
The availability of personal allowances
The timing of tax payments.
Whether the tax is deducted at source or has to be paid subsequently.
The extent to which tax deducted at source can be reclaimed by the investor.
To what extent losses or gains can be aggregated between different
investments or over different time periods for tax purposes.

(ii)

Compared to the previous system, capital gains and income will be treated
equally in terms of the rate of taxation, although there will be some deferral of
taxation if capital gains are unrealised. Thus, there may be different effects
depending on the individual investors personal tax position and their
awareness of the impact of taxation.
Under the new regime, no specific savings wrapper (e.g. pension, insurance,
deposit) would be tax favoured. This may lead to behavioural changes and
disincentivise saving for long-term needs (e.g. retirement or care).
Due to a simplified tax system, it is likely that product designs will become
simpler and administration costs may fall. However, where a product has now
become taxable, additional features may be needed to attract customers.
With the only allowance being the annual personal allowance (covering all
sources of income), product sales are unlikely to have any strong seasonal
effects arising from a desire to use up allowances in the current tax year.
Managers will respond by restructuring existing products where possible, and
by launching new product designs to maximise demand. Some existing
investments will not be amenable to restructuring.
Individuals are likely to find borrowing relatively more attractive since interest
payments are deductible against savings or earned income.
The change to the taxation system may influence attitudes to overseas
investment.
Other valid points raised were given credit.

Many candidates did not focus sufficiently on the personal investment marketplace as
specified in the question. Instead, they wasted time discussing more general economic issues
(which were not required).

Page 5

Subject ST5 (Finance and Investment Specialist Technical A) April 2014 Examiners Report

(i)

First we have to calculate s of the two investment trusts


i

A =

Cov( Ri , Rm )
Vm

(0.36)(0.135)(0.065)
(0.065)

0.003159
0.004225

0.004631
0.004225

= 0.74769
B =

(0.75)(0.095)(0.065)
(0.065)

= 1.0962
Investment trust A
Treynor measure

0.09 0.04
0.74769

Sharpe measure

(0.09 0.04)
= 0.37037
0.135

Jensen measure

= 0.09 (0.04 + 0.74769 (0.07 0.04))

= 0.06687

= 0.02757
Prespecified SD

0.07 0.04

0.09 0.04 +
0.135
0.065

= 0.01231
Investment trust B
Treynor measure

0.08 0.04
= 0.03649
1.0962

Sharpe measure

0.08 0.04
= 0.42105
0.095

Jensen measure

= 0.08 (0.04 + 1.0962 (0.07 0.04))


= 0.007114

Page 6

Subject ST5 (Finance and Investment Specialist Technical A) April 2014 Examiners Report

Prespecified SD

0.07 0.04

0.08 0.04 +
0.095
0.065

= 0.00385
(ii)

Comments
(a)

On the basis of SD of return (Sharpe and Prespecified SD)


Trust B outperforms A.

(b)

On the basis of systemic risk (Treynor and Jensen)


Trust A outperforms B.

Limitations
(a)

The data is based only on 3 years. There is no guarantee that the same
will hold in future.

(b)

It is not known whether the returns are gross or net.

(c)

We have not considered the suitability relative to any liabilities.

(d)

The Treynor and Jenson measures are based on the validity of the
Capital Asset Pricing Model.

Generally well answered, although some candidates did not calculate the trust betas, but
rather used the correlation coefficients directly. Not all candidates addressed the limitations
in part (ii).

(i)

The main problems in an asset allocation change of this size are:

The possibility of shifting market prices (both on the sale of the existing
portfolio and the purchase of new assets).

The time needed to effect the change and the difficulty of making sure that
the timing of trades is advantageous.

The dealing costs (commission, dealing spreads, purchase taxes, etc.)


involved.

The possibility of the crystallisation of capital gains leading to a tax


liability.

These problems are particularly acute in the inflation-linked bond market due
to the relatively low liquidity of these bonds, both in the UK and the US
markets.

Page 7

Subject ST5 (Finance and Investment Specialist Technical A) April 2014 Examiners Report

This reflects that a large proportion of the bonds in issue are held by investors
as hedges against inflation-linked liabilities.
As both UK index-linked gilts and US Treasury Inflation-Protected Securities
have T +1 settlement cycles, it is possible (but unlikely) that there would be
significant out of market exposure .
(ii)

Total return swaps (TRS) can be helpful in a transaction of this nature for the
following reasons:
Dealing costs should be significantly mitigated.
There may be a tax advantage where there is no need to crystallise gains on
the portfolio being swapped.
Implementing a TRS should not cause asset prices to move.
A TRS on a large allocation can be executed quickly with a bank, unlike a
physical asset sale. Given the size of the switch involved here, this could be
significant.
Under a TRS the price of paying or receiving an asset return is transparent
(quoted as LIBOR plus or minus a spread). Therefore if paying one asset
return and receiving another asset return, it is very clear what the switch costs
are.
Conversely, with a physical switch, it is unclear what the transaction costs will
be for the return switch until it takes place. Thus, the use of TRS can be very
helpful from a portfolio management point of view.
Disadvantages
The main disadvantage of a TRS is its fixed term. Since we only have the
expectation that the swap will be amended in three to six months' time there is
the prospect that the TRS arranged will have to be rolled over or terminated
prematurely. To break a TRS mid-term can be expensive.
Additionally, it is not certain that a TRS will result in lower costs than a
physical switch, particularly if cash settled .
Counterparty risk is introduced, since the TRS will only deliver the required
cash flows if the counterparty honours its commitments. Given the size of the
switch involved here, this could again be a significant issue.
The requirement to provide collateral for a TRS is also a disadvantage.
It may not be possible to synthesise the underlying portfolio (as the TRS
probably based on an index).

Page 8

Subject ST5 (Finance and Investment Specialist Technical A) April 2014 Examiners Report

Generally well answered, but some candidates failed to appreciate the implications of the
fixed term swap contract. Rather, they stated that such contracts could be easily closed-out.
The specific points relating to inflation-linked bonds (in the US and the UK) were not
generally appreciated.

(i)

The price of an individual companys shares is affected by the level of supply


and demand for those shares. The key factors affecting relative demand for
individual shares are investors expectations of:

future dividend payments


future capital growth
the risks of the business and thus the uncertainty of estimates of the above

Factors that drive expectations for capital and dividend growth are estimates
of profits, free cash flow, and total enterprise value.
(ii)

Oil and Gas companies large companies, global and risky


Consumer goods companies brand names
Industrial companies profits move ahead of trade cycle, volatile profits
Utilities high dividend yield
Financials high gearing, volatile profits
Other sectors which could be used are:
Technology stocks risky, global
Consumer services volatile profits, brand names
Additional factors:
Oil and Gas commodity price dependent
Consumer goods capital intensive, low profit margin, high gearing
Industrials cyclical, dependent on government spending, high profits when
conditions good, low gearing, overseas exposure
Utilities natural monopolies, low growth, heavily government regulated
Financials labour costs, brand names
Consumer services poss. high gearing
There are other ways of structuring the portfolio and these were given credit.

Generally well answered.


Page 9

Subject ST5 (Finance and Investment Specialist Technical A) April 2014 Examiners Report

(i)

(ii)

The principal aims of regulation are:

to correct market inefficiencies and to promote efficient and orderly


markets

to protect consumers of financial products

to maintain confidence in the financial system

to help reduce financial crime

An important aspect of the legislative framework is to maintain confidence in


the financial system and to avoid a systemic financial collapse. To avoid a
collapse in this scenario, liquidity was provided to the bank.
Bank A should have observed high standards of integrity and fair dealing
when deciding to purchase Building Society B. If this had happened then the
transaction may not have completed.
A particular problem with this scenario was the banks decision to undertake
the due diligence itself. It appears that bank did not act with due skill, care
and diligence in considering whether to acquire the building society otherwise
the bad debts should have been identified.
In undertaking the due diligence itself, the bank was exposed to potential
conflicts of interest which could have been avoided. The bank may have
intended to manage these conflicts internally, but decision makers could have
interests in the transaction proceeding, despite the bad debt issue. The bank
should not unfairly place its interests above others, e.g. its customers and the
wider population who may ultimately need to bail out the bank.
Good corporate governance requires management to make decisions based on
the interests of relevant stakeholders rather than on their own personal
interests. It is possible that these bad debts were discovered but then not
disclosed. Full disclosure requirements may therefore have avoided the crisis.
Bank A should organise and control its internal affairs in a responsible manner
and keep proper records, so that this type of scenario could be avoided. Staff
involved with the transaction should be suitably qualified, adequately trained
and properly supervised. Well defined compliance procedures should be
embedded as part of day to day activity.
The fact that a capital injection was required shows that the bank was not fully
monitoring the risks it was exposed to. The bank should have ensured that it
maintained adequate financial resources to meet its investment commitments
and to withstand the risks to which it is faced.
The bank should be required to disclose all relevant information and be ready
to provide a full and fair account of the fulfilment of its responsibilities to
them. The regulator should expect the bank to deal with it in an open and co-

Page 10

Subject ST5 (Finance and Investment Specialist Technical A) April 2014 Examiners Report

operative manner and to keep it fully informed of anything concerning the


bank which may cause problems in the future. A strong regulator is likely to
be needed to ensure orderly markets and reduce fraudulent activity.
Discussion of the application of these principals to either Bank A or Building Society B was
given credit. Simply regurgitating the material in the Core Reading regarding financial
services legislation, without referencing the specifics of the question, was not sufficient.

(i)

Each company needs to be considered individually, but factors that will


generally be investigated include:

management ability
quality of products
prospects for market growth
competition
input costs
retained profits
history

Thus topics to be investigated include:

the financial accounts and accounting ratios


dividend and earnings cover
profit variability and growth (by looking at all sources of revenue and
expenditure)
the level of borrowing
the level of liquidity
growth in asset values
comparative figures for other similar companies

It will be necessary to use all the available sources of information. The


primary source is likely to be the companys published accounts but there are
many other sources of information which include:

the financial press and other commercial information providers


the trade press
public statements by the company
the exchange where the securities are listed
government sources of statutory information that a company has to
provide
visits to the company
discussions with company management
discussions with competitors
stockbrokers publications

Page 11

Subject ST5 (Finance and Investment Specialist Technical A) April 2014 Examiners Report

(ii)

When investigating a recently formed company, some of the factors in (i) will
not be available.
Not much info on current management ability look at their experience from
previous companies, if they have any.
Input costs difficult to have a good understanding as they have not been
running long enough to have stable costs. Look to compare against a similar
company with longer track record.
Retained profits dont have any so would need to model expected profits
stating assumptions.
Financial and commercial press given company is so young there is likely
not to be excessive information. Need to find trade press and fashion articles.
Public statements by company likely to be very few.
Online fashion is a fairly new industry so could struggle to find information on
other firms to draw comparisons. Could try and use other online industries
that appear to have similar characteristics.
Low barriers to entry which means competition could increase rapidly which
is difficult to factor-in to analysis.
Intern has not completed a report before and might lack the knowledge of how
to complete analysis. He should ask for help and possibly a mentor.
Intern junior status might mean senior management would not be willing to
meet to discuss. The intern should ask someone with market experience to
join them on the visit.

Candidates needed to use the specific details given in the question to answer this part well.
(iii)

Investigation of a recently formed company will be an equal problem for both.


The differences and similarities between carrying out analysis for investment
and the rating agencies approach to rating companies:
Similarities
Both will focus on:

Page 12

the competitive position (relative to peers)


the downside risk of investment vs. the upside potential of the company
the quality of profitability of the company and. EPS growth
cash flow generation vs. book profitability
forward looking analysis
strategy, management track record and risk appetite

Subject ST5 (Finance and Investment Specialist Technical A) April 2014 Examiners Report

Differences

The credit agency focuses on fundamental risks of the companys industry


whereas the asset management analysis is more company focussed from
the outset

Credit agencies give a public ratings and release information on their


analysis, whereas asset management company ratings tend to be internal
only with less client friendly analysis (higher order)

Credit agencies are supposed to be more conservative in their approach to


ratings than an asset management company (higher order)

The credit agencies will have more emphasis on the capital structure and
financial flexibility although asset management will carry out some
analysis

With respect to the credit rating agencies the review is based on


comprehensive information, both public and private (background and
supplemental rating questionnaires). An important component is frequent
meetings and discussions between rating analysts and company management,
providing crucial information and understanding of the companys operations,
financial condition, competitive market position and future business plans.
Although asset managers will focus on similar things credit agencies arguably
have greater access to information.
Part (iii) was generally poorly answered. Many candidates gave relevant details, but few
focussed sufficiently on comparing the similarities and differences.

END OF EXAMINERS REPORT

Page 13

INSTITUTE AND FACULTY OF ACTUARIES

EXAMINATION
1 October 2014 (pm)

Subject ST5 Finance and Investment


Specialist Technical A
Time allowed: Three hours
INSTRUCTIONS TO THE CANDIDATE
1.

Enter all the candidate and examination details as requested on the front of your answer
booklet.

2.

You have 15 minutes before the start of the examination in which to read the
questions. You are strongly encouraged to use this time for reading only, but notes
may be made. You then have three hours to complete the paper.

3.

You must not start writing your answers in the booklet until instructed to do so by the
supervisor.

4.

Mark allocations are shown in brackets.

5.

Attempt all seven questions, beginning your answer to each question on a new page.

6.

Candidates should show calculations where this is appropriate.


AT THE END OF THE EXAMINATION

Hand in BOTH your answer booklet, with any additional sheets firmly attached, and this
question paper.
In addition to this paper you should have available the 2002 edition of the Formulae
and Tables and your own electronic calculator from the approved list.

ST5 S2014

Institute and Faculty of Actuaries

(i)

Define the term risk budgeting in the context of portfolio construction.

(ii)

Outline how the process of risk budgeting is carried out.

[5]
[Total 7]

The following information has been provided for an investment portfolio:


Year 1

Year 2

Benchmark asset allocation:


Equities index
Bonds index

30%
70%

60%
40%

Actual asset allocation of fund:


Equities
Bonds

50%
50%

50%
50%

Investment returns:
Average return for similar funds
Equity index return
Bond index return
Equity return in fund
Bond return in fund

6.0%
10.0%
2.0%
12.0%
3.0%

(i)

[2]

5.0%
8.0%
3.0%
7.0%
4.0%

(a)

List three approaches that could be used to assess relative portfolio


performance.

(b)

Outline the merits and shortcomings of each approach.

[6]

(ii)

Calculate the past performance for the fund as a whole over the two year
period, using the three approaches.
[5]
[Total 11]

(i)

State the principal aims of financial regulation.

[2]

The sales team in an investment bank has developed a new investment product that
offers returns linked to a global equity index. Customer assets are to be pooled
centrally to provide economies of scale. The bank will directly invest the funds into
equity holdings and will also use derivatives.
The product is being marketed as a savings product via the internet and through direct
selling. The sales team will be incentivised through commission payments after each
completed sale.
(ii)

ST5 S20142

Outline the key issues within this scenario that financial regulation aims to
address.
[11]
[Total 13]

In the country of Actuaria, the Society of Actuarian Actuaries has decided to change
the format of some of its actuarial examinations to a multiple-choice format (with no
negative marking for incorrect answers).
An examiner for the Society is in the process of setting questions and is seeking
advice regarding the ordering of the correct and incorrect answers.
The examiner has heard about the theories of behavioural finance and wishes to
ensure that the placement of the correct answers will minimise the chances of the
students simply guessing the correct answers. He is seeking recommendations as to
whether to put the correct answers as the first choice, the last choice or somewhere in
the middle.
Based on the theory of behavioural finance:
(i)

Suggest where the examiner should place the correct answer in the list of
choices (first, last, 2nd, 3rd or 4th) in multiple choice questions with five
options, all outlined in considerable detail.

[2]

(ii)

Suggest where the examiner should place the correct answer (first or last) in
multiple choice questions with two options, both outlined in considerable
detail.
[2]

(iii)

Explain the reasoning behind your answers.

[9]
[Total 13]

A pension fund has seen its funding level improve in recent years from 85% to 102%.
The pension fund has decided to de-risk its investment strategy by reducing its
equity exposure and increasing its exposure to corporate bonds.
(i)

Discuss the relative advantages and disadvantages of direct investment by the


pension fund in corporate bonds versus indirect investment using credit
derivatives.
[12]

(ii)

Discuss to what extent these advantages and disadvantages would be different


if the fund was a hedge fund rather than a pension fund.
[3]
[Total 15]

ST5 S20143

PLEASE TURN OVER

The following market data and information has been provided about a pension fund
wholly invested in US equities:
Date

31 Dec 2012
31 Mar 2013
30 June 2013
30 Sept 2013
31 Dec 2013
Period
(2013)
Q1
Q2
Q3
Q4

Market Value of Domestic Share Index


fund ($000s)
(Capital only)
3,600
3,600
4,050
4,500
4,200

Dividend Yield on
Domestic Share Index
(% per annum)

1500
1603
1776
1797
1680

Contribution Income
(Outgo if negative)
($000s)
56
30
187
52

4.3
4.2
3.9
4.2

Investment Income
($000s)
52
60
60
68

Contributions and investment income all occur on the last day of each quarter.
(i)

Calculate for each period and over the full year:


(a)
(b)
(c)

the time-weighted return.


the money-weighted return.
the index return.

State any assumptions made.

[9]

(ii)

Comment on these returns.

[3]

(iii)

Compare the investment income actually received by the fund with the
investment income that would have been received if the fund had been
invested in the index.

[2]

(iv)

ST5 S20144

Explain the conclusions that might be drawn about the stock selection policy
of the fund, using the information from parts (ii) and (iii).
[3]
[Total 17]

A wealthy individual has decided to diversify her portfolio and wishes to gain
exposure to property returns.
(i)

Outline the main characteristics of direct property investment.

[4]

(ii)

Outline the different ways in which the individual could gain exposure to
property returns.

[8]

The individual decides to invest directly in property and has narrowed down the
investment choice to the following alternatives:

a commercial office block in a major financial centre of a developed country

a residential housing block on the outskirts of a city in a developing emerging


market

a portfolio of hotel rooms situated in, and managed by, a well known hotel chain

(iii)

Compare and contrast the potential investments in respect of:


(a)
(b)
(c)

the overall characteristics each investment exhibits.


the stability of the income stream each investment is likely to produce.
the on-going management of each investment.
[12]
[Total 24]

END OF PAPER

ST5 S20145

INSTITUTE AND FACULTY OF ACTUARIES

EXAMINERS REPORT
September 2014 examinations

Subject ST5 Finance and Investment


Specialist Technical A
Introduction
The Examiners Report is written by the Principal Examiner with the aim of helping
candidates, both those who are sitting the examination for the first time and using past papers
as a revision aid and also those who have previously failed the subject.
The Examiners are charged by Council with examining the published syllabus. The
Examiners have access to the Core Reading, which is designed to interpret the syllabus, and
will generally base questions around it but are not required to examine the content of Core
Reading specifically or exclusively.
For numerical questions the Examiners preferred approach to the solution is reproduced in
this report; other valid approaches are given appropriate credit. For essay-style questions,
particularly the open-ended questions in the later subjects, the report may contain more points
than the Examiners will expect from a solution that scores full marks.
The report is written based on the legislative and regulatory context at the date the
examination was set. Candidates should take into account the possibility that circumstances
may have changed if using these reports for revision.
F Layton
Chairman of the Board of Examiners
December 2014

Institute and Faculty of Actuaries

Subject ST5 (Finance and Investment Specialist Technical A) September 2014 Examiners Report

General comments on Subject ST5


Candidates are reminded of a bias in the paper towards recognising higher level skills and
practical application this is intentional and will continue. Likewise the examination system
does properly allow for prior subject knowledge to be assumed. Investment is a necessarily
practical subject and, at this level, the examiners expect candidates to demonstrate a breadth
and depth of competency as would be expected from a senior student in a frequently evolving
discipline. Hence simple regurgitation of bookwork will never be sufficient to ensure a Pass
grade and this was evident from the dispersion of candidates responses in the more
differentiating questions.
Whilst the examiners accept bullet point style responses, handwriting that is too difficult to
read will lose marks. It should also be stressed that text speak abbreviations are not
appropriate for professional communications, including exam solutions, and will not be
accepted.
Specific comments on the September 2014 paper
Comments on individual questions are incorporated in the solutions below.
Many questions represented opportunities to demonstrate higher level skills in terms of nonstandard/practical application of theory to current or unusual issues in investment. Most
candidates seemed to identify and understand the key issues being examined and appreciated
the general content of solutions that the examiners were looking for however those that
were unsuccessful will find their solutions lacked sufficient (and often the most basic) detail
or application of knowledge and scored lower accordingly. Thus, weaker candidates found
difficulties with Question 4 and 5, and the later parts of Questions 3 and 7.
Whilst some candidates are too narrow in their responses, a greater number still deviate from
the topic and include irrelevant material or over emphasise minor points. Although candidates
will not be explicitly penalised for this, it gives an impression of a lack of understanding and,
more importantly, wastes limited time. Time and priority management are key skills
actuaries need to have.
Weaker candidates often fail to respond to the specific issues included in the question.
Instead, they regurgitate a generic answer based on the syllabus topic. More care needs to be
given to crafting answers that directly address the points raised in the question.
Where candidates made relevant points in other parts of their solutions, the examiners have
used their discretion as to whether to recognise these answers or not. Likewise the examiners
share and agree alternative possible solutions to questions alongside the approach outlined
below.

Page 2

Subject ST5 (Finance and Investment Specialist Technical A) September 2014 Examiners Report

(i)

The term risk budgeting refers to the process of establishing how much
investment risk should be taken and where it is most efficient to take risk in
order to maximise return.
[2]

(ii)

A feasible set of asset classes that could be included in the portfolio (subject
to any constraints specified in the mandate / investment agreement) are first
analysed. This will consider the expected returns, volatilities and the
covariances between asset class returns.
Some risk / return optimisation process is then used to select an initial asset
allocation between the asset classes. A Value at Risk assessment will be used
to determine the total risk budget the risk tolerance in respect of the
exposure to potential loss on the portfolio. The total risk budget is then
allocated between strategic risk and (total) active risk, and finally the total
active risk is allocated among the various asset managers.
It is important that the developing position of the chosen portfolio is
monitored to assess risk exposures (increases and decreases in the value of the
positions) and changes in volatilities and correlations. The portfolio will need
to be rebalanced in the light of such changes, in order to keep the overall
portfolio risk at the level defined as tolerable.
[5]
[Total 7]

In answering part (ii), weaker candidates did not pay sufficient attention to HOW the risk
budget would be determined. Instead, they focussed on how the budget would be
administered once it had been determined.

(i)

Portfolio
relative to:

Published Indices

Other Portfolios

Benchmark portfolio

Pros

Easy to do
Data readily available,
and accurate

Gives an indication
of the cost or benefit
of a strategy, relative
to those adopted by
other funds

Benchmark portfolio
can be constructed to
reflect fund
objectives

Shows relative
manager skill in
stock / sector
selection

Can be helpful in
aligning fund
managers interests
with liability
requirements

Page 3

Subject ST5 (Finance and Investment Specialist Technical A) September 2014 Examiners Report

Cons

Index may be
inappropriate for
investors objectives

Comparison may be
inappropriate if other
funds have very
different objectives
or are exposed to
very different
conditions
Lack of available
data

General cons

All methods look at past performance only, so are not a reliable guide
to the future
Assessments do not take account of risks taken by managers
[6]

Candidates who suggested the use of risk-adjusted performance measures were also given credit.
(ii)
Year 1

Year 2

Total

ActualExpected

Actual

7.50%

5.50%

13.41%

Index return

6.00%

5.50%

11.83%

1.58%

Average return for


similar funds

6.00%

5.00%

11.30%

2.11%

Benchmark

4.40%

6.00%

10.66%

2.75%
[5]
[Total 11]

Candidates who analysed stock and sector selection performance attribution were also given
credit.
Most candidates scored well on this question. However, it was very disappointing to see that
some candidates added together the annual performance returns in order to calculate the
two-year result.

Page 4

Subject ST5 (Finance and Investment Specialist Technical A) September 2014 Examiners Report

(i)

The principal aims of regulation are:

to correct market inefficiencies and to promote efficient and orderly


markets
to protect consumers of financial products
to maintain confidence in the financial system
to help reduce financial crime
[2]

(ii)

Specific issues within this scenario include:


A new investment product. The new product will need to comply with the
existing legislative framework. As this is a new product there will need to be
appropriate documentation describing the purpose and operation of the fund to
investors.
Returns linked to the global equity market. This objective must be clearly
set out and defined. Precisely what measure of global equity market
performance will be used to determine the return on the new product? Will
this be a capital only or a total return? Will it be gross or net of tax if so,
at what rate?
The treatment of income earned needs to be set out. Will the product provide
regular income payments, or will these be reinvested?
Will the product be issued for a specified term, or in open-ended form?
What will the charges be on early redemption?
What overall charges (initial and recurring) will be levied? Will there be any
performance guarantees?

Customer assets to be pooled centrally. What arrangements will the bank make for
the proper protection of customer assets (e.g. by segregation and identification of
those assets)?
Use derivatives. Are there to be any limitations on the use of derivatives? How will
associated risks (e.g. counterparty risk) be addressed? How will any associated
margins and collateral payments be funded? In the event of losses (relative to the
global equity market returns that the product will track), how will these be recouped?
Does the firm have adequate financial resources (now and in the future)?
The staff responsible for the investment decisions related to the product should be
adequately trained and properly supervised through well-defined compliance
procedures.
Marketed as a savings product. The product appears to be marketed ambiguously as
a savings product but could have a high level of risk attached, which needs to be
clearly explained to potential customers.

Page 5

Subject ST5 (Finance and Investment Specialist Technical A) September 2014 Examiners Report

Marketed via the internet. To market via the internet means that customers do not
necessarily receive appropriate advice. This means firms should seek information
from their customers about their circumstances and investment objectives. How will
the bank seek this information from customers? How will information be provided to
customers in an ongoing, comprehensible and timely way? What additional advice
services will the bank offer to customers (and what charges will be levied for this)?
How will the bank monitor against fraud, money-laundering and tax avoidance by
customers? Customers should be allowed a cooling-off period to withdraw from the
contract.
Marketed via direct selling. The staff responsible for speaking to potential
customers should be adequately trained and properly supervised through well-defined
compliance procedures.
...incentivised through commission. How will possible conflicts of interest be
avoided? How will the bank/sales team demonstrate that it is acting with due skill,
care and diligence? What arrangements will be made for ensuring that the staff are
suitable, adequately trained and properly supervised?
Within all the above points there should be an overarching principle that the firm
should act with integrity and observe high standards of market conduct.
[11]
[Total 13]
As with most case study questions, candidates were required to use the specific
information given in the question to frame their answer. Some candidates, however,
produced a generic answer on the general subject of financial regulation with little
reference to the points set out in the question.

(i)

Recommend placing the correct answer as the 1st (or possibly the 2nd answer)
because the last answer is probably most likely to be chosen (assuming the
individual is attempting to read each answer).
If candidates are randomly choosing answers without reading them, then
behavioural finance may not be very relevant.

(ii)

Recommend placing the correct answer as the last answer because the first
answer is probably most likely to be chosen (assuming the individual is
attempting to read each answer).
If candidates are randomly choosing answers without reading them, then
behavioural finance may not be very relevant.

(iii)

The reasoning is based on the primacy effect, recency effect, anchoring and
the effect of options.
Primary effect people are more likely to choose the first option presented

Page 6

[2]

[2]

Subject ST5 (Finance and Investment Specialist Technical A) September 2014 Examiners Report

Recency effect in some instances, the final option that is discussed may be
preferred.
Anchoring is a term used to explain how people will produce estimates. They
start with an initial idea of the answer (the anchor). They then adjust away
from this initial anchor to arrive at their final judgement.
A greater range of options tends to discourage decision-making.
In (i) the candidate's anchor could be considered to be the first answer,
assuming that he/she read each answer and guessing, but they are likely to
adjust away from it with each successive answer, making it less likely to be
chosen.
The primary effect may be worn-off from this adjusting away. With the
number of options being five, and with each requiring significant
consideration due to the detail in each, it is likely that the effect of the greater
range of options will discourage decision making further reducing the
primary effect.
This might leave the recency effect to be dominant and result in candidates
who read each answer and just make a guess, choosing the last answer. So
placing the correct answer towards the start is likely to minimise the chances
of the candidate just guessing the right answer.
In (ii) the candidate's anchor could again be considered the first answer
shown assuming that he/she read each answer and was guessing. There is
only one further answer so the effect of adjusting away from the answer will
be smaller than in (i). There is a lower number of options compared to (i)
this would mean decision making will be discouraged to a lesser extent so
impacting to a lesser degree on the primary effect and the anchoring effect.
The recency effect is likely to encourage the candidate to choose the last
answer. However, the combined effect from both the primary effect and
anchoring is likely to outweigh it, consequently the dominant bias would
probably be to choose the first answer.
So placing the correct answer at the end is likely to minimise the chances of
the candidate just guessing the right answer.
Other answers that showed application of the principles of behavioural
finance were also awarded marks (e.g. regarding negative answers being less
likely to be chosen, imaginable answers being more likely to be chosen,
answers involving change being less likely to be chosen etc.). However, only
points relevant to the described scenario were credited. For example, the
question stated that the options were all outlined in considerable detail.
Thus, references to framing were not generally relevant.
[9]
[Total 13]

Page 7

Subject ST5 (Finance and Investment Specialist Technical A) September 2014 Examiners Report

(i)

It is more likely that the pension fund and/or their investment managers will
have the experience and expertise to invest in corporate bonds than to invest in
credit derivatives. However, they might not have the expertise to directly
invest in corporate bonds, e.g. they may not have individual corporate bond
stock-picking expertise, and instead be relying on investing in a corporate
bond fund.
Direct investment in corporate bond could be regarded as being easier to
understand and cheaper to manage.
Direct investment in corporate bonds may be limited by restrictions, e.g. based
on foreign ownership.
Administration arising from investing using credit derivatives is likely to be
more involved and require additional administrative expertise, e.g. setting up
an ISDA agreement.
The indirect investment using credit derivatives could in theory be carried
using a pooled vehicle which invested directly using credit derivatives, e.g. an
ETF. Alternatively, cash could be invested in government bonds with a credit
derivative overlay to provide the required credit exposure.
Investing directly in corporate bonds, will likely result in a regular income in
the form of coupon payments investing in credit derivatives will usually not
result in such cash flows. This may be an advantage or a disadvantage
depending on the cash flow requirements of the pension fund. Where the
pension fund does not require the cash flow, it may necessitate coupon
reinvestment. The same consideration applies with regard to a government
bond / credit derivative overlay structure.
Investing directly in individual corporate bonds may result in less
diversification than investing using credit derivatives, unless the credit
derivatives are also based on individual corporate bonds. Smaller pension
funds may not be able to achieve adequate diversification directing investing
in corporate bonds and instead may opt to invest in corporate bond funds for
diversification reasons. The minimum unit size of the latter is likely to be
significantly lower.
Alternatively, investment in corporate bonds may allow greater choice and
diversification of exposure.
In recent years, marketability and liquidity in credit derivatives markets has
been better than in the underlying corporate bond markets making them
more attractive from this perspective.
The greater marketability and liquidity is also likely to mean that transaction
costs (bid/offer spreads etc.) are lower for investing using credit derivatives. If
short-dated credit derivatives are used, any necessary rollover of contracts is

Page 8

Subject ST5 (Finance and Investment Specialist Technical A) September 2014 Examiners Report

likely to result in additional costs. However, longer dated contracts are more
likely for a pension fund.
Using credit derivatives would result in an additional counterparty risk
versus direct investment in corporate bonds. The size of the counterparty risk
is linked to the credit rating of the intermediary involved.
Credit derivatives may enable longer durations to be available which might be
more attractive for a pension fund with liabilities of long durations.
Credit derivatives may be short term in nature and be exposed to roll risk
due to the costs and risks associated with rolling the positions.
There may be tax implications depending on the country of origin of the
pension fund and could result in either strategy being advantageous over the
other from a tax perspective.
Credit derivatives may be subject to more onerous regulation than direct
investment in corporate bonds.
Credit derivatives can allow gearing and leverage but may not be desirable
for pension funds. The investment mandate may not allow use of credit
derivatives due to say restrictions on the use of derivatives.
Over-the-counter credit derivatives could be customised to suit the specific
requirements of the pension funds investment strategy e.g. regarding
duration.
In general, indirect investment is particularly suitable for small funds,
although even large funds can sometimes benefit from vehicles investing in
specialist areas which are outside the funds own areas of expertise.
[12]
The question related explicitly to the new asset class. Some candidates discussed the use of
derivatives in managing transitions between asset classes, but this was generally irrelevant,
as was any suggestion that it was planned to reverse the switch in the short term. It was not
generally appreciated that the use of credit derivatives in this scenario was to ADD credit
exposure e.g. to a core holding in government bonds, in order to replicate the returns that
would be expected from a portfolio of corporate bonds.
(ii)

The gearing and leverage available from using credit derivatives is likely to be
of more interest to a hedge fund than to a pension fund. This is because the
pension fund has underlying funds to invest, while hedge funds typically
attempt to leverage their clients funds.
Since investment mandate of the hedge fund clients is likely to be less
restrictive than that of the pension fund investing in credit derivatives maybe
more feasible.

Page 9

Subject ST5 (Finance and Investment Specialist Technical A) September 2014 Examiners Report

OTC customisation could be regarded as less important to a hedge fund than


to a pension fund unless it was part of a hedging strategy or as part of an
arbitrage trade.
[3]
[Total 15]

(i)

As contributions and investment income all occur on last day of each quarter,
the index returns need to be calculated on a similar basis using the yield at the
end of each quarter. Assumptions underlying the calculations relate to the
impact of tax, dealing costs and the accuracy of the data used.
Time weighted return (%)
Q1

Q2

Q3

Q4

(1.56)%

11.67%

6.49%

(5.51%)

10.61%

Money weighted return (%)


The quarterly returns are the same but the annual return is derived from
3,600 (1 + i) + 56(1 + i)+ 30(1 + i) + 187(1 + i) = 4,252
This can be approximated by
3,600 (1 + i) + 56(1 + 3i/4)+ 30(1 + i/2) + 187(1 + i/4) = 4,252
(1.56)%

11.67%

6.49%

(5.51)%

10.23%

Index time weighted return (%)


R(t) = ((I(t)*(1 + Y(t)/4))/I(t 1) 1)*100
where R(t)
I(t)
Y(t)
8.01%

= total return for period t


= index value at time t
= yield on index at time t
11.96%

2.17%

(5.53)%

16.72%
[9]

Not well answered, in general. The question explicitly specifies that contributions and
investment income all occur on the last day of each quarter but some candidates ignored
this and made alternative assumptions. Many failed to appreciate that the fund values given
in the question therefore included the income items.
(ii)

Money and time-weighted are same for each quarter because cash flows occur
at the end of each quarter, but annual is different and reflects time of cash flow
v market movements.
Both under performed the index by a considerable amount.

Page 10

Subject ST5 (Finance and Investment Specialist Technical A) September 2014 Examiners Report

The first quarter is the period accounting for all the under performance.
There is strong out performance in Q3.
Given the difference in income, capital return for the fund has been very poor.
[3]
(iii)

Assuming that investment income is received at the end of the quarter (as
specified in the question):
Period

Q1
Q2
Q3
Q4
Total

Fund
Income
52
60
60
68
240

Index
Income
41.4
41.9
40.0
44.2
167.5

[3,600*1,603/1,500*0.043/4]

[2]
Again, this part of the question was not well answered. Weaker candidates applied the
ANNUAL dividend yield to calculate the index income for the QUARTER. Few candidates
calculated the quarter-end fund value based on the index performance in order to project the
income.
(iv)

As can be seen the fund was invested in stocks that yielded 40% more than the
average for the index.
It is likely that high yield stocks under performed in the year in question as
overall the fund under performed the index by a considerable margin.
The fund manager may have a yield requirement. If this is the case then
perhaps a different index should be used to monitor performance.
[3]
[Total 17]

(i)

Illiquid
Large size
Each property is unique
Non-exchange traded
Expensive to purchase and dispose of
Difficult to value
Purchase prices are often not disclosed
Requires a lot of on-going management
Provides real returns
Offers diversification from other asset classes

Page 11

Subject ST5 (Finance and Investment Specialist Technical A) September 2014 Examiners Report

Risk of voids
Risk of obsolescence
[4]
(ii)

Direct buying property outright


Investing in specialist property fund e.g. REITs
Investing in a property index (passive) fund e.g. ETF
Equities where underlying company has significant exposure to property (i.e.
property developer)
Structured product with link to property returns
MBS
ABS
Swaps
[8]

(iii)
Commercial office
block
(a)

Residential housing
block

Hotel rooms

Overall
characteristics
Liquidity

Illiquid

Most illiquid (nonprime)

Depends on location

Size

Sizeable

Sizeable

May be resold in
smaller lots more
marketable

Uniqueness

Not particularly

Driven by location
and infrastructure
factors

Very location
dependent (local
transport links,
visitor attractions)

Expensive to
trade

Less high (unless a


high tax economy)

Highest

Lower purchase costs

Difficult to
value

Stable (stronger
demand / more
stable economy).
Better price data
available

Less stable
(developing
economy).
Less reliable price
data.

No actual physical
property.
Volatile

Stability of
value

Most stable
stronger demand /
stable economy

Less stable

Driven by economic
environment more
volatile

Real returns

Yes

Largely, but depends


on local conditions

No volatile returns

Page 12

Subject ST5 (Finance and Investment Specialist Technical A) September 2014 Examiners Report

Commercial office
block

Residential housing
block

Hotel rooms

Risk of voids

Professional /
company tenants
most stable

Retail tenants
greater risk of voids

Volatile /
unpredictable. May
be seasonal (and
weather dependant)

(b)

Stability of
income stream

Most stable.
Long leases with
upward rent reviews

Will depend on
location and
economic conditions.
Shorter leases.
May be affected by
political
considerations.

Affected by local
and international
economic conditions
(including exchange
rates).
Least stable.

(c)

Ongoing
management

Least expenditure
needed mainly
maintenance of
shared services.

Significant. Estate
management costs
plus resale / reletting
of empty units.

Considerable
expenditure needed
to maintain quality
and standards.
Most expensive (but
this may be borne by
the hotel chain).
[12]
[Total 24]

Part (iii) was not well answered by weaker candidates, who failed to apply many of the
characteristics identified in part (i) in assessing the alternative investments. There was
insufficient appreciation that the portfolio of hotel rooms represented a source of rental
income (akin to sale-and-leaseback) rather than a physical property investment.

END OF EXAMINERS REPORT

Page 13

S-ar putea să vă placă și